1

GOLDEN FILE# 3 INCLYDES SEPTEMBER, OCTOBER AND NOVEMBER PAPERS- by Dr. Rafi ullah

Author Notes about the controversy regarding his & Dr Salah-ud-din Kamal book(s) is removed as the aim of file sharing is to help juniors. Although I appreciate the author and all those complied these tests for upcoming juniors.

2

SEPTEMBER 6, 2016 EVENING 1. 2. 3. 4. 5. 6. 7. 8. 9. 10. 11. 12. 13. 14. 15. 16. 17. 18. 19. 20. 21. 22. 23. 24. 25. 26. 27. 28. 29. 30. 31. 32. 33.

Regarding Aphasia; Lesion in =Temporal lobe Fascia enclosing masseter muscle =Deep Cervical Fascia There was a scenario female with webbed neck and short stature = turner syndrome A Scenario of Male with XXY =klinfilter syndrome Decrease in core body temperature results in = Shivering Virus cause disease by = Alteration in protein synthesis Regarding node of Ranvier = no Myelin Highest TGs concentration = Chylomicrons Regarding hemorrhagic infarct = Lungs Type of collagen present DURING wound healing = Type 3 Regarding reticular connective tissue = Spleen Smoking working in tyre factory, cause of trAns:itional cell carcinoma = Smoking > Aromatic.(ref Goljan) Supernumerary ribs = compression of Brachial plexus and subclavian Diseased vs non diseased = Case control Gardener with linear lesion = Cutaneous larva migrAns: Repeat question of a diabetic who died despite aggressive measure = Mucormycosis Insulin inhibited by = B blockers Vitamin in fatty acid oxidation = biotin Aggressive sympathetic drive = Bronchodilation Most important response in shock as a whole = CNS ischemic response Increased BT is observed in = vWF deficiency Regarding Renal vein = Anterior to Aorta Left adrenal vein drainage = Left Renal Vein Retromandibular vein = Maxillary with superficial temporal Chromatids align at equatorial plain = Metaphase Child presented with proteinuria of >3gm, cause on edema? = Decrease colloid osmotic pressure Natural Defense Against Cancer = Apoptosis Step in Apoptosis = Activation of caspases Child with macrocytic type CBC = B12 level Ans: Elastic cartilage present in = Epiglottis Regarding Hyaline cartilage = Articular surfaces Mother having measles, child at risk of =Cataract Mother delivered a baby with cataract, cause? = Measles

3 34. 35. 36. 37. 38. 39. 40. 41. 42. 43. 44. 45. 46. 47. 48. 49. 50. 51. 52. 53. 54. 55. 56. 57. 58. 59. 60. 61. 62. 63. 64. 65. 66. 67. 68. 69. 70. 71. 72. 73. 74. 75. 76.

Sound produced in Ventricular filling = S3 Which of the following decreases serum osmolality?= antidiuretic hormone Thiazide diuretic = Hypokalemia Fracture of Lumbar vertebra = Conus medularis syndrome True about osmosis? = its osmolality depends on particles number Regarding nucleus? = Euchromatin is trAns:criptionally active In severe dehydration = decrease in total body water A patient with severe dehydration drinks 2L tap water: inc ICF (reference ganong) Global cause on infective blindness: = Chlamydia > bacteria Internal carotid artery: anterior choroidal branch to cavernous sinus Leukocyte Adhesion = LFA1 True about Leukocytes = Phagocytic in blood Repeat case of= EXTRAHEPATIC CHOLESTASIS Preganglionic autonomic Fibers = B type T cell Receptor bind with = MHC 2 Secondary ossification center present in? = lower end of femur Associated with intestinal malignancy = H. Pylori Decrease ESR = Increase Albumin Patient aspired on lying down = Right lower lobe Irreversible step of glucose metabolism? = G6P to fructose 1,6 po4. In second degree heart block = Progressive lenghthening of Pr interval until a beat is dropped(type1) 41% hematocrit = 41% RBC WBC Platelets Structure embedded in cerebral cortex? = Insula SA node location = upper part of sulcus terminalis Ureteric bud development = Mesonephric duct Patient presented with chest pain of 2-4 hours’ duration, all baseline was normal next investigation = CK-MB Mechanoreceptor of Warm = Long receptive fields Gracilis and cuneatus damage = Astereognosia PICA supply = dorsolateral medulla Cells having most telomerase activity = Germ Cells Most drugs are metabolized in = Liver Pseudomembrane colitis = C. difficile Reason of tuberculin test negative = taking immunosuppressant Case of PARKISON with all associated symptoms like tremor, bradykinesia, rigidity gait problem, lesion in = Substantia Nigra Heart Rate of 120 = RR interval 0.5sec 60 years old farmer with Cushing symptoms = Adrenal Adenoma Malignant tumor = Metastasis 2*2 table = Chi Square Which augments the effect of phagocytes? = Complement Metastasis occurs because = loss of E cadherin Allele = Non identical genes Hypovolemia results in = Decrease carotid sinus nerve firing Typhoid investigation in first week = Blood Culture

4 77. 78. 79. 80. 81. 82. 83. 84. 85. 86. 87. 88. 89. 90. 91. 92. 93. 94. 95. 96. 97. 98. 99. 100. 101. 102. 103. 104. 105. 106. 107. 108. 109. 110. 111. 112. 113. 114. 115. 116. 117. 118.

Thiamine deficiency causes = Peripheral neuropathy Increase in GFR and RPF= afferent arteriolar dilatation Cimetidine effect = Inhibit hepatic enzymes Inducers of liver enzymes = Phenobarbital 50 years’ post-menopausal lady = Increase FSH and LH Severe trAns:fusion reaction occurs = A+ to O+ Which drug increases LES tone and gastric motility = Metoclopramide NK Cells = Kills virally infected cells without sensitization Cushing features with Raise ACTH = Cushing disease Mediators of Pain = Bradykinin and PG Young patient dyspnea on lying down = Retrosternal goiter Breaking bad news = Information in crisp and formal sessionin exclusivity . QRS complex occur = Prior to ventricular systole Alpha and gamma motor neuron = UMN supply Atheroma development major factor = HTN Response to bleeding due to dermal abrasion = vasoconstriction Increase thyroid hormone cause = increased in gluconeogenesis There was a female with malar rash = Anti DSDNA Regarding somite’s = Arise from paraxial mesoderm Early sign of Vitamin A deficient= Night blindness Partial mole = 69XXY Lady presented with uterine bleed due to coagulation failure secondary to miscarriage=Evacuation . Regarding sensory fibers = May pass or may not pass reticular depending upon modality Motor Fibers =Supply alpha and beta simultaneously Young female with decrease Hb, Dec mch, hbA 94% hbA2 was 4% = Thalassemia trait A smoker patient, was having interstitial lung disease and pleural plaque= Asbestosis Compression of sciatic nerve level was asked = L4,L5 Best measure of GFR estimation = Creatinine( if best measure asked:inulin). A patient with BP200/120 and glucose >200, what to give = Lisinopril FEV1/FVC = 0.8 Bronchial asthma = decrease FEV1 4th heart sound correlates with = Atrial contraction against stiff ventricles Rapid acting hypoglycemic drug = second generation sulfonylurias. Autonomic supply to mid gut = T8-t12 100cm of ileum resected = decrease absorption of bile salts How to monitor a patient on warfarin = PT, APtt, INr Increase Bt and Aptt normal PT = vWF defect Ganglion found at c7 vertebra = Cervicothoracic Patient having constricted pupil, unable to dilate it = cervical sympathetic Regarding esophagus = upper 1/3rd striated Color Blindness due to = Cones defect In severely dehydrated child, venesection was performed; most likely nerve to be damaged = Saphenous

5 119. 120.

Osmotic diuretics act upon = PCT A known case of HCV, histology of liver shows eosinophilic cytoplasm, nuclear clumping = Councilman bodies. 121. A patient presented with fatigue, mcv104, alt 74 ast 80 = B12 122. A patient presented with irregularly irregular pulse, p wave was replaced by irregular waves = Atrial Fibrillation 123. Digoxin given in =Atrial Fibrillation 124. Atonic bladder =Damage to parasympathetic below sacral 125. Most radiosensitive tumor =Lymph node 126. A patient presented with acute exacerbation of COPD, Po2 45, Pco2 51, pH 7.31, Bicarbcarbonates =Type II respiratory failure. 127. A patient doing exercise, increased respiration. Which of the following will activate, that remains inactivated at normal respiration = Ventral respiratory group 128. Mechanism of action of Aspirin = inhibit COX( irreversibily both cox1 n 2) 129. OndAns:etron control vomiting by = antagonism pf 5HT3 130. Itching is carried by = Type C 131. Viral infection and purpura = antibodies to Platelets (ITp) 132. Abduction and medial rotation or arm = Teres Major 133. Microscopic feature of malignancy = Invasion> pleomorphism 134. Brain perfusion at rest = 50ml/100gm 135. Brain auto regulatory pressure? = 50-150mm Hg 136. Coccygeal ligament = L1 137. On right lung bronchoscopy first section to be visualized? = inferior bronchus 138. Narrowest point of esophagus = Cricopharynx 139. Point mutation occurs in = Sickle cell disease 140. A patient diagnosed as amyloidosis having plasma cell disease, amyloidosis is due to deposition of = Amyloid light chain 141. Which of the following function through cAMP = Glucagon 142. Which of the following passes through aortic hiatus? = Azygous vein 143. Norepinephrine secreted at = post ganglionic sympathetic 144. Patient from Baluchistan presented with fever, hepatosplenomegaly, and has history of fly bite = Kala Azar 145. Intracranial veins connected to extracranial= Emissary veins 146. A child with congenitally missing dorsalis pedis artery, blood supply to dorsum of foot maintained through = peroneal artery 147. A child got blow at elbow, elbow joint displaced, now he is having loss of sensation in little finger = ulnar collateral ligament. 148. 35 years old male patient died suddenly, autopsy finding nil, tox screen shows cocaine = contraction band necrosis 149. Edrophoneum most common side effect =Nausea 150. In females, neck of bladder lies = above urogenital diaphragm 151. Perineal innervation = S2,3,4 152. Death due to C. Botulinum = Respiratory Muscles Paralysis 153. A female having blurred vision, decreased muscle activity, other neurologic nerve involvements = demyelination of nerves 154. Antibodies to post synaptic Ach receptors = Myasthenia Gravis

6 155. 156. 157. 158. 159. 160. 161. 162. 163. 164. 165. 166. 167.

168. 169. 170. 171. 172. 173. 174. 175. 176. 177. 178. 179. 180.

Loss of sensation of lower teeth and chin = Inferior Alveolar Nerve Alpha 2 agonist used in ICU for sedation= Dexmedetomidine Decrease release of Oxygen from Hb = decrease in Temp Hyperuricemia is side effect of ATT = Pyrazinamide AIHA diagnosed by = Anti Globulin Antibody Test (coomb ‘s test). One question was on = OCCULOMOTOR Loop C. Diphtheriae = secretes Exotoxin Benign tumor is = Warthin On biopsy, endocervix show squamous cells = Metaplasia Which structure loops around arch or aorta = Left recurrent laryngeal nerve Sound heard in left 3rd Intercostal space = Aortic A2 Regarding kidney trAns:plant = HLa1 more important than HLA2 Scenario was given 5 years’ arthritis history with congestive cardiac failure features cardiomegaly and glucose was 167 what lab finding in this patient you will find = raised Ferritin Benzodiazepine cause =Dependence Highest pH is seen in = Pancreatic Secretion A child presents with perianal pruritus; which cells will be elevated = Eosinophils Gastric HCL is necessary for = Converting pepsinogen to pepsin In pleural fluid aspiration in mid axillary line which structure will not be pierced = Levator Costrum Left Ventricular Failure cause = Aortic Valve pathologies A patient having a lot of pizza and burgers = Hypertrophy of Steatocytes There was one question on coronary circulation = run in respective grooves sciatic Nerve root = L4 L5 S1,2,3 Which of the following cause paraneoplastic syndrome = Small Cell CA A patient was presented with Malar Rash and other symptoms of SLE what other finding can be found? = Pericarditis Blood flow to muscle during exercise = Inc. local metabolite Exophthalmos occurs due to = TSI

7

8

7 SEPTEMBER MORNING SHIFT…….. MED AND ALLIed 1.theophyline interaction with which causes decREASE its clearance a.adenosine b.cimitidine c.beta bloker d. litium Ans:: aa 2.most common premalignnt condiotion… a.leukoplakia b.erythroplakia c.lichen planus d.apthos ulcer Ans:: cc 3.painful vesicular eruption at chin and dermatomes involve diagnosis a.trigeminal neuralgia b.herpes Ans:: bb 4.Immunoglobulins produced by: a. plasma cells b. hepatocytes Ans:: aa 5. following produce by plasma cells a. albumin b. firinogen c. gama globulin Ans:: cc 6. an enzyme which combine two and make a single compound a. synthetase b. isomerase c.trAns:ferase Ans:: aa 7.Intercoastal nerves a.only supply skin b.12 in number c.Supply only intercostal muscles d.Associated with sympathetic chain Ans:: cc 8.percentage of lichen planus chance to covert in malignancy a.1-10% b10-15 c.15-20 d.20-25 e.25-30

Ans:: aa 9 . after tooth extraction cause of sub acute endo carditis a.viridins goup of streptococci b staph aurus c.fecalis Ans:: aa 10.Most common cause of subacute endocartis a.viridiAns: streptococi b. staphloccus aureus Ans:: aa 11. Blood born infection A. staphlococus b.hepatitis B Ans:: bb 12.Child with sore throat and fever you gave antibiotics...after 3 weeks hematuria occurs a. good pasture syndrome b.poststreptococal glomerulonephritis Ans:: bb 13.Increase LES tone and speeds up gastric emptying ? a. Metoclopramide. b.OndAns:etron. Ans:: aa 14.there is achalasia of lower esophagus a. no myentric plexus in lower esophagus b.food rapidly goes through esophagus Ans:: aa 15. Dignosis for Hashimotos a.antimitochondraial antibody b.Antimicrosomal and anti thyroglobulin Ans:: bb 16. thyrocervical trunk a. suprascapular branch b. arise from second part of subclavian c.superior intercostal is branch Ans:: aa 17. High carbohydrate diet taking should have in diet or deficientin a.thiamine b.riboflavin Ans:: aa 18. Muscle of back supplied? a. dorsal rami of cervical spinal. b. ventral rami of spinal nerves. Ans:: aa 19.Cavernous sinus contain A. abducent nerve

9 b. trochlear c. oculomotor Ans:: aa 20.premalingnat nevus is a.compound nevus b.intradermal nevus. c.junctional nevus. Ans:: cc……as There is no option of dysplastic or congenital giant nevus 21.hyperthyroid ,goiter and exophthalmos a.toxic goiter b. graves disease Ans:: bb 22.Alchohol cause deficiency of which vitamin A, vitamin b1 b.vitamin a c.vitamin d Ans:: aa 23.Sle diagnostic test a.anti double strand antibody( anti DsDNA Ab) b. Anti Ach Ab. c. AMA. Ans:: aa 24.Sjogren associated with? a.sle b. RA Ans:: bb. 25.lumber puncture site a. above l4 b. below l5 Ans:: aa 26. baby has abdominal pain and anemia diagnosis.....repeated scenario of sickle a.hbss b.hb a c.hb cs Ans:: aa 27. Hyperbaric oxygen use in treatment of sea divers.. a.barotrauma b.decompresion sickness Ans:: bb 28. optic chiasma damage with also hypothalamus damage will result in a.dibetes inspidus b. hyperphgia c.hypertension. Ans:: bb

29. Man has non lethal MI what happens to him in few minutes? A. complete recovery without any complication b. ventricular arrhythmia Ans:: bb 30 What decreases penicillin excretion from kidney a.Probenecid b.Erythromycin c.Gentamicin d.Benzathin penicillin e.Penicillin G Ans:: aa 31.Dead space decrease in A.standing b.tracheostomy c. deep inspiration d. shallow breathing Ans:: bb 32. SA node action potential due to a.Na influx b.Ca influx Ans:: bb. 33. Large intestine most accurate a.Caecum may have a diameter of 9cm b.TrAns:verse colon supplied by right colic c.Appendix is rarely retro caecal d.20% have mesentry attached to ascending colon e. Descending colon lies medial to left kidney Cecum Ans:: dd 34. Mechanism of action of methyldopa a.Binds adrenergic receptors in cns b.Acts directly on arteriolar smooth muscles Ans:: aa 35. Left axis deviation in A .Right sided myocardial damage b.RBBB Ans:: aa 36. while taking blood sample at cubital fossa which nerve injured? a. median b.ulner c.radial Ans:: aa 37. Common peroneal nerve lesion a. unable to evert b .unable to invert c. unable to planter flex ankle and big toe

10 d Loss of sensation on medial side of leg. Ans:: aa 38.A child has bald patch on scalp since birth. Which is leathery. On microscopic exam there is papillomatous projections in epidermis. Dermis main abundance of sebaceous glands What is the serious complication a.Basal cell carcinoma b.Squamous cell carcinoma c.Keratocanthoma d.scaling alopecia Ans:: cc 39. Mgso4 action inhibited by a. Na b. Cacl2 c. K Ans:: bb 40.Thiazides decreses clearence of a.Class 2 antidepressants b.Phenothiazide antipsychotics c.TCA d. lithum Ans:: dd 41. patient cant flex index finger and loss sensation of lateral palmer surface .which nerve damage a. median b. radial c. ulnar Ans:: aa 42. Heparin primary action achieved by binding to a. Antithrombin 3 b. Xa c. Thrombin Ans:: aa 43. Skin incision with sharp object redness due to a. Axonal reflex b. Histamine Ans:: aa 44. Angina may be worsened by A. Atropine B. Salbutamol C. Theophyline D. Vassopressin E. Verapamil Ans:: cc 45. Most important feature of shock a.tissue hypoxia

b.hypotension Ans:: aa 46.bar body in xxx a. 1 b. 2 c. 3 Ans:: bb 47.Short arms /legs . Lordosis . With normal mental health . Disease mode of trAns:mission.. (achondroplasia) a. autosomal dominant b. autosomal recessive c. x-linked Ans:: aa 48. adult Polycystic kidney disease mode of trAns:mission a. autosomal dominant b. autosomal recessive Ans:: aa 49. L dopa given in Parkinsonism a. decrease drug inactivation in periphery b. increase peripheral utilization c. increased upake into plasma Ans:: aa. 50.Vomiting present and absent a.Ratio b.Nominal c.Ordinal d.Pametrics Ans: : bb 51.hormone increase in response of meal in duodenum a.cck b.secretin c. lipase Ans:: aa 52.increase gall bladder contractility a.cck b.gastrin Ans:: aa 53. Heparin acts by binding to a.Factor xa and thrombin b.Factor xa only c.. factor 8 d.Factor 7 Ans:: aa 54. In pancreatic fistula action of somatostatin??? a. Inhibit enzymes b Inhibit secretion Ans:: bb

11 55. Inversion and eversion of foot on which joint a.subtalar joint b. calcaneocuboid. c.talonavicular Ans:: aa 56.Phrenic nerve relation to ant to which muscle a.scalnues anterior b.scalneus medius Ans:: aa 57.Phrenic nerve root value a.C3c4c5 b.C2,c3,c4 Ans:: aa 58. Pregnant lady go into obstructed labour and reach hospital 50 miles away when reached hospital she died....cause? A.Fat emboli b.amniotic emboli Ans:: bb 59. Aspirin should be stopped before surgery. How many days the effect of aspirin remains? A. 7-9 days B. 3-4 days C. 10-12 days D. 14-16 days Ans:: aa 60. Heart chamber related to sternum (costochondral surface) a. right ventricle b. right atrium c. left ventricle Ans:: aa 61. Regarding blood supply of bones a. Small bones supplied by nutrient artery b. Irregular bone supplied by nutrient artery c. Flat bones bone supplied by nutrient d. Shaft of long bone supplied by Nutrient artery only. e. ends of long bones supplied by epiphyseal arteries. Ans:: ee. 62.urinary bladder mass there and features towards squamous cell ca….cause a.smoking b. cotton industry worker Ans:: aa 63. mcv raised 107 and gut problem….. ?

a.Megaloblastic anemia b.IDA c. AOCD Ans:: aa 64. severe steatorrhea a.pancreatic resection b. stomach resection c.duodenal resection Ans:: aa 65.nutmeg liver. a.chronic passive congestion b.active hyperemia c. reactive hyperemia Ans:: aa 66.3rd heart sound murmr occur due to a. rapid inflow in ventricles b. slow inflow in ventricles. Ans:: aa 67. esophgeal ca and middle 3rd is squamous diagnose a. alcoholism b. smoking Ans:: aa 68.EEG change a.alpha waves disappear on eyes open b.beta waves when eyes are closed. Ans:: aa 69.hyoid bone level a. C3 b. c2 c. c1 Ans:: aa 70.human intermediate host of A.Hydatid disease . b. ameobic dysentery c.liver fluke Ans:: aa 71. patient died on brain biopsy which type of necrosis… a.liqufactive necrosis b.coagulative necrosis c. fat necrosis Ans:: aa 72.about pesudomAns: a.endotoxin b. exotoxin. Ans:: aa 73.aplastic anemia… a.hypocelluar tap on trephine biopsy b. bloody tap Ans:: aa

12 74.prophylaxis agaist tetnus to population a.toxoid b. antitoxin Ans:: aa 75.thamalic stoke or hamrohge like scnerio … a. hperasthesia b.Synesthesia Ans: : aa 76.red infarct a.organ have dual blood supply b. organ having single blood flow Ans:: aa 77.yellow fever in non endemic area a. epidemic b. sporadic c.endemic Ans:: aa 78.ventricles completely depolarized at a.qrs b. st Ans:: bb 79. patient cant speak or utter words damage to a. wernickes b.brocas Ans:: bb. 80.. hemartoma… A. mass of mature but disorganised tissue indiginous to a location. b. mass of well differentiated tissue outside a particular location. Ans:: aa 81.left precentral gyri damage a.right hemiplegia b. left hemiplegia Ans:: aa 82.adenoma a.Glandular pattern. b. histologic variation Ans:: aaa 83.phledelphia chromosomes A.cml B. AML Ans:: aa. 84. Atrial flutter a.Treated with defibrillation shock therapy b. rate is so rapid above 400beats/min c.Heart beat between 200 and 350beats /min Ans: : cc

85.creatinine clearance Values 144 s creatinine 1.4 And plasma creatinine =1.2 a.1.68 b.168 c.163 d.cannot be calculated Ans:: dd. 86.true hermaphrodite,… a.xy b.xyy c.xxy Ans:: cc 87.xerostomia , and keratitis patient is having? a.sjorjen syndrome. b.Mukilicks syndrome Ans:: aa 88 .. patient with mydriasis and ptosis. Whats the cause a.occulomotor damage b.horner syndrome Ans:: aaa as in horner there is miosis ,ptosis n enaphthalmos 89.In G6PD what is present? a.comb test , b.osmotic fragility c. heinz body d.microcytosis Ans:: cc 90.In venous blood rbc have? a.Inc cl conc b.Inc hco3 Ans:: aa 91. In deep sleep hormone secreted is a. serotonin b.melatonin Ans: : bb ( melatonin secreted in deep sleep while serotonin mediates deep sleep) 92.diabetic patient wound healing impaired due to a.neuropathy and angiopathy. b. angiopathy c.neuropathy. Ans: : aa 93.Mg store in a.bone b.muscle Ans:: aa 94. Diabetic patient after gram negative infection present with shock after 6 days sugar level normal

13 a. acute tubular necrosis b. increase permeability Ans:: aa 95. Patient stands from supine position what will happen? a. increase heart rate b. decrease venous return Ans:: bb. 96.boat shaped organism in case of pneumonia… a. pneumocystitis jervoci b. cmv Ans:: aa 97.Patient smoker psychotic with vomiting abdominal pain n visible perstalsis in upper portion of stomach a.Pyloric stenosis b.Psychogenic. c.achlasia Ans:: aa 98.fisherman with gums hypertrophy and bleeding …deficiency of which a.vitamin c b.vitamin k Ans:: aa 99. Bilateral ptosis at the end of day is conformatory test? A. Tensilon B.antibodies (Ach Receptor Ab) c.emg Ans:: bb 100.isthamus of thyroid… a.2,3,4 rings b. 3,4 rings c. 1,2 rings Ans:: aa 101.corticospinal tract decussate at… a.pyramid b.medulla c.pons Ans:: aa 102.pus contain a.dead bacteria b.dead neutrophil Ans:: bb 103.which cause malignancy in git … a.h.pylori b. cmv Ans:: aa 104.which condition predispose to cancer a. barretes esophagitis

b. chronic ulcer Ans:: aa 105.hypophysis cerebri related with a.sphenoid sinus b.ethmoid sinus Ans:: aa 106.soft palate lined by a. stratified squamous non cornified b. stratified squamous cornified. Ans:: aa. 107.pt died after 6 days of MI on autopsy large amount of fluid seen in pericadia cavity a.cardiac tamponade b. arrythmias Ans:: aa 108.gfr increase by a. afferent arteriolar dilation b. afferent arteriolar constriction. Ans:: aa 109.post 1/3 interventrcular sptum supplied by a. marginal b. right coronary artery c. circumflex Ans: : bb . 110.thirst is stimulated by a .increase osmolarty and decrease volume b. decrease osmolarty and increase volume Ans:: aa 111.prolactin regulation by a.dopamine b. serotonin Ans:: aa 112.about pulse pressure depends. a. Stroke volume b. venous return Ans:: aa 113.heparin ,histamine and protease release from a. mast cells b. eosionphils c. basophils Ans:: aa 114.glucocorticoid causes dec which cell.. a. lymphocyte b. neutrophil Ans:: aa 115.gluconeogenesis is caused by a. insulin b. glucagon

14 Ans:: bb 116.resection of ilium a. decreased bile salt absorption. b.increased water content of feaces. Ans:: aa 117.ventral corticospinal end at a.mid thoracic b.upper thoracic c.all spinal cord Ans:: aaa 118.muscle stretch then relaxation by a. muscle spindle b. gto Ans:: bb 119. muscle stretch to extreme which prevent from damage or tear like that a. muscle spindle b. GTO Ans:: bb 120. Hyaline cartilage A. present in nasal septum B pinna C TMJ articulation D epiglottis E.two menisci Ans:: aa 121.antimetabolite use in cancer treatment a.methotrexate b.azathioprine c.flourouracil Ans:: aa 122.left shift of oxygen hb curve a.increase temp b.co poisononing Ans:: bb 123. about left renal a.ant to both aorta and left renal artery b.post. to aorta Ans:: aa 124.one of delayed wound healing a.vitamin c deficiency b. vitamin A exc ess. Ans:: aa 125.intermediate filament for cancer is a.cytokeratin b.vimentin Ans::aa 126.diffusion depends upon like stem… a.total surface area . b.saturation kinetics,

Ans:: aa 127.in myasthenia gravis treatment affected by a. Choline acetyl trAns:ferase b. acetyl coA c, acetylcholine d. acetylcholinesterase e. cholinestrase. Ans:: dd ( should be anti Ach esterase but no option there). 128. definitive diagnosis of a patient with Tb asked a.caseous necrosis b.identification of acid fast bacili Ans: : bb 129. length of right bronchus a.2.5 cm b.5 cm c.10 cm Ans::aa 130. in mountain the po2 and nitrogen from 700 was asked.. a.pO2 144 and pNo2 546 b.po2 110 and pno2 590 c. po2 100 pno2 600 d. po2 98 and Pno2 602. Ans:: cc 131.in copd patient a.Vt 350.fev .20000. fvc 36000 132. univeral recipient a. AB -ve B.AB +ve C. O+ve D.O-ve Ans:: bb 133.about cerebellum a. lies below tentorum cerebelli b.middle cerebellar connect with medulla or spinal was given…. c. only supply by basilar artery Ans:: aa 134.glucagon secretion increase by. a.exercise b.insulin Ans:: aa. 135 Patient have malar rash and anti ds DNA positive now having proteinuria and hematuria a.lupus nephritis b.sore throat . Ans:: aa

15 136 . edema of kidney origin AAlbuminuria and Na retention B.Only Na retention . C.Decrease Ca. Ans:: aa 137 A patient has injury red flare was seen due to a. Histamine b. bradykinin Ans:: aa 138 Decrease Na secretion due to .?? a. maximum Na reasbsorption b. maximum Cl reabsorption. Ans:: aa 139. One was asked about N REM A. night mares b. on EEg delata waves Ans:: bb 140. hormone of stress a. cortisol b. norepinephrine. Ans:: aa 141. gluocorticoid. a. decrease utilization of glucose b. increase utilization of glucose. Ans:: aa. 142. thyroid hormone increases what? a. free fatty acid b.cholesterol Ans:: aa 143. micturation reflex. a.once start then self regenerative b.control by sacral segments of spinal cord. Ans:: bb. 144.post inter ventricular septum1/3 a.right coronary artery b.circumflex c. left anterior descending artery. Ans:: aa. 145. Essential fatty acid a.linolenic acid b. lithocholic c.cholic acid Ans:: aa 146.Coarctation of aorta is associated with A.Constriction just after the subclavian artery B.Dilatation below the coarctation Ans:: aa.

147.The most common changes which occur in a case of uterine prolapse a.chronic cervicitis with squamous metaplasia b. dysplasia of cervix c. hypertrophy of cervix d. squamous metaplasia , endocervix e.vaginal discharge Ans:: aa. 148.a 2 year boy having mcv raised .. and choice of investigation a.vitamin b12 b. intrinsic factor c. anti intrinsic factor antibody. Ans:: aa 149.heparin given affect on a.factor X B.fctor x and thrombin c c.factor vii Ans::aa 150.Christmas disease a.Factor 9 deficient b.Factor 8 deficient. Ans:: aa. 151. By the End of 5th week a.heart definite form b.stomach rotates c.limbs bid appear Ans:: bb 152. Internal carotid artery a.Lateral to external carotid at origion then goes POST to come medialy b. Give ophthalmic artery that enter the eye through superior orbital fissure Ans:: aa. 153.renin decrease by…. ?? a.antidiuretic hormone b. angiotension 2 Ans:: bb 154.fascial nerve a. supplies stapedius b. posterior belly of diagastric Ans: :bb 155.Visual Accomodation a.Contraction of ciliary muscle b.Sphincter pupillae relax c. Tension of ligaments to lens Ans:: aa 156 A protein that binds oxygen in skeletal muscle a.Myoglobin

16 b. Actin c. Myosin Ans:: aa 157 Germ cells arise in yolk sac at 3rd week of Intrauterine life ..they originate from ? a. Endoderm b. Mesoderm c. Ectoderm Ans:: aa 158. Melanocytes derived from a. Dermis b. Neural crest cells c. Mesoderm d. Endoderm Ans:: b. 159. Which is absorbed passively by kidney a. Urea b. Cl c. K Ans:: bb 160.Epitheliod cells of tubercle are a.Mononuclear phagocytes b.LangerhAns: giant cells c.PMN leukocytes Ans::aa 161.Diffusion of a solute across cell membrane depends on a.Saturation kinetics with nernst equation b.Carrier protein c.Surface area of membrane Ans:: cc 162.Most abundant in serum ? a.IgG b.IgM Ans:: aa 163.A normal n healthy individual is a.Who eats sleeps n exercises regularly n lead a disciplined life b. Economically Occupationally socially acceptable Free o f discomfort .c-Who is famous in his society Ans:: aa 164. A pt with hypoxia ..with po2 :60 mmhg n HB :15g/dl...reason of hypoxia a.Histotoxic b. Anemia c. Hypoxia Ans:: cc

165.Somatostatin helps in pancreatic fistula by a.decreasing secretions b.Inhibiting enzymes Ans::aa 166At upper part of lung /zone 1 a.Shunting is increased b.Dead space is increased c.Circulation highest Ans:: bb 167Which part of brain receives major excitatory input to from cerebral cortex but it itself doesn't send axons to cortex a.Substantia nigra b.Globes pallidus c.Caudate d.VL thalamus e.VA thalamus Ans:: aa

SEPTEMBER 7 EVENING MEDICINE 2016

1. Which part of brain receives major excitatory input to from cerebral cortex bt it itself doesn't send axons to cortex a.Substantia nigra b.Globus pallidus c.Caudate d.Ventrolateral thalamus e.Ventro anterior thalamus Ans::a. 2. MgSO4 effects can b overcome by ? a.Calcium chloride b. Labetalol c.Atropine Ans::a. 3.Most severe steatorrhoea in a. Gall bladder resection b...Pancrease resection c..Terminal ileum resection d..Colon resection Ans::b. 4.At upper part of lung /zone a Shunting is increased b.Dead space is increased c.Circulation highest Ans::b. 5.Somatostatin helps in pancreatic fistula by

17 a.decreasing secretions b.Inhibiting enzymes Ans::a. 6. A patient with hypoxia ..with po2 :60 mmhg n HB :15g/dl...reason of hypoxia a.Histotoxic b.Anemia c.Hypoxia Ans::c. 7. True about blood supply : a.Flat bones supplied by nutrient Artery b.Short bones supplied by nutrient Artery c.End of long bones in Adult by epiphyseal Artery d.Shaft by metaphysial artery e.Vertebrae supplied by nutrient Artery Ans: : cc 8. Pin stroke on skin red flare cause ? a.Histamine b.Kinins Ans::a. 9. compensation on standing from sitting a.Decrease stroke volume b.Decrease Venous return c.Increase Heart rate d.Decrease Cardiac output Ans:: bb 10. Which drug increases LES of esophagus n speeds up gastric emptying a.Metochlopramide b.ondensaron Ans::a 11. Effect of sympathetic system by Alpha discharge a.Contraction of radial muscles of iris b.Glycogenolysis Ans::a. 12. A patient with a bald patch on scalp .which on examination is yellow velvety .. It can lead to which dangerous condition a.Squamous cell carcinoma b.Basal cell carcinoma c.Keratoacanthoma Ans::c. 13. Left deviation of oxygen dissociation curve a.Carbon monooxide poisoning b.Increased ph Ans::a. 14. A normal and healthy individual is

a.-Who eats sleeps n exercises regularly n lead a disciplined life b.-Economically Occupationally socially acceptable Free of discomfort c.-Who is famous in his society Ans::a. 15. Most common premalignant condition of mouth a.Leukoplakia b.Erythroplakia c.Lichen planus d.Apthous ulcer Ans::c 16.Epithelium of oral surface of soft palate a.Stratified columnar epithelium b.Stratified squamous non cornfield epithelium c Stratified cuboidal epithelium d.Pseudo stratified columnar epithelium Ans: bb 17.Hyaline cartilage a.Precursor of endochondral ossification b.Epiglottis Ans::a. 18.Cortico spinal tract crosses at a.Base of pons b.Pyramids c.Midbrain peduncle d.Ant horn cells e.Internal capsule Ans::b. 19.Intercostal nerve a.12 pairs b.Connection with sympathetic nerves c.Only supply skin d.Only supply intercostal muscles Ans::d 20.Beginning of 5th week a.Limb bud appear b.gut rotation begins Ans:: bb 21.Hormone decreasing stomach contractility upon fatty food in duodenum a.Secretin b.Cck Ans::b. 22.Pt with chronic asthama for 5 yrs ..vt? Fev1 ? Fvc? a.Vt 300 fev 1 : 2000. Fvc:3200 b.Vt 450. Fev1 : 2100. Fvc:2600

18 c.Vt :350 fev1 : 3000. Fvc. 3700 23.A man with short limbs but normal trunk n lumbar lordosis ..which type of inheritance? a.Autosomal recessive b.Autosomal dominant Ans::b 24.Pulse pressure : a.Is more in peripheral artries than aorta b.Depends on venous compliance Ans::a. 25.Most abundant in serum ? a.IgG b.IgM Ans::a 26.Diffusion of a solute across cell membrane depends on a.Saturation kinetics with nernst equation b.Surface area of membrane Ans::b 27.True about large intestine a.Descending colon is medial to left kidney b.Right colic supplies trAns:verse colon c.Ascending colon has mesentery in 20% cases Ans: cc 28.Lichen planus malignant trAns:formation in ? a.10-15% b.20-25% Ans::a. 29.When to stop aspirin before surgery a.10-12 days b.7-9 days c.1-3 days Ans::b. 30.Atmospheric pressure 700 ..humidity zero ..partial pressure of nitrogen and oxygen ? 31.Epitheliod cells of tubercle are a.Mononuclear phagocytes b.LangerhAns: giant cells c.PMN leukocytes Ans::a 32.Which is absorbed passively by kidney a.Urea b.Cl c.K Ans:.a 33.Decreased clearance of Na in ... a.Osmotic diuresis

b.Water diuresis c.Increased absorption of Na Ans:.c 34.Melanocytes derived from a.Dermis b.Neural crest cells c.Mesoderm d.Endoderm Ans::b. 35.Germ cells arise in yolk sac at 3rd week of Intrauterine life.they originate from ? a.Endoderm b.Mesoderm c.Ectoderm Ans::a 36.Protein that binds oxygen in skeletal muscle ? a.Myoglobin b.Actin c.Myosin Ans::a. 37.Hemorrhage in thalamus ..sensory changes ? a.Numbness b.Hyperesthesia Ans::b. 38.Patient with fatigue which worsens towards evening ..most appropriate investigation a.Acetylcholine receptor antibody b.Tensilon test c.Emg d.Ct brain Ans::a. 39.Posterosuperior relation of pituitary a.Sphenoid air cells b.Ethmoid air cells c.Maxillary sinus d.Mastoid air cells Ans::a. 40.True about cerebellum a.Lies below tentorium selle b.Connected to midbrain via middle peduncle Ans:: aa 41.True about internal carotid a.Lies lateral to external carotid at origin and latter turns to medial side b.Enters skull through squamotympanic fissure

19 c.Lies lateral to external carotid at origin n latter turns to medial side Ans::a. 42.Visual Accomodation a.Contraction of ciliary muscle b.Sphincter pupillary relax c.Tension of ligaments to lens Ans::a. 43.Angina worsened by a.Virapamil b.Theophylline Ans:.b 44.Tubular excretion of penicillin decreased by ? a.Probenecid b.Gentamycin Ans::a 45.Theophyllin action slowed down by concurrent use of a.Atropine b.Beta blocker c.Adenosine Ans::c 46.Mechanism of action of methyldopa a.Directly causes relaxation of arteriolar sm muscles b.Stimulation of adrenergic system in CNS c.inhibition of adrenergic system in CNS Ans:: cc 47.Post 1/3 of Interventricular septum damaged ..artery ? a.posterior interventricular artery b.Right coronary artery Ans::a 48.Patient had non lethal MI .what can develop in few minutes ? a.Cardiac arrythmias b.Left ventricular failure c.Nothing Ans::c 49.linear growth of bone is affected if following structure is fractured a. metaphysis b. epiphyseal line c. epiphyseal plate d. diaphysis Ans: c 50..Abscess anterior to pretracheal fascia.infection can spread to a. anterior mediastinum b.post.mediastinum

Ans::a 51.. Beta endorphins a. hypothalamus b.pituatary Ans::a. 52.. Typical scenario of sickle cell anemia with abdominal pain jaundice etc... A. HB ss B. HB sc Ans:.a 53. Difference b/w sarcoma and carcinoma a.. pleomorphism b. increase vascularity Ans::b. 54. ITP scenario was given splenectomy done which organism will cause postsplenectomy recurrent infections a.. hemophilis influenza b. streptococus pneumonia Ans: :b ( Shin : strept pneumonia > h.influenza type b > Nisessria meningitidis ) 55. Hepatitis scenario viral markers negative ALT slightly raised on examination there were some eye findings what is the suitable diagnostic test a. serum ceruloplasmin b. urinary copper c. HbsAg d. anti mitochondrial antibody Ans::a. 56. Patient comes with tender right hypochondrium stool with occult blood on colonoscopy, ascending colon was involved, showing ulcers while other parts of colon were spared on histology/ microscopy what will be seen a. necrotizing vasculitis b. crypt abscess c.entamoeba histolytica d. carcinoma Ans:: bb 57.microscopic findings were epithelium and rete ridges a.verrocus carcinoma b.squamous cell carcinoma Ans:: bb 58. Child with history of bed wetting though toilet trained investigations done at one side duplication of ureter was

20 found and one of them was opening in vagina. what is the cause a. early division of ureteric bud. b. Inability of mesonephric duct to join to ureteric bud Ans::a. 59.Sand blaster a. asbestosis b. silicosis C. anthracosis Ans::b 60.Anxiety is decrease by the activation of which receptors a.GABA b. glutamate c. glucocorticoid d.nicotinic cholinergic e. dopamine Ans::a. 61. Turner syndrome a. autosomal recessive b. autosomal dominant c.gynaecomastia d.short stature Ans::d 62. Chronic renal failure patient died on autopsy which organ will show hypertrophy a. thyroid b. parathyroid c.adrenal Ans::b. 63. Which cell organelle contain double membrane a. nucleolus b. golgi apparatus c. ribosomes d. Rough endoplasmic reticulum Ans:: dd 64. Fisherman with gingival hyperplasia and ecchymosis which one is deficient a. vitamin B12 b.vitamin K c.vitamin c Ans:: cc 65 Patient on immunosuppressant’s abscess is formed on upper outer half of arm after drainage healing process is very slow though a month has been passed reason a. decrease collagen formation

b. decrease neutrophil migration c.infection Ans:: aa 66. Slowest growing malignant thyroid Carcinoma a. papillary b. follicular c.lymphoma Ans:: aa. 67.Which infusion will increase Extracellular fluid to maximum extent a. hypertonic fluid b. isotonic fluid c.hypotonic Ans:: aa 68.. Steady pressure is detected by a. meissners b. ruffini c.pacinian Ans:: bb. 69.Patient with myocardial infarction with typical sign and symptoms.st elevation in leads II, III and AVF a. anterior wall MI b.inferior wall MI c.anerioseptalMI Ans::b. 70.Patient with AML undergone bone marrow trAns:plant got pneumonia histology showing large cells with intracellular inclusions cause is a. CMV b.candida c. pneumocystic Ans: : aa 71. Child brought by mother suffering from diarrhea on examination rectal prolapsed was found. worm with small anterior end a. entrobius vermicularis b. trichuria trichuris c. whip worm Ans:: bb. 72.Best indicator of venous return a. end diastolic volume b.end systolic volume c.capillary filling pressure Ans: : aa 73. myocardial infarction patient after 48 hour found collapsed no pulse. on ECG

21 irregular waves were seen. whats the pathophysiology? a.decrease cardiac output b. ventilation perfusion defect c.decrease capillary filling . d.venous pooling Ans:: bb 74.Subcapsular afferent a. lymph nodes b. spleen c. thymus Ans: : aa. 75. Alveoli are kept dry because of a. alveolar macrophages b. tight junction between capillaries c. surfactants d.negative interstitial pressure Ans:: dd 76.Dead space does not change in a .shallow breathing B deep inspiration c.standing d. re-entery current Ans:: aa 77.Least amount of minerals are found in a. roots b. Tubers c. cereal d. pulses e. vegetables(leaves) Ans: : bb 78. Erythropoietin secretion is inhibited by a. cobalt b. hypoxia c.theophylline Ans: : cc 79. Which is low in csf as compare to plasma a. Na b. Osmolarity c. Cl d. Mg e. protein Ans: : e 80.Cells of chronic inflammation a.macrophages b.lymphocytes c.eiosionphils d.basophils Ans: : aa

81. Middle aged man presented with meningitis after having lung abscess a.Staph. Aureus b. Streptococcus fecalis c.Streptococus viridiAns: Ans: : aa. 82. Which toxin is responsible for scarlet fever a. exotoxin b. erythrogenic c. endotoxin Ans: : bb 83. Serum sodium is regulated by a. osmoreceptors b.Volume of ECF c.ICF volume Ans: : aa 84. Highest sodium channel concentration a. initial segment b. node of ranvir c. dendrites Ans: : bb 85. in chronic liver disease which histological finding suggests chronicity a. fibrosis b. councilman bodies c. lymphocytes Ans: :aa 86. FFPs used for the acute management of a. factor 8 deficiency b. warfarin overdose c. PLT function defects. Ans: : bb 87. intense trAns:fusion reaction occurs if we trAns:fuse a. A- to A+ b. A+ to O+ c. A to AAns: : bb 88. which prevents muscle from tearing under pressure a. GTO b. muscle spindle c. Ligaments Ans: : aa 90. flexor withdrawal reflex a.multisynaptic b.monosynaptic c.disynaptic

22 Ans: : aa 91. right atrial pressure is increased a. increases cardiac output b. increased venous return c. increases intrathoracic pressure Ans: : bb 92. After adrenalectomy which is preferred a. glucose b. NaCl c. Mg cl Ans: : bb 93. in cell membrane of RBCs CL- and HCO3- exchange occurs through a. ankyrin b. band3 c. spectrin Ans: : bb 94. patient with discoid rash arthritis and hemolytic anemia whats most specific investigation a. ANA b. anti ds DNA antibdy c. RNA studies Ans: : bb 95. patient with malaise and raised BP died because of hemorrhagic stroke in basal ganglia. on autopsy bilateral small kidneys with petechial hemorrhages, hyperplastic arteriosclerosis and fibrinoid necrosis. whats the diagnosis a. fibromuscular dysplasia b. DM type II c. systemic sclerosis d. NSAID induced Ans: : aa 96. which one is most pre malignant a. compound nevus b. intradermal nevus c. seborrhaic keratosis d. dysplastic nevus Ans: : dd 97. Old male patient has history of headache in temporal region thick cord like vessels in that area on biopsy giant cells are seen a. ESR more than 110 . b. Diagnosis is made on biopsy . Ans: : bb 98. presynaptic sympathetic fibers release a. ACH

b. nor epinephrine c. dopamine Ans: : aa 99. saliva prevents iron utilization by microbes via a. Lactoferrin b. lysozymes c. Lactase. Ans: : aa 100. benign tumor a.Leiomyoma b.Hamartoma c.Basal cell carcinoma Ans: : aa

14 OCTOBER MED N ALIED EVENING PAPER

1. A patient undergoes kidney transplant on first day he was unable to produce urine there is also tenderness at transplant site A} Graft versus host rejection {b} hyper acute graft rejection {c} blockage of ureter {d} renal artery stenosis {E} thrombus at the side of transplant ANS.D 2. Blood supply interrupted to a tissue which type of necrosis will occur {a} coagulative necrosis {b} fibrinoid necrosis {c} caseous necrosis ANS.A 3. what are present at thin loop of henle {a} brush border {b} mitochondria {C} flat epithelium ANS.C Flat epithelium is correct as there are no brush borders in thin descending loop but few mitochondria are present 4. one scenario about dislocation of acetabulum posterio superior which bone dislocate {a} sacrum ilium {b} pubis sacrum {c} ischium sacrum

23 {D} ilium ischium ANS.D 5. At which point in ecg more Ca ions enter in ventricular muscle cell {a} p wave {b} r wave {c} s-t segment {D} p-r interval ANS.C 6. even in the absence of ADH 65% of water is absorbed where this absorption occur {a] pct {B} thick loop of henle {C} thin loop of henle {d} collecting ducts ANS.A( yaad rakhen bcqs me absence ho ya presence of ADH max water PCT me he absorb hoga ref ganong) 7. A diabetic patient has unhealing ulcer on the foot what is correct {A} angiopathy {B} neuropathy {C} poor sugar control {D} neuropathy and angiopathy ANS.D 8. spinal part of accessory nerve {A} passes below transverse process of atlas {B} passes through the substance of sternocledomastoid ANS.B 9. patient had severe dehydration what is the best treatment {A} 5% dextrose {B} 10% dextrose {C} albumin {D} normal saline ANS.A ( ganong 22nd edition bcq key is A there) 10. a patient presented with hypotension he is cool and tachycardiac he was asked to stand up on standing his heart rate is increases why {A} decrease in total peripheral resistance {B} decrease after load {C} decrease venous return

{D} increase venous compliance ANS.C 11. vagal trunk cut what happens {A} incomplete gastric emptying {B} augmented gastrin secretion {C} augmented pepsin {D} augmented enterogastric reflex ANS.A 12. most abundant IG in adult body {A} igG {B} igA {C}igM ANS.A 13. in 3rd degree block what is true {A} atrium beat independently of ventricles {B} there fixed ratio of between atria and ventricles Ans:A 14. ECG of patient showed absent p wave normal QRS complex n T wave , where should b the pacemaker? {A} SA node {B} AV node {C} purkinji fibers {D}ventricular muscle ANS.B( ideally rt ventricle is ans but it was not in options) 15. Gravida 2 first pregnancy normal , 2nd boy delivered at 32 weeks with hydrops fetalis , icteric phase , exchange transfusion done what is the type of immunity? {A} type 4 hypersensitivity {B} complement mediated hypersensitivity {C} antireceptor antibody {D} self tolerance {E} autoimmune complex formation ANS.B 16. which one is essential amino acid? {A} serine {B} tyrosine {C} therionine {D} proline ANS.C 17. regarding cemitidine effect ? {Ans } inhibit the hepatic enzyme

24 18. A diabetic patient smoker with myxedema which drug u will give for hypertension? {A} nifidipine {b} b bloker {C} captopril {D} varapamil ANS.C 19. commn cause of thromboembolism? {Ans} prolonged immobilization 20. female with infection pap smear shows prominent nucleoli n increased nuclear size irregular architecture? {A} dysplasia {B} metaplasia ANS.A 21. a group of people studied for effect of a drug and compared to a control group they are followed longitudinally over a period of time what type of study is? {A} case control {B} cohort study {C} randomized control trials ANS.C 22. edema due to renal cause can be due to? {A} albuminurea {B} glomerulonephropathies {C} Na retension {D} Na retension and albuminurea ANS.D 23. Which organelle inside the cell can replicate? {a} mitochondria {B} centriole {C} nucleolus ANS.A 24. defecation carried out by? {A} sacral parasympathetic nerves {B} myentric plexes {C} mass movements ANS.C 25. A patient undergoes tonsillectomy he became hypotensive after 2 hours why? {A} blood loss {B} vasovagal shock

{C} hypovolumic shock {D} due to trauma Ans:C 26.. diagnostic test for diabetic nephropathy? {A} cretinine clearance rate {B} serum cretinine {C} blood urea nitrogen {D} microalbuminurea ANS.D 27. A patient with thyrotoxicosis heart rate 145 per mint BP 160/90 which drug should be given {A} B-blocker {B} ca channel blocker {C} digoxin ANS.A 28. EBV cause which carcinoma {A} bronchoalveolar {B} nasopharryngial ANS.B 29. difference between skeletal and smooth muscle? {A} scattered myofibrils {B} ca calmodulin present in smooth muscle {C} specialized cells for forceful contraction ANS.B 30. which hormone is responsible for multiple fracture in pt on dialysis? {A} PTH {B} cortisol {C} low vitamin D {D} epiandristenedione ANS.A 31. which enzyme responsible for degradation of norepinephrine and serotonin? {A} COMT {B} MOA ANS.B 32. on chest p/A view sternocostal surface is formed by? {A} right atrium {B} right ventricle {C} left ventricle ANS.B

25 33. true about blood? {A} wbc most abundant cells {B} pletelets are more then rbcs {C} iron in blood is mainly in hemoglobin ANS.C 34. which of the following donot causes acid secretion {A}stretching of antrum {B} alcohol {C} acidic antrum {D} peptides ANS.C 35. Ansa cervicalis supplies to? {A} omohyoid {B} strnohyoid ANS.A 36. CLD with upper GI bleeding best treatment? {A} transamine {B} vit k {C} octriotide ANS.C (if terlipressin is in option then click it hre its c) 37. one question was about that there was no elevation of neural tissue below lumbosacral region what the condition called {A} meningomylocele {B} Rachishesis with myloschisis {C} occulta ANS.C 38. A patient after accident talks excessively but talk irrelevant and purpose less which area damaged? {A} brochas area {B} wernick {C} motor sensory ANS.B 39. posterior surface of heart is supplied by how many arteries? {A} usually two in numbers {B} usually three in numbers {C} ant descending supply anterior surface {D} interventricular artery {E} right coronary artery ANS.A

40. Organophosphate poisoning treatment? {Ans} atropine {pralidoxime was not in option 41. true about left renal vein? Ans. anterior to aorta 42. in carbon monoxide poisoning hyperpnoea does not occur why? {A} paO2 is normal {B} paCO2 is normal {C} blood flow increased ANS.A 43. systemic arterioles vasoconstriction occurs if local concentration rises? {A} ANP {B} NO {C} angiotensin2 ANS.C 44. 4 n 5 brachial arch give rise to? {A} hyoid {B} thyroid {C} cricoid cartilage ANS.C 45. which drug if given intravenous causes 60% thrombophlebitis {A} oxazepam {B} i/v diazepam {C} midalozam {D} triazolam ANS.B 46. which drug can be given preoperatively to a patient with liver decompensation? {A} lorazepam {B} oxazepam {C} diazepam {D} Phenobarbital {E} fentanyl ANS.A 47. A boy with recurrent chest infection , lymphadenopathy, klebsiella at each occasion diagnosis? {A} chronic granulomatous disease {B} x-linked agammaglobulinemia ANS.A 48. hyperthyroid patient with pulse 180, syncope, palpitation treatment ? {A} digoxin {B} varapamil {C} lignocaine

26 {D} propranolol ANS.D 49. A patient developed atrial fibrillation he was treated accordingly but same patient died due to occlusion of vessel in brain why? {A} arrhythmia {B} thrombus {C} embolism {D} hypotension ANS.C 50. one question was there is a coin lesion in lung , how to differentiate between granuloma and tumor? {A} autonomous growth {B} growth at increased rate ANS.A 51. A multipara women presented to doctor with pulsatile mass below inguinal ligament what is it ? {A} psoas abscess {B} femoral hernia {C} femoral artery aneurysm {D} saphenous varix {E} lymphadenopathy ANS.C 52. again question was about nasopharyngeal Ca caused by? {A} EBV {B} HPV ANS.A 53. A rash first started on a child near ears then spread to face neck and trunk diagnosis? {A} herpes virus {B} measles {C} mumps {D} varicella ANS.B 54. one scenario about hemophilia platelets 350000,family history of increased bleeding ,what investigation first to performed? {A} coagulation studies {B} platelet function test {C} iron studies ANS.A 55. second heart sound produced by? {A} closure of mitral and tricuspid {B} closure of aortic and pulmonary {C} rapid filling of ventricles

ANS.B 56. Again scenario on hemophilia, pattern of inheritance? {A} autosomal dominant {B} autosomal recessive {C} x linked recessive {D} x linked dominant {E} multifactorial ANS.C 57. which vitamin take part in the formation of clotting factors? {A} vit A {B} VIT K {C} Vit E {D} vit C ANS.B 58. A farmer visited a clinic for some chest related problem on x-ray a round ball was seen in chest? {A} aspergilloma {B} bronchogenic carcinoma ANS.A 59. vital capacity 5L, inspiratory capacity 3.5,tidal volume 500 expiratory reserve volume was asked? {Ans} 1.5 60. IN tabes dorsalis urinary bladder? {A} atonic {B} hypersensitive {C} hyposensitive {D} spastic neurogenic ANS.A 61. A patient comes to u with chest pain what question will give u maximum information? {A} tell me about pain {B} any history of heart disease in your family {C} point to the location of pain in your chest {D} have you ever visited doctor for heart complain in past 6 months ANS.C 62. in complete heart block pace maker is placed? {A} left atrium

27 {B} Right ventricle {C} right atrium {D} SA node {E} AV groove ANS.B 63. confidentiality can be breached? {A} when patient authorizes you {B} insurance claim ANS.A 64. A male complains of pain at penis, urine culture negative for N.gonorrhea but leukocyte are positive in urine D/R which organism can cause this infection? {A} E.coli {B} Chlamydia {C} HPV ANS.A 65. organophosphorus poisoning , characteristic feature? {A} dry skin {B} flaccid paralysis {C} pinpoint pupil ANS.C 66. lesion at the median region in optic chiasma causes which lesion? {A} bitemporal hetronomous hemianopia {b} bitemporal homonomous hemianopia ANS.B 67. a pregnant lady visited gynecologist for dilatation n curettage on microscopic examination which type of precancerous change will be indication for hysterectomy? {A} squamous {B} complex hyperplasia {C} simple hyperplasia {D} metaplasia ANS.B 68. lesion at left optic tract causes which lesion? {A} right homonomous hemianopia {b} left homonomous hemianopia ANS.A 69. A young girl finds it hard to hear at low frequencies of sound which type of conduction ?

{A}sensory {B} due to ear wax {C} conductive {D} mixed ANS.C 70. patient presented to dentist with tooth problem the dentist referred him back to the physician coz pt had resting tremors where is problem? {A} hippocampus {B} substancia nigra ANS.B 71. A patient presented with mental retardation, protruded tongue , umbilical hernia and he is hypermobile and dystonic which trisomy this is? {A} 13 {B} 21 DOWN SYNDRO Ans B 72. saesamoid bone is present where? {A} ala of nose {B} epiglottis ANS.A 73. oneconfusing scenario about vertebral fracture? {A} axis {B} atlas {c} c7 vertebra fracture ANS.B 74. bifurcation of trachea? {Ans} t4,5 in normal supine person 75. ventricular filling in which phase? {A} atrial systole {B} rapid inflow {C} slow ejection phase ANS.B 76. which substance in earth crust causes pulmonary fibrosis? {A} silicon 77. one question was about middle meningeal artery ? {A} it passes through foramen spinosum {B} causes subdural hematoma ANS.A 78. what arises from intermediate mesoderm {A} mesonephrone and mesonephric duct {B} epithelium of gut ANS.A 79. true hermaphrodite ? {Ans} XXy

28 80. A patient visit Pakistan first time develop fever ,headache n then coma {A} plasmodium falciparum {B} dengue ANS.A 81. most widest part of Dural space? {A} L2 {B} s2 ANS.A 82. jaw trauma cant open mouth something like that scenario {A} orbicularis oris {B} lateral pterygoid ANS.B 83. agammaglobulinemia person is more prone to? {A} pyogenic infection {B} viral infection ANS.A 84. young sexually active male presented with painful micturation urine showed 20 to 30 wbc with no penile lesion ,N gonorrhea negative most likely organism is {A} HPV {B} CMV {C} herpes {D} Chlamydia ANS.D 85. which drug does not cause gynaecomestia ? {A} griseofulvin {B} Androgen ANS.B 86. hydroxlation of proline and lysine done by? {Ans} vit C 87. pregnant lady with jaundice went to remote village to see her parents. {A} hep E {B} hep A {c} hep D ANS.A 89. salicylate toxicity causes what? {A} alkalosis {B} acidosis {C} hyperventilation ANS.A 90. which one occur on breast? {A} hypertrophy {B} hyperplasia

{C} hypertrophy and hyperplasia ANS.B 91. which one is more sensitive to pain something like that scenario {A} visceral {B} parietal ANS.B 92. which intercellular junction is like a spot weld between cell and the base? {A} hemi desmosome {B} adhering junction {C} desmosome ANS.A 93. In a person with PH 7.49, hco3 level 30, pCO2 is 50, pO2 95 {A} compensated respiratory acidosis {B} partially compensated respiratory acidosis {C} partially compensated metabolic acidosis {D} partially compensated metabolic alkalosis ANS.D 94. A male patient having unilateral undescended testis if not treated what will occur? {Ans} neoplastic change 95. Regarding cimetidine? {Ans} inhibit hepatic enzymes 96. a big scenario patient having aplastic anemia which type of hypersensitivity? {A} type 4 {B} type 3 {C} type 2 {D} type 1 ANS.C 97. first sacral spine level is at {A} c1 {B} T1 {c} l2 {d} s1 ANS.D 98. the histology of thin ascending limb of loop of henle? {A} brush border {B} flat epithelium {c} mitochondria ANS.B 99. one question was about CSF finding meningitis? 100. high fiber diet benefit?

29 {A} causes diarrhea {B} decrease GI motility {C} causes constipation {D} Lowers cholesterol ANS.D 101. Neural crest cell arise from? {A} neural tube {b} neural plate {C} ectoderm ANS.C 102. coccygeal segment of spinal cord lie at the level of? {A} L1 {B} c1 {c} s2 ANS.A 103. maximum clearance in kidneys {A} creatnine {B} PAH {C} inulin ANS.B 104. inversion and eversion at a joint? {A} ankle joint {B} subtalar joint ANS .B 105. which structure passes through esophageal hiatus? {A} vagus nerve {B} azygous vein {C} thoracic duct ANS .A 106. function of fibroblast ? {A} secrete collagen only {B} secrete elastin only {C} amorphous extracellular matrix ANS.C 107. child eat coin Its location on x-ray? (a) Right lower segment (b) right Lower bronchus. ANS. A 108. One bcq related with golgi tendon??? Ans. causes relaxation 109.. Patient awake eyes closed waves on ECG? Ans: Alpha waves 110. A climatize patient increase HB due to ?? Ans: Secondary polycythemia 111. branches of right coronary artery Ans: Sa nodal artery

MEDICINE 7 NOVEMBER MORNING 1) Articular cartilage most commonly? A. Belongs to category of elastic cartilage B. Can easily repair after injury C. Cells orderly arranged in column D. Covered by perichondrium E. On epiphysis of long bones in synovial joints Ans is E 2)Most imp in irreversible cell injury ??? A. Damage to cell memb B. Loss of co enzymes C. Massive Ca ion influx D. Lysosome swelling E. Mitochondrial swelling Ans is C 3)Acute MI,post 1/3rd Interventricular septum,artery involved A. Ant descending B. Circumflex C. Left coronary D. Marginal E. RCA ANS IS E 4)Lesion at rt precentral gyrus A. Exaggerated knee jerk on left B. Rr hemiplegia C. Loss of voluntary control on left D. Normal plantars on rt E. Rt side sensory loss Ans:A 5)Multinucleated giant cells least likely present in A. Blastomycosis B. Cat scratch disease C. Sarcoidosis D. Leprosy E. Strep pneumoniae Ans is E 6)Urine flow rate 1ml/min. Lowest osmolarity in which part A. Beginning or pct B. Dct C. End of cortical collecting tubule D. End of cortical papillary collecting tubule E. Tip of loop of henle Ans B 7)At the end of ventricular

30 contraction,what happens A. Aortic valve opens B. Av valve close C. Pulm valve close D. Av valve open E. Aortic opens Ans C 8)BT increased in A. Vwb B. Hemophilia Ans:aaaaaaaaaaaa 9)Drug tolerance due to ??? A. Dec degradation of enzymes in liver B. Inc degradation of enzymes in blood C. Failure to take med on time D. Dec effect of drug upon repeated administration Ans D 10)What stimulates thirst? A. Dec plasma osmolarity and inc plasma vol B. Inc plasma osmolarity n dec plasma vol C. Dec plasma osm n vol D. Inc plasma osm n vol E. Injecting vasopressin into hypothalamus Ans:B 11)55 yrs old female,non insulin dependant diabetes. Had cholecystectomy. 3rd post op day she developed sudden dyspnea, chest pain n hemoptysis. Cause A. hiatus hernia B. MI C. pneumonia D. Pulmonary embolism E. Spontaneous pneumothorax Ans:D 12)Horners syndrome. If sympathetic fibers to eyes are involved. What happens ?? A. Absent sweating on same side B. Eye lid drooping same side C face flushing same side D. Same side xerosis n anhydrosis E. Common in younger age group Ans:A 13)Lysosomes?? A. Oxidation of long chain fatty acids B. Formed by smooth endoplasmic reti C. Cant digest worn out cell components

D. Variety of Hydrolytic enzymes E. Oxidative enzyme Ans:D 14)Long scenario regarding rectal carcinoma. Or end pe pucha tha most imp microscopic finding for malignant ca ?? A. Pleomorphism B. Inc N/C ratio C. Invasion Ans:C 15)Oral (buccopharyngeal) membrane separates??? a. Amniotic sac from yolk sac B. Larynx from pharynx C. Nasal cavity frim oral cavity D. Proctoderm from oral hindgut E. Stomodeum from foregut Ans is E 16)Thoracolumbar outflow composed of A. ANS B. CNs C. Parasympathetic D. PNS E. Sympathetic Ans is E 17)Difference of primary n secondary healing Ans. Contraction Of wound 18)Corpus callosum? A. Connected to fornix by septum pallucidum B. Most fibers interconnect symmetrical areas of cerebral cortex C. Related superiorly to falx cerebi D. Fibers of genu curve into frontal lobe as forceps minor E. Rostrum connects genu to lamina terminalis Ans:B 19)Achondroplasia ka scenario tha with short limbs waghera . Genetics puchi thi >? Ans:Autosomal dominant is 20)Dorsal ramus of 2nd cervial nerve? A. Greater auriculad B. Greater occipital C. Lesser occipital D. Supraclavicular E. Tranverse cervical Ans:B 21)Which receptors undergo rapid

31 adaptation? A. Baroreceptors B. Cold receptors C. Muscle spindles D. Pain rec E. Touch Ans:A 22)Baroreceptors ka function tha A. Respond to inc b.p B. Respond to.dec b.p Baki kuch respond to heart rate Ans:B 23)Parasympathetic ka effect on heart A. Inc CO B. Inc SV C. Inc PR interval Ans:ccccccccccccc 24)PTH ka effect on kidneys Ans: Increased ca reabsorption i marked Dont remember the site 25) Ek tha EEG records electrical activity of brain during sleep area of brain?? Ans:Reticular formation 25) Max speed on conduction in heart tissue----??? Ans Purkinje Fiber 26)Ek tha pt on total parenteral nutrition. Develops alopecia and dermatitis. Likely cause ? A. Riboflavin B. Zinc Ans:B 27)Spinal se parasympathetic outflow A. L1,L2 B. S2 3 4 C. S3 Ans:B 28)Clically GFR is calculated by?? Asn:creatinine Clearance 29)Shift to left due to hypocarbia. Which effect A. Bohr effect B. Haldane effect C. Ficks law Ans:B 30)Ek tha us men bht sari values thi. Meds ki prices thi like 2.5, 4, 4.5,9. Is tarah ker k. Us men pucha tha best measure of central tendency A. Mean B. Median

C. Mode D. Standard deviation Aaaaa or bbbb stll to confirm 31)Duchene muscular dystrophy ki defective protein??? Ans:dystrophin 32)Exposure of what causes demylination and injury ti peripheral nerves ?? A. CO B. Lead C. Benzene D. Silica E. Asbestosis Ans: B 33)Ek inorganic dust wala scenario men. Old baba. No cough. B/L lung men kuch. ? Ans :was inorganic duct (From past papers) 34)Post MI death. On 4th day findings on biopsy of lefr myocardium ??? A. Edema, loss of striation B. Transmural collagen deposit C. Lymhocytes D. Necrosis, neutophils,macrophages E. Perivascular n interstitial amyloid deposits Ans:D (macrophages replace neutrophils) 35)Most common cause of death in rheumatic fever??? Ans:myocarditis 37)Increase BT and PTT………?? Ans:vWD 36)Iron deficiency anemia ………??? Ans: hypochromic microcytic can occur 38)Pregnant female. Anemic. MCV n MCH low. HbA 96% or HbA2 4%?? A. Alpha thalassemia B. Beta thalassemia trait C. Sickle cell waghera Ans:B 39)Wohi hypochromic wala scenario. Elder bro has blood transfusion hx Best investigation ?? Ans:Hb electrophoresis 40)Hemophilia ka characteristic bleeding pattern A. Ecchymosis B. Petechiae C. Bleeding in tissuesAns Ans: C 41)16 yrs old girl. Epistaxis.

32 Menorrhagia. Raised BT. Bht zeada raised aptt tha. Normal pt. Platelets 280. Dx A. Vwb B. Glanzman thrombasthenia C. Factor 9 deficiency Ans:A 42) . Old age man. Epigastric pain for 3 yrs not relieved by antacid. Gastric antrum ki biopsy men kuch erythema waghera tha. Non specific gastritis. Cause ??? A. Adenocarcinoma B. Zollinger ellison C. H.pylori D. Duodenal ulcer ANs Cccccc (hint h pylori is present in antrum) 43) Drug which alters thyroid hormone levels in pregnancy , crosses placenta? Ans:. PTU for sure 44)Child with rectal prolapse. Worm with some chota ant end. ?? Ans:Trichuris 45)Chronic cough for more than 5 yrs. Copious amounts of sputum. ?? Ans:Chronic bronchitis 46)Characteristic feature of sarcoidosis ?? A. Diabetes insipidus B. Erhythema nodosum C. Hepatic granuloma D. Hypercalcemia E. Pleural effusiom Ans:D 47)12 yrs old afghan boy. Fever. Night sweats,On rt side neck discharging sinus. No growth revealed on routine culture. Most appropriate initial single step investigation ?? A. Biopsy B. ZN staining C. Mycobacterium culture Ans: B 48)Drug for actinobactee boumanii ?? Ans: imipenem 49)Captopril. Can cause ?? Ans:hyperkalemia 50)Gluteus maximus ?? A. Covers pyriformis B. Arises from gluteal tubercle

C. Supplied by inferior gluteal nerve Ans:C 51)Duodenum?? A. Ampula of vater opens in posterolateral B. 25 cm long C. Feathery something (which cant be...ye jejunum men hota shayad) D. Retroperitoneal parts are 1st,3rd n ye wo Ans: B 52)Filum terminale?? A. Ends at 2nd sacral vertebra B. Composed of pia C. Composed of arachnoid D. Has some central canal thing Ans:A It is also a modification of pia 53)Hemisection spinal cord rt side??? A. Same side ki proprioception lost B. Same side ki pain temp lost Ans:A 54) Man has atrophy of left leg. Atrophied calf muscles and lateral side of foot.?? Sensations ka kuch nai mentioned tha. Lesion A. LMN B. UMN C. LMN Ans:C 55)70 yrs female diabetic. Difficulty in seeing distant objects. VA reduced in both eyes ???? A. Cataract B. Hypermetropia Ans: Aaaaaaaaaa 56)PDA ??? A. Inc arterial pco2 B. Dec arterial po2 C. Peripheral cyanosis D. Inc systemic pressure E. Inc pulmonary pressure Ans:C 57)Regarding split of S2. Split can be enhanced by ?? A. Early closure of aortic valve B. Delayed closure of aortic valve C. Early closure of pulm valve( also delayed closure of pulmonary valve but it was not in options both r rite) Ans:A 58)40 yrs ka obese male with sedentary lifestyle. Bp 140/100. Pulse 80. Cause???

33 A. Increased diastolic pressure B. Inc SV C. Inc CO D. Inc vascular tone Ans:D 59). Child with itching. Worms. Raised cells-??? Ans:eosinophilis 60)3 yrs old child with repeated bacterial infections since infancy. At 32 yrs he developed SLE. Wbc 7850. IgG ki koi values theen. Lymph node biopsy ki ajeeb si findings thi i dont remember those alien terms. Cause???? A. Something NADPH B. Dec of adenine deaminase may be. Dont remember C. Decreased cd4 lymphocytes D. Failure of b cells to become plasma cells Ans:D ( past paper mcq ) 61)Ek classical scenario abt cerebellar lesion.??? A Gait disturbance. B .Past pointing. C.Rapid movements Ans:A 62)Degradation of deoxynucleic acid releases? A. Phosphoric acid B. Ribose C. Uracil Ans:A 63). Damage to tibial nerve. But some flexion still present at knee joint. due to which muscle ??? A. Long head of biceps femoris B. Short head of biceps femoris Ans: B 64)Macule of utricle ka function. ?? Ans:Acceleration Linear / horizontal ans. 65)Breast atrophy in old age due to A. Dec estrogen,progest B. Dec estrogen C. Dec progesterone Ans: B 66) The scenario was 40 yrs old female. Pregnancy. Inc vol of urine (most probably). Inc thirst. Relatives also compaint of her Behavioural changes (jo k unka kaam hy))...Fasting glucose was 108 (normal). Passing large amounts of

dilute urine. Lesion will be at???? A. Thalamus B. Hypothalamus C. Caudate D. Basal ganglia Ans:B 67)Boy with fracture arm after bout of fight with sister ( kuch is tarah tha) withBlue sclera ?? A. Osteogenesis imperfecta B. Marfans Ans:aaaaaaaaaaaaaaa 68)Basic electrical rhythm of brain is located in???? A. Reticular formation B. Thalamus C. Cerebellum Ans: A 69)Deep cervical artery arises from which Ans:Costocervical trunk 70)Right brachiocephalic is a branch of??? Ans Right aortic arch 70)toxoplasmosis pregnant female. She delivered a baby. To check acute infection in neonate, which Ig to be done On elisa ??? A. IgG B. IgG4 C. IgM Ans:C 71)Minutes after bee sting a woman develops wheeze, sob n ye wo. It is due to which??? A. Histamine B. Bradykinin Ans:A 72) Typical scenario post streptococcal. 12 yrs old girl,pehle sore throat. Uske baad post streptococcal walay sign n symptoms. This is due to??? A. Antibody to immune complexes B. Antigen-antibody complexes deposit in tissues Ans:B 73). Neutropenic patient. Admitted to oncology ward. C/o ear pain n fever. Causative organism pseudomonas. Which of the following shows why these symptoms have developed??? A IL1

34 B TNF Asn :A 74) Aeobaric vertigo in divers is due to?? A.BAro trauma B. Perforated tympanic membrane C.Some ear plugging thing D. Unequal pressures in middle ear during ascent Ans D 75)Haan ek wo cricoid cartilage wala ?? A. Incomplete ring B. At c4 C. Vocal cords attached to it D. Inferior pharyngeal constrictor attached to it Ans:D 76 trachea ki bifurcation A. At t2 B. Trachea in is midline but bifurcates a little to the right ans: B 80) right bronchiole ka size pucha tha?? ANs: 2.5 cm 77) Goblet cells change to clara cells in?? Ans: terminal bronchiole 78) Giving large amounts of normal saline will cause?? A. Dec urinary Na excretion B. Inc plasma vol Ans: Bbbbbbbbbb 79)ADH suppressed by.?? Ans:Alcohol (imp Bcq) 80)Morphine ?? A Increased GIT secretions B.Inc gut peristalsis C.Constipation Ans: cccccccccccccc (All opiods causes constipation) 81)Endotoxin?? A. Heat labile B. Released when pathogen dies Ans:bbbbbbbbb 82)In pus. Ans: Dead Neutrophils 83)In acute renal transplant rejection. On microscopy which of the following will show something worse? A. Interstitial lymphocytes B. Vasculitis and thrombosis Ans B

84)Goodpasteurs. ?? Ans:IgG k depositions in basement membranes 85) Normal fev1/fvc ki value ?? Ans:80% 86)Actinomyces? Ans:Talcum granules 87)Female with vaginal discharge. Metaplasia of cervix due to ?? A.HPV B HSV Ans: Aaaaaaa 88.Irreversible step of glucose metabolism??? Ans:G6p to f6p 89) Benzodiazepenes …… ?? Ans:Lead to dependence 90) which drug shud be avoided in angina??? A. Beta blockers B.CCBs C. Theophylline Ans:C 91)Increased t3 and t4 in pregnancy due to ?? A. Inc TBG B. Inc TSH C. Inc TRH Ans:A 92)Regarding CO2 ??? A. High solubility in plasma B. High diffusion coefficient Ans:B 93) Muscle membrane is relatively permeable to??? A. Water B. Electrolytes C. Amino acids Ans: C 94)Ek tabes dorsalis ka typical scenario tha. Dribbling urine. Unable to see in dark. Something with light reflex and pupils Cause?? Ans: syphilis 95)Hypercalcemia effects the formation of? A. 1,25 dihydroxycholecalciferol B. 24,25 dihyroxycholecalciferol C. 25 cholecalciferol Ans:B

35 96)Tumor metastasis ?? Ans:loss of e cathedrins . 97 which of them causes adhesion AnsA. Integrins 98)Dietary fibers contain what?? A. Starch B. Glycoprotein Ans:AA 99.Organelle which has cisterns n concave surface faces nucleus???? A. Golgi B. Smooth endoplasmic reticulum C. Mitochondria Ans:A 100)Regarding mitochondrial mutations ??? A. Can cause fragile x syndrome B. Only transmitted from mother C. From paternal side D. Prader wili syndrome Ans:B 101)Neuron ……..?? Ans: Axonal hillocks where axons leave from cell body 102)Spermatogenesis ?? Ans:Spermatids into spermatozoa 103). Most imp and acute effect on testosterone production is of?? A. LH B. FSH C. Prolactin Asn:A 104)About ovaries ?? A. Lymphatic drainage into para aortic lymph nodes B. Koi 6 something million ova at the time of birth C. Supplied by ovarian artery Asn:A 105)Atrial systole A. During “a” wave B. PR segment C. dicrotic notch Ans:A 106)About thymus ?? A. Regresses shortly after birth B. Contains lymphocytes C. Derived from thyroid Ans:bbbbbbbbbbbb

107)Ectopic Thyroid hormone produced by ?? Ans:Sq cell CA larynx (ref MRCP notes) 108)Pre malignant lesion Ans. Leukoplakia 109)Medial lamniscus men decussation ?? Ans:Post limb of internal capsules 110)Patient on ATT pins and niddle sensation?? Ans:isoniazid 124)ADH n oxytocin?? Ans: originated from hypothalamus 111)Decreased intestinal motility ? A. Secretin B. CCK C. Gastrin D. Somatostasin Ans:D 112)Vagus nerve leaves skull through ?? A. Foramen spinosum B. Jugular foramen Ans:BBBB 129)Severe transfusion reaction ??? Ans: A+ to O+ 113)Secretin relased in response to??? A. Hcl in stomach B. Hcl in duodenum C. Fat in duodenum Ans:BBBBB 114)Effect of hyperactivity of sympathetic system ??? A. Inc gut motility B. Inc gut secretions C. Bronchodilation D. Construction of pupils Ans:CCCCCCCCCCC 115)Hormone increased in stress??? Ans: Cortisol 116)Middle thyroid vein drains into……….. ??? Ans:internal jugular vein 133)Pellegra treatment ?? Ans: niacin vit b3 117)Superexcitability (hyperexcitability) of heart is ???? A. Longer than relative refractory period B. Ventricles are hyperexcitable C. Ventricle are less excitable

36 Stem not cleared … aaaa may be right 118.)Within 12 hrs of MI best marker?????. A. ALT B. ALP C. AST D. LDH E. CK Asn:E 119:Creatinine released from…………….. Asn:skeletal muscles 120:Insulin inhibited by ??? Ans:Beta receptor antagonist 121)Best about kidneys and regarding structures?? A. Perinephric fat is outside renal fascia B. Adrenals not covered in fascia C. Left renal vein is anterior to aorta and renal artery Ans:CCCCCCCCCCC 122)Pain referred from diaphragm to shoulder ANs: C3 4 5 138.Pt Aptt raised by antibiotic use treatment a..FFP b.Vitamin k Ans:B 123:Medullary thyroid cancer marker Ans:Calcitonin 124)Dead space is decreased in??? A Emphysema B. Tracheostomy C. Pleural effusion Ans:BBBBBBB 125)Red infarcts occur in?? A. Where venous blockage alongwith arterial B. Only arterial blockage C. Supplied by more than one arteries Ans:C 126)Gangrene in diabetes ?? A. Dry B. Wet Ans: A (if asked complication then b) 127) Parasitic infection spread by mosquito bite?? A. Filariasis B. Onchocerca C. Tapeworm

Ans: A 128)Which drug/antipsychotic enhances the efficiency/activity (dont remember exact word ) of others A. Valproate B. Phenytoin C. Phenobarbital Ans: Controversial 129) Warfarin effect increased.? Ans: Cimetidine 130)Vasodilation with hypotension ?? Ans:Septic shock 131:Drug tolerance due to ??? Ans:decrease effect of drug upon repeated administration 132:Metabolic alkalosis is feauture of?? a.pancreatic fistula b.hyperaldosteronism Ans:B 132:Anterior cerebral artery supply?? A.brocas b.wernickes c.paracentral lobule Ans:C 133:Blood supply of thalamus ?? a.anterior crebral artery b.middle cerebral artery c.posterior communicating n Ans: cc 134:Patient on antihypertensive drug ..ER collapse.. Potassium 6 cause is 1 cause?? A.Diuretic intake B.Acute renal failure Ans:B 135: years old boy..generalized edema..first line of investigation???? a.albumin globulin ratio b.serum creatinine c.urinary protein Ans:C (exclude nephrotic syndrome)

9-NOV-2016 MED & ALLIED (MORNING SHIFT) 1.Taenia solium pork worm doc tx? a Niclosamide b praziquental c albendazole d mebendazole Ans B (ref Katzung)

37 2. Hyaline cartilage A present in nasal septum B pinna C TMJ articulation D epiglottis e. menisci of knee Ans A 3. Propranol contraindication Ans. Asthma 4. MAP Ans .Diastolic plus 1\3rd of Pulse pressure 5. Blood suply of head of humerus A.Arcuate B.anterior circumflex c post circumflex ANS B>A 6. Aluminium Hydrooxide Ans.Dec Gastric Emptying 7. A structure that is formed by aponeurosis of External oblique Muscle Extending from Anterior superior iliac spine upto pubic tubercle is called Ans. Inguinal Ligament 8. Common in CRF +Conns + pheochromocytoma Ans: HTN 9. Patient having Plt count 1 lac 8000 with increased bleeding time??? Ans ws Platelet Functional Defect ( Qualitative Disorder ) 10. .A drug that in given in imunocompromised patients causes peripheral Neuropathy and Pancreatitis Ans.Diadinosine 11. Epithelium of small intestine a simple columner b simple coloumner non ciliated ans is b 12. Utricle detects?? A.horizontal acceleration B.velocity of horizontal acceleration C linear accelaration Ans is C 13. Regarding Active transport A.require carrier proteins that are enzymatic in nature B.uphill transport Ans is B 14. Imunologic test diagnostic for which ....???? a.amebic liver abcess ...

b.hydatid disease Ans is B 15. Dopamine n ach imbalance in corpus striatm leads??? a.Chorea b.Hemibellismus c,Parkimson d.athetosis ans is c 16.Minimun amount of caloric required for man?? Ans 25_30 17. Heparin acts by binding to?? a.anti thrombin3 factor 10 b.Factor xa only c.. V||| ans a 18.Tumour in brain compressing oculomotor nerve that area venous drainage into a.straight sinus b superior sagital sinus c inferior sagital sinus d cavernous sinus Ans is D 19.Internal jugular vein when it passes from jugular foramen its imediate relation with A. Sternocledomastoid B.Trocher nerve C.Acessory nerve ans C 20.Dec/shortest ventricular muscle fiber length in which one of the following ?? a.Inc venous return b.contracetion of priphral muscles c.Dec intra thoracic pressure d.Inc intra pericardia pressure Ans D 21. Submandibular gland location? Ans mendible above n ant post belly of digastric muscle 22. A 30 year old woman in azad kashmir was injured mildly in an earthquakethat killed many people apart from her distant uncle. She would most probably be in A. Social crisis B. Personal crisis C. Situational crisis

38 D. Developmental crisis ans is c 23. 4x4 table? a.student t test, b.chi square tes Ans B 24.Shivering???? Ans Post hypothalamus 25. Patients whose small gut is resected due to any disease, they present e "hyperoxaluria". The cause of this cond is...? A. Dec blood calcium to form calcium oxalate B. There is increaed abs of oxalate from gut C. There is increased phosphate in blood Ans C 26.CNS myelination by which cells... Ans . oligodendrocyte 27.Prostatic CA metastize to skull through. Ans.Vertebral plexus 28.Internal jugular vein just coming out of skull whats anterior to it. a.ICA b. phrenic nerve c.. sternocleidomastoid d accessory nerve Ans is c 29.Cause of edema in renal dysfunction. A. Salt retention B. Hypoalbunemia nd salt retention ans is b 30.Which is high amount in tear ? a.Na b.K ans a 31.Tall T wave Ans Hyperkalemia 32. long term bp control by Ans.renin angiotensin system 33.Pt had cut on dorsal surface of wrist and now failure to abduct n adduct fingers with intact sensations?? Ans deep branch of ulnar 34. 5 yr old girl wid Sob no cough..for three months..presente with temp 38.7 F ..R/R 25.sputum clear negativ culture with ordinary media..Ana neg..On Cxr hilar Lymphadenopathy and right lowr lobe infiltrates.

A.Bronchogenic carcinoid CA B.Pnemonia C.Tb D. Sle. ans is b 35. S1 loud Ans. Lower limit of pr interval 36.”V” wave on cardiac cycle? A .diastole B.isovolumic contraction C.isovolumic relaxtion ans is c 37. Old patient Excretion of drug is decreased bcz of ? Ans. dec renal threshhold of drugs excretion 38. Cardiac muscle cant be teranized Ans long refrectry period 39..Stress hormone Ans acth (no cortisol in option) 40.Thyroxine causes Ans. Fetal brain development 41.Insulin Ans. anabolic for cho, lipid nd protein 42.Councilmen bodies Ans apoptosis 43. Microscopic feature of compact bone A haversian canal arranged obliquely B lacunae have osteoblast C haversian canal are not interconnected collagen D lamellae are regularly arranged Ans is D 44.Mask like faces Ans. Parkinson disease 45. Scalenius ant grooves over ….. ans 1st rib 46. Patient serum glucose 100 mg/dl but inspite of that glucose apears in urine Ans.low renal threshold 47,. C3a and C5a both A..opsonins B..anaphylotaxins C..chemotactic factors ans is B 48. Tb patient pleural effusion what to do.drain from?? A. 4 to 6th intercostal upper border mid axial line B. 9th intercostal lower border anter axilary line

39 C.9th upr border mid clavicular D.9th uwpr border ant axillary Ans aa ref oxford med 49.Denticulate ligament………??? Ans.Lateral extension of pia matter btw ventral n dorsal roots 50.Subacute endocarditis organism……………?? Ans..Strep viridans 51.Transitional cell ca risk factor of bladder ………..??? a.smoking b. schistosomiasis Both r right go wid bbb 53.Diabtes inspidius treatment. ……??? Ans.Desmopressin 54. Hep B and C caused by ……….. ?? Ans.Contaminated needles 55. Platelets formed from ………..?? Ans.Megakaryocytes 56.Erythropoietin secretion is inhibited by a. cobalt b. hypoxia c. theophylline ans is C 57. Erythropoeuin secretion increased by Ans hypoxia 58.True hermaphrodite A.xx/xy B.xxy C.xx ans B 59. On complete removal of colon wat happens death A..malnutrition B.electrolyte imbalance Ans is B 60. Patient having zollision ellisin syndrome for that gastrectomy done , which will occur? Ans B12 deficiency 61.Btructure lying with LAD having risk of damage during surgery……… ?? Ans.Great cardiac vein. 62. Sodium nitropruside mechanism of action Ans.dilation of vessels 63. Life span of sperm at body temp A.24-48 hr

B.24-72 Hr ans is A 64.Cerebral blood flow regulated by a.pco2 b.ph Ans Is A 65. Hypoglosal nerve damage……… ?? Ans. Ttongue deviate same side 66. Adrenal gland..supply…… ?? Ans.Thoraxic sympathetic plexus 67.Tyre workshop worker and smoker, has squamous cell carcinoma cause??? a.aromatic amine b.smoking Ans is B ref Goljan pathology 68.One regarding atlas vertrbra a.Have no spinous process b.Have no ant arch c.Hve no lateral mass Ans A 69.Regarding arithematic average. . Ans mean 70. Pt have Headache Vomiting Stiff Neck On Lp Csf contain Blood A.Epidural B.SubDural C.Subarachniod ANs is C 71. Hemophilia Ans. X linked reccesive 72. Inc ejection fraction what dec….. ?? Ans.esv 73. Stomach circular muscle become more thicker a fundus b antrum C cardiac d Pylorus Ans D 74. Down Syndrome…. ? ans.Trisomy 21 75. Man had Fasting Lipid Profile After eating junk food, after centrifugation plasma becomes milky white?? A.Chylomicron B.VLDL C.HDL D.LDL E.IDL Ans A

40 76. Preganglionic sympathetic secrete…??. Ans.Ach 77.Golgi tendon decreases ….?? Ans.Tension 78. Ankylsing spondylitis ……..? Ans.HLA B27 79. About cholera ?? A..Treated by parentral antibiotics B..treated by fluids only C..ransmitted by water n food d.Cl- ans CAMP has role Ans is D 80.SA node located ………… Ans.Epicardium sulcus terminalis 81 Strontium 90 is associated with which ca……. Ans.ousteosarcoma 82. Chronic rhinitis a.mast cells b. eosinophill c.lymphocytes ans A 82. Histamine type 2 receptor antagonist.. …… ansCimetidine 83. Macrophage...... ans.scavenger cells 84. Homonomous hemianopis A.gwniculate body B.Optc tract c.Optc chaisma Ans b 85. A child wid reccurnt respiraty tract infctn n etc etc also having kartnegr syndrome?? A.dynin arm B.actin C.mysoain D.cell microtube ans is A 86.A surgicaly removed ovary showed aggregate of epitheloid with lympho these aggregate called A.Graffian follicle b.Granulation tissue c.Granuloma pyogenicum D. Granulomata formation E.Granulosa cell tumor Ans is d 87. Malignant Quarten Malaria

A. plasmodium Falciparum B.plasmodium vivax c.plasmodium malaria d.pasmodium ovale Ans is A 88. K level 3.0 n ph 7.54 due to whch hormone effct A.cortisol B.aldo C.testo D.progstrn Ans is b (aldo causes decrease in potassium dat leads to alkalosis) 89. Factry worker..neuropathy …… ?? Ans lead 90. HIV drug causing pancreatitis??? Ans …..Didanosine 91. 29 yr old went remot area develop mild watry diarrhoea for 1 wk.no accult blood.r resolved after 2 wks couses?? A.Amoeba B.Vibrio C.Rota D.Shigella E.Giardia Ans A 92. Hyaline cartilage.. Ans endochondrial ossification 93. Viral neoplasm…... Ans.T cell leukemia 94. Pain , temperature, from face a anterlateral spinal b trigeminothalamic c dorsal culumn medial leminscs d trigeminal leminuscus ans is A 95. Dignostic creteria for sof tissu tumour ?? a.Inc vascularity b.Local invasion Ans B 96. ACE inhibitor produce bradykinin by?? a Dec angiotensin2 b increase/activate kallikrein Ans b 97. Mast cell a produce heparin,histamin,protease b function like eosinophils C Have IgA surface receptor Ans A

41 98. Local anesthetic…. Ans.A delta fibre 99. In carcinoma which filaments are most likely present a. keratin filaments b.vimentin filaments c.desmin filaments Ans A 100 Gastric emptying is observed thru A.Radioisotope scan B.Repeated xray after barium enema Ans b 101. Child had generalized edema due to A..Dec hydrostatic pressure B>Dec plasma colloid oncotic pressure C.Lymphatic obstruction Ans B 102. Parietal cell secrete A.Hcl+ intrinsic factor B.Hco3+ intrinsic factor ans A 103. Iron deficiency A.Persistent blood loss B.Defected globin chain CTreated by iv iron DMegaloblast Ans IS A 104. 70 kg male 30 yr old hypertensive. Na 140.k 2.5.urae normal .s creatinine normal. Total cholesterol 200. A.Essential hypertension B.Primary hyperaldosteronism Ans is B (Conns syndrome) 105. Pt had accidental damage to neck of fibula and now presnt with foot drop? Ans common peroneal nerve 106. Patient having blood group A Ans:.B Antibody 107. Defect of thalasemia? Ans defect in chain 108. Sjgorens sydrome patient..ANA positive..most serious long term complication? A.Glomerulonephritis B.Pulmonary interstitial fibrosis Ans B 109. .A lady who got rash on face after going in sun.the biopsy of lesion show igG with surrounding area clear. No other abnormality. Ana negative.what is the cause?

A.Contact dermatitis B.Focal Sle C.Discoid sle d. Dermatomyositis.. Ans D (controversial bcq) 110. Brush border in which part of nephron A.Pct B.Thck loop C.Thin loop D.Dct Ans is A 111. Nalbuphine A.Effect comparable to morphine B.1/7 of morphine Ans A 112. Pulmonary emboli of left lung.what finding???? A. Ventilation perfusion ratio infinity B. Vent pro ratio dec C. Alveolar o2 equal to atmospheric o2 D. Alveolar o2 equal venous o2 ans A 113. Dec/shortest ventricular muscle fiber length in which one of the following A.Inc venous return B.Contraction of priphral muscles C.Dec intra thoracic pressure D.Inc intra pericardia pressure ans is A 114. Vastus medialis? A. Supplied by femoral branch B. Attacdfhed to intertrichantetic line C.attached to petala D.attached to medial tubrcl with vasts intermedius and adductr magns Ans is A 115. A young girl during exercise develops dyspnea,she has developed the left sided heart failure n havnig pulmonary edema wat is the cause of developing dyspnea during exercise a.ryt heart failure b.left ventricles pump failure Ans b 116. What could be damaged in surgery of superficial submandibular gland A.Hypogastric nerve B.Mandibular nerve CTrigeminal nerve

42 D.Marginal Brach of facial Ans is D 117. Cytokeratin pigment is used to diagnose?? A.Diff sarcoma and carcinoma B.Diff be metaplasia and dysplasia ans is B 118. Fat necrosis due to ….. ans .breast trauma 119.Coagulatie necrosis ..... ans due to ischemia… 120. In brain type of necrosis ………… ans.liquefactive necrosis 121.Hairy leukoplakia A.fungal growth B. smoking c associated with hiv D leukemia Ans is c 122.The most common cause of infarction in vicera???? A.Air embolism B.amebic ulcer C.Anemia D.Atheroma E.Thrombosis Ans is D 123. . What is time period of absolute refractory resting period of heart A.100milli sec B. 150 msec C. 250 msec D.10-20 msec ans c 124. Scenario of pt with completely blocked blood flow to left lung, which of following is true A. v/Q ratio in left lung will be zero B. systemic arterial Po2 will be elevated C. V/Q ratio in left lung will be lower than Rt lung D. Alveolar Po2 in left lung will be approx. equal to Po2 in inspired air E. Alveolar Po2 in rt lung will be equal to Po2 in venous blood Ans is d 125. An old man playing with his grandson in garden got trauma, pt fibrile, redness in the toe, C- reative protein positive, culture negative, negative biferengence, some values of uric acid

given diagnosis A. Septic Arthritis B. Acute Gout C. chronic gout D. pseudogout Ans B (Hint is inc CRP which is always in acute condition) 126..Peroxisome contain ………….??? Ans.oxidases 127 Regarding Beta thalassemia A.mutation in promoter region of beta gene B.Point mutation in beta gene C.Large deletion in beta gene D. Small deletion in beta gene Ans A reference Robins 128. . Patient of renal transplant on immunosuppressive drug.cns symptoms culture negative.lympho increased sugar normal.oval cell with halo found A.Cryptococus B.Tb C.Influenza Ans A 129. A pregnant diabetic lady gave a birth to a child apgar score 4 and 6 in 1 and 5 min. Child with respiratory distress. What will you give patient before delivery to prevent the condition? A.Hydricostisone b.Testosterone CProstaglandin D.Estrogen Ans A 130. Survey of pts having melanoma, blood sample taken which of the following cells would be present initially(First) A. NK Cells B. Machrophages C. Lyfmpho D. Eosinophills Ans A 131. Dec unloading of o2 in peripharl tissue duet A.pH BBmr CTemp D2,3bpg Ans A 132. Complete trasaction of brain stem above pons would A. Result in cessation of all breathing

43 movements B. Prevent any voluntary holding of breath C. Prevent control of central chemoreceptors from exerting any control over ventilation D. Prevent peripheral chemoreceptors from exerting any control over ventilation E. Abolish hering breuer reflex Ans B 133. Which of following is not related to pain transmission??? A. Thalamus B. Substantia gelatinosa Ans A 134. Hypothalamic hypophysial portal syestem connects hypothalamus to?? A. Adenohypophysis B. Neurohypophysis Ans A 135. Scenario of pt with congestive cardiac failure with pulmonary edema, now in acute distress appropriate treatement at this stage would be?? A. High flow O B. Fuoseamide C. Nitroglycerin D. Dopamine E. Adrenaline Ans B 136. Movement of body in response to visual stimulus...structure involved? A.Sup colliculus BInf colliculus CSup olivary D.Inf olivary Ans A 137. Chlorpromazine exhibits anti pyschotic effects by??? A..alpha agonist action B..act on dopamine receptors ans B 138. Which doesnt cause esoniphilia A...igM B.igE C.parastic infestations D.asthma Ans A 139. One qst k person while fighting gets a blow on temporal side of the head,internal bleeding occurs due to damage to which vessels?? A.middle meningeal atery

B.MCA Ans B 140. .local factor causing delayed wound healing a infection b vitmin c Ans A 141. lymphatic nodule not present in Ans. Thymus 142.Pyogenic bacterial peritonitis………..??? ANS .Bacteroid 143.lipoma on thigh ,local anesthetic xylocaine is given which will be 1 affected?? a.B fibers B.c fibers c.A delta fibers Ans C (for details of general anesthesia see section in Gynae obs theory) 144. Presence of st squamous epithelium in endocervix is called????? A..neoplasia B..metaplasia C..dysplasia D..anaplasia Ans C 145.Patient having hemoptysis and developed heamaturia cause….. ??? Ans .Goodpasture syndrome 146.Skeletal muscle protein synthesis………… ?? Ans CPK 153.Pus contain… a. Dead Neutrophil b.active bacteria Ans A 147.Vit c causes….. ??? ans.hydroxiltion of proline 148. Supra renal gland relation .. a.rt suprarenal is one centrla tendon b.Left one is realted anteriorly to pancreas c.Left is on central tendon.(poor recall) Ans is Bb 149. Protrusion of mandible ………….? ans .lateral ptyrgoid 150. Which one dec blood supply to brain A.Inta ventricular epinephrine B.Inhalation off 17% co2 C.Siezures

44 D.Hyeperbaric O2 Ans is B 151. Which harmone is NORMALLY secreted in pregnanc A.Pth BGH CThyroxin D.Cortiosl asn is D 152 Enterococcus fecls tx A.Ampicilin B.Metronidazole C.Co trimezol D.Aztreonem E.Imepenem Ans A 153. In cardiac cycle atrial fibrillation occurs A at start of QRS B at end of QRS C at peak of QRS D.at the lower end of PR interval Ans D 154. Hypoxic pulmonary vasoconstriction is accentuate by??/?? A. Increase Po2 B. Decrease Po2 C. Increase pco2 D. Decreased pco2 Both B and C r right choice is yours 155. Ascaries causes………….?? Ans. respiratory system(causes pneumonia) 156. Transplant best donor….??? Ans.Identicle twin 157. Steroid mechanism of action……….. ?? Ansexpression on genes 158. Gfr clinically measured by,…… ? Anscreatinine clearance 159. Cholestrol synthesis …………. ?? Ans:Liver 160. Tractus solitarius …… ?? Ans. Taste sesation 161. Secondary ossification center………?? AnsEpiphysis 162. Recurent larngeal nerve hooks around………. ??? AnsLigamentum arteriosum 163. Benign tumour ………..?? Ans WarthinTumor

164. Pt undrgone tonsillectomy and presented with signs of shock its due to(cant recall exact but it was also easy one) Ans Hypovolemia.. 165. Acetylcholine is formed by A acetyl co A nd choline B preganlianic nerve end Ans A 166. Cricoid level? A. At division of common carotid B. Cervical 6 vertebrae C.Junction of pharax and oesophagus D. Isthumus of thyroid Ans B 167. Patient after RTA losses 2 liters of blood, which of the following will occur??? A. Increase venous tone B. slight stimulation of sympathetic syestem c. Increase heart rate d. Negative feedback compensation All are right Best is AAAA 168. phrenic nerve damage resulting in: (RABIA ALI BCQ) A.loss of sensations from central part of diaphragm.. B.loss of respiratory reflex Ans A 169 linear deposit……….. ?? Ans.good pasture syndrome

Medicine 9th November, 2016 Evening Shift (194 MCQs)

1. HLA-B27 associated with A. Type 1 DM B. Juvenile rheumatoid arthritis C. Addison disease D. Graves disease E. Multiple sclerosis ANS. B 2. Longest acting insulin A. Human lente B. Procine NPH C. Procine lente D. Bovine ultralente E. Bovine semilente ANS. D 3. Information care session is for: A. Take informed consent for the procedure the patient will undergo

45 B. Clear the myths and misconceptions of the patient and relatives about the disease C. Tell the patient and family about new advances in treatment of the disease D. Take informations from the family and friends of the patient about the disease E. Tell the patients about the facilities available to in the hospital for the treatment of the disease ANS. B 4. A boy while playing was hit to right elbow. He developed swelling and bruise. What is the cause of swelling? A. Carbohydrates B. Proteins C. Fats D. LDL E. Water and electrolytes ANS. B 5. For a substance X GFR 125mg/ml, Tm 125mg/mint, plasma concentration is 200mg/100ml, What will be the filtration, reabsorption and excretion of substance X? A. 250 filtration, 125 reabsorption and 125 excretions B. 125 filtration, 125 reabsorption and 0 excreted C. 200 filtration, 125 reabsorption and 75 excreted ANS. A 6. Patient with backpacks, diarrhea, fatigue. Stool culture shows binucleate organism with 5 flagella. Which drug you'll give? A. Stibogluconate B. Metronidazole C. Sulphadoxine D. Quanidine ANS. B 7. Fenestration in choriocapillaries numerous and large in A. Equatorial plate B. Around optic disc C. Ora serata D. Sub macular kuch tha ANS. C 8. Latent tetany seen in A. Hyperparathyroidism B. Graves disease C. Addison disease

D. Con's disease E. Cushing disease ANS. D 9. A 50 years patient who used to drinks from 20 years. His labs are Bilirubin 5 times raised than normal, ALT 2 times and ALP 3 times. His histopathology of liver will show? A. Piecemeal necrosis B. Bridging necrosis C. Councilman bodies D. Periportal Lymphocytosis ANS. A 10. Antipsychotic effect of phenothiazine due to: A.Block muscarinic receptors in corpus striatum B. Block dopamine receptors in corpus striatum C. Block dopamine receptors in limbic system D. Block adrenrgic receptors E. Block Serotonin receptors in limbic system ANS.C 11. Rexed lamina 3, 4 present in A. Lateral Gray horn B. Posterior Gray horn C. Anterior Gray horn D. Gray commissure of spinal cord ANS. B 12. A patient had non-productive cough for 2 months. Now presented with blood streaked sputum. Left Lung has 5 cm hilar mass. Sputum shows hyper chromatic, pleomorphic cells with scanty cytoplasm. Cause of this condition? A. Silicosis B. Asbestosis C. Smoking D. Passive smoking E. Radon exposure ANS. C 13. A person visited Thailand, now presented with fever, chills, body aches and postural hypotension. What is the most probable diagnosis? A. Syphilis B. Malaria C. Dengue D. Chagas disease

46 ANS. B 14. A tourist 2 weeks after returning from another country presented with fever, generalized lymphadenopathy, and rash. Atypical Lymphocytes on peripheral smear. Most probable diagnosis? A. HIV (seroconversion) B. Falciparum Malaria C. Chagas disease D. Lyme disease ANS. A 15. Fibres arises from main sensory nucleus of spinal cord A. Fasciculus gracilis B. Lateral spinothalamic tract C. Posterior spinothalamic tract D. Anterior spinothalamic tract ANS. A 16. Regarding HPV: A. Serotype 16,18, 31 associated with dysplasia and cervical carcinoma B. Serotype 11 associated with ca cervix C. More than 100 genotypes D. Vaccination effective only after infection E. Vaccination at 20-23 years age is recommended ANS. A 17. Nifedipine causes refectory hypotension when given with halothane due to A. Causes tachycardia B. Causes vasoconstriction C. Has long action when given orally D. Has synergetic effect on calcium channels ANS. C 18. Normal ECG do not record electrical activity of A. AV node B. SA node C. Ventricles D. Left atrium E. Right atrium ANS. B 19. Counter part of mesonephric duct A. Appendix B. Ejaculatory duct C. Round ligament D. Seminal vesicle ANS. B 20. Axons:

A. Carry impulses toward the cell bodies B. Carry impulses away from the cell bodies C. Carry pain impulses D. Are several in numbers for multipolar neurons? E. Are found primarily in the gray matter ANS. D 21. Basement membrane of lung alveoli: A. Comprises of type 2 collagen B. Contributes in the formation of surfactant C. Has chondrotin sulphate as a constituent component D. It is a mucopolysaccharide E. Separates epithelium from endothelium in alveolar wall ANS. E 22. Cardiovascular change in geriatric A. Systolic hypotension B. Diastolic hypotension C. Decrease elasticity of myocardial ANS. C 23. Blood vessel valve present in A. Large size veins B. Medium size veins C. Muscular arteries D. Fenestrated capillaries ABS. B 24. Highest oxygen tension in: A. Pulmonary capillaries B. Arterial blood C. Venous blood D. Left ventricle E. Right Atrium ANS. A 25. Circumduction is: A. Abduction/adduction & flexion/extension B. Medial rotation/lateral rotation & flexion/extension C. Flexion and extension D. One axis movement ANS. A 26. Plateau occurs due to A. Na influx B. Na and calcium influx C. Potassium channels opens D. Chloride channels ANS. B 27. Ischio-pubic ramus fuse at the age of A. 7 years B. 1 years C. 24 years

47 D. 21 years E. 14 years ANS. E 28. During isovolumetric contraction what will happen: A. Decrease coronary blood supply B. First heart sound splits C. Semilunar valve open ANS. A 29. True about Venous return: A. Will become zero when all circulatory responses abolish B. Will increase when mean systolic pressure falls C. Is decrease when right atrium pressure decreases? ANS. A 30. The axillary artery: A. Lies posterior to pectoralis minor B. Lies lateral to the lateral cord of brachial plexus C. Lies lateral to the short head of bicep D. Extends to the lowest border of pectoralis major ANS. A 31. Obese person working as a Clerk with sedentary life style, was routinely checking his blood pressure and blood pressure was raised. He was investigated but no cause was found, his raised blood pressure is due to? A. Increase Vessels tone B. Decrease diastolic blood pressure C. Increase heart rate ANS. A 32. Lesion of caudate, putamen and loss of GABA neuron in substantia nigra and gollibus pallidus. Patient is suffering from which disease A. Parkinsonism B. Hemiballismus C. Chorea D. Athetosis E. Ballismus ANS. C 33. Retic count decrease in A. Chronic renal disease B. Hereditary spherocytosis C. Bleeding from gut D. Polycystic kidney disease ANS. A

34. A patient after CVA has loss of appetite to food and drink and tendency to throw. Cause of his decrease thirst? A. Vasopressin B. Stimulation of lateral hypothalamus ANS. A 35. Muscle tone of axial ans paraxial antigravity muscles control by A. Rubrospinal tract B. Pontine reticulospnal C. Medullory reticulospinal ANS. A 36. A 12-year-old boy with hemophilia A has had episodes of pain about his knees for the past 6 years. Over time, there has been an increase in size of his knee joints, with deformity. Laboratory studies show decreased levels of coagulation factor VIII activity. Which of the following materials is most likely to be seen within the joint space following episodes of pain? A. Lipofuscin B. Russell bodies C. Neutrophils D. Cholesterol crystals E. Anthracotic pigment ANS. D 37. Enterococcus faecium: A. Resistant to ampicilin B. Resistant to carbapenams C. Usually Sensitivity to aminoglycosides D. Catalase positive E. Frequent cause of pyogenic infections ANS. A 38. Vertigo with nausea and vomiting associated with? A. Dorsal vagus nucleus juxtaposted to vestibular nuclei B. Vertebral artery ischemia C. Vagus and glossopharyngeal nucleus ANS. A 39. Secretry Granules of neurohypophysis are present at A. Nerve endings B. Pituocytes C. Sinusoids ANS. A 40. A case of Androgen insensitivity syndrome was referred to your clinical. What investigation you Wil NOT do in this case?

48 A. Serum 17 OH progesterone B. Testosterone level after stimulation with HCG C. Dihydrotestosterone after stimulation with HCG D. FSH and LH E. Ultrasound abdomen ANS.C 41. Polysaccharide antigen conjugated to increased immunity with vaccine A. Meningococcal B. Pertusis C. Chlamydia D. Vibrio ANS. A 42. A neutropenia, hypotensive patient was admitted in oncology ward with history of dysuria and high grade fever. His blood culture yielded growth of pseudomonas aeruginosa. The disease as a result of excessive release of the most likely. Bacterial product in the blood is? (Asim Shoaib Medicine BCQ) A. Retiulocyte macrophage stimulating factor B. Granulocyte stimulating factor C. Tumor necrosis factor(TNF) D. IL 1 ANS. C 43. Proto oncogene converted to oncogene A. Point mutation B. Translation C. DNA transcription ANS. A 44. Viruses cause malignant lesion by alternation in: A. Oncogene B. Proto oncogene C. DNA ANS. B 45. Which fibres sensation will be lost first after administration of local anesthetic agent? A. A delta B. B fibers C. C fibers D. A beta ANS. A 46. Alkalosis can cause: A. Hypokalemia B. Hyperkalemia

C. Hyponatremia D. Hypernatremia ANS. A 47. An isolated complete rupture of anterior cruciate ligament will result in instability of tibia over femur. The direction of instability will be A. Anterior B. Antero medial C. Antero lateral D. Postero lateral E. Postero medial ANS. A 48. Which of following tumor in children is due to gene amplification? A. Neuroblastoma B. Retinoblastoma C. Wilms tumor D. Nasopyryngeal carcinoma E. Burkitt lymphoma ANS. A 49. From a population 2 groups were selected on age basis and then subgrouped are assigned randomly for study. Type of sampling is: A. Random stratified sampling B. Simple random sampling ANS. A 50. Most effective in preventing DVT? A. Intermittent Pneumatic compressions B. Low dose of LMWH C. Low dose of unfractioned heparin ANS. A 51. Vitamin A deficiency most important A. Xerophthamia B. Increase predisposition to skin cancer C. Increase predisposition to lung cancer D. Infertility ANS. A 52. Ribosomes A. Protein synthesis B. In human composed of two equal subunits of S50 and S40 C. Attach to endoplasmic reticulum and synthesize cytoplasmic protein ANS. C 53. Characteristic feature of infarction: A. Frequently occurs due to pressure B. In kidney is associated with red infarct C. In liver it produces suppuration D. Is a usual consequence of TB

49 E. Usually seen with the arterial occlusion ANS. E 54. Basillar artery give rise to? A. Posterior cerebral arteries B. Anteriorly cerebral arteries C. Posterior communicating arteries D. Anterior communicating arteries ANS.A 55. Most abundant glial cells in gray matter A. Fibrous astrocytes B. Protoplasmic astrocytes C. Microglial D. Oligodendrocyte E. Ependymal ANS.B 56. Which test you'll do for diagnosis of Typhoid fever during the second week? A. Blood culture only B. Blood culture plus Widal C. Stool culture D. Urine culture ANS.B 57. Test for definitive diagnosis of TB? A. Demonstration of AFB B. Caseous granuloma C. Epitheliod cells ANS.A 58. TB is important because it is: A. Type 4 Hypersensitivity reaction B. Type 3 HSR C. Type 1 HSR D. Type 2 HSR ANS.A 59. Which vessel is involved in portal hypertension? A. Portal vein B. SMA C. IMA D. IVC ANS.A 60. Symphysis pubis is which type of joint? A. Secondary cartilaginous joint B. Primary cartilaginous joint C. Ghamphosis D. Syndesmosis E. Plane synovial joint ANS.A 61. Which of the following acts through cAMP?

A. TSH B. Insulin C. Thyroxine D. GnRH E. Aldosterone ANS.A 62. In Normal physiological conditions Thyroxine causes? A. Dec fatty acid oxidation B. Dec gluconeogenisis C. Inc fatty acid oxidation D. Ca deposition to bone E. Inc protein synthesis ANS.A 63. Most common pattern of inherited diseases? A. X linked recessive B. Autosomal dominant C. Multifactorial D. Autosomal recessive E. X linked dominant ANS.C 64. Child with generalized edema and proteinurea 6g per day. Which part of the kidney involved? A. Interstitium B. Basement membrane C. Collecting tubules D. Proximal convulated tubules E. Distal tubules ANS.B 65. True about Alpha motor neuron A. Myelinated B. Unmyelinated C. Ends at motor end plate D. Terminate in intrafusal fibre of neuromuscular spindle ANS.A 66. If root of left lung is injured which structure will be spared? A. Vagus nerve B. Azygous vein C. Phrenic nerve D. Pulmonary ligament ANS.A 67. The structure which arches over the root of left lung. A. Azygous vein B. Left recurrent l nerve C. Phrenic nerve D. Arch of aorta

50 ANS.D 68. Vagal stimulation of the heart causes: A. Increased heart rate B. Increase PR interval C .Increase force of heart contraction D. Increased cardiac output ANS.B 69. Only nerve affected in Multiple Sclerosis is : A. Optic B. Trigeminal C. Olfactory D. Occulomotor E. Abducent ANS.A 70. Most important event in IRREVERSIBLE cell injury A. Damage to cell membrane B. Loss of co enzyme C. Massive calcium influx D. Swelling of lysosomes E. Swelling of mitochondria ANS.C 71. Child with recureent knee joint swelling and pain. Factor 8 was 90% and factor 9 was 8%. What is the disease child suffering from? A. Christmas disease B. Hemophilia A C. VWb disease ANS.A 72. A person working in coal mine. His work is stone polishing and sand blasting. He might can develop which of the following disease in the future? A. Asbestosis B. Silicosis C. Anthracosis ANS.B 73. Transient adhesion is caused by? A. Integrin B. Selectin ANS.B 74. A person goes from standing to supine position. What will happen? A. Increase in stroke volume B. Increase leg muscles pressure C. Dec firing of baroreceptors ANS.A 75. An African girl who has history of anemia since birth and history of multiple

blood transfusion and episodes of chest and abdominal pain..presented with non healing leg ulcer, What will her blood shows? A. Sickle cells B. Schistocytes C. Heinz bodies ANS.A 76. Jejunum is recognised by? A. Single or double arcade arteries B. Narrow luman C. More Fat in the mesentery D. Thin mucus with less blood supply ANS.A 77. Which of the following shows an initial Left to Right shunt before reversal? A. Eisenmenger complex B. Transposition of great vessels C. Persistent Truncus arteriosis D. Ebstein anomaly E. TOF ANS.A 78. During which phase of the cardiac cycle 2/3 of ventricular filling occurs and 3rd heart sound is produced? A. Rapid inflow B. Atrial contraction C. Slow ejection D. Isovolumetric relaxation ANS.A 79. Which heart sound is produced due vibration of the ventricles during rapid inflow? A. 3rd heart sound B. 4th heart sound C. Ist heart sound D. 2nd heart sound ANS.A 80. Most abundant spingolipids in Gray matter of the brain? A. Spingomyelin B. Cerebroside C. Globoside ANS.A 81. Parasympthetic flow to heart via???? A. vagus nerve B. Superficial cardiac plexus C. Deep cardiac plexus D. T1-T5 E. T1-T4

51 ANS.A 82. Drug given in HOCM A. Verapamil B. Diltiazem C. Amlodipine D. Nifedipine ANS.A 83. Axillary Fascia derived from? A. Prevertebral fascia B. Pretracheal fascia C. Superficial layer of Cervical fascia ANS.A 84. Most potent pain mediator? A. Prostaglandin B. Histamine C. Bradykinin ANS.C 85. Denticulate ligament: A. Lateral extension of pia matter B. Extension of filum terminale ANS.A 86. Superficial inguinal ring present in? A. External oblique aponeurosis B. External and internal oblique muscles C. Fascia transversalis D. Conjoined tendon ANS.A 87. Paraneoplastic syndrome is caused by: A. Small cell carcinoma of lung B. Renal cell carcinoma Other options were irrelevant ANS.A 88. Most important cause of death in Rheumatic fever? A. Myocarditis B. Pericarditis C. Mitral stenosis ANS.A 89. Thyroidectomy was done in left lower lobe. Artery was ligated which supply this area. Which nerve have more chances to be damaged during the operation? A. Recurrent laryngeal nerve B. External branch of superior laryngeal nerve C. Internal branch of superior laryngeal nerve ANS.A 90. Prolong use of which of the following will cause increase in gastrin level A. Omeprazole/PPIs

B. H2 blockers C. Sucralfate D. H1 blockers ANS.A 91. Which of the following disease with raised ALT most commonly? A. Acute viral hepatitis B. Hepatic malignancy C. Sclerosing cholangitis ANS.A 92. A child having purpura in all over the body, otherwise normal. Cause of pupura? A. Decrease platelate count by 20,000 B. Decrease platelets count by 90,000 C. Factor 8 deficiency D. VwB disease ANS.A 93. Ventral spinothalmic tract carry? A. Itch and tickle B. Pain C. Temperature D. Vibration E. Proprioception ANS.A 94. QRS complex produced prior to? A. Ventricular contraction B. Atrial contraction C. Isovolumetric relaxation ANS.A 95. Married man had no children for few years and was diagnosed as sterile with azospermia. what is investigation of choice? A. Serum FSH and serum LH B. Serum prolactin C. Serum testosterone D. Semen analysis ANS.A 96. Which of the following drains into jugulodiagastric nodes? A. Palatine tonsils B. Pharyngeal tonsils C. Submandibular gland D. Sublingual glands ANS.A 97. A person lying naked in a room at temperature of 25°C. His heat loss will be through? A. Radiation plus conduction B. Convection

52 C. Conduction D. Evaporation E. Sweating ANS.A ref Ganong 98. Formation of Interstitial fluid increases with decreased in? A. Capillary hydrostatic pressure B. Capillary permeability C. Interstitial fluid colloid osmotic pressure D. Lymphatic flow E. Plasma colloid osmotic pressure ANS.E 99. A patient has nodule on vocal cord. The least likely mechanism? A. Hyperplasia B. Atrophy C. Hypertrophy ANS.B 100. 30 yrs old person has stab wound on right side of chest. what will happen? A. Ipsilateral lung collapse and ipsilateral chest wall spring out B. Ipsilateral lung collapse and contralateral chest wall collapse C. No change in chest wall D. Contralateral lung collapse and ipsilateral chest wall collapse E. Ipsilateral lung collapse and contralateral spring out ANS.A 101. Deficiency of pulmonary surfactant: A. increases surface tension, increases compliance B. increases surface tension, decreases compliance C. decreases surface tension, decreases compliance D. decreases surface tension, increases compliance E. increases surface tension, no effect on compliance ANS.B 102. A patient presents to the emergency department with 6 hours history of severe central chest pain and dyspnea. Which of the enzymes would be elevated? A. SGOT B. SGPT C. LDH D. Alkaline phosphatase E. CPK

(CK-MB, Trop were not in the options) ANS.E 103. Organism associated with bladder squamous cell carcinoma A. Schistosoma hematobium B. Schistosoma Mansoni C. Schistosoma Japanicom ANS.A 104. Not a part of natural immunity A. Plasminogen B. Interferon ANS.A 105. Smoking affect organogenesis during which period of gestation? A. 3-8 weeks B. 4-10 weeks C. 12-15 weeks D. 1-2 weeks ANS.A 106. Open wound healing includes: A. Contraction of myofibroblast B. Fibroblast C. Macrophages D. Lymphocytes ANS.A 107. Basal rhythm of small intestine? A. Synchronized with stomach B. Decrease when mechanical activity start C. Variable effect by hormone D. Constant in duodenum variable else where ANS.D 108. Exotoxin: A. Corynebacterium diphtheria B. Staph aureus C. Strep pyogen D. Clostridium tetani ANS.A 109. Which among following is a chemical carcinogen? A. Alpha 1 antitripsin B. Benzidine C. Ethyl alchohol D. Alfatoxin ANS.B 110. Regarding Glycolysis which one is correct: A. Reversible B. Hexose phosphate converted to lactate and pyruvate C. Same as Hexose monophosphate shunt

53 D. Includes hexose sugars to pyruvate ans>D 111. 55-year-old woman with insulin dependent diabetes of 40 years’ duration complains of severe bloating and abdominal distress, especially after meals. Evaluation is consistent with diabetic gastroparesis. The drug you would be most likely to recommend is A. Docusate B. Dopamine C. Loperamide D. Metoclopramide E. Sucralfate ANS.D 112. Most specific for diagnosis of SLE A. Anti dsDNA B. ANA C. Anti SSA D. Anti histone antibodies ANS.A 113. Patient presented with dryness of eyes and mouth. Which of the following would be most helpful in the diagnosis? A. Anti SSA B. Anti drNA C. ANA ANS.A 114. Frequency distribution A. Tells distribution of value appear in data B. Tells distribution of value which is missing C. Gives percentage of values D. Can’t be used for data from different categories. ANS.A 115. Most potent response in massive hemorrhage? A. CNS ischemic response B. Baroreceptors C. Peripheral chemoreceptors D. Brain bridge reflex ANS.A 116. Difference between septic and hypovolumic shock? A. Temperature B. Cardiac output C. Heart rate ANS.A 117. Regarding pharyngeal phase of deglutition which is appropriate?

A. Vocal cords strongly approximate B. Larynx moves downward C. Esophageal sphincter is contracted D. Palatopharyngeal fold moves downward ANS.A 118. The effect which a drug causes on body is called? A. Pharmacodynamics B. Pharmacokinetics C. Bioavailability ANS.A 119. Parasympathetic origin in spinal cord A. S2 S3 S4 B. S1 S2 S3 C. S3 S4 S5 D. L1-L2 ANS.A 120. True about Ulnar nerve A. Loss of abduction and adduction of fingers B. Loss of opposition of thumb C. Wrist drop D. Thenar muscles wasting ANS.A 121. Epileptic drug which increases the effect of other Antiepileptics when given together is A. Valproic B. Phenytoin C. Carbamazepine ANS.A 122.Drain is inserted in chest wall at the lower border of rib. Which structure will be damaged first ? A. Intercostal nerve B. Intercostal vein C. Intercostal artery ANS.A 123. Which vein drains into the left brachiocephalic vein? A. Inferior thyroid vein B. Superior thyroid vein C. Middle thyroid vein ANS.A 124. Preganglionic autonomic fibres are? A. B fibres B. C fibers C. A beta D. A delta ANS.A

54 125. Avidin binds with A. Biotin B. Vit B12 C. Vit C D. Vit D E. Riboflavin ANS.A 126. In pancreatic fistula somatostatin helps by A. Decreasing output of secretions B. Increasing ph of secretions C. Increasing blood flow of gland D. Inhibiting enzymes ANS.A 127. Which is present just lateral to the trachea? A. Vagus nerve B. Phrenic nerve C. RLN ANS.A 128. Usual cause of atrophy of breast in adult female is deficiency of? A. Gonadotropin B. Growth hormone C. Estrogen D. Estrogen nd progesterone E. Progesterone ANS.C 129. Women using OCPs are most common at risk for A. Thromboembolism B. Osteoporosis C. Osteomalesia ANS.A 130.One of the following is related to the diagnosis of asthma in 20 yr old male? A. FEV1/FVC <65 B. PO2 =40 mmHg C. Respiratory rate=12 D. Tidal volume 500 ml E. Vital capacity= 3.0 ANS.A 131. Male infertility due to testicular cause? A. Cryptorchidism B. Hypospadiasis C. Kliinefilter syndrome D. Varicocele ANS.A 132. Metabolic rate is increased by: A. Thyroxine

B. Decrease body temperature ANS A 133. A 65 Year old mate patient Presented with chest pain for last 30 mins. And has ST segment elevation in leads V1 to V4. He is most likely to have A. Anterior infarction B. Anterolateral infraction C. Lateral infarction D. Inferior infarction E. Posterior Infarction ANS.A 134. Which of the following is not directly caused by cortisol in Cushing's syndrome? A. Buffalo hump B. Thin lower limb C. Hirsutism D. Pendulous Abdomen ANS.C 135. In bone new matrix and fibres are form by? A. Fibroblast B. Osteoclast C. Osteoblast D. Osteocytes ANS.C 136. Blood Antigens: A. Present in saliva B. Called agglutinins ANS.A 137. Anemia of chronic disease A. Serum transferin saturation is low B. Iron is mirror for Bonemarrow ferritin C. Iron not coming out of monocytic macrophage Cant recall other 2 options ANS.C 138. Q fever is caused by which organism? A. Rickettsiae B. Rick C. Virus D. Parasite ANS.A 139. Primary Center of Ossification at Epiphysis of Long bone at Birth? A. Lower end of Femur B. Lower End of Humerus C. Upper End of Fibula D. Upper End of Tib

55 ANS.A 140. Patient with Hypotension, shouldnot give benzodiazepine in which case? A. Hypovolumic B. Obese C. Age > 65 D.Pregnancy ANS.A 141. Autoimmune hemolytic anemia is diagnosed by A. Positive antiglobin B. Splenomegaly C. Spherocytosis D. Reticulocystosis E. Erythroid hyperplasia ANS.A 142. Right iliac fossa pain reffered to umbilicus??? A. T 10 B. T 8 C. L 1 ANS.A 143. Regarding Thymus A. Supplied by internal thoracic and inferior thyroid artery B. T Lymphocytes enter into the blood stream through post capillary venules C. Is a lobulated organ D. One option was about something collagen (Can’t recall) ANS.B 144. Small emboli from veins is most commonly lodged in A. Lungs B. Heart C. Brain D. Kidneys E. Liver ANS.A 145. Lobar pneumonia diagnosis A. Sputum culture B. Sputum staining C. Blood culture ANS.A 146.True about Kidney: A. Cuboidal epithelium in parietal layer of bowman capsule B. Podocytes on visceral layer of bowman capsule C. DCT longer than PCT

D. Glomerulus has single arteriole Ans>B 147. Aphasia due to lesion in A. Temporal lobe B. Parietal C. Occipital ANS.A 148. Plasma half life of a drug depends upon: A. Rate of clearance B. Volume of distribution C. Dose D. Rate of absorption E. Route of administration ANS.A 149. Prozone phenomenon? A. Antigen excess B. Antibody excess C. Antigen depletion D. Antibody depletion ANS.B 150. Gonorrhea DOC? A. Penicillin G B. Ceftriaxone C. Gentamycin ANS.A 151. Dose of which of the following drug should be reduced in renal impairment: A. Gentamicin B. Rifampicin C. Penicillamine D. Phenytoin ANS.A 152. Primary brain vesicle A. Mesencephalon B. Telencephalon C. Diencephalon D. Metencephalon E. Mylencephalon ANS.A 153. Diabetic gangrene: A. Atheromas in arteries B. Always start as moist necrosis C. A/V fistula develop D. Large arteries are always responsible ANS.A 154. Most common cause of sex chromosome abnormality???? A. Hermaphrodite B. Klinefelter syndrome C. Downs Syndome

56 D. Multi X female Ans>C 155. Which of the following veins drains into coronary sinus and itself becomes part of coronary sinus? A.Great cardiac vein B. Middle cardiac vein C. Anterior cardiac vein D. Small cardiac veins ANS.A 156. A 24 years old female with 12 weeks of amenorrhea presents with frequent burning micturation and high grade fever. Diagnosis? A. DM B. Early pregnancy C. Urinary bladder stone D. UTI E. Urethral stricture ANS.D 157. Sodium reabsorption in baso lateral membrane occurs through: A. Gucose co transport B. Na K pump ANS.B 158. Parasympathetic effect on heart increases permeability in SA node to A. Sodium B. Potassium C. Calcium D. Potasium and sodium E. Sodium and calcium ANS.B 159. A hematocrit of 41% means: A. 41% formed elements in blood are RBCs B. 41% blood in plasma C. 41% of blood in serum D. 41% of formed elements in blood compromises of RBCs, WBCs, platelets ANS.D ref ganong 160. GFR is decreased by A. stone in ureter B. Angiotensin 2 C. Afferent arterioles dilation ANS.A 161. Ziinc is important essential element in one of the following: A. Carbonic anhydrase B. Cellular oxidation C. Vitamin B12 D. Glycolysis

E. Ketogenesis ANS.A 162. The most usual complication of wound healing in black population is????? A. Hemorrhage B. Keloid formation C. Malignant change D. Metaplasia E. Premalignant change ANS.B 163. In a healthy alert adult sitting with eyes closed, the dominant EEG pattern with electrodes over the occipital lobe? A. Alpha B. Theta C. Delta D. Beta E. Fast irregular low voltage activity ANS.A 164. Paracetamol A. Weak anti inflammatory property B. Nephrotoxic ANS.A 165. Cerebellar lesion A. Ataxia B. Rigidty C. Difficulty in initiating movement D. Resting tremors ANS.A 166. Cause of cyanosis: A. Decrease O2 in the blood B. Decrease Hb C. Increase concentration of deoxy hemoglobin ANS.C 167. Secondary oocyte is surrounded by corona radiate, Which of the following layer contribute to the formation of corona radiate? A. Granulosa cells B. Theca interna cells C. Theca externa cells ANS.A 168. Gas gangrene. Cause of death? A. Septecemia B. Bacteremia C. Toxemia ANS.C 169. Decreased ESR is due to increase in A. Fibrinogen

57 B. Plasma albumin ANS.B 170. Which of the following protein binds free hemoglobin in plasma ??? A. Alpha 2 macroglobulin B. Transferin C. Heptoglobin ANS.C 171. Difference between malignant and normal cells such that malignant cells repair the shortening of chromosome so that they can continue dividing. Enzyme responsible? A. Telomerase B. Reverse transcriptase C. Isomerase ANS.A 172. Cimetidine is not used frequently because it causes??? A. Decrease hepatic metabolism B. Nausea and vomiting ANS.A 173. Gonorrhea can be easily diagnosed by: A. Gram stain B. ZN stain C. Culture D. PCR ANS.A 174. Glucagon secretion is increased by? A. Exercise B. secretin C. Somatostatin D. Fatty acid E. Increase Glucose ANS.A 175. Most common premalignant lesion of the skin? A.Sebhorric keratosis B.Intrdermal Nevus C.Junctional Nevus Ans>A 176. Inhibitory factor released by hypothalamus against which one of the following? A. Prolactin B.Oxytocin C. ADH D. Growth Hormone ANS.A 177. Medial lemiscus formed by

ANS. Nucleus gracilis and cuneatus 178. One question about Autosomal dominant A. Incomplete penetrance B. Both genes are mutated C. Early presentation ANS. A 179. Patient with eyelid dropping, muscle weakness, anterior mediastinal mass. What is seen? ANS. Antibodies against acetylcholine receptors 180. After hemorrhage which will return last to normal? ANS. Red blood cells in peripheral smear 181.Hearing is best perceived at? Ans> Superior temporal gyrus 182.Stress Hormone? Ans>Cortisol 183. Most common cause of gastric malignancies? Ans>H pylori 184.After feeding the baby, diarrhea develops. Cause? Ans>Gastrocolic reflex 185. Ectoparasite? Ans?Lice* 186. Pituitary mass? Ans>Bitemporal hemonopia 187.One about Graft rejection.. ?? Ans>Associated with HLA 188. One was about knee jerk/When hit patellar tendon?? Ans>Contraction of quadricep muscles 189. Highest renal clearance? Ans>PAH 190. Potassium gradient inside and outside the cell is maintained by? Ans> Na/K pump 191. Which cells form collagen and amorphous substance? Ans> Fibroblasts 192. Fatal disease of a patient A. Should not be told to the patient and his family B. Tell the patient and his family as soon as possible C. Crisp clear information should b provide to the patient according to his need D. Should be told the relatives of that patient

58 ANS.C 193. One was about meningitis..MCC? Ans>Streptococcus (No age etc were given) 194. Sternum ossify at the age of? Ans>21 Years Regards Dr. Irshad Hussain Remember me in your prayers. November 12, 2016

10TH NOVEMBER, 2016 MORNING SHIFT MEDICINE & ALLIED (PAPER I & II)

1. Amount of urine excreted by a normal adult in a day a. 500 ml b. 1000 ml c. 3 litre d. 2200 ml e. 5 litre Ans: B here ( Normal range is 800-2000ml per 24 hrs) 2. Narcotic with the lowest pKa is: a. Sufentanil b. Morphine c. Fentanyl d. Meperidine Ans: B (Alfentanil > Remifentanil > Morphine > Sufentanil > Fentanil > Meperidine) Ref. Lippincott Clinical Anesthesia, 6th ed, chp 19, page 469 3. 19 year old girl presents with history of weakness and lethargy. On examination she is pale. Hb is 6.2g/dl MCV 62 MCHC (---reduced---). Diagnosis? a. Anemia of chronic disease b. Iron deficiency anemia c. Thalassemia Ans: B 4. Pregnant lady with yellow sclera and dark colored urine… Obstructive jaundice… What is the appropriate investigation in this patient? a. Serum glutamyltransferase b. alkaline phosphatase c. Gamma glutamyl transferase Ans: C 5. A young boy underwent spleenectomy. After the procedure he gets repeated

infections. The most likely organism involved is: a. Streptococcus pneumonia b. HemophilusInfluenzae c. Staphylococcus aureus d. Streptococcus Pyogenes Ans: A 6. Dermatome distribution around umbilicus is at the spinal level of a. T7 b. T12 c. T10 d. L1 Ans: C 7. Thoracolumbar outflow is: a. Sympathetic b. Parasympathetic c. Cranial nerves Ans: A 8. Pudendal nerve block can be given by feeling which of the following landmarks? a. Ischial tuberosity b. Ischial spine Ans: B 9. True about facial nerve a. Arises from upper part of pons b. Lesion compresses vestibular part of cranial nerve 8 c. exits skull through foramen rotundum d. receives upper motor fibers from contralateral cortex only e. contains secretomotor fibers. Ans: E 10. levato rani muscle takes motor supply from a. Pudendal nerve b. Iliohypogastric nerve Ans: A 11. Cervical rib a. compresses brachial plexus and Subclavian artery ans A 12. True about descending thoracic aorta a. supplies lower 8 intercostal spaces b. supplies pericardium, lungs and esophagus c. pierces diaphragm at L1 and unites with abdominal aorta Ans: B 13. Progesterone works in contraceptive pills by

59 a. spermicidal action? b. induces uterine contraction c. inhibits ovulation d. inhibits maturation of ovarian follicles Ans: C here (The progesterone only pills work by thickening the mucus in the neck of the womb, so it is harder for sperm to penetrate into the womb and reach an egg and sometimes it may also prevent ovulation-depends upon the type of progesterone pills) 14. Regularly placed lamella are seen on a histopathological slide. Such an arrangement is present in a. Cancellous bone b. Compact bone c. Dentine Ans: B 15. When activated, C3a and C5a cause?? a. Vasodilation and anaphylactogenic response. b. cytolysis by MAC c. PhagocytoseIgG coated bacteria Ans: A 16. True about neutrophils: a. Produced in spleen and lymph nodes b. Migrate in and out of blood c. Actively phagocytose microbes in plasma. Ans: C 17. History of abdominal pain, bloating, fatigue, weakness, pallor. Stool examination reveals some eggs. Most likely organism is:?? a. Ascaris lumricoides b. Ankylostoma duodenale Ans: A (Their eggs are visible in feces under microscope) 18. An 8 month old baby boy has history of repeated attacks of sinusitis, rhinitis, diarrhea and 2 episodes of pneumonia. On investigations his B lymphocytes are reduced with deficiency of all types of immunoglobulins. Diagnosis? a. Isolated IgA deficiency b. Common variable immunodefeciency c. X linked agammaglobulinemia Ans: C 19. Out of these, a benign tumor is a. Wilm’s tumor b. Neuroblastoma

c. Hepatoma d. Warthin tumor Ans: D 20. Secretion of gastrin is inhibited by a. Acetylcholine b. Somatostatin Ans: B 21. A man after long marathon race will show blood level changes as follows a. Increase Insulin and decrease glucagon b. increase Insulin and increase glucagon c. decrease insulin and decrease glucagon d. decrease insulin and increase glucagon e. no change Ans: D 22. Vitamin required for the dietary fatty acids to undergo beta oxidation in cells a. Thiamine b. Biotin Ans: B 23. Hormone which uncouples oxidative phosphorylation is: a. Growth hormone ? b. Cortisol c. Glucagon d. thyroxine e. epinephrine Ans: D 24. A female patient has history of thromboembolism/DVT. She is started on oral anticoagulant therapy. Which test will be helpful in monitoring the effects of oral therapy? a. Platelet count b. Prothrombin time c. clotting time d. activated partial thromboplastin time e. bleeding time Ans: B 25. Patient has a lesion of the L4 nerve root. He will feel pain at: a. Anterior upper part of thigh b. Back of thigh c. Front of knee d. Upper part of calf e. lower calf Ans: C 26. Only nerve involved in multiple sclerosis a. Optic nerve b. occulomotor nerve

60 Ans: A 27. Different conditions are associated with different HLA types. For instance a disease of the spine which leads to fusion of vertebrae i.e. Ankylosing spondylitis is associated with which HLA type? a. HLA DR3 b. HLA B8 c. HLA B27 d. HLA DR2 e. HLA DR4 Ans: C 28. A malignant tumor is seen on microscopy. It will show following features: a. Invasion b. Increase nuclear cytoplasmic ratio c. Pleomorphism Ans: A 29. Scanty Barr bodies will be present in: a. Testicular feminization syndrome b. Klinefelter syndrome c. Turner syndrome Ans: C 30. A 20 year old gardener with symptoms of respiratory tract. Chest xray shows a 4 x 3 cm mass in Right lobe of the lung. Most likely diagnosis is: ans. Aspergillosis 31. A student carries a study in a population comparing symptoms in diseased individuals against lack of those symptoms in non-diseased people. This study is called a. Cohort b. Cross sectional c. Interventional Cohort d. Case control Ans: D 32. A young farmer was working continuously on a sunny day for several hours. He collapsed suddenly, labs show sodium 120 mEq/L. Reason? ans. Excessive sweating 33. High pitch frequency in relation to basilar membrane strikes a. round window b. intermittently throughout the length of basilar membrane c. Base of cochlea d. Apex of cochlea

Ans: C 34. Structure affected by a mass in the posterior mediastinum will compress a. Arch of aorta b. ascending aorta c. descending aorta Ans: C 35. A neonate was brought to the emergency with history of fever, neck rigidity. Organism likely to be the causative agent: a. Hemophilus influenza b. Streptococcus Pyogenes c. E. coli d. Staph aureus e. Streptococcus pneumonae Ans: C (MCC of meningitis in new born/neonates is Group B strep and 2nd MC is E.coli) 36. Left shift of the hemoglobin dissociation curve occurs due to a. hypothermia b. increase in temperature c. acidosis Ans: A 37. In a person with meningococcal meningitis and in the event of no known drug allergy the medicine preferred will be ans. Penicillin G 38. cAMP is formed in the cell by the action of a. Enzymes b. Receptors c. Cholesterol Ans: A 39. Substance which induces the flow of saliva a. Motilin b.VIP c. Substance P DAcetylcholine Ans: d 40. Low pressure in the carotid sinus will cause? ans. increase in heart rate and contractility 41. Bleeding time is prolonged in?? a. Hemophilia b. von Willibrand disease Ans: B

61 42. Atrial fibrillation shows following clinical signs: ans. Pulse deficit 43. True about Rh blood group system, Rh agglutinins a. Are absent in Rh negative mother in first pregnancy with Rh positive baby b. All agglutinins are IgM variety c. All Rh negative people have anti Rh antibodies. Ans: A 44. Cardiac output is mainly determined by which of the following? a. Venous Return. b. Stroke Volume. c. Heart Rate d. EDV in the left ventricle e. ESV in the left ventricle Ans: A 45. At what point in the cardiac cycle does T wave appear on ECG? a. Beginning of the refractory period. b. Ventricular repolarization. c. End of an action potential. d. Ventricular depolarization Ans: B 46. Which of the following statements is correct regarding middle meningeal artery? a. can be felt at pterion b. Enters the skull through foramen spinosum Ans: B 47. Heart failure is commonly (associated with/due to?) which of the following conditions? a. Fibrinous pericarditis b. Hemopericardium c. Pericardial effusion d. Chylopericarditis Ans: A 48. Main arterial supply of the Interventricular septum is: a. Anterior Interventricular artery b. Posterior Interventricular artery c. left coronary artery d. left circumflex artery e. Right coronary artery Ans: A 49. A young boy presented with a globular swelling in the midline of neck.

The swelling moved up & down with swallowing. This is because the structure that caused this swelling is in: ans. Pre-tracheal fascia 50. Histamine release causes: a. Venodilation b. Vasoconstriction c. Muscarinic receptor activation d. Arteriolar dilation Ans: A 51. Left recurrent laryngeal nerve is related to: a. Brachiocephalic trunk b. Arch of Aorta c. Azygous vein d. Subclavian e. Esophagus Ans: D 52. Acrosome cap of the sperm is formed by which of the following organelles? a. Peroxisome b. Golgi Bodies c. Lysosomes Ans: B 53. A patient for --- underwent liver transplant surgery. After 5 years this patient would be at a risk of developing a. Skin Cancer b. Lympho proliferative disorders Ans: A (In the first yr after transplant  Lymphoproliferative disorders and After 5 yrs  Skin cancer common. Ref Bailay & Love) 54. One of the following fungus acts by involvement of the Reticulo endothelial system a. Histoplasma b. Blastomycosis c. Cryptococcus d. Candida Ans: A 55. Characteristic feature of the cerebellar disease is: a. Shuffling gait b. Ataxia Ans: B 56. Propranolol acts on: a. Muscarinic receptors b. Beta 1 receptors c. Beta 2 receptors d. Alpha receptors

62 Ans: B (Although it is a non-selective beta blocker and acts on both B1 and B2) 57. Notochord gives strength to the post natal adult body as it becomes one of the following structures a. Vertebral bodies b. Annulus fibrosis c. Nucleus pulposus Ans: C 58. A 2 years old child has the history of premature birth. He is most likely to have the followingcongenital anomaly except a. Patent ductus venosus b. Patent ductus arteriosus c. Patent foramen ovale Ans: A 59. Potassium level in blood is most likely to be increased due to a. Diarrhea b. Spironolactone therapy c. Metabolic Acidosis d. Metabolic Alkalosis Ans: C > B 60. Class III anti-arrythmics include: a. Procainamide b. Disopyramide c. Flecainide d. Quinidine e. Dofetilide Ans: E (Other drugs of this group are Amiodarone, Ibutilide and Sotalol) 61. Why is cimetidine given before surgery? a. to decrease gastric motility b.to block H2 receptors c.to decrease gastric acid secretion Ans: C 62. A patient was taking cimetidine &sucralfate simultaneously for GERD but he was advised to stop taking them both together. Why was he advised this? a. Cimetidine decreases absorption of Sucralfate b.Cimetidine increases the potency of sucralfate C. Cimetidine increases the levels of Sucralfate because of decrease degradation Ans: C 63. A neonate born prematurely has a deficiency of surfactant. She is at a risk of developing

a. Lung hypoplasia b. Respiratory distress syndrome c. Asthma Ans: B 64. Superficial temporal artery is related to which of the following nerves? ans. Auriculotemporal nerve 65. Nerve that supplies Trapezius muscle a. Radial Nerve b. Cranial part of Accessory nerve c. Spinal part of Accessory nerve Ans: C 66. Which nerve supplies the parietal & outer layer of serous pericardium? a. Phrenic b.Vagus nerve Ans: A 67. Which of the following enzymes is involved in the transfer of radicals from one compound to another? a. isomerases b. oxidoreductases c. transferases Ans: C 68. Which of the following is a must for the diagnosis of sepsis? a. TLC <2000 or >15000 b. SBP <70 mmHg c. Blood culture positive for bacteria d. HR >120/min e. HR >100/min Ans: C ( SIRS criteria is absolete now so ans is CC) 69. The most common cause for formation of deep venous thrombosis is: a. Prolonged immobilization b. Protein C deficiency Ans: A 70. Which of the following statements is unlikely to be true about H+ in the renal tubules? a. it can bind with NH4+ b. It can bind with HPO4c. It can bind with HCO3d. It is secreted by H+ ATPase pump e. It can exist as free H+ Ans: A ( H+ can bind with NH3+ not NH4+) 71. Granulocyte cell is: a. Neutrophil b. Platelet

63 c. Lymphocyte d. Monocyte e. Red blood cell Ans: A 72. What will be the labs in primary hypothyroidism? a. Low TSH b. High TSH c. High TSH & high T3, T4 Ans: B 73. Which of the following statements is correct regarding CSF? a. it is reabsorbed by ependymal cells b. same pH & constitution as plasma c. Flows above dura mater d. Reabsorbed by arachnoidal villi Ans: D 74. Which of the following is the characteristic feature of irreversible cell injury? a. Hydropic degeneration b. Clumping of nuclear chromatin c. Cytoplasmic contraction bands d. Cell swelling Ans: C 75. Which of these is a malignant brain tumor? a. Meningioma b. Ependymoma c. Pseudolymphoma d. Glioma Ans: D 76. When a stimulus reaches an excitable membrane, it causes: a. Always generates action potential b. reverses the resting membrane potential c. Chloride influx d. Potassium influx Ans: B 77. True regarding a normal adult heart is that purkinje fibers have fastest speed of conduction. This is because these fibers have a. high no. of gap junctions b. threshold near resting membrane potential c. highest no. of intercalated discs d. largest diameter e. maximum number of sodium channels Ans: C 78. Regarding doctor – patient relationship, confidentiality of the patient

can be breached in which scenario? a. after the death of the patient b. for an insurance claim c. when the patient authorizes you Ans: C..( if u r taking USMLE then prefer B coz insurance has got more importance in USA) 79. A doctor can ensure better communication with his patient by a. Prescribing many medications b. Decorating his office beautifully c. Asking many questions d. Active listening Ans: D 80. Which is true regarding the withdrawal reflex? a. it is a monosynaptic reflex b. includes input from higher centres c. Multisynaptic Ans: C 81. Role of lidocaine as an anti-arrythmic drug is due to its action on? a. prolonging action potential b. Inhibition of the generation of a new potential Ans: B 82. Opioids cause vomiting by? a. Direct activation of vomiting center b. Activating the chemoreceptor trigger zone Ans: B 83. A chronic smoker is working in tire factory for (>10 years) develops hematuria. A diagnosis of transitional cell carcinoma is made on histology. Which carcinogen led to the development of this cancer in him? a. Nicotine b. Nitrates & Nitrites c. Aromatic Amines d. Aniline dyes e. Smoking Ans: E (Tar in cigarette smoke causes cancer while nicotine doesn’t cause cancer, it is an addictive agent which may keep you smoking.) 84. A patient underwent surgery for mitral valve replacement. After 8-12 months of the surgery, she develops complains of pallor & dark colored urine. What is the likely cause? a. Anemia of chronic disease

64 b. Iron deficiency anemia c. Infective endocarditis d. Microangiopathic hemolytic anemia Ans: D 85. Regarding black water fever, the correct statement is: ansz. associated with falciparum malaria 86. A young patient presents to you with fever, neck rigidity. Diagnosis of meningitis is made. Gram stain of CSF reveals pus cells & cocci. Likely organism is: a. Streptococcus pneumonia b. Streptococcus Pyogenes c. Neisseria meningitides d. Staphylococcus Aureus Ans: A 87. The strongest inhibitory signals in central nervous system come from which neurons? a. Substantia nigra b. Pyramidal cells of cerebral cortex c. Purkinje cells of cerebellar cortex Ans: C 88. Malaria is associated with? a. Lymphocytosis b. Hemolytic anemia c. Thrombocytosis Ans: B 89. An old lady has 5 years history of fatigue. Investigations reveal Hb = 12 g/dl, MCV = 120. Bilirubin = 12, AST = 70, ALT = 80.Haptoglobin = 80. Cause of her lethargy is? a. Iron deficiency anemia b. Megaloblastic anemia c. Hepatitis C Ans: B 90. A 9 year old boy presents to you with the history of yellow sclera & dark colored urine. Most appropriate investigation would be? a. SGPT b. SGOT c. Bilirubin d. SGPT & bilirubin Ans: D 91. On antero-posterior view of chest radiograph, the sternocostal surface of heart is made of: a. Right atrium

b. Right ventricle c. Left ventricle d. Left atrium Ans: B 92. An old man developed severe chest pain in an event & died in a few minutes. Cause of death was established to be dissecting aortic aneurysm. The autopsy finding in this man would be: ans.Medial necrosis 93. Which structure passes in front of the root of Right lung? a. Azygous vein b. Hemiazygous c. Phrenic Nerve Ans: A 94. A 12 year old child has dwarfism due to short limbs & normal height of the trunk. There are no signs or symptoms suggestive of mental retardation. Pattern of development of dwarfism in this child would have been: a. Autosomal dominant inheritance b. Autosomal recessive inheritance c. X linked dominant d. X linked recessive e. Not inherited Ans: A (Achondrioplasia) 95. Linear growth of the body will be affected by a fracture of the bone that passes through: a. Epiphyseal plate b. Metaphysis c. Diaphysis d. Epiphyseal line Ans: A 96. A diagnosis of malignant melanoma is made on a patient. The first line cells which fight this tumor are: a. Monocytes b. B cells c. Macrophages d. Cytotoxic Lymphocytes e. NK cells Ans: E 97. Athetosis is a. due to defect in chromosome 4 B. slow continuous writhing movements c. Jerky movements Ans: B

65 98. Thrombocytopenia develops in a child who has suffered an acute viral infection recently. The cause of low platelet count would be: a.Platelets were consumed during the infection b. antibodies are formed against platelets Ans: B 99. Which of the following amino acids is not formed in the body & has to be taken in diet? a. Phenylalanine b. alanine c. tyrosine d. glycine Ans: A 100. Shaft of hair is mainly composed of a. Stratum corneum b. Stratum basale c. Corneum &basale d. Cortex of hair Ans: D 101. The distinctive arrangement of collagen fibers in the skin is predominantly found in a. Reticular layer b. Dermis c. Hypodermis d. Papillary Layer e. Epidermis Ans: B 102. Which vaccine shall be given to the patient who has undergone spleenectomy? a. H. influenzae b. Hepatitis B c. Streptococcal d. Pneumococcal Ans: D 103. During quite breathing, the expenditure out of total body energy is: a. 1% b. 5% c. 10% d. 15% e. 20% Ans: B 104. Most appropriate statement regarding middle meningeal artery a. can be located at the pterion b. Is commonly involved in the extradural hematoma

c. Leaves impression on skull through its course d. Enters skull through foramen spinosum Ans: D 105. After suffering head trauma, patient is being examined. Pulsations appear in the orbit with heartbeat, this phenomenon is due to relation of: a. Cavernous sinus & ICA b. cavernous sinus & ECA c. ophthalmic nerve & ophthalmic artery d. cavernous sinus & optic artery e. ICA & ophthalmic nerve Ans: A ( Ref. Snell Neuroanatomy) 106. Old man has a history of stroke. Later he develops reduced motivation & depression. He has a lesion in a. Basal Ganglia b. temporal lobe c. parietal lobe d. occipital lobe e. frontal lobe Ans: E 107. Which of the following tracts is involved in carrying out finely controlled point to pint movements by a miniature artist? a. Cortico-spinal tract b. Cortico-rubro-spinal tract c. Vestibulo-spinal tract d. Cerebello-spinal tract e.olivo-pontine tract Ans: A 108. Left half of pons is related to a. left olivary nucleus & b. left cerebellar hemisphere & right basal ganglia c. right cerebellar hemisphere & left basal ganglia d. anterior vestibular nuclei e. right cortical tract & left olivary nucleus Ans: E 109. Anterior & posterior spinal arteries are the branches of: a. PICA & vertebral arteries respectively b. AICA & vertebral arteries respectively c. ICA & vertebral arteries respectively d. vertebral & posterior cerebral arteries respectively Ans: A ( Ref. RJ Last) 110. Maximum percentage of carbon dioxide in the blood is found as

66 a. Bicarbonate b. Carbaminohemoglobin c. Carboxyhemoglobin Ans: A 111. Inhibitors of carbonic anhydrase will cause: a. Reduced urine b. Increased formation & reabsorption of HCO3c. Acidosis d. Decreased Potassium e. Increase pH Ans: C 112. Surfactant deficiency causes respiratory distress. It was either “due to” or “because normal action of surfactant is” a. Decrease alveolar pressure b. Role in immunity c.. Reduce surface tension over alveoli Ans: C 113. One of the following is a premalignant condition: a. Solar keratosis of skin b. Nodular hyperplasia of liver c. Pneumonia d. Bronchial asthma Ans: A ( aka actinic keratosis) 114. Characteristic feature of malignancy? a. Invasion b. Vascularity c. Pleomorphism d. Increase nuclear-cytoplasmic ratio Ans: A 115. Diagnostic feature of a dead cell is: a. Pyknosis of nucleus b. Karyorrhexis c. Karyolysis d. Cell shrinkage e. Hydropic change Ans: A 116. GFR in a normal adult male is: a. 90 b. 125 c. 300 Ans: GFR over 90mls/min/1.73m2 is normal. 117. “foot drop” is due to injury of: a. common peroneal nerve b. deep peroneal nerve Ans: A

118. Right suprarenal vein drains into: a. IVC b. Right renal vein c. Right hepatic vein Ans: A 119. Which of the following is absorbed in ileum? (bile wasn’t in the options) a. fatty acids b. vitamin B12 c. amino acids d. water Ans: B 120. After successfully performing two adrenalectomies (removal of the adrenal gland), the surgical resident was disappointed to learn that he would be merely assisting at the next one. The chief of surgery told him: "I'm doing this one, since the one on the right side may be a little too difficult for you." The difficulty he envisioned stems from the fact that the right suprarenal gland is partly overlain anteriorly by the: A. aorta B. inferior vena cava C. left hepatic vein D. right crus of the diaphragm E. right renal artery Ans: B 121. True statement regarding kidneys is: a. is wrapped in a fascial sheath alongwith its adrenal gland b. Hilum is at the level of L4 c. pelvis of ureter lies between the renal artery & renal vein at the hilum. d. related posteriorly to ribs Ans: A (Ref RJ Last) 122. Diameter of bronchi & bronchioles is: a. mainly under the control of sympathetic nervous system b. under muscarinic control c. narrowed due to local stimuli Ans: A 123. Which of the following tumors of CNS is caused by radiation? a. Glioma b. Meningioma c. Lymphoma d. Ependymoma Ans: B

67 124. Weil felix test is done for the identification of? a. Klebsiella b. Rickettsia c. Pseudomonas ANs: B 125. A patient has irregularly irregular pulse on examination. ECG shows many abrupt P waves & the QRS complexes occur irregularly. What is the diagnosis? a. Ventricular fibrillation b. WPW syndrome c. Atrial fibrillation d. Atrial flutter e. Ventricular tachycardia Ans: C 126. Best index of preload is: a. Pulmonary artery wedge pressure b. Central venous (pressure/volume) c. EDV in left ventricle d. ESV in left ventricle Ans: C (LVEDV) 127. Lymphatic drainage of the lower lip is into a. Jugulo digastric nodes b. Submental& submandibular nodes c. Submandibular &Jugulo-digastric nodes d. Parotid nodes e.Submandibular nodes Ans: B ( Central part of the lower lip drains into submental while remaining into submandibular nodes) 128. True about the sensation of taste: a. salty taste is perceived at the lateral aspect of tongue b. adapts within seconds c. is a chemical sensation Ans: A a. person ascends > 1500 feet b. expiration to residual volume c. moderate exercise d. inspiration to total lung capacity Ans: C (Ref. Guyton) 130. True statement about the physiological dead space is: a. Is not diffusion dependent. b. is equal to anatomical dead space in a normal adult c. does not include anatomical dead space d. Decreases with exercise Ans: B

131. First part of duodenum is posteriorly related to: a. Gastroduodenal artery b. Hepatic artery c. Gastric artery Ans: A 132. During pregnancy, obliteration of uterine cavity occurs due to fusion of: A. Amnion & decidua capsularis b. Decidua Parietalis& decidua capsularis c. Chorion& amnion d. Decidua basale& Capsularis Ans: B 133. A drug is given by error that causes further increase in calcium level in blood. The drug is: a. Loop diuretic b. Thiazide diuretic c. Calcitonin d. Etodine Ans: B (Thiazide diuretics cause Hypercalcemia while Loop diuretics are used in Hyperclacemia) 134. Which of the following substance is more concentrated at the end of proximal convulated tubule as compared to beginning of proximal tubules? a. bicarbonate b. glucose c. calcium d. sodium e. creatinine Ans: E (because no reabsorption of creatinine occur in PCT but others occur along with water so creat conc increases) 135. Eosinophilia is likely to occur in: a. Leishmaniasis b. Atopy c. Hodgkin’s lymphoma Ans: C 136. True about laxatives a. Lactulose is digested by gut bacteria b. Ispaghol husk is bulk forming laxative c. Senna stimulates GI within 30 minutes d. Lactulose is a sugar e. Excessive use can lead to metabolic alkalosis & colonic atony Ans: B 137. A young boy has complaint of diarrhea & abdominal bloating. Stool examination shows rod shaped bacteria that are slightly curved. Organism

68 involved is: a. Salmonella b. Shigella c. E coli d. Campylobacter jejuni Ans: D 138. 50 years old diabetic woman complains of abdominal bloating after meals. A diagnosis of diabetic gastroparesis is made. What is the best treatment for her symptoms? a. Antacids b. Loperamide c. Omeprazole d. Metochlopramide Ans: D 139. Anterior tibial artery runs inferiorly down the leg along with a. Medial plantar nerve b. Lateral plantar nerve c. Sural nerve d. Deep peroneal nerve e. Tibial nerve Ans: D 140. A 50 years old man who has previous history of diabetic retinopathy & MI presented with chest pain… coronary occlusion occurs commonly due to: a. Vasospasm b. Embolism c. Stasis of blood d.Hypercoagulable state e. Thrombosis Ans: E 141. A 25 year old man presented with the complaint of post prandial bloating & abdominal cramping. He has history of passing foul smelling stool. Which investigation will give us multiple information regarding the problem? a. Stool C & S b. Stool DR c. Serum lipid profile d. Ultrasound abdomen Ans: B (Stool R/E) 142. A young girl (15-25 years) develops rash on cheeks & nose when she goes in sun. There is no other complaint & the rash subsides when she is indoors. Biopsy of the skin reveals IgG deposits in the rash but the surrounding skin is normal. Serum ANA is negative. Most probable

diagnosis is: a. Discoid lupus b. SLE c. Progressive systemic sclerosis d. Contact dermatitis Ans: A 143. 45 years old lady developed pericardial effusion & pleural effusion. Effusion fluid was examined & it turned out to be Transudative. (Normal baseline investigations)… Anti-DS titer was 1:560. Urine nitrogen was > 55. (Renal) biopsy will show: a. Amyloidosis b. anti-GBM antibodies c. Glomerular deposits d. Immune complex deposition e. Chronic Vasculitis Ans: D 144. When a person encounters an obnoxious stimulus, he reflexively withdraws his limbs & body away from the stimulus. Most appropriate statement regarding this reflex is: a. is monosynaptic b. is absent in lower animals c. Remains for many months after complete cord transection d. weak stimulus prolongs it e. is a superficial reflex Ans: C 145. 70 years old man presents with myalgia, rash & arthralgia. Joints examination reveal no abnormality. Motor power is 5/5 . ANA is positive with speckeled appearance. If antiribonucleoprotein RNP is also positive then the diagnosis is a. Mixed connective tissue disorder b. Dermatomyositis c. SLE d. Polymyositis Ans: A 146. During last seven months of pregnancy, estrogen and progesterone are secreted by 2.? ans. placenta 147. A patient with history of chronic liver disease is brought to the emergency with the complaint of upper GI bleeding. Drug of choice in the emergency is: a. Octreotide

69 b. Vitamin K infusion c. Tranexamic acid Ans: A 148. Hypoglossal i.e. CN XII injury on the right side will cause a. Left tongue paralysis with atrophy b. Left tongue paralysis without atrophy c. Right tongue paralysis with atrophy d. Right tongue paralysis without atrophy Ans: C 149. True regarding sperms of men above 70 years of age a. are asthenospermic b. many lack forward progressive motility c. count ">" (some figure) as they are oligospermic d. reduced quantity of semen because their seminal vesicles atrophy e. can fertilize ovum Ans: A 150. During which stage of sleep do theta waves appear on the EEG? a. Stage I b. Stage II c. REM d. Stage III e. Stage IV & V Ans: A 151. A middle aged man came back from Cairo & developed painless hematuria… Likely diagnosis is? a. Squamous cell carcinoma of the urinary bladder b. Transitional cell carcinoma of the urinary bladder c. Transitional cell carcinoma of the prostate Ans: A ( Cairo is the capital of Egypt and is endemic for schistosomiasis which causes SCC of the urinary bladder) 152. Difference between primary & secondary intention wound healing is? ans. Contraction of myoblast 153. Feature of primary shock is a.hypovolemia b.bradycardia c. tachycardia d. initial hypertension e. lower peripheral vascular resistance Ans: C 154. Total peripheral resistance is determined by? a. Venous compliance

b. Vasomotor tone Ans: B 155. Acute inflammation is a. Local response of the tissue b. Systemic response Ans: A 156. A young boy was playing in the ground. He fell down & got abrasions on the knee. Important local response will be: a. Chemotaxis b. Platelet plug formation c. Vasoconstriction Ans: C 157. Most immediate step that occurs in the scenario of bleeding is: a. vasoconstriction b. increase tissue permeability c. drop in blood pressure Ans: A 158. Triple assessment for congenital fetal anomalies include one of the following: a. Estradione E2 b. Estrione E1 c. Estriol E3 Ans: C 159. Important cause of epithelial metaplasia is: a. Ultraviolet rays b. Gamma radiation c. Chronic irritation Ans: C 160. One of the following drugs is found to play a beneficial role in subarachnoid hemorrhage a. Propranolol b. Nifedipine c. Verapamil d. Nimodipine e. Labetalol Ans: D 161. Carbon laden macrophages develop in a. Silicosis b. Pneumoconiosis c. Anthracosis d. Asbestosis Ans: C (Ref Goljan) 162. Characteristic of TB granuloma is: a. non- caseating b. caseation

70 c. cavitation Ans: B 163. Temperature center of the body is located in a. Hippocampus b. Hypothalamus Ans: B 164. Severe dehydration will bring about following change: a. Total body water will decrease b. Only ECF will decrease c. Only ICF will decrease d. Systolic BP will rise e. Heart rate will decrease Ans: A 165. Unilateral undescended testes should be placed in the scrotum surgically because if not corrected & left in the abdomen, it can lead to: a. Infertility b. Malignancy Ans: B 166. A young girl has history of SOB & wheezing. Investigations reveal C1 deficiency & she lacks an esterase required by the body. Most likely diagnosis is: a. Angioedema b. Myeloperoxidase deficiency c. SLE 167. Which of the following structures cannot regenerate? a. Lens b. Skin c. Intestinal mucosa Ans: A 168. Regarding Anti-tuberculosis therapy, correct statement is: a. . INH produces adverse effects that can be reversed with pyridoxine 169. Pulse pressure is determined by (confusing options) a. Mean arterial pressure – central venous pressure b. Stroke volume Ans: B 170. A man injured his wrist by falling with outstretched hand over pieces of broken glass. Which structures of the wrist will be damaged that run superficial to flexor retinaculum? a. Ulnar nerve & artery

b. Flexor digitorumprofundus c. Median nerve &? d. Ulnar & flexor digitorumsuperficialis Ans: A 171. true about conjoint tendon: a. formed by fused aponeuroses of internal oblique &transversus muscles b. Forms medial most fibers of inguinal ligament c. prevents indirect inguinal hernia Ans: A ( Ref RJ Last) 172. Which of these statements is correct about triglycerides? a. are carried by lipoproteins b. contain fatty acids attached to glucose Ans: A 173. In the chest wall, neurovascular bundle lies between: a. External & internal intercostal muscles b. Internal & innermost intercostal muscles Ans: B 174. Adult hemoglobin is composed of: a. 2 alpha & 2 gamma chains b. 2 beta & 2 gamma chains c. 2 alpha & 2 beta chains d. 2 alpha & 2 delta chains Ans: C 175. Which of the following structures prevents flow of unwanted substances between cells? a. Tight junctions b. Gap junctions c. Desmosomes Ans: A 176. (History of TB, ATT & some eye signs. Drug involved was asked) a. Streptomycin b. Ethambutol c. INH d. Pyrizinamide e. Rifampicin Ans: B 177. (Something about the most beneficial drug in the event of acute coronary syndrome) a. Heparin b. Warfarin c. Aspirin d. Streptokinase e. Atorvastatin Ans: C

71 178. A tumor that contains all germ layers is: a. Teratoma 179. Radial Nerve supplies which of the following muscles: a. Biceps brachii b. Triceps c. Flexor digitorumprofundus d. Palmaris longus Ans: B 180. Prolonged use of corticosteroids lead to: a. Neutrophilia b. Lymphopenia c. Eosinopenia d. Thrombocytopenia Ans: A 181. What happens in a patient suffering from HIV? a. Decrease in helper T lymphocytes 182. True about the features of JVP: a. ‘a’ wave occurs during ventricular systole b. ‘c’ wave occurs during atrial systole c. ‘a’ waves appear as giant Ans: C ( Read the full topic, Very Imp) 184. Increase in bleeding time. Probable diagnosis? a. von Willibrand disease b. Christmas disease c. Hemophilia A d. Vitamin K deficiency Ans: A 185. Pregnant lady gets pain, swelling & redness in the left leg. Which drug should be prescribed to prevent the adverse complications? a. Aspirin b. Clopidogrel c. Warfarin d. Heparin Ans: D (IV Heparin) 186. A young boy has history of painful burning micturition. Urine contains pus cells& few epithelial cells. What further investigation will you advise? a. Urine culture b. Urinalysis c. Serum unconjugated bilirubin Ans: B 187. During wound healing, collagen is laid down. The process that goes on within the cell includes:

a. Laminin b. Collagen c. Fibronectin d. Tyrosine kinase Ans>Controversial

PAPER - I MEDICINE & ALLIED 10 NOVEMBER 2016 EVENING 1. Potassium is mainly regulated by Aldosterone. 2. Tip of scapula at level – T7. 3. Oblique fissure of the lung at level - T3 to T6 Costochondral. 4. Decussating of medial lemniscuses Internal Arcuate Fibers. 5. Superficial temporal artery relation with which nerve - Auriculotemporal nerve. 6. Correct about Thyroid gland - lymph drainage to deep cervical lymph nodes. 7. Anemic hypoxia occur in – Methemoglobulenemia. 8. In cerebral circulation brain arteries do not anastomise - once entered in the brain subtance 9. Phase 1 of transformation of drug metabolism – Oxidation. 10. Standard deviation shows - Variability of individual observation. 11. Counseling in patients is - To help themselves. 12. A young girl who is going to die and asks you “Am I going to die?” Doctor response should be – “What your parents have told you?” 13. In whole wheat – Thiamine. 14. Cholesterol enriched diet – Egg 15. 1g protein gives energy – 4 kcalories 16. Organelle where protein combines with carbohydrates, packed and released - Golgi complex. 17. Correct about DNA – Euchromatin is transcriptionally active. 18. ADPKD associated with – ) Cerebral haemorrhage 19. Cause of delay in healing – Infections. 20. PaO2 decreased, PCO2 increased, hydrogen ion increased; manifestationHypoventilation. 21. PCO2 31, HCO3 19, pH increased (Metabolic alkalosis scenario) Hyperventilation

72 22. Person with tachycardia, and heat intolerance with low level of TSH, on giving TRH; level of TSH and thyroid hormones increases. Diagnosis – Hyperthyroidism with thyroid problem. 23. Origin of oxytocin and ADH – Hypothalamus. 24. Difference between systemic and pulmonary circulation - Low resistance in pulmonary circulation. 25. Mean systemic filling pressure is regulated by - Venous return. 26. Systolic pressure is directly related to which one of the following – Renin. 27. ADH responds to – Osmolarity. 28. Osmoreceptors – ADH. 29. Right border of heart on X-ray also visible a part of – SVC. 30. In MI sensitive cardio marker – Tropinin T. 31. Diabetic nephropathy investigation Urine albumin. 32. Young boy with generalized edema and proteinuria - Lesion of basement membrane. 33. Lesion of parasympathetic system affects mostly - GI muscles. 34. Stress hormone of our body - ACTH. 35. S2 sound heard on - Closure of aortic and pulmonary valve. 36. A patient with history hemorrhage (trauma) receives a bag stored for 2 weeks mainly contains – RBCs. 37. Due to inspiration – Decreased negative intrapleural pressure. 38. Important buffer of blood - HCO339. Max increase in ECF due to infusion of Hypertonic NaCl. 40. Auscultation of tricuspid valve best heard at – Right lower end of the body of sternum. 41. GVE vagus nerve for preganglionic fiber arises from – Dorsal nucleus. 42. Thorn prick in left lower limb caused abscess - Staph aureus. 43. Diabetic female after abdominal surgery; dyspnea and cough - Pulmonary embolism. 44. MCC of pulmonary embolism – DVT. 45. Typhoid fever 1st week test – Blood culture 46. Typhoid fever 2nd week test – Blood culture and Widal test.

47. Typical feature of falciparum - Black water fever. 48. Alcoholic patient with deranged LFTs; on biopsy – Mallory bodies. 49. Councilman bodies seen in – Apoptosis. 50. Natural self-defense against tumors – Apoptosis. 51. P53 gene absent results in - cell survival. 52. Pain mediator - Bradykinins. 53. Metaplasia – Functional change in cells. 54. Female with infection of HPV, comes after 2 years, Pap smear shows prominent nucleoli and increased nucleus size – Dysplasia. 55. Gas exchange occur - Simple squamous epithelial layer. 56. Patient with granulomatous disease, biopsy done. Microscopic finding that suggests TB – Epitheloid cells. 57. The spindle fibers will decrease in discharge of impulses when - Muscle contracts (vs) When efferent gamma discharge occur -muscle contracts 58. Diagnosis for leprosy, initial investigation - Nasal scrapping. 59. Benign neoplasm – Adenoma. 60. 3 germ layers tumor – Teratoma. 61. When adrenalin release from medulla, causes vasodilation by acting on - Beta 2 adrenergic receptors. 62. Increased GFR and increased plasma flow occur due to - Dilation of afferent arteriole. 63. Charateristic of cerebellar lesion Dymetria 64. Emax of a drug depends on – Efficacy 65. Study in which every person of a population has equal chances of being selected – Random sampling. 66. Amniocenthesis is done - After 14th weeks. 67. 1st response against acute inflammation in tissue – Macrophages. 68. In dark granules containing cells; IgE attaches to – Basophils. 69. Opsonization - C3b. 70. Exudate - more than 3g of proteins. 71. About active transport of drug all are true except - All drugs pass via active transport.

73 72. Pulmonary artery supply to – Alveoli. 73. Muscles of back innervated by - Dorsal rami. 74. In young boy dyspnea produced on lying - Retrosternal goiter. 75. Most important cause of bronchogenic cancer – Smoking. 76. Edema caused by - Increased hydrostatic pressure. 77. Edema caused by - Lymphatic blockage. 78. BP 210/180mmHg and creatinine 8% damaged part – Juxtaglomerular apparatus. 79. Female with blood group A, have 2 children one with O and other with AB, blood group of father is –B 80. Genetically true hermaphrodite – XX/XY. 81. DNA replication occur in – Interphase. 82. Glycogenolysis caused by deficiency of which hormone - Insulin 83. Investigation for liver amoebic abscess – Serology. 84. Adult female with breast atrophy due to dec. in - Estrogen and progesterone 85. Child with yellow sclera and coloured urine, best investigation - Bilirubin & ALT 86. Patient with all symptoms of cushing with increased ACTH cause- Cushing Disease 87.Typical symptom of food intoxication by Clostridium Botulinum - Flaccid paralysis 88. If left circumflex artery occluded Infarction of left atrium and left ventricle. 89. MCC of multiple fractures in adult – Osteoporosis. 90. Patient with fracture of many bones and low BP immediate treatment - Volume replacement. 91. Most common fracture of long bone Tibia. 92. Collagen fibers –lightly stained with Eosin 93. Gamma efferent supply to – Intrafusal muscle spindles.. 94. Isotonic and isometric contraction difference is that isotonic contraction – consumes more phosphate bond. 95. Autonomic nervous system – parasympathetic increase salivary secretion.

96. Protrusion of mandible – Lateral pterygoid 97. peroxisome made from ---------------> SER 98 clot retraction is mediated by ----------------> Thrombosthenin

PAPER II MEDICINE & ALLIED 10 NOVEMBER 2016 EVENING

1. Acute warfarin toxicity immediately treated with – a. FFP b.Vit k Ans A 2. Patient unable to supinate forearm, injury to: A.Radial nerve B. Median Nerve, C. Musculoskletal and Radial nerve D. Ulnar Nerve Ans c 3. Thumb plus Deltoid Plus Bicep tendon affected, Nerve root: a. C4 b. C5 c. C5,6 d. C6 ans c 4. Back Muscles Supplied By A Dorsal primary Rami B ventral primary Rami Ans a 5. During Inspiration, there is decerease in A arterial Plus volume , B. Heart rate , C- Negative Intrapleural pressure Ans c 6 Woman with respiratory symptoms that improve on her vacations, bronchoplumonary biopsy shows increase in lymphocytes, some esosinophils and nuetrophils, also allergic alveolitis and some hilar nodes, what will be the history findings a. she crack cocain b. She has pigeons/parrots in basement c. tobacco smoke d. house has lead composition ans b 7 Woman goes to Sunlight, Malar rash appears with proteinuria, hematuria (

74 SLE case), what other finding you will see in this patient: A. glomerulonephritis b. Pericarditis C. Pulmonary fibrosis Ans A 8. Creatinine Clearance when urinary []=196mg/ml , Urine flow rate = 1ml/min & Plasma [] = 1.4mg/ml A 140 B 1.96 C 125 Ans a 9 MAO inhobitors cause fatal excitation with which Opioid : A. morphine B. Pethidine C. fentanyl , D. sufentanyl E. Alfentanyl Ans b 10 About fetal Heart: A. Foramen ovale blood regurgitate from left atrium to right atrium B. Oxygenated blood comes from Arteries like that C. Majority blood through truncus arteriosus from pulmonary trunk to aorta D. large Muscular and small membranous interventricular septum Ans d 11 A young Boy with dyspnea on lying down A Retrosternal goiter B multinodular goiter C colloid goiter Ans a 12 How to differentiate between obstructive lung disease and restrictive lung disease A. Fev1/FVC B PFFR C FEv Ans a 13 Anterior Intercostal arteries are Branches of: A. Posterior thoracic arteries B. internal thoracic arteries Ans B 14 In anesthesized patient most common nerve damage: A. femoral Nerve

B. Sural Nerve C. Common peroneal Nerve D. Obturator Nerve Ans C 15. Thoracic duct passes through A Aortic Opening in diaphragm B inferior vena caval opening C esophageal opening Ans a 16. The slow growing Tumor: A. papillary ca B. Medullary ca C. Follicular ca Ans a 17. Pathogenesis of Diphtheria a Exotoxin b enterotoxin c endotoxemia ans a 18. Reinfection with what causes Fever and Hemorrhagic manifestations: A. CMV B. Meseals C. Dengue D. Mumps Ans c 19. Only helminth that spreads through Mosquito Bite a Filariasis b ascariasis c ochronosis ans a 20 .Ingestion Eggs of what Causes Cystecercosis A. Taenia Solium B. T. Saginata Ans a 21 IV Drug Abuser : A. Infective endocarditis B. Limb sacks Endocarditis Ans a 22. T4 in the blood Mostly binds with which plasma protien A TBG B thyroglobulin Ans a 23. Merkel Diverticulum Ans 2 feet from ileocecal Valve 24. ALT raised with fever and malaise A Viral Hepatitis B alcoholic hepatitis

75 Ans a 25. Bile salts Absorb in: A. Ileum B. Stomach C. Jejunum D. Ascending Colon Ans A 26. Drug , 1st order reaction like that A. Oral B. IV C. I.M Ans a 27. FRC : A. ERV + RV B. Cant be measured by spirometer Ans a 28. Not secreted from zona Reticularis A. cortisol B. Aldosterone C. testosterone D. Cortisone E. Androtendione Ans a 29. A diagnosis of Complete hyadatiform mole is done what further to know more: A. triploid like 2 ovum 1 sperm B moderate HCG C genotype. 46xx parental origin Ans b 30. Mid-leuteal Hormone increase A. LH B. FSH C. estrogen D. Progesterone E. Estrogen + progesterone Ans d 31. Hypotension due to Benzodiazepenes with given in A. Diabetic B. Hypovolemia C. Old age > 65 D. Pregnancy Ans b 32. Afferent of Carotid Sinus: A. Vagus Nerve B. Glosopharyngeal Nerve C. acessay Nerve Ans b 33. Hypersensitivity reaction in hydrops fetals a type 2 b type 3

c type 4 ans a 34. Acute treatment of acture severe Ulcerative colitis: A. Methotrexate B. Sulphasalazine C. c IV Hydrocortisone D. Azathioprine Ans c 35. Trochlear Nerve damage will cause: A. Loss of upward gaze and adduction B. Loss of downward gaze and abduction c. Loss of downward gaze and adduction d. loss of lateral gaze ans b 36. Most effective disease modifying agent in Rheumatic Arthritis A. Azathoprine B. Methotrexate C. Cyclophosphamide Ans B 37. A boy with respiratory and GIT infections with normal lymphocytes but decrease or absent plasma cells: A. Isolated IgA Def B. Common variable immunodeficiency c. DeGorge ans b 38. Patient works in a mine with all symptoms of TB : A Asbestosis B. Silicosis Ans c 39. Brunner Glands present in A submucosa of Duedenum B ileum Ans a 40. H2 Blocker is A Cimetidine B metronidazole C carbimazole Ans a 41. Rx of Nausea and Vomiting: A. H1 Blockers Only B. Steroides C. Metochlorpomide is less effective in cytotoxic induced emesis as compared to Ondasterone Ans c 44. Chenoxycholic acid decrease turnover in:

76 A. Cholesteryamine B. ileal resection Ans b 45. Blood Transfusion : A. Hep C B. HIV c Hep B ans b 46. Abscess of tracheal area and deep investing layer drains into: A. Retropharyngeal space B. In thorax cavity anterior to precordium C. at mid of sternal manubrium Ans a 47. Female 16 week pregnant with Hb 8, MCH & MCV Low, HbA=96% HbA2=4%: A. Iron deficiency anemia B. Sideroblastic Anemia C. Sicklemia D. Thalasemic Trait Ana d 48 QRS complex A Ventricular Depolarization B ventricular repplarization Ans a 49. Feeding Centre Located In : A. Temporal B Lateral hypothalamus Ans b 50. A patient with Pleural Effusion on standing u will see effusion in: A costodiaphragmatic recess B. Cardiac Notch C. Oblique fissure D. Horizental Fissure Ans a 51 Pernicious Anemia dx : A antismith antibodies B. Antinuclear antibodies C. Anti intrinsic factor antibodies Ans c 52. Palmar Arch Innervation : A. Median Nerve B. Deep branch radial C. Deep branch ulnar . D Superficial Ulnar Ans c 53. Surfacant : A. Secreted after 36th week B. Colesterol C Dec. surface tension of water in alveoli

ans c 54. Zollinger Ellison Syndrome : A. Dec. gastrin B. increase Insulin C. Peptic ulcer disease Ans C 55. Truncal vagotomy will cause: A. Gastrin secretion augmented B. HCl Secretion augmented C. Delay gastric emptying Ans C 56. Parasympathetic Action of vagus on SA node due to : A. Potassium B. Calcium C. Sodium D. Potassium and Sodium Ans a 57. “ A” Wave A Atrial Depolarization Ans A 56. Postural Tone of antigravity Muscles is maintained by constant stimulus by: A. Cerebellum Purkinji fibers B. Cortocospinal fibers C. Lateral vestibular fibers D. Fibers of Red Nucleaus Ans d 57. Oral propanolol contraindicated In : A. Asthma B. HTN Ans a 56. 2 year Boy from AZAD kashmir with failure to thrive for 6 month comes with Hepatosplenomegaly and pallor, plus pancytopenia and ESR 80, dx A. Visceral Lesihmaniasis B. Gaucher disease C. A.L.L Ans a 57. Structure that connects Hipocampus to hypothalamus : A. Fornix B. Amygdala c. Sria Terminalis D. Superachiasmatic Ans a 58 Patient with HTN preoperative drug decrease BP rapidly A. Hyralazine B. Metoprolol C. Glyverl Trinitrate

77 D. Nicardipin Ans a 59. Digoxin effect increased by: A. K _ Loosing diuretics B. Hypocalcemia Ans a 60. V/ Q Increase : A. inadequate Ventilation B. Increase dead space C. Change in composition of alveolar gas Ans c 61. A obstructed labour will damage in Pelvic Diaphragm structure: A. Lavator ani B Sphincter urethae C anal sphincter Ans a 62. Hb synthesis is active in A. prerthroblast B. early Normoblast C. Intermediate Normoblast D. late Normoblast E. reticylocyte Ans d 63. Hormone have short half life in blood A. Aldosterone B. LH C. Fsh D. Testosterone E. GnRH Ans e 64. Regarding testosterone : A. regulated by FSH B. has receptors on membrane C. Decend testis inlate fetal life D abundentaly secreted by adrenal cortex Ans c 65 cortical oocyte release : A. Lh B. FSH C. Progesterone D. Estrogen E. Act as lysosomal vesicle Ans e 66. Cornea Innervation : A. Optic Nerve B. Abducen Nerve . C. Occulmotor Nerve D. Tochlear Nerve E. Trigeminal Nerve Ans e

67 Patient with problem in posterior tongue taste and in elevation of shoulders, these affected nerves pass through: A. foramen rotundum B. formen ovale C. foramen spinosum D. Jugular Foramen Ans D (hint acessary nerve is damaged) 68. A 32 year old patient with fever 100 fh, dry cough, joint pain WBCs 7.5 with 70% neutrophils, cause : A. Strep. Pneumoniae B. Stap. Aureus C. Mycoplasma Pneumoniae Ans c 69. small saphenous vein runs along with : A. Saphenous Nerve B. Radial Nerve C. Sural nerve D. Common peroneal Nerve Ans c 70. Regarding Ear and Hearing: A. High frequency waves/ sounds in apical portion of choclea like that B. Stapedial foot at oval membrane C. Pressure in ear affects sound Ans b 71 Bladder carcinoma in workers exposed to: A. zinc B. Niclke C. Arylamine D silicon Ans c 72. Initial marker of MI : A. CKMb B. Myoglobin C LDH Ans b 73. S2 sound heard a Closure of aortic and pulmonary valve b closure of mitral and tricuspid valve ans a 74. GVE carried in vagus nerve for preganglionic fibers arise from A dorsal nucleus B solitary nucleus Ans A 75. Old RBCs are removed in: A. Capillaries and red pupl B. capillaries and splenic cord

78 c. Capillaries and sinusides d. Cord and sinusides ans a 76 Release of Neurotransmittor due to A Influx of calcium in presynaptic fibers Ans a 77. Half Life of Kanamycin in perilymph A 2 hours B 4 hours C 30mint, D 12hours, E 20hours Ans b 78. Digoxin act by A block Na/Ca exchanger B increase intracellular ca+ Ans a 79. A patient with low hb with pancytopenia investigation A Bone marrow study B cbc Ans a 80. Absent P wave, pacemaker is in A AV node B purkinje fibers Ans a 81. A patient with Pneumonia cell present A Neutrophils Ans A 82. G6Pd A Self Limiting B autosomal dominant Ans a 83. Systemic Fungi treatment A Amphotericin B B ketoconazole Ans a 84. A patient is on radiotherapy after that DNA damage A gene which arrest in G1 phase progression defective unregulated growth occurs due to which A. K-ras B. n-myc c. P53 D. Bcl-2 Ans d 85. In Nucelus A Bar-body is inactive x chromosome in male B . Hematoxylin stains the nucleus pruple or blue Ans b

86. Dosral Coulmn fibers of lower body related to upper body at cervical level A. Medial to lateral B. Lateral to media C. Dosral to ventral D. Rsotral to caudal Ans a 87. About Climbing Fiber A Inferior olivary Nucelus ANS A 88 Rheumatoid arthritis in pregnancy A MR B MS C pulmonary regurgitation Ans B 90 Bile salts absorbe from A terminal iieum B colon C duodenum Ans (a) 91; Concentrated solution of quinidine applied on tongue as compared to distild water sensation via A trigeminal nucleus B VPM of thalamus Ans B (controversia Bcql)

79

SURGERY PAPERS 2016 SEPTEMBER, OCTOBER, NOVEMBER

80

5TH SEPTEMBER SURGERY MORNING 1. 1/3 of total body water= extracellular fluid. 2. Which hormone will be less after drinking water =antidiuretic hormone. 3. Phospholipase c is activated by which hormone = antidiuretic hormone (action on blood vessels by Gq proteins and causes vasoconstriction. 4. Maximum blood pressure in =renal arteries. 5. Basic drug binds with alphaglycoprotein. 6. Left shift of oxygen saturation curve = HbF 7. Maximum potassium shift from intracellular to extra cellular compartment = strenuous exercise. 8. C nerve fibers = carry temperature. 9. All preganglionic fibers = carry b fibers. 10. Esophagus starts at level of = cricoid cartilage. 11. Nerve supply of posterior 1/3 of tongue = 9th cranial nerve. 12. Thyroid lymph drainage into = deep cervical lymph nodes. 13. Dislocation of temporo mandibular joint = anteriorly. 14. Glucose is main and only source of energy for = RBC if asked organ than brain if asked cell or as a whole glucose is main and only source of energy so its rbc and if its not in option click neuron. 15. Extra embryonic coelom forms from =hypoblast. 16. Main parasympathetic supply to =parotid gland. 17. Right sided tongue atrophy and wrinkling =12th cranial nerve. 18. Main myelinating cells in CNS are =oligodendrocytes. 19. Oxytocin and antidiuretic hormone originate in =hypothalamus.

20. Conduction aphasia = arcuate fasciculus. 21. Delayed language issue, can speak but not propulsive speech= lesion of angular gyrus. 22. Apoptosis show = councilman bodies. 23. Irreversible injury sign-= contraction band necrosis. 24. Brain perfusion depends on =Pco2. 25. Middle meningeal artery = foramen spinosum. 26. Labyrinthine artery is branch of =anterior inferior cerebellar artery. 27. Association of adult polycystic kidney disease is with = cerebral hemorrhage. 28. A patient came in night 2o’clock with chest pain from last 4 hours all baseline was normal what to do next= ckmb. 29. Middle wall , roof formed by =tegmen tympani. 30. Position of vocal cords after bilateral recurrent laryngeal nerve damage= Paramedian 31. Muscles not supplied by bilateral recurrent laryngeal nerve = cricothyroid muscle. 32. Contents of posterior triangle =subclavian artery and 3trunks of brachial plexus. 33. Muscles of anxiety and sorrow that pulls the angle of mouth downwards. = platysma muscle. 34. Contents of internal auditory canal = facial nerve and vestibulo cochlear nerve. 35. Atropine cause =dry mouth. 36. Subarachnoid space ends at =S2. 37. Upper motor neuron type of lesion Babinski positive = damage to medullar pyramid. 38. Clostridium botulism= Respiratory muscle failure. 39. At c7 level = Cervico thoracic ganglia located. 40. Anemia causes= Atrophic glossitis. 41. Hyper magnesemia causes = Neuromuscular junction problem.

81 42. Patient with bleeding time increase and anemia= Platelet function defect. 43. Case of achondroplasia = Autosomal dominant. 44. Most common type of genetic disease = Autosomal dominant. 45. Long term steroid use = Osteoporosis and fractures. 46. GMP or GTP related hormone = Atrial natriuretic peptide. 47. Metastatic calcification of kidney = Hyperparathyroidism. 48. Nerve supply to maxillary sinus = Superior alveolar branch of maxillary division of trigeminal. 49. Nerve supply to pinna= Greater auricular nerve. 50. Which nerve accompany superficial temporal artery = Auriculotemporal nerve. 51. Patient with acute asthma 1st treatment = Inhalational salbutamol. 52. Case of pneumonia after bone marrow trAns:plant inclusion bodies in cell = Cytomegalovirus. 53. Tumor that stains with vementin = Sarcoma. 54. Septic meningitis caused by = Streptococcus. 55. 4ml of 2% lidocaine = 80 mg of lidocaine. 56. Patient on table without clothes loss of heat by = Radiation & Conduction. 57. Mother gave birth to a child having congenital cataract = Rubella infection during pregnancy. 58. Grave disease scenario = Anti thyroid stimulating antibodies. 59. Animal as reservoir = Influenza. 60. Action of propylthiouracil stop oxidation of= Iodide. 61. Kid with edema problem in kidney = Basement membrane damage. 62. H+ ions more in urine = Secretes plasma secretion. 63. Carbon laden macrophages = Anthracosis.

64. Bacteriostatic Tb drug = Ethambutol 65. Prostaglandins inhibit by = Corticosteroids. 66. Left adrenal vein drain into= Left renal vein. 67. Case of renal failure after postpartum hemorrhage damage in kidney= Proximal convoluted tubules. 68. More in endolymph in ear = Potassium. 69. Nerve supply to infra hyoid muscles = Ans:a cervicalis. 70. Inferior thyroid artery = Related to recurrent laryngeal nerve. 71. 2nd pharyngeal arch derivative = Stlyohyoid ligament. 72. Nerve supply to fibrous pericardium = Phrenic nerve. 73. Commonest presentation of vitamin a deficiency = Night blindness. 74. Acrosome of sperm formed by = Golgi complex. 75. Simian crease= Trisomy 21. 76. Goblet cells replaced by clara cells in = Terminal bronchioles. 77. Case of aspergillous eosinophilia in blood and bronchial secretion = Noninvasive aspergillious. 78. Glycocalyx = Carbohydrate moiety. 79. Resistance in inspiration = 2/3 by small bronchioles and bronchi ( intermediate size bronchioles was not in option) 80. Supply of esophagus = Inferior thyroid artery. 81. Valve less structure = Superior vena cava. 82. Trachea = little shift to right side is normal. 83. Root of lung relation = Descending aorta. 84. Pancreatic secretion increase and increase calcium influx = Hormone. 85. Enzymes needed to activate trypsinogen = Enterokinase. 86. Maxillary sinus opens in = Middle meatus. 87. Case of COPD but no history of smoking or any risk factor fev1

82 decrease neutrophils will clear which particles = Carbon. 88. Heme binds in blood with = Hepatoglobin. 89. Cardiac reserve decreases in =Athletes. 90. Systemic sclerosis = Excessive fibrosis. 91. Pleural effusion case from where to take sample of fluid = Upper border of the rib at area of highest dullness. 92. H2o2 is present in = Peroxisomes. 93. Wound healing strength given by = Collagen type 1 . 94. Commonest etiology of genetic disease = Multifactorial. 95. Difference between malignant and benign tumor = Metastasis. 96. On histology of malignant tumor = Pleomorphisum. 97. Cause of pyogenic peritonitis = Bacteriods. 98. Mycobacterium is = Acid fast bacilli. 99. LMN type of lesion = Flaccid paralysis. 100. Caseof Tb granuloma was given. Best way to confirm the diagnosis = Acid fast bacilli. 101. Case of two years child with fever and anemia Hb 5 mcv 107. How to confirm diagnosis = vit B12 102. Angular cheliosis in patient with renal trAns:plant cause in = candida albicAns: 103. A 10 yr. old kid with neck swelling solid mass present = lymphoma 104. Beetle nut history girl 20 years trismus = sub mucosal fibrosis 105. site of antigen binding in antibody = hyper variable regions on l and h chains 106. in pregnant female which will be increased =TIBC 107. immunity effected in = HIV infected helper t cells 108. posterior triangle is supplied by = c2 c3.

109. type of necrosis in brain = liquefactive necrosis 110. function of ICAM and VCAM = neutrophils adhesions 111. viruses effect cells by = altering protein synthesis 112. metaplasia = replacement of one type of epithelium with another type of normal epithelium. 113. papillary carcinoma of thyroid spread by = lymphatic invasion 114. atrial fibrillation = many p waves followed by QRS 115. GFR will be more when protein in blood is= less 116. which tissue does not regenerate = lens 117. labor working in heat sodium is 125 lost his consciousness cause = excessive sweating 118. ionizes calcium = parathormone secretion regulated 119. somatostatin inhibit insulin secretion = paracrine way 120. pregnant lady with jaundice = ggt 121. corrosive intake = squamous cell carcinoma of esophagus 122. iron stored in tissue in the parenchyma as = Ferritin. 123. red blood cell a and b antigens = glycoproteins 124. hydrocarbons cause = bronchial carcinoma 125. hyperplastic bone marrow = leukemia 126. polycythemia vera = neoplastic proliferation in bone marrow 127. trapezoid body = role in hearing

Surgery paper 2016 – 5th Sep 2016 EVENING 1. Lymphatic from the lateral quadrant of the breast mainly drain into the A. Inferior deep cervical lymph node B. Inter pectoral lymph node C. Pectoral (anterior) lymph node D. Retrosternal lymph node E. Supra clavicular lymph node

83 Ans:: cc 2. A student wanted to study relationship between increase in temperature and duration of surgery, to evaluate results which statistical test he should apply? A. Student t test B. Anova test C. Mc witneys D. Chi squared E. Regression Analysis Ans:: ee 3. Action of Gluteus Medius & Minimus A. Abduct the hip joint & Rotate the thigh medially B. Adduct the hip joint and rotate the thigh laterally C. extend the hip joint D. flex the hip joint E. helps in sitting in squatting position Ans:: aa

84 4. Heavy smoker complains of dyspnea chest pain cough hemoptysis since 3 months. Bronchoscopy reveals fungating mass obstructing medium sized bronchus. Which type of tumor? A. squamous cell carcinoma B. small cell carcinoma C. large cell carcinoma D. Bronchiolar carcinoma Ans:: aaa 5. Skeletal Muscles are attached to Bones by: a. Anchoring filaments b. Fascia c. Ligaments d. Tendons Ans:: dd 6. Sugar in Urine will be detected if its concentration in blood becomes a. 250mg/dl b. 200mg/dl c.180 mg/dl d. 150 mg/dl e. 375mg/dl Ans:: bb (refrence ganong) 7. Dorsal Nucleus of Vagus Nerve lies at a. Pons Lower Lower Part b. Medulla c. Pons Upper part d. Midbrain Ans:: bb 8. 2% solution of 4ml lidocaine will contain How much Lidocaine a. 80 mg b. 160 mg C. 8 gm. D. 40 mg Ans:: aa 9. 0.85% Nacl Solution contains how much NACL a. 85mg/100ml b. 85 mg / 1000 ml c. 850 mg/100ml d. 8.5 mg/100 ml Ans:: cc 10. A person with Chest pain having ST Elevation in Lead V4 only. which artery involved a. LAD b. RCA

c. Anterior Interventricular d. LCX e. Diagonal Ans:: aa 11. Which of the following is Ectoparasite a. Lice b. Fleas c. Ticks d. Bed Bugs Ans:: aa 12. Before final composition of Concentrated Urine, which part of Tubular fluid contains least osmolar solution? a. PCT b. Collecting Duct c. Ascending loop of Henle d. Descending loop of henle. e. Distal Convoluted Tubule. Ans:: eee 13. Best Term to describe to describe a condition when a cell gets extra number of chromosome other than haploid a. Non disjunction b. Trisomy c. Euploid d. Aneuploid e.Polyploidy Ans:: bb 14. Alveolar Surface is kept dry because of a. Tight Junctions b. Surfactant c. Negative Interstitial Pressure d. Macrophages Ans:: cc 15. Histoplasmosis affects which system mainly a. Respiratory b. Reticuloendothelial c. Cardiac System d. Nervous System Ans:: bb 16. A female delivered a baby and she was started on warfarin as she developed dvt. The next day she has a purplish patch on her right thigh what can be the cause A. lupus anticoagulant b. warfarin overdose c.protein c deficiency d.proteins deficiency

85 Ans:: cc 17. Grade 3 Tumor, % of undifferentiated cells is: a. 25 % b. 50-75% c. >75% Ans:: bb 18. In a Pre-OP patient after inducing General Anesthesia, heat will not be produced due to: a. Muscle tone is lost b. Na/K Pump Blocked Ans:: aa 19. Initial symptoms of Bupivacaine Overdose a. Arrhythmia b. Unconsciousness c. ringing of ears Ans:: cc 20. Cytoskeleton communicates with Extracellular environment through a. Intermediate Filament b. Integrins c. Cadherins Ans:: bb 21. True about Glycocalyx: a. Carrier Protien b. Carbohydrate Moiety c. Lipid Membrane Ans:: bb 22. Acoustic Neuroma is a tumor of a. Cochlear part of CN 8 b. Vestibular part of CN 8 c. Facial Nerve d. Trigeminal nerve Ans:: bb 23. In young age, Limb growth will be affected if fracture passes through a. Epiphysial Plate b. Epiphysial Line d. Diaphysis d. Metaphysis Ans:; aa 24. Cause of Death after Liver trAns:plantation due to which pathogen a. CMV b. HBV c. HDV d. HCV e. HEV Ans::aa

25. In VIT K deficiency, which factor will be depleted first and foremost a. Factor 7 b. Factor IX c. Factor 10 d. Protein C e. Protein S Ans:: aa if asked factor then its factor 8 and if asked protein then its protein c) 26. SKIN GRAFT will be most successful in which of the following conditions a. Excessive Burns b. Agammaglobenemia c. Terminal cancer patient d. Liver failure Ans:: bb 27. Which cancer invades large blood vessels? a. Clear cell ca of Lungs b. Clear cell ca of Kidney c. Bronchogenic Ca d. TrAns:itional cell Ca Ans:: bb 28. In Old what happens A. Decrease Vital Capacity B. Increase Cardiac Output Ans:: aa 29. Neutral absorption of sodium occurs at which part a. Proximal convuated tubule b. Thick ascending limb of loop of hence c. Thick descending limb d. Cortical collecting duct e. Distal convulated tubule Ans:: aa 30. Breast Atrophy in elderly females, Major cause a. Estrogen & Progesterone deficiency B. estrogen deficiency Ans:: bb 31. Major cause of fatty liver in our society a. Hep B & Hep C b. alcohol c. increase fat intake Ans:: aa 32. Unable to evert and dorsiflex, which structure injured a. Common peroneal nerve

86 b. deep peroneal nerve Ans:: aa 33. Scrotal Swelling after injury, contents of swelling extends up to abdomen but not thigh, most common cause of this swelling in scrotum a. Injury to Bulbar Urethra b. injury to penile urethra c. membranous urethra d. neck of urethra Ans:: aaa ( urine extravasation rembered by NMEMONIC BUS : Bulbar urethra rupture  Scrotum MUD: Memranous urethra rupture DEEp pereaneal pouch PUS : Penile urethra  superficial pereneal pouch ) 34. Mediolateral Episiotomy, which part at risk of being getting injured a. External Anal sphincter b. Bulbospongiosis c. levator ani muscle d. perineal body Ans:: bb 35. Axillary Artery A it lies post to pectoralis minor muscle B lateral chord lies posterior to it C starts at the lateral border of pectoralis minor D all the chords are lateral to it Ans:: aa 36. Ecchymosis and bleeding history in fisherman a. Vit C Deficiency b.Vit b12 def c. Folic acid deficiency d. Iron deficiency Ans:: aa 37. Chromosome 9:22 abnormality a. CML b. CLL c. AML d. ALL Ans:: aaa 38. Function of Vitamin A a. Forms Rhodopsin b. Not required for night vision . Ans:: aa 39. Lymph drainage in Cervical CA a. External & Internal Illiac lymph nodes b. Internal ilaic

c. External Iliac Ans:: aa if the option doesnot contains both then select Internal Iliac nodes. 40. Dec in ESR due to a. Increase Fibrinogen b. Increase Albumin Ans:: bb 41. Recurrent Laryngeal nerve injured on both sides, which muscle still intact a.thyrohyoid b.postcricio thyroid c.cricothyroid d.cricoarytenoid Ans:; cc 42. Gastric Lymphoma due to a. H Pylori b. CMV Ans::aa 43. The features of BCC a. metastasis b. benign tumor C. borders are undermined d. Locally invasive. Ans:: dd 44. Posterior wall of Rectus sheath b/w Symphisis pubis & Abdomen a. Arcuate line b. Linea semilunaris c. Linea semicircularis Ans:: aa 45. Bundle of His Blood supply a. RCA b. LCA c. LAD d. Circumflex Ans:: cc Reference : snell 46. Head of Femur Blood supply a. Medial & Lateral Circumflex Arteries . B . Retinacular. Ans:: B> A 47. Major Cause of Epiphyseal Tumor a. Osteoblastoma b. Osteosarcoma c. Osteoid Osteoma d. Ewing Sarcoma Ans:: aa 48. DIC is initiated by a. Tissue Thromboplastin b. fibrinogen Ans:: aa

87 49. Seventy years old male develops sudden loss of power in both right upper n lower limb with angle of mouth deviated to the left during talking. The most likely site of lesion is in a.Internal capsule b.pons c.medullaoblongata d.midbrain Ans:: aa 50. Sciatic nerve injured which structure will still flex the knee a. Short head of biceps femoris b. long head of bicep femoris c. popliteus Ans:: aa 51. Patient died of Stroke, which type of necrosis a. Liquificative necrosis b. Caseous necrosis c. coagulative necrosis Ans:: aa 52. Median Umbilical Ligament is remnant of a. Urachus b. umbilical artery c. umbilical vein d. allantois Ans:: aa 53. Which structure will be damaged in Lumbar Puncture a. Ligamentum Flavum b. Posterior longitudinal ligament c. Anterior longitudinal ligament d. Pia matter Ans:: aa 54. Scenario with Vitamen K deficiency, Vitamin K not absorbed, which factor decreased a. Prothrombin b. Protein C c. Protein S d. Factor VIII Ans:; aa 55. Low estrogen pills will cause a. Breast Ca b. Lungs Ca c. Endometrial Ca d. Deep venous thrombosis e. Hepatic Adenoma Ans:: ee

56. Pregnant lady with raised SGPT, history of visit to Remote village, which hepatitis a. Hep A b. Hep E c. Hep B d .Hep C Ans:: bb 57. Which Ca will not metastasize a. Basal Cell Ca b. Squamous cell ca Ans:: aa 58. Isthmus of trachea lies in front of A. 2nd 3rd and 4th Tracheal Ring B. Rings 3, 4 C. Rings 2, 3 Ans:: aa 59. 50 years old hypertensive patient presented with severe chest pain radiating to back suddenly collapsed On autopsy what can be seen a. Medial Necrosis of Aorta b.ischemic necrosis c..atherosclerosis Ans:: aa 60. Abdominal Aorta a. Gives Renal Arteries at L2 b. bifurcates at L5 c. bifurcates at T5 d. lies at the right side of IVC Ans:: aa 61. Loss of Supination & Flexion after Stab Injury to Inner Arm a. Musculocutaneous b. ulnar c. radial artery d. axillary artery Ans:: aa 62. Regarding Ciliary body A. Pigment epithelium produces aqueous humor. b. Pigment epithelium of retina is continuous With pigment epithelium of ciliary body C. Non pigment epithelium produces aq. Humor D. Supplied by short ciliary arteries Ans:: cc 63. Minerals are least found in a. Pulses

88 b. Cereals c. Tubers d. Green Vegetables. Ans:: cc 64. Patient had previously Episodes of Dizziness and Diplopia, patient presented now in Acute Comatosed state, Cause : a. Carotid Artery Thrombosis b. Basilar Artery Thrombosis c. Arachnoid Hemorrhage Ans:: bb 65. Which of the following lymphoid organ receives lymph via SUBCAPSULAR SINUS a. Thymus b. spleen c. Lymph node Ans:: cc 66. True regarding rectum is : a. lies in front of S1. b. is 9 inches long c. is biconvex wall in front of s2,3,4 d. also supplied by median sacral Artery Ans:: dd 67. Infection anterior to pretracheal fascia will spread to: a. Anterior Mediastinum b. Superior mediAns:tinum c. Middle Midiastinum Ans:: aa 68. Regarding abdominal angina which artery is involved a..inferior mesenteric artery b..aorta c..superior mesenteric artery Ans:: cc 69. premature baby born with a..PDA b.patent foramen ovale c.right atrial pressure increases Ans:: aa 70. Which hormone causes fatty acid synthesis and protein synthesis a.cortisol b.thyroid c.glucagon d..insulin Ans:: dd 71.Most common salivary gland tumor a.salivary b.submandibular

c.parotid Ans:: cc 72. Regarding anal canal a.supplied by both superior n inferior rectal arteries b.inferior rectal artery part of portal system c.external anal sphincter is involuntary d.has two lateral curves e.sup inguinal lymph nodes drain the upper half of anal canal Ans:: aa 73. Patient started on anti coagulant how to monitor further A. Aptt B. Clotting time C. Pt Ans:: cc 74. Trachea commences at A. C4 B.C5 C. C6 D. C1 Ans::cc ( lower border of C6) 75. A female of reproductive age presented with DVT. She gave history Of taking some kind of pill what is the cause of her condition a. Thrombo embolism b. Analgesics c..OCPs Ans:: cc 76. 45 years old man presented with chest pain not related to respiration a. Myocardium b. Pericarditis c. chostochondritis Ans:: aaa,,, if th pain aggravates eith changing posture its pericarditis,, if it is related to respiration that feels pain with breathing then its costochondritis 77. Tongue is deviated towards right side , lesion is a. Right hypoglossal nerve b. Left hypoglossal nerve c. Rt. Vagus d. Lt. vagus Ans:: aa 78. Deep part of the parotid fascia forms a. bucco pharyngeal membrane b.stylomandibular ligament c.sheno mandibular ligament

89 d.stylo hyoid ligament Ans:: bb 79. first Lumbrical a.bipennate b.attached to proximal phalynx c.origin near the radial side of first meta carpal bone Ans:: cc 80. In a terminal resection of ileum 100 cms of ileum was resected. What will be the out come? a.less absorption of bile salts b.less absorption of bile acids c.more absorption of bile salts d.increase sodium loss Ans:: aa 81. Regarding inguinal canal A. Absent in infants. B. Extends from anterior superior iliac spine to pubic tubercle. C. Roof formed by conjoined tendon Ans:: cc 82. Greatest clearance is of a. PAH b. insulin c. Glucose d. Creatinine Ans:: aa 83. Regarding cardiac plexus a.formed by both symathetic n parasympathetic nerves b.thoracic splanchnic nerves are the main nerves c.starts at trachea bifurcation d.present at the base of heart e.anterior cardiac nerves are the main nerves Ans:: aa 84. Baroreceptors a.hypotention b.respond to rapidly increasing arteriolar pressure c.respond to rapidly decreasing arteriolar pressure Ans:: cc refrence BRS phsiology 85. Insulin is inhibited by a. secretin b. glycogen c. beta agonists d. beta blockers Ans::dd

86. Bioavailibity of minerals is lowest in a.roots b.green leaves c cereals d pulses e.Tubers Ans:: ee 87. Drug drug interaction by a. Both pharmacokinetics and pharmacodynamics b. only pharmackokinetics c. only pharmacodynamics Ans: : aa,, as they havnt mentioned whether the interaction is by the drug on the body or by the body on the drug so we select both. 88. Regarding sweat glands a. Cuboidal epithelium b. Columnar epithelium Ans:: aa 89. Bioavailibility is not affected by a.age b.liverdisease c sex/gender Ans:: cc 90. Dimercaprol a.increases clotting b.decreases clotting c.decreases bleeding time D. Increase bleeding time Ans:: dd 91. For sodium neutralization a. k b. organic ions c. Cl d. mg e. hco3 Ans::cc 92. Amino acid containing sulphur a. Cysteine b. Phenylalanine c. Leucine. Ans:: aa 93. Sodium regulation is through A osmoreceptors b.sodium absorption c.Adh d.Reninangiotensin e.Aldosteron Ans:: aa 94. Circumflex artery supplies a.anterior surface of heart

90 b.left atrium n ventricle c.right atrium d.left atrium e.left ventricle Ans:: bb 95. 41%haematocrit contains a.rbcs b.rbcs,wbcs,platelets c.platelets Ans:: bb 96. Venous blood as compare to arterial blood has a.low ph b.more packed cell volume c.increasepo2 d.increase ph. Ans:: bb 97. During stab injury a. Ipsilateral lung collapse n ipsilateral chest wall spring out. b. ipsilateral lung collapse n contralateral chest wall spring out c. contralateral chest collapse and contralateral chest wall spring out Ans:: aa 98. A person got lacerated injury on his elbow what will be the initial affect of blood a. haemostasis b. vasoconstriction Ans:: bb 99. during a one km running the venous return to the heart is by a.valves in the leg veins valves b.pumps in the legs c.skeletal muscles contraction Ans:: cc 100. right atrial pressure is increased this will lead to a.inc cardiac output b.heart rate inc . c.inc contraction of right ventricle Ans:: aa 101. Regarding occupational diseases increase risk of problems with exposure to a.coalmines b.asbestos c.silicons d.textile industry Ans:: bb

102. Eversion of foot leads to ligamentum fractured a.lateral ligament b.planataris ruptured c.Deltoid ligament Ans:: cc 103. A farmer while spraying suddenly got collapsed with salivation from his mouth and broncho spasm. What is the specific anti dote a.atropine b.neostigmine cphysostigmine d.Pralidoxime Ans:: dd 104. A person was walking bare feet suddenly stepped upon some sharp object. He quickly removed his leg due to which reflex a.flexor response b.mono synaptic reflex c.multi synaptic reflex Ans:: cc 105. Opsonization due to interaction of a. FC portion of igG and C3b b. C5a c. C3b d. C3a Ans:: aa 106. Morphine is used in a. Terminal cancer illness b. Pain of gastric ulcer c. UTI Ans:: aa 107. Inferior wall MI a. Right marginal b. RCA c. LAD d. LCA Ans:: aa (cpsp demo) 108. Most appropriate about Middle meningeal Artery a.pterion is the surface marking b.its branches form grooves on the under surface of skull c. Involved most commonly in Subdural Hematoma d.enters skull thru foramen spinosum . Ans:: dd 109. vagotomy done for

91 a.decrease motility of gut b.decrease gastric acidity c.decrease pepsin d. decrease gastric acidity n pepsin motility of gut Ans:: dd 110. pain of Gastric ulcer is due a. Greater splanchnic nerve b.Lesser Splanchnic nerve Ans:: aa 111. medial wall of ischiorectal fossa is formed by a. Levator ani muscles b. external anal sphincter Ans:: bb both are correct refrence rj last 112. Double membrane bounded Organelle a. Nucleolus b. RER C. Mitochondria d. ribosomes Ans:: bb ( no option of mitochondria and nucleus was present) 113. Most radio sensitive tumor is a. seminoma b. glialglioma c. craniopharyngioma Ans:: aa 114. Man is intermediate host in A. Hydatid cyst B. Beef tapeworm infestation c. trichomoniasis d. filiriasis e. hook worm Ans:: aa 115. a young man fell from two storey building and now he has urinary incontinence. lesion is at a. s2s3s4 b. l2 l3 l4 c. s1,s2,s3 Ans:: aa 116. S2 differs from S1 by a..s2 has increase duration b.s2 has decrease pitch c. s2 has increase frequency d s1 has increase pitch e s1 has decrease duration Ans:: cc

117. a child presented to opd with 104 fever for the last 4 days. He has been passing cola colored urine for the last one day. He has bee n on anti malarials too. His LFTs are deranged with increase unconjugated bilirubin. What is the most probable diagnosis a.drug induced jaundice b.black water fever c.paroxysmal nocturnal haemaglobinuria dhaemolytic uremic syndrome e G6pd Ans:: ee 118. 46XX associated most probably with which of the following conditions a. Adrenogenital Syndrome b. Mixed Gonadal dysgenisis. c. Congenital Adrenal Hyperplasia Ans:: aa 119. Turnor syndrome with a.Autosomal dominant b.Autosomal recessive c shows secondary infertility d stunted growth is the main feature Ans:: dd 120. ECF is increased by infusion of A hypotonic saline B norma saline C hypertonic saline d dextrose water Ans:: cc 121. Peri capsular sinuses in a.lymph node b thymus c palatine tonsils Ans:: aa 122. Weight bearing line passes through a. Ischial tuberosities b. pubic crest c. ischial spines Ans:: aa 123. A 14 years old girl presented with right iliac fossa pain. pain associated with vomiting and was radiated to peri umbilical region. Which segment is involved/or pain radiation due to a. ilioinguinal nerve b.iliohypogastric nerve c.t12level d L2level

92 e T10 level Ans:: ee 124. microscopic slide of liver shows portal vein hepatic artery and bile duct at periphery and central vein in the centre. What is this called a.hepatic lobule b portal acinus c hepatic acinus (also called liver acinus) d Simple acinus Ans:: cc 125. EEG of a patient shows A increase frequency of waves in active mind than in deep sleep B is symmetrical in both hemispheres C small amplitude of waves in intense thinking D determine intelligence Ans:: aa 126. what is produced in muscles A. phospholipids B. creatinine C. urea D. uric acid E. glycogen Ans:: bb 127. About sertoli cells A produce testosterons B rapidly dividing cells C release androgen inhibiting factor D break blood testis barrier E reinkes crystals Ans:: cc 128. In Chronic renal failure what is least likely to be presented? A .low ph b . hypo phosphatemia c . enlargedparathyroids d. metabolicacidosis e. hypocalcemia Ans:: bb 129. On autopsy of CRF patient which organ was hypertrophied A. Para thyroid glands b. adrenal glands c. thyroid glands d. pituitary gland Ans::aa 130. Strongest layer of small bowel A submucosa B musoca

C longitudinal layer D circular layer E lamina propria Ans:: aa 131. chronic hepatitis histology on microscope shows A fibrosis b neutrophills c lymphocytes d lymphokines Ans:: aa 132. sarcoma as compare to carcinoma a. spread to bones early b. in the form of clusters of cells c. more sensitive to radiotherapy d. Clumps/Clusters e.Aggressive Ans:: ee 133. Systemic antifungal is a.Amphoterecin b. b.Ketokonazole c.Nystatin Ans:: aa 134. Heparin inhibits A organization of clot B retraction of cot C dissolution of clot D propagation of clot Ans:: dd 135. During pregnancy in which period teratogen affects fetus mostly A 1 to 2weeks B 3 to 8 weeks C 22 to 36weeks D last trimester E 8 to 12weeks Ans:: bb 136. muscle is prevented from tearing under action of which receptor a.muscle spindles b GTO c ruffinis d meissners e phosphate bonds Ans:: bb 137. maximum sodium entry in nerve fibre takes place at A during depolarization B during repolarization C at nodes of Ranvier

93 D as the impulse propagates along length of nerve fibre Ans:: cc 138. The valve which has more propensity toward diseases is a. Mitral Valve b. tricuspid valve c. pulmonary valve Ans:: aa 139. Non bacterial endocarditis due to A rheumatic fever B SLE C streptococcal glumerulonephritis D thrombo embolism E neoplastic diseases Ans:: ee 140. 23 years old boy came with mild jaundice. His hbsAg is negative, anti Hbc igM is Neg, anti Hbe positive. Whats the current situation a Acute resolving phase b acute hepatitis c window period d chronic hepatitis Ans:: aa 141. Difference between CSF and cerebral capillary is a. increase ph b carbohydrates c increase pco2 d less proteins Ans:: dd 142. Benign tumor among following is a. Leiomyoma b. Hamartoma c.Choristoma Ans:: aa 143. Hypotensive pt admitted in ward wd h/o dysuria and high grade fever. His blood culture is positive for pseudomonas. The disease is due to release of A. endotoxin B. exotoxin C proteases D. Toxic shock syndrome toxin. Ans:: aa 144. Highest diffusing capacity across respiratory membrane and in body fluids is A helium B carbon monooxide

C carbon dioxide D nitrogen E oxygen Ans:: cc highest diffusion capacity across respiratory membrane is of carbon dioxide while affinity of carbon monoxide is greatest 145. Tumor suppresser gene a. p53 b. ras c. c-myc Ans:: aa 146. upper motor neuron lesion leads to A decrease tendon reflexes B fasiculations C inc muscle tone D flaccid paralysis Ans:: cc 147. which nerve passes through cavernous sinus A. abducent nerve b. trigeminal nerve c. occulomotor nerve Ans:: aa 148. premalignant growth A sebborhic keratosis B compound nevus C intradermal nevus D dysplastic nevus syndrome. e junctional nevus Ans:: dd 149. 35 years old girl has been using analgesic for backache. She presented with platelets in normal range.wbc 7000 neutropenia. Her hb was 9 with small immature cells in peripheral smear. Diagnosis a acute leukemia b ITP c aplastic anemia d drug induced Ans:: aa 150. Gastric emptying increases by a.secretin b.Gastrin c.Cck d.ach inhibition Ans:: bb 151. local edema cause a.Allergy b. Flare

94 c. hyperresposiveness Ans:: aa 152. Exogenous antigens acted by interacting with a. MHC class II b. APC Ans:: aaa 153. A lady presented with yellow eyes and she has got hepato megaly. Also xanthomas present. What can be the cause in her A AMA B ANA C sclerosing cholangitis D lipidemia Ans:: aa 154. Regarding PseudoHYPERparathyroidism? A. because of Hypocalcemia B. Because of PTHrP by various tumor C. associated with Nephritic syndrome D. because of Vit D deficiency Ans:: bb 155. Right ovarian vein drains into a. IVC b.Lt. adrenal vein Ans:: aa 156. Regarding biceps brachi a. insertion on humerus b. Arises from supraglenoid tubercle c. pronation d. extention of elbow Ans:: bb 157. A 22 years old man came to Opd with ALT raised viral markers were negative. He has got some problem in the eye and Immunological tests were derranged. Whats the most useful investigation for him A liver biopsy B HbsAg C HCV virology D serum copper level E Sr. cerulo plasmin level Ans:: ee 158. A diabetic patient presented with increasing degree of progressive illness involving respiratory system. Gastrointestinal. What can be the cause a histoplasma b acid base balance

c Mucormycosis d hyperkalemia e blood glucose Ans:: cc 159. A person got an accident and there was severe bleeding from his body. He got multiple trAns:fusios but Ecg showed Changes after trAns:fusion due to A hypocalcemia B .hyperkalemia C hypokalemia D hypermagnesemia E PH changes Ans:: bb 160. Doctor patient relationship a. Active listening b. Having sound medical knowledge c. Updating skills d. good bedside manners Ans:: aaa 161. after adrenelectomy taste preference increases for A sweets b.Nacl c sour Ans:: bb 162. 60% of ventricular filling is because of A Rapid inflow B slow inflow C Atrial systole D slow ejection phase E iso volumetric relaxation Ans:: aa 163. about follicular cancer of thyroid a.vascular invasion b psammoma bodies c lymphatic spread d hyperchromatism Ans:: aa 164. protein on Rbc membrane for chloride exchange a.spectrin b. band 3 c.glycocalyx Ans:: bb 165. Regarding cholangio carcinoma a. Chlinorcis sinensis b. Tinea saginata. Ans:: aa 166. primary syphilis initial diagnosis

95 a. mouth ulcer b.genital sore c.tabes dorsalis Ans:: bbb 167. Dead space does not decrease in a.shallow breathing b.deep breathing c.lung diseases d.gravity e.tracheostomy Ans:: aa 168. chemical mediator of inflammation a.histamine b.serotonine c.lymphokines Ans:: aa ,,, prostaglandin is prefered if avaialble in options 169. Adverse blood trAns:fusion reaction occurs in A o+ to A+ B o neg to A+ C A neg to A+ d. AB+ to A+ e Ab neg to A+ Ans:: ddd 170. Regarding un coupling of Oxygen that’s produced by a.thyroid b.norepinephrine c.epiephrine Ans:: bbb reference ganong physiology 171. If a 70kg healthy adult loses one litre of blood within 5 minutes there would be a.decrease TPR bi.ncrease GFR c.increase venous tone d.decreasse diastolic pressure e.splanchnic vasodilatation Ans:: cc 172. injury to pulmonary alveoli causing constriction of alveoli by a.hypoxia b.atherosclerosis c.adenosine d.prolonged oxygen therapy Ans:: dd 173. Primitive streak mesoderm cells migrate Round cloacal membrane to form anterior abdominal wall cells. If this migration does not happen around cloacal

membrane will rupture and open cranially as a extrophy of bladder b imperforate anus c ectopic anal canal opening d hypospadias Ans:: cc 174. Diptheria toxin will affect most lethally on which organ a. Larynx b. Heart c. Kidney Ans:: bb 175. Sparing of the lateral part of the thenar eminence with conspicuous wasting of other small muscles of the hand suggests a lesion of: A. Ulnar B. Median c. C5 and C6 spinal cord segments D. C8 T1 spinal cord segments e. Radial nerve Ans:: aa 176. Blockage of beta adrenergics will result in a. Decrease heart rate b. heart contractility c. insulin production from islet of pancreas d. lipogenesis Ans:: aa 177. Due to stress of surgery what will increase and cause vaso constriction A. ACTH B. Serotonin C. Cytokines. D. increase level of circulating catecholamines ANS:: AA 178. Intraarticular disc of tmj involved.. Muscle paralyzed. A. Lateral pterygoid B. Medial pterygoid . C.Temporalis. Ans:: aa 179. Femoral arterial pulse. A. Mid inguinal point B. Mid inguinal ligament. C. Adductor canal. Ans:: aa 180. Regarding posterior Communicating artery

96 A. Connects PCA to ICA above occulomotor nerve B.Connect Pca below occulomotor nerve C. Connect ica to basilar artery. Ans:: aa 181. Child has episodes of bleeding and echymosis since birth hb 9.8 mg/dl And platelets count was normal 180 x 10^3 BT prolonged A. Hemophilia B. itp C. platelet functional defect Ans:: cc 182. Corona radiata forms from A. Germ cells. B.theca interna. C. Theca externa d. Granulosa Ans:: dd 183. All preganglionics neurotrAns:mitter A. Acetylcholine Ans:: aa 184. Ppd false negative A. Malnutrition B. Immunosuppressant Ans:: bb 185. Most important cell of chronic inflammation A. Lymphocytes B. Macrophages c. neutrophils Ans:: bb 186. patient with lung abscess develops meningitis, causative organism.? A. Staphylococcus B. Streptococcus Ans:: aa 187. 2x2 table A. T test. B. Anova . C Chi square Ans:: cc 188. How can doctor provide better management to patients: a. By better communication Ans:: aa 189. Which form of thyroxin is an active form A. Reverse t3 B. Unbound form C. Bound to albumin

D. Bound to prealbumin Ans:: bb 190. Parasympathetic effect on Heart A. Increase PR interval Ans: : aa 191. which of the following augment the effect of immune system? A. Complement System B.opsonins Ans:: aa 192. For chemotaxis A. Lymphokines B. serotonin Ans::aa 193. Testosterone main function a. Grows hair on scalp B. Inc muscle and bone growth Ans:: bb 194. Sweat gland by. a.sympathetic Ans:: aa 195. Primary active trAns:port A. Enzymes b. Carriers c. Proteins d. Pumps Ans:: dd 196. Question regarding edema--A. Lymphatic blockage Ans: : aa 197. Primary Brain vesicle is a. Mesen cephalon Ans::aa 198. Test for typhoid fever in first week a. Blood culture Ans:: aa

97

Surgery paper 2016 – 5th Sep 2016 1. Diptheria toxin will affect most lethally on which organ a. Larynx b. Heart c. Kidney Ans:: bb 2. Histolplasmosis affect which system a. Reticulo endothelial 3. 2% of 4m Lidocaine solution, how much lidocaine a. 80 ml b. 160 ml c. 400 ml d. 200 ml Ans:: aa 4. 0.85% of Nacl contains how much Nacl a 850mg/100 ml Ans:: aa 5. Prevents muscle from tearing a. GTO b. muscle spindle Ans:: aa 6. Skin Graft successful in a. Excessive Burns b. Agammaglobenemia Ans:: bb 7. Follicular Thyroid Ca, Microscopic feature a. Psamomma bodies b. Vascular Invasion Ans:: bb 8. Which cancer invades large blood vessels a. Clear cell ca of Lungs b. Clear cell ca of Kidney c. Bronchogenic Ca Ans:: bb 9. In Old what happens a. Decrease Vital Capacity b. Increase Cardiac Output Ans:: aa 10. Neutral absorption of sodium occurs at which part a. PCT b.DCT Ans:: aa

11. Breast Atrophy in females, Most common cause a. Estrogen deficiency b. Estrogen & Progesterone deficiency Ans:: aa 12. Most Common cause of fatty liver in our country a. Hep B & Hep C b. HCV c. Hbv Ans:: aa 13. Glycocalyx : a. Carbohydrate Moiety b. Helps in defence against microbes Ans:: aa 14. Unable to evert and dorsiflex, which structure injured a. Common Peroneal Nerve b. Superficial peroneal nerve. Ans:: aa 15. Scortal Swelling after injury, contents of swelling extends upto abdomen but not thigh, Most common cause of this swelling in scrotum a. Injury to Bulbar Urethra b. Injury to Penile urethra Ans:: aa 16. Episiotomy, which part at risk of being getting injured a. External Anal sphincter b. Bulbospongiosis Ans:: bb 17. Axillary Artery a. Lies lateral to Biceps tendon b. Present behind pectoralis minor c. Extends to lateral border of Pectoralis minor Ans:: bb 18. Anal Canal Blood supply a. Superior & Inferior Rectal Arteries b. Internal Iliac artery Ans:: aa 19. Ecchymosis and bleeding history in fisherman a. Vit C Deficieny b. Vit e deficincy Ans:: aa

98 20. Which clotting factors Depletes early a Factor 7 b. Factor 5 Ans:: aa 21. Chromosome 9:22 abnormality a. CML b. ALL Ans:: aa 22. Function of Vitamin A a . Forms Rhodopsin b. Not essenial for dark adaptation Ans::aa 23. Lymph drainage in Cervical Ca a. External & Internal Illiac Ans:: aa 24. Decrease in ESR due to a. Increase Fibrinogen b. increase Albumin Ans:: bb 25. Double membrane bounded Organelle a. Nucleolus B RER c.SER Ans:: bb ( there was no option of mitochondria or nucleus in cpsp exam ) 26. Recurrent Laryngeal nerve injured on both sides, Which muscle still intact a. Cricothyroid Ans::aa 27. Gastric Lymphome due to a. H Pylori Ans:: aa 28. Locally Malignant Cancer a. Basal Cell Ca Ans:: aa 29. Posterior wall of Rectus sheath b/w Symphisis pubis & Abdomen a. Arcuate line b.Linea semilunaris Ans:: aa 30. Bundle of His Blood supply a. LAD b. RCA Ans:: aa refrence Kaplan. 31. Head of Femur Blood supply a.Medial & Lateral Circumflex Artries b. Retinacular Ans:: B> A 32. Most common cause of Epiphyseal Tumor

a. Osteoblastoma b. Osteosarcoma c. Osteoid Osteoma d.Ewing Sarcoma Ans:: aa ,, if osteoclastoma ( giant cell tumor) in options prefer that. 33. DIC by a. Tissue Thromboplastin b. Factor 3 Ans:: aa 34. Jaw deviation to Left and paralysis on Right Side, Lesion is in a. Pons b. Internal Capsule Ans:: bb 35. Dorsal Nucleus of Vagus a. Medulla b. Pyramid Ans:: aa 36. Sciatic nerve injured which structure will still flex the knee a. Short head of biceps femoris b. Long head of biceps. Ans:: aa 37. Patient died of Stroke, which type of necrosis a. Liquificative ncrosis b. Coagulative necrosis Ans:: aa 38. Median Umbilical Ligament is remnant of a. Urachus b. Umblical arteries Ans:: aa 39. Which structure will be damaged in Lumbar Puncture a. Ligamentum Flavum b. Posterior Longitudnal ligament. Ans:: aa 40. Scenario with Vit K def, Vitamin K not absorbed , which factor decreased a. Prothrombin b. Protein C c. Protein S d. Factor VIII Ans:: aa. If factor 7 in options prefer that. 41. Low estrogen pills will cause a. Breast Ca b. Lungs Ca c. Endometrial Ca d. Hepatic Adenoma

99 Ans:: dd 42. Pregnant lady with raised SGPT, hiatory of visit to Remote village, which hepatitis a. Hep A b. Hep E c. Hep B d. Hep C Ans:: bb 43. Which Ca will not metastasize a. Basal Cell Ca b. SCC Ans:: aa 44. Which of the following in most unlikely to occur in CRF a. Hypophospatemia b. Anemia Ans:: aa 45. Action of Gluteus Medius & Minimus a. Abduct & Medially Rotate the thigh at hip b. Adduct & Laaterally rotate Ans:: aa 46. Isthmus of trachea lies in front of a. 2nd 3rd and 4th Tracheal Ring b. 2nd and 3rd Tracheal ring. Ans:: aa 47. ECG changes only in lead v4, due to which Artery a. Left anterior descending b. RCA Ans:: aa 48. Bund of HIS, blood supply a. Right coronary artery b. Left anterior descending c, Left circumflex artery d, Left Main Coronary Artery Ans:: bb reference Kaplan B.D churasia 49. History of Hypertension & Chest pain radiating to back, patient died. Findings on Autopsy in such patient a. Medial Necrosis of Aorta b. MVP Ans:: aa 50. Sertoli Cells inhibit what a. Androgen binding protein b. FSH Ans:: aa 51. Abdominal Aorta a. Gives Renal Arteries at L2 b. gives Lumbar branches at T 8

Ans:: aaa 52. Radiosensitive Tumor a. Seminoma b.Lymphoma Ans:: aa 53. T cell Receptor Binds with a. MHC b. Antigen presenting cells Ans:: aa 54. Breast Atrophy in adult female is due to a. Estrogen b. Estrogen & Progesterone Ans:: aa 55. Outer quadrant of Breast is drained by which lymph nodes a. Pectoral (Anterior Axillary) b. Central group Ans:: aa 56. Loss of Supination & Flexion after Stab Injury to Inner Arm a. Musculocutaneous b. Axillary Ans:: aa 57 Chemical mediator of inflammation a. Lymphokine b. Serotonin Ans:: aa 58 Femoral artery palpated a. Mid inguinal point b. Mid inguinal ligament Ans:: aa 59 Pseudomonas causing fever etc a. Endotoxins b. Erythrotoxin Ans:: aa 60 Cholangiocarcinoma a. Choloronicus sineses Ans:: aa 61 Lateral Thenar muscles spared but short muscles of the hand paralyzed .. where is the Lesion? a. C8, T1, b. Ulnar Nerve c. Median Nerve d. Radial Nerve Ans:: bb 62 which of the following lymphoid organ receives lymph via SUBCAPSULAR SINUS a. Thymu

100 b. spleen c. Lymph node Ans:: cc 63 Regarding first lumbrical a. Bipennate b. attached to proximal phalanx of index finger c..attach to radial side of first phalanx Ans:: cc 64 Patient having severe chest pain died, Autopsy says it was dissecting aortic aneurysm... Most likely cause: a. Medial Necrosis of Aorta b. Saccular Berry Aneurysm Ans:: aa 65 heparin prevents a. propagation b. embolization c. organization Ans:: aa 66 Patient had previously Episodes of Dizziness and Diplopia, patient presented now in Acute Comatosed state.. Cause : a. Carotid Artery Thrombosis b. Basilar Artery Thrombosis Ans:: bb 67 Regarding Ciliary body a. Outer Epithelium Pigmented and Inner Not b. Pigmented layer is a continuation of pigmented layer of Retina. Ans:: bb ,, if there is option : nonpigmented epithelium secretes aqueous humour then select that … 68 Superficial Cardiac Plexus contain. a. Both sympathetic and parasympathetic b. Sympathetic c. Parasympathetic Ans:: aaa 69 In VIT K deficiency, which factor will be depleted first and foremost a. Factor 7 b. Factor IX c. Factor 10 d.Protein S Ans:: aa

101

SURGERY 6 SEP Morning 1. low frequency – meissner’s corpuscle 2 .256 HZ – pacinian corpuscle 3. pudendal block – ischial spine 4 phrenic nerve – pericardiophrenic artery 5 trigerminal nerve – temporalis 6 brain vein – drain into dural sinuses 7 ant relation of ant scalene muscle –phrenic nerve 8 sub acute endocarditis – strep viridiAns: 9 carotid sinus – decrease discharge in hypovolemia 10 CNS (CSF) – decrease glucose 11 parathyroid – post medial of thyroid 12 tongue ant 2/3 nodule – metastasis to submental? 13 prevent unwanted trAns:port- tight junctions 14 protrusions – lateral pterygoid 15 superior parathyroid artery – external laryngeal nerve 16 myxiod degeneration- mitral valve prolapse 17 medial necrosis- dissection of aorta 18 thyroid- epithelial changes with activity 19 myxedema- complete lack of thyroxine 20 smokers epithelium –squamous metaplasia 21 free nerve ending – unencapsulated 22 fracture – avascular necrosis of femur head 23 haversion lacunae- osteoclasts. 24 mid trimester hematopoises- liver 25 polycythemia vera cbc shows – lymphocytes count are normal 26 ADH inhibited by –alcohol 27 anemia in pregnancy – dilutional. 28 maxium absorption in kidney pct – glucose 29 secondary active trAns:port – amino acid in pct 30 microtubules – centrioles 31 neck pain of young girl specially during study which nerve damage – spinal accessory 32 cause of hyperthyroidism – graves 33 short muscle of hand – t1 34 levator ani – pudendal + s4 35 sigmoid colon – s234 36 pheochromocytoma – alpha 1 + beta blocker 37 meckal diverticulum - vitelline duct 38 ureter post relation – genitofemoral nerve/internal ilaic artery 39 pudendal nerve – pudendal canal 40 post duodenal ulcer- gastrodueodenal artery 41 breast- pregnancy changes alveoli secretory 42 hipjoint inferior relation – obtruator externus 43 antibiotic use, causing bleeding – give vitamin k 44 benign – hemartoma 45 hemarthorosis – factor 8 assay 46 b- thalassemia major – HBF 47 chondrocytes to osteocytes – sammatomamotrophin.(GH)

102 48 normal basal metabolic rate by – liver 49 myleoperoxidase – neutrophil 50 glycoytic loaded – mitochondria. 51 hereditary spherocytosis – membranous defect 52 metastasis - loss of e cadherin’s 53 fish tap worm diphylobotharium Latium – fish in diet of fisher man 54 antioxidant – vitamin e 55 adenocarcinoma – adenomatous polyp 57 increase bilirubin 16 alkaline phosphatase 300 carcinoembryonic antigen 200, diarrhea due to psc and colon ca, - ca colon 58 virus – oncogenes 59 pus – dead neutrophils 60 t3 n2 m0 breast – stage 3 61 throne prick – stap. aureus 62 sulphur containing – cysteine methionine, valine (cmv) 63 most common breast ca – invasive ductal 64 cervix stratified squamous epithelium - metaplasia 65 estrogen in post menopausal women - endometrial ca 66 artery remain in pelvis – middle rectal 67 rbc nucleus absent in – reticulocytes 68 left 4th intercostal space stabe- intercostal membrane 69 accompany lad – great cardiac vein 70 ascaris – respiratory symptom 71 amoebic liver abscess – serology 72 mc Burney point – ilio hypogastric nerve – direct inguinal hernia 73 hassel bechs triangle– direct inguinal hernia 74 bulbous urethra – superficial perianal pouch 75 aortic aneurysm at opening – thoracic duct azygous vein 75 aorta – branches to esophagus, pleura pericardium 76 abducted arm drops at once – deltoid damaged 77 loss of abduction upto 30 – supraspinatus 78 thumb tingling – c6 nerve 79 young boy with fracture decrease serum calcium increase urine calcium – vitamin d deficiency 80 medial meniscus attached to – medial cruciate ligament 81 below umbilicus rectus sheath – arcuate line 82 bile salts decrease – after resection of ileum 83 pseudomonas colitis – clindamycin 84 vancomycin – drug monitoring 85 atropine – decrease acetylcholine binding 86 sweat glands – cholinergic symptoms 87 parasympathetic effect – ciliary muscle contraction 88 radial muscle contraction – alpha 1 89 not secreted of nerve terminal in ANS:- l- dopa 90 allergy – bradykinin (histamine was not in option ) 91 prozone phenomena – increase antibody titer 92 Rbc antigen – secreted in saliva 93 infective endocarditis – damaged valves > bacteremia> thrombus>perforation 94 cataract+ deafness + cardiac defects – rubella 95 Sexually trAns:mitted disease leading to neonatal conjectivitis after vaginal delivery – gonorrhea 96 most common malignancy after radiotherapy – hematopoeitic.

103 97 for trAns:plant specimen – wbc 98 old man cough with thick sputum – klebsiella 99 teratogen – alcohol 100 septic meningitis ( fatal) – streptococcus 101 smoking teratogen during – 3-8 weeks 102 isograft ( identical twins ) late rejection due to minor hla antigen 103 csf different from plasma – dec glucose 104 septum supply mainly – LAD (2/3) 105 trachoma – chlamydia 106 myocardial infarction sign on ecg – st elevation 107 after MI patient in ICU suddenly become dyspneic and cyanosis – vent perfusion mismatch 108 vein effected in portal hypertension - portal vein 109 cirrhotic patient with diated umblical veins developed edema in lower limb most effected – inferior vena cava. 110 right heart border on x ray – superior vena cava 111 lobar pneumonia – sputum culture 112 specific of shock – tissue hypoxia 113 lidocaine side effect – dizziness and light headedness 114 difference between anaphylactic shock and hypovolemic shock – temperature. 115 barret esophagus – adenocarcinoma esophagus 116 steatorrhea most due to – pancreatic resection 117 5cm painless purpura in old lady ( senile purpura) – due to capillary wall fragility and defect 118 primary brain vesicle – mesencephalon 119 posterior cruciate ligament – prevent ant dislocation of femur 120 same in skeletal and smooth muscle – calcium influx before contraction 121 collagen atrophy 122 temporal lobe – memory 123 external carotid difference to internal carotid – gives off branches in neck 124 partially alleviate thirst – GIT distention 125 ADH regulation – osmoreceptors 126 Parathyroid surgically surgically removed – tetany + troussieurs sign 127 increase parathyroid hormone – chronic renal failure 128 residual volume – left air 129 left shift of – HBF 130 normal blood pressure 120/80 mmhg 131 second peristalsis – esophageal distention 132 anthrax – 90% skin lesion 133 negros – keloid 134 lymph nodes with stratified squamous epithelium – palatine tonsils 135 scanty barr body – turner syndrome 136 edema – lymphatic obstruction 137 clara cells – terminal bronchioles 138 pulmonary artery supply – alveoli 139 vertebral disc – ¼ of column 140 cubital fossa injection which nerve will be damaged- median nerve 141 most common fracture – tibia 142 first bone to ossify -clavicle 143 venous cut down – great saphenous 144 iron stores as – ferritin 145 aplastic anemia – fat in bone marrow

104

Surgery paper 1 & 2 Mix 14th Oct 2016 1. Ectoderm and endodermunite at A. Cloacal membrane B. Coelom Ans:A 2. Suprapubic catheterization done because bladder is not covered by peritoneum at A. Dome B. Posterior C. Anterior Ans:C 3. Corneal opacity is due to A. Ethambutol B. Phenobarbital C. Chloroquine Ans:C 4. A difficult question regarding pons options were A. Facial colliculus B. Facial nuclei ventrolateral surface facial motor nucleus is located ventrolateral to abducent nucleus and facial colliculous. The facial colliculous is an elevated area located on the dorsal pons in the floor of the 4th ventricle. It is formed by fibers from the motor nucleus of facial nerve. So both correct 5. Scenario regarding lateral rectus muscle damage…lesion at Ans. Facial colliculus Facial colliculous is facial nerve which loops around abducent nerve nucleus. Patient with facial colliculous syndrome will demonstrate an ipsilateral horizontal gaze palsy with an ipsilateral lower motor neuron pattern of facial weakness. The horizontal gaze palsy is secondary to a lesion that is effecting the CN 6 necleus that is responsible for abduction of the ipsilateral eye by sending projections to the ipsislateral lateral rectus muscle. 6. Prevertebral fascia encloses A. Sternocledomastoid and trapezius B. Muscle of cervical vertebrae C. Trachea and esophagus D. Trachea and thyroid Ans:B

7. Aphasia…damage or lesion at A. Parietal B. Occipital C. Temporal Ans:C 8. Na is actively absorbed from A. Jejunum B. Ileum C. Colon D. Stomach E. Duodenum Ans:C 9. Long scenario…..Bladder ca gene was asked A. C-neu B. Kras C. Nmyc HRAS>KRAS>NRAS Ans:B 10. Parents have 6 or 7 children. only 1 has retinoblastoma…which gene is responsible A. P53 B. Rb C. Kras Ans:B 11. Melanoma most common site A. Soft palate B. Hard palate C. Tongue Ans:B 12. Recurrent infections…diarrhea… IgA, igG, IgE levels given..all were low..cause? A. Chronic variable agammaglobinemia B. Bruton’s agammaglobinemia Ans:B 13. Cell to cell adhesion A. Integrins B. Selectins C- cadherin>selectin Ans:A 14. Ejection Fraction decreases. what increases A. ESV B. EDV Ans:A 15. Single dose for streptococcal infection A. Ampicillin B. Benzyl penicillin C. Benzathine penicillin

105 Ans:C 16. Thyroid hormone at physiological condition causes A. Fatty acid oxidation B. Protein synthesis Ans:A 17. Depression and low motivation lesion at A. Midbrain B. Temporal C. Basal ganglia Ans:C Ans:A 20. If all values fall on one point or lie within given range it is? A. Rate B. Frequency C. Prevalence D. Data Ans:B 21. Glandular acini pseudostratified columnar epithelium with smooth muscle calcification A. Prostate B. Seminal vesicle C. Parotid D. Breast Ans:A 22. Regarding Glycolysis A. Hexose monophosphate shunt B. Includes hexose sugars to pyruvate Asn:A 23. No. of somites at 30th day A. 42-44 B. 30-32 C. 26-28 D. 32-36 Ans.D 24. Linea aspera of femur joins upward and medially to A. Pectineal line B. Intertrochanteric line C. Intratrochanteric crest Ans:A 25. Main pillars of medical ethics Ans. Autonomy, beneficence, nonmaleficence, justice (correct) 26. School students play game of breathing in a bag…boy starts hyperventilating after few mins, reason? A. Decrease p02

18. Another difficult question regarding anatomy of basal ganglia options were A.Below tentorium cerebelli B.Midbrain pretectum part of basal ganglia Ans( Controversial) 19. Standard deviation is calculated along mean because it shows A. Variability among individual values B. Sample size B. Increase pco2 Ans:B 27. Final motor pathway A. Corticospinal tract B. Alphamotor neurons emerging from anterior horn C. Inhibiting fibers along alpha motor neurons Ans.B 28. Egg contains protein avidin… eating raw eggs will lead to deficiency which causes A. Increase transamination B. Decrease phosphorylation C. Decrease carboxylation D. Decrease coA formation Ans.C 29. Creatinine 8mg….B.P 210/110… damage to which is a cause of this A. Glomerular cells B. JG cells C. Macula densa D. PCT Ans.B 30. Eversion done by A. Peroneus tertius B. Peroneus longus C. Tibialis anterior Ans.B 31. Loss of dorsiflexion and sensation on foot..damage to A. Tibial nerve B. Common peroneal nerve C. Femoral nerve Ans.B 32. Axillary vein formation A. Basilica vein and vena communicante of brachial artery ANS:a

106 33. Patient during surgery transfused with blood… she develops high grade fever, hypotension and shock in post operative period…cause? A. Graft versus host reaction B. Febrile non hemolytic reaction C. Anaphylactic shock D. Bacterial contaminated blood ANS.D 34. Pyruvate between A. Glucose and acetyl coA (correct) 35. Long scenario…cotton wool sensation.. MCV raised…which deficiency A. Vit b6 B. Vit c C. Vitb12 ANS .C 36. A question regarding coronary supply..options dnt remember..difficult one 37. Stroke volume increases…what decreases A. ESV B. EDV Ans.A 38. A patient of sickle cell anemia develops hemolysis acutely hb 4… cause? A. Hemolytic crisis B. Sequestration crisis Ans.A 39. Increase bleeding time and appt cause A. Low platelet B. Hemophilia C. Vit k deficiency D. Vwd ANS.D 40. Patient on ATT develops hyperuricemia..which drug A. Isoniazid B. Pyrazinamide C. Rifampin D. Streptomycin Ans.B 41. Lymphoid follicles are present in A. Thymus B. Cortex of lymph node C. Spleen Ans.B 42. Antibodies formed by

A. Plasma cells (correct) Remember Antibodies formed IN spleen formed BY plasma cells. 43. 1.5L blood loss…low GFR and urinary output…part of nephron responsible in bringing back GFR and Urinary output A. PCT B. DCT C. Thick ascending limb D. Thin ascending limb E. Collecting tubule Ans.C 44. Type of Ig in Myasthenia gravis A. IgA B. IgM C. IgG D. IgE Ans.C 45. 2L sweating..drinking 2L pure water causes A. Increase ICF volume B. Increase ECF volume C. Decrease ECF osmolarity D. Decrease ICF osmolarity Ans.A 46. Loss of flexion and sensation of little finger. damage to A. Radial nerve B. Ulnar nerve C. Median nerve Ans.B 47. Superior thyroid artery ligation. damage to A. External laryngeal nerve B. Recurrent laryngeal nerve Ans.A 48. Regarding infrahyoid muscles A. Cause upward movement of larynx B. Between sternum and hyoid C. Supplied by ansa cervicalis ANS.C 49. Regarding Trachea A. C4-t3 B. C6-T4 C. C6-t7 Ans.B 50. Anterior suture closes at A. 18-24mnths B. 12-18 C. 6-12

107 Ans.B 51. Medullary branch of vertebral artery occlusion causes damage to Ans:pyramids 52. Vibration sensed by A. Pacinian B. Meckel C. Meissner D. Ruffini Ans.A 53. Regarding virus infection A. Proto oncogenes B. Oncogenes Ans.B 54. DNA virus A. RSV B. Picorna virus C. Adeno virus ANS.C 55. Yellowish discharge from lesion A. Staph aureus B. Actinomyces C. Pseudomonas Ans.B 56. 3yrs old boy having swollen right knee with pain and redness… A. Streptococcus fecalis B. Pseudomonas C. Staph aureus D. Strep viridans Ans.C 57. Greenish discharge from lesion A. Staph aureus B. Pseudomonas C. Actinomyces Ans.B 58. Long scenario…parasite having narrow anterior end A. Tenia solium B. Trichuris trichura C. Ascaris Ans.B 59. Type of hypersensitivity in TB A. Type 1 B. Type 2 C. Type 3 D. Type 4 ANS.D 60. Most important investigation in TB A. Caseous granuloma on biopsy B. AFB staining

ANS.B( discussed thousands time in group ans was b) 61. During bronchoscopy first structure visible is A. Upper lobe B. Upper basal lobe C. Lower basal lobe D. Middle lobe Ans.C 62. Angular cheilitis caused by A. Chlamydia B. Cryptosporidium C. Candida ANS.C 63. CD4 cells are A. Cytotoxic B. Helper ANS.B 64. Subacute endocarditis caused by A. Staph aureus B. Strep viridans C. Strep pyogenes Ans.B 65. Cysticercosis caused by ingestion of A. Ova of tenia solium B. Eating raw beef C. Eating raw mutton Ans.A 66. HIV confirmation test in baby of HIV pos mother A. Western blot B. Elisa C. HIV PCR ANS.C 67. Light on retina causes A. Increase cGMP B. Increase neurotransmitter release C. Increase conversion of cis retinal to alltrans retinal ANS .C 68. Vit D3 is associated with A. Storage form B. Animal source C. Plant source D. Active form Ans.B 69. Which of the following is essential aminoacid A. Tyrosine B. Cysteine C. Phenylalanine

108 ANS.C 70. Most common inherited defect in hemostasis A. Factor 2 deficiency B. Factor 5 deficiency C. Factor 1 deficiency D. Factor 9 deficiency Ans.B 71. Pseudomembranous colitis caused by A. C. botulinum B. C. difficle C. C. tetani Ans.B 72. Regarding Vibrio cholera A. Grows at 42 C B. Grows in alkaline medium C. Grows in acidic medium Ans.B 73. Surfactant contains compound Ans. Dipalmitoylphosphatidylcholine(correct) 74. Regarding surfactant A. Decreases compliance B. Increases surface tension C. Decreases surface tension ANS.C 75. Lesion on nose…proteinuria..vasculitis.. A. Leprosy B. Wegeners granulomatosis ANS.B 76. Most imp test for gonococci A. Blood culture B. Gram stain C. ZN stain D. AFB Ans.B 77. Baby having cataract cause A. Toxoplasmosis B. Rubella C. Syphilis Ans.B 78. Right gastric branch of A. Gastroduodenal B. Splenic C. Hepatic ANS.C 79. Free edge of lesser omentum contains A. CBD B. portal vein C. hepatic artery

ANS .C 80. LVF occurs due to malfunction or damage of A. Mitral stenosis B. Aortic C. Tricuspid Ans.B 81. Artificial pacemaker leads should be in A. SA node B. AV node C. Bundle of his Ans.B( ideal ans is right ventricle which is CPSP key,but it was not in our paper) 82. Regarding esophagus A. It lies on right side throughout its course B. Supplied by thoracic aorta only . C. Joins stomach at 10th costochondral junction D. Has thick serosa on lower end Ans.C 83. Shortest pre –erythrocytic phase of which plasmodium A. Malariae B. Falciparum C. Vivax D. Ovale Ans.B 84. History of pharyngitis..enlarged cervical lymph nodes..cause A. Lymphoma B. Burkitts lymphoma C. Infectious mononucleosis ANS.C 85. Most common cause of nasopharyngeal Ca A. CMV B. EBV C. RSV Ans.B 86. RCA supplies Ans. was all SA,AV node and bundle(correct) 87. Femoral hernia strangulation due A. Inguinal ligament compression B. Tight boundaries C. Narrow canal ANS .C 88. Inferior rectal artery branch of A. Internal carotid B. Internal pudendal

109 C. External carotid D. Inferior mesenteric Ans.B 89. Flexion of knee and extension of hip A. Sartorius B. Semitendinosus C. Biceps femoris Ans.B 90. Regarding medulla Ans. Decussation of pyramids 91. Superior mesenteric supplies Ans. Till proximal 2/3rd of transverse colon 92. Regarding defecation reflex Ans. Opening of internal anal sphincter by parasympathetic 93. Regarding autonomic nervous system A. All sympathetic nerves are adrenergic B. All sympathetic nerve are cholinergic C. All parasympathetic nerves are cholinergic ANS.C 94. Sympathetic stimulation causes A. Salivation B. Mydriasis C. Miosis Ans.B 95. Parasympathetic nucleus of vagus A. Nucleus ambiguous B. Solitary nucleus C. Dorsal nucleus ANS.C 96. 25yrs old female has weakness, malaise and pallor…occult blood in stools. What is the cause? A. Iron deficiency anemia B. Hemolytic anemia C. Sideroblastic anemia Ans.A 97. Cervical Epithelium Stratified Squamous A. Metaplasia B. Anaplasia C. Dysplasia Ans: A 98. Regarding collagen A. Most abundant protein in body B. Is acidophilic Ans.A 99. Scarlet fever

Ans. Erythrogenic toxin(correct) 100. dry mouth ..dry eyes Ans. Anti ssa..ssb(correct) 101. protein not filtered due to Ans. Pore size and neg charge(correct) 102. long scenario…brain necrosis was asked Ans. Liquefactive(correct) 103. long scenario regarding anti cancerous drugs… tumor regresses due to A. Apoptosis B. Necrosis Ans.A 104. blood supply of head of humerus A. Anterior circumflex B. Posterior circumflex C. Arcuate Ans.A 105. cardiac muscle cannot b tetanized due to Ans. Due to long refractory period(correct) 106. blood vessels innervated by A. Parasympathetic B. Somatic C. Sympathetic ANS.C 107. high catecholamines and blue cell tumor A. Pheochromocytoma B. Neuroblastoma C. Pituitary adenoma Ans.B 108. ACTH secreted through A. Hypophyseal portal system B. Arterial supply of ant. Pituitary C. Venous supply of ant. Pituitary ANS.C 109. suprachiasmatic nucleus function A. ADH production B. Circadian rhythm C. Temperature regulation Ans.B 110. testis lymphatic drainage A. Bilateral para aortic B. Unilateral para aortic C. Inguinal nodes Ans.A 111. regarding internal capsule Ans. Medial to globus pallidus 112. 4th Arch derivartives Ans:Cricothyroid Muscle

110 113. Small cell Ca produces A. ACTH B. PTHrp C. B-HCG Ans.A 114. sterilization( CPSP mcq) Ans. Autoclave for 3min option correct.. 115. parotid gland supplied by A. GVA B. SVE C. GVE D. SVA Ans.C 116. cold intolerance, constipation, weight gain A. Hypothyroidism B. Hyperthyroidism Ans.A 117. which of the following is related to cancer A. Silicosis B. Asbestosis ANS.B 118. Hydrocarbons cause A. Small cell ca B. Mesothelioma C. Bronchogenic ca ANS.C 119. hypocalcemia and hyperphosphatemia A. PTH B. CRF C. Vit D Ans.B 120. structure pierced during lumber puncture Ans:Ligamentum flavum(correct) 121. patient has fever..chest pain..not related to respiration A. Costochondritis B. Myocardium ANS.B 122. medial lymphatic drainage of breast A. Pectoral B. Supraclavicular C. Internal thoracic D. Inferior phrenic Ans.C 123. patient cannot lower his jaw muscle involved A. Medial pterygoid

B. Lateral pterygoid C. Temporalis Ans.B 124. regarding popliteus A. Medially rotated femur on tibia B. Attached to lateral ligament C. laterally rotates femur on tibia ANS.C 125. patient lying on ot table at 25 c…80 % humidity..heat loss due to A. Conduction B. Radiation + conduction C. Evaporation Ans.B 126. selective inhibiter of COX-2 A. Aspirin B. Celecoxib C. Ibuprofen Ans.B 127. GFR and renal blood flow increases by A. Efferent dilatation B. Efferent contraction C. Afferent dilatation ANS.C 128. cervical and I think supra clavicular nodes palpable were given..stage was asked A. 2a B. 2b C. 3 ANS.C 129. phrenic nerve accompanied with A. Musculophrenic artery B. Pericardiophrenic artery ANS.B 130. movement of glucose from higher to lower conc… this type is called A. Na glucose cotransport B. Simple diffusion C. Facilitated diffusion Ans.B 131. RER involved in A. Toxin removal B. Protein synthesis ANS.B 132. tricuspid valve auscultation Ans. Lower right border of sternum(correct) 133. dyspnea on lying down Ans. Retrosternal goitre(correct)

111 134. therapeutic and adverse affect of loop diuretics A. Alkalosis B. Low blood volume C. Acidosis Ans.B 135. cause of menstruation A. LH..FSH B. Estrogen C. Hormone withdrawl D. Progesterone ANS.C 136. in pheochromocytoma drugs given A. B-blockers B. A-blockers C. Ca channel blockers D. Both alpha and beta blockers ANS .D 137. investigation in pregnant patient A.GGT B. ALT ANS.A 138. claw hand due to Ans.Unopposed action of extensor digitorum and flexor digitorum profundus(correct) 139. MELAS syndrome histological finding Ans. Red ragged fibers(correct) 140. Cardiac fibrillation can occur if stimulus is given at A. Peak of action potential B. Start of action potential C. End of action potential ANS.C 141. Regarding asthma Ans. FEV1 142. Regarding nitroglycerine except A. Blood vessel dilatation B. Decrease pre load C. Decrease HR ANS.C 143. ischial spine inverted Ans. Android pelvis 144. Temporal lesion causes loss in A. Olfaction B. Memory C. Vision ANS.B 145. Short gastric artery branch of A. Hepatic

B. Celiac C. Splenic D. Gastroduodenal ANS.C 146. left gastroepiploic artery branch of A. Gastroduodenal B. Splenic C.Hepatic ANS.B 147.Pancreatic fluid leakage accumulates in A. Subhepatic space B. Lesser sac C. Paracolic gutter ANS.B 148. optic chiasma lesion causes Ans. Bitemporal heminopia 149. pubic symphysis A. Primary cartilaginous B. Secondary cartilaginous ANS.B 150. malformation of diaphragm due to A. Failure of fusion of lateral buds with pleuroperitoneal membrane A. act on genes B. act on cytoplasm C. act on receptors in cell membrane ANS.A 151. feco oral route A. Hep b B. Hep c C. Hep e D. Hiv ANS.C 152. aortic opening in diaphragm Ans. Azygous vein and thoracic duct 153. drug-drug interaction due to A. Pharmacokinetics B. Pharmacodynamics C. Both ANS.C 154. dislocation of shoulder which nerve damaged Ans. Axillary(correct) 155. regarding thyroid gland A. Its lobes are pyramidal in shape B. Lobes are symmetrical C. Lies infront of tracheal ring 1,2,3 ANS.A 156. maximum contribution in power of eye

112 A. Anterior surface of cornea B. Lens C. Iris ANS.A 157. organ least likely to get infarction A. Lungs B. Kidneys C. Liver D. Spleen ANS.C (cpsp key although lungs is also correct) 158. ascent of horse shoe kidney A. Superior mesenteric B. Inferior mesenteric ANS.B 159. phenol as compared to alcohol(poor recalled bcq) Ans. Less painful analgesia 160. tachyphylaxis Ans. Diminishing action of drug developing rapidly 161. stress hormone of body A. GH B. Testosterone C. ACTH D. Cortisol was not in options ANS.C 162. heat intolerance palpitations Ans. Hyperthyroidism 163. start of 5th week A. Heart tube B. Stomach rotates C. Limb buds ANS.B 164. starling law Ans. Increase cardiac output with increase venous return(correct) 165. anatomical question Ans. tail of pancreas in splenorenal ligament 166. patient on thyroxine what should b monitored A. T3 B. TSH C. T4 ANS.B 167. trachea bifurcates at A. T4-t5 B. T5-T6 C. T6-T7 ANS.A

168. HCO3 reabsorption in A. PCT+DCT B. PCT+CT C. DCT D. CT ANS.B 169. Functional residual capacity A. RV+ERV B. ERV+tidal C. RV+ERV+tidal ANS.A 170. patient in CCU A. Anxiety B. sense of loss ANS.A 171. edema in nephrotic syndrome due to A. Increase blood volume B. Hypoalbuminemia C. Lymphatic obstruction ANS.B 172. shock by injection…Ig involved A. IgA B. IgG C. IgE D. IgM ANS.C 173. question regarding parts of duodenum A. Retroperitoneal B. 1st2nd intraperitoneal ANS.B 174.most common fractured carpal bone A. Lunate B. Scaphoid C. Pisiform ANS.B 175. immediate reflex after acute blood loss A. Baroreceptor B. CNS ischemic C. Renin angiotensin D. ADH ANS.A 176. regarding staging Ans. Used to detect spread of tumor 177.difference btw interstitial and plasma fluid A. Protein B. Electrolytes C. Osmolarity ANS.A

113 178. open wound healing A. Contraction of myofibroblasts 179. highest mitochondria at apex Ans. Cilia(correct) 180. calcitonin secreted by Ans. Thyroid ca

7TH NOVEMBER MORNING SURGERY 1)Cervical branch facial nerve supply. A)Plytsma b)Masseter Ans:: a 2)Lateral half of thenar spared, root injury involved. a) ULNAR Ans:: A 3)Retrograde mamory loss.. a)Brain stem lesion b)Precentral frontal lobatomy_ Ans:: B 4.Pendular knee jerk?? Ans:: Cerebellar lesion 5)Chorda tympanea nerve.. a)autonomous function b).innervate ant 2 3rd of tongue .. .C)branch of mandibular nerve Ans:: B 6)Supinator muscle paralysis...function replace by which muscle ( A)bicep brachii B)brachialis C)brachiradialis Ans:: A 7)I/V drug abuser most likely develop A)Marantic Endocarditis B)Libman Sac Endocarditis C)Rheumatic Disease D)Infective endocarditis Ans:: D) 8)Wat causes inc aldosteron secretion A)Hyperkalemia b)FLUID OVERLOAD ANS:A 9)-The Characteristic Lesion Of Prmiary Heridator Telangectesia Is On • A) Palate • B)gingivae • C)Lips

181.free nerve ending Ans. Un encapsulated(correct) 182. Raised ICP which drug cannot be given Ans. Ketamine • •

D)Nose E)Tongue Ans:: C 10)Maximum reabsorbed from renal tubule to plasma? A)chloride sodium potassium glucose Ans:: A 11)Sensory supply of uterus travels in A)Round Ligament B)Uterosacral Ligament C)Tranverse Cervical Ligament D)Broad Ligament Ans:: D 12) 62 years old female with change in bowel habits. DRE negative and sigmoidoscopy shows 6cm mass near caecum. Diagnosed by pathologist as Adenocarcinoma. Most likely a) -K-RAS with neoplastic cells b) I CAM VCAM Ans:: A 13)Subcostal incision. Rectus abdominis not necrosed even after damage of superior epigastric artery because it anastamoses with ?? A) deep circumflex iliac : B) inf Epigastric artery C)subcostal artery external pudendal artery Ans:: B 14) 256Hz tuning fork Wala. A) Pacinian B) Meissonier Ans: A 16.Which of the following receive max drainage from arachnoid granulations. A) Straight sinus B) Sup sagital sinus C) cavernous sinus Ans: B 17. 8 years old boy with difficulty in swallowing and mass on left side of esophagus. Desmin negative and Vimentin + . He had a history of fetal alcohol

114 syndrome and not doing well in school. Most likely A)-Wilms tumour B)-Rhabdomyosarcoma C)-Metaplastic Lymphoma D)-Retinoblastoma Ans: B 18.Rejection of trAns:plantion WITIHIN MINUTES? A) cell mediated b) preformed antibody c) immune complex d ) blood group Ans: B 19..Endothelum histolgy? simple squamous cells Ans:: A 20..Feco oral route which ? a)hep E b)hep C c) hep b Ans:: A 21..In Emergency deprtmnt ? a)Doctr relation patient options given? b)Social skills c)Family shld b outside ER d)Active listening Ans:: c 22.Pt wth pulmonary t.b case ...change in urine nd one symptom given..which of following drug is responsible ? a) RIFAMPICIN b) ethambutol c) pyrazynamide d) inh Ans: A 23.Pt hve 1 week typhoid ? Which investigation wil do? a) urine b)stool c) blood cultue d)widal Ans: C 24.Neurons in Spinothalamic are not involved in ? a)temp b) pain, c) cordination Ans: C 25.Morphine used ? a)In terminal stage of cancer pt

b)appendicitis c) minor surgery Ans:: A 26) Trichnomus vaginalis?? a)STD b)blood borne Ans: A 27) Lateral rectus muscle ?question A) abducent nerve b) trochler c) optc Ans: A 28.Which of following is tetrogenic? a) coffee b) alcohol Ans: B 29.Grl wth type 1 diabetes hve mostly hyperglycemia in morning .whixh is appropriate option? a)regular insulin before meal shrt acting insulin b)twice b4 meal long acting insulin twice a day c)b4 meal long acting nd intermediate insulin twice a day d) intermediate insulin twice a day Ans: B 30..3 yr old child had fever ,flushy face nd tachycardia aftr eye drops instillation ...which drug cause above characteristics? a) atropine ) b)phenlnephrine related wth othr c)tropine trponide Ans: ;A 31)Fatty acid oxidation by which Vitamin a)Biotin b)vit d c) riboflavin Ans:; A 32..mechanism of action blockage of steroids ? A;Cyclocygenase B)Phospholipase A2 Ans: b ) 33.urine coming out of umblical vein A)Patent vitalline duct b) PDA c}malrotation d) bladder extrophy Ans: ;A

115 34.There was a fistula after 12 day of c sec between bladder and vagina ? A)vv fustula due to obstructed labour Ans:;a 35..women with edema till knees, lungs had pleural effusion,heart border visible,liver b kharab tha a) recurrent thrombophlebitis b) itp c) copartment syndome Ans: A 36..Tumour of unrinary baldder cause by.. A ) schistosomiasis b) trichomonas c) h pylori Ans:; A 37)Which hormone is responsible to change proto oncogene to oncogene.? ------------controversial bcq--------------------------------38( A scenario was like a man need blood trAns:fusion but no cross match with any type of blood. Which antigen is present in?? A) DUFFY B)kelly C) RH Ans: ; A 39) Standard deviation,,, A) Individual observation b)Variation among individual Ans: b ) 40) high cardiac output failure A) Thiamine deficiency b) vit c excess c) b12 def D).iron overload Ans: ;A 41)Area of gaseous exchange .... A)respiratory bronchiole till alveoli B ) bronchiole c} trachea Ans: ; A 42,,Which moves potassium out of cell... A) insulin b)salbutamol c)exercise Ans: ;c 43..Boy was busy in a well ventilated room temp 26 degree C , sweating.,bp was 150 /95,pulse 120. he was

a)Exercise b)in stress c) severe pain Ans: 'A 44.Superficial inguinal ring.. ..Ans:..in fascia of ext oblique 45) Microscopic characteristic feature of metastatic carcinoma??? a)Invasion b)Pleomorphism c) shape Ans: ; A 46..Post op wound infection aftr 72 hrs what will help neutrophlis in process of inflamation? a)C3b b)C5a c)Complements c5 to c9 d)Fc portion of IgG Ans:;C 47..Heparin prevents.. a)propagation of thrmbus b)clot formation c) coagulation Ans:;A 48..Hormone therapy given size of tumor reduced by the process of a) Atrophy b)Apoptosis Ans: B 49..Myxoid degeneration in? a)Mitral valve prolapse b) aortic regurgitation c) tricuspid stenosis Ans: ;A 50..After hepatic reaection which helps in regeneration ? A)Hepatocyte growth factor b)TGF Ans:A 52)Ek tha jis ka Ans: was a}caseating granuloma Ans: a 52) Glycolysis .. a)hmp shunt ......reversible Ans: A 53)Colon carcinoma metastasis first to a) mesocolic lymph node b)liver Ans:; B 54)Achondroplasia scenario

116 a) autosomal dominant b)autsomal recessasive c) not heridatary Ans: ; A 55) Cancer causing a)benzidine b) cooking oil Ans: A 56)Accomudation lost scenerio??? A) mid brain Ans: A 57)Left leg UMNL, LEFT face LMNL,left homonymous hemianopia,,lesion at a}Lat pons (aica) b)cerebellum Ans: A .. ref FA 58)Spleen feature A) accesory spleen in 25% b)12 cm normal size c)lax ligament cause wandering spleen Ans: B 59..Steroid like mechanism hormone a) glucagon b)insulin c) THYROXIN Ans: C 60)Uterus major support to A)PELVIC DIAPHRGM Ans: a 61)Unilateral orchidectomy Lymph will drain to A)bilateral paraaortic B)ipsilateral para aortic and C)ipsilateral pelvic Ans: A 62..Basilar artery thrombosis jis ma fainting comatose patient tha aik wo bhi A. locked in syndrome B . cerebellum Ans: A 63,,Adult female breast atrophy A ;estrogen estrogen b)progesterone Ans: A) 64) Breast lies in front of A)pect minor B)pect major fascia .Ans: B 65) Regarding esophagus A) t12

b) posterior surface of lower end has right vagus c) upper end has smooth muscle nonkeratinized d) stratified squamous epi (Ans: d ref kaplan) 66)Wilson disease???) Ans:>Ceruloplasmin level 67)Which Pleura supplied by phrenic only ??? A)Diaphragmatic parietal pluera B) viscreal Ans: A 68)Disease vs non disease A) nominal Ans: A 69) Elastic fibres with interlinked collagen fibers A A)dermis b)epidermis reticular layer Ans: B 70)Severe trAns:fusion reaction a) A+ to O+ b) o neg to b pos Ans: A 71)Good analgesic but not a good anesthetic A)nitrous oxide b)ketamine Ans: A 72)true about heart.. A)Great cardiac vein starts at interventricular sulcus b)small veins drain to rt atrium c)Other options not remembered Ans: B 73)One was about some peripheral smear spherocytes n polychromasia , A)G6pd b) IDA Ans: A 75..Blue cells catecholamine high abdominal mass< A) nephroblastoma b)Neuroblastoma Ans: A 76)Ca head of pancreas compress?? a)Portal Vein b) duodenum Ans: A 77)4th part of dudonum

117 A)Retroperitoneum b) intraperitoneal Ans: ..A 78)Caecum cancer feature A) keratin Ans: a 79)Qrs A) before ventricular systole B ) during sytole c) after systole Ans: A 80)wound on Toe lympyh nodes involved A)Vertcal inguinal group B )horizontal inguinal group Ans: ;A 81)Ak tha diagnostic test tb stem aur tha but matlab yhi tha😎 A) Afb b) blood culture c ) x ray Ans: A 82)A typical senario About pulmnry artry athetosclerosis. ((( An old aged pt having increased trAns:luency of lungs in upper fileds flattened diaphgrams no evidenece of infiltration prominent both pulmonary arteries and prominent right border of heart no hx of fever what is likely findings in pulmonary arteries? a)granulomatous vasculitis b)atherosclerosis c)aneurysm d)medial dissection Ans: B )) 83)Some defect with vision ct done occulsion of what artry ? A) central retinal artery b) optic Ans: A 84)Sub dural hematoma a).briding vein b)Superior cv c)Inf cv Ans: A 85)EFFECT ON CARDIAC CYCLE? A)HEART RATE CHANGE CHANGES LENGTH OF CYCLE Ans: A

86)ulceratd mass in cecum with tubular glads what is relevantMembr of family has adenomatus polip and aur option A) FAP b)crohns c) uc Ans: A 87)Friction rub on auscultation,creatitine 9 ,urea bhi raise tha kafi pericarditis ?? a)Uremic pericarditis> fibrinous Ans: ;A 88.Night secretion increases and inhibition of gnrh >a)tsh b)melatonin Ans: B 89)Pericarditis type in arf? A)Fibrinous (Ans: .uremic coverts to fibrinous) 90)Extra embryonic coelome A)Hypoblast Ans: A 91)Physiologic herniation > A) clockwise b) 4 6 week c) around celiac trunk Ans: A 92)Peristalsis of pharynx slow as compared to small intestine a) troposin vs calmodulin b) k vs ca Ans: A 93)Level of thyroxin altered by > A) TRH B) GH Ans: A 94)Thyroid epithelium > A)change with change in secretory function b) constant lifetime Ans: A 95)Farmer with cushing syndrome a)adrenal adenoma b) copd Ans: A 96)Virus alters > a)Dna b) RNA C) gene Ans: A 97)Kidney activates> A)cholicalciferol b) ACE

118 c) riboflavin Ans: ;a 98)Meningitis after lung abscess > a)staphylococcus aures b) s pneumona Ans: A 99)Severe dehydration A) nacl 0.9 % B) 5 % dextrose c) hemacell Ans: ref Ganong Bcq 100)Nasopharyngeal CA a)rhinovirus b)adenovirus c) ebv Ans: C 101)Receptor drug effect > A)bradycardia causing atenolol B) hyperglycemia due to insulin Ans: A 102)Abt parasympathetic A) All parasympthtc post all muscurinic b)all sympathetic post all muscurinic Ans: A 103}Sarcomere A) Z band b) i band Ans: a • 104)Intraorbital • a) br of maxillary • Ans: A • 105)Atonic bladder > • a) parasympathetic efferent damage • b) symp out flow • Ans: A 106)Filiform > A)cover whole dorsum of tongue Ans: A 107)rubella in pregnancy?? A) cataract b) river blindness Ans: A 108)Patient with acute hepatitis trAns:fused with 1 pint cross matched,,after week trAns:fused again with 1 pint cross matched Week later present with HB 7,coombs positive,reticulocyte 10% A) drug induced hemolysis B ) delayed hemolytic trAns:fusion reaction Ans: A

109)Difference BT interstitium n cell tha kuch A) I marked 1 mosm Ans: 110)One was about cecum adenocarcinoma? A)K-RAS Mutation Ans: A 111)Black water fever wala scenario > A)falciparum b) ovale c) plasmodium malaria Ans: 'A 112)G6pd A) Heinz body Ans: A 113)Hco3 calculation > A)urine flow × hco3 Ans: A 114)TrAns:fer of radicals> A)trAns:ferase b) mutases Ans: A 115)Yolk sac tumor >> A) AFP raised b) b hcg raised Ans: A 116)Portal hepatis tumor > Ans:>portal vein obstruction 117)Icam vcam > a)leucocyte adhesion b) RBC adhesion Ans: A 118) Child coronary aneurysm > A)kawasaki b) wegner Ans: A 119) Median atlanto axial joint? a)pivot Ans: a 120)Colon carcinoma > A) kras b) afp Ans: A 121)Inversion> A)tibialis ant n post Ans: A 122)Gaba wala substantial nigra damage >A)chorea b) rigidity Ans: A 123)Chromatid allign<

119 a) metaphase b) anaphase c)telophase Ans: A 124)One was hx of family bleeding bt raisd and aptt raisd A) .vwd b) vit k def c)thalasemia Ans: A 125).nerve danage in tonsilectomy a ) 8th b) 11 c) 9 nerve Ans: C 126) Anorectal reflex > a)defecation b) urination Ans: a 127)Thiamine cause A)Subacute degeneration of neurons b) choreoathetosis Ans: a 128)Defecation reflex< a)rectoanal Ans: A 129) Asthma a)fev/fvc more than 75 b)fev/fvc less than 75 Ans: B 130)oral ulcers > a)candidiasis b) ebv c) kissing disease Ans: A 131)Kidney difference a)structure in hilum b) arteries Ans: a 132) Lateral thenar spared, short muscles of hand paralysed a) radial b)long thoracic c) ulner n damaged Ans: C 133) Overhead abduction > a)subscapular nerve b) infraspinatous Ans: A 134)LONG THORACIC NERVE supplies A)SERRATUS ANTERIOR

b)serratous pos Ans: A 135.Cervical facial > a) platysma b)stylohyoid Ans: A 136.Left side fall > a)left cerebellum b) right hemisphere Ans: A 137.Medullary pyramid lesion A) propioception B ) taste Ans:. a 138.. 51 years old female having iron def anemia and esophageal carcinoma... wheres lymph drainage ?? A)celiac nodes.. B)deep cervical Ans: B{upper 1/3 esophagus} 139..old man admitted in oncology ward has ear ache and fever pata nae doosra b kuch infection hua tha head n neck region ka.. culture revealed pseudomonas aeruginosa and cbc neutropenia.. this is because of. . a IL5, b.TNF c. granulocyte stimulating factor d. granulocyte macrophage factor Ans: B140)Microscopic feature of compact bones a)Haversian canals are arranged obliquely b) lacunae have osteoblasts c)Haversian canals are not interconnected collagen imparts colour d).lamellae are regularly arranged Ans: d 142.. Food entring in larynx prevented by A)Pharyngeal muscle contraction b)Approximation of vocal cords Ans: B 143..study on sexually trAns:mitted disease,one group of boys had penile ulcers and the other group had no ulcers,non ulcerating leisons were d/t? A.treponema paliidum B..chlamydia C.HPV D.ducreyi E.Gonorhea

120 Ans:; B 144..Regarding physiological herniation A)On IMA axis B)Clockwise C)Herniates in extra embryonic celoem DMidgut connected to allantois Ans: B 145.Infra orbital artery is terminal branch of a)Facial b),Maxillary c)TrAns:verse facial Ans: B 146.Anterior surface of duodenum perforate, fluid will accumulated in? a) Right iliac fossa b)Right posterior subphrenic space Ans: A 147.Testicular ca drains into a)Ipsilateral pevic and contralateral p.a nodes b).lateral pelvis c)Ipsilateral p.a nodes d)Bilateral paraaortic nodes Ans: D 148)Femoral pulsation cab be feel at a)Mid inguinal point b).Mid point of inguinal ligament Ans: A 149)Regarding Sephanous vein A)Ascend in front of medial malleolus b)Pierce s.f faci a c)Connect with deep veins of calf d)Start from medial venous arch Ans: A 151)One q characteristics feture of well def sq cell ca A)Keratin perls b)Pleomorphism c)Polarity Ans: A 152.Regarding heart supply A)great cardiac vein start from apex run in interventricular grove Ans: A 153..Which harmone released at night cause block of gnrh A)Melatonin B) tsh Ans: A

154)Anterior compartment syndrome of lower limb nerve involved A) common perronal b)femorral c) sciatic Ans: A 155) Spleen • A)12 cm • b)accessory spleen in 25 % • Ans: A 156) Left leg UMNL,left face LMNL,left homonymous hemianopia area damaged ?? • a)cerebrum • b).lateral pons • c)medulla • Ans: b 157) Steroid acts on?? Ans:> via Gene 158) Post triangle biopsy> a)spinal acessry b) long thoracic c) 12th Ans: A 159)Carcinogenic> a)benzidine b) alcohol Ans: A 160)uterus main support > a)trAns:verse cervical b)broad lig Ans: A 161)River blindness A)Oncoceriosis b) schistosomiasis Ans: a 162)Microscopic feature Tb lymph node > A)caseous necrosis Ans: A 163)Factor 8 > a)cryoprecipitate b) ffp Ans: A 164.Immunologic test related question A)Amobic liver disease b)Hydatid cyst Ans: a 165.Human is intermediate host of. A) hydatid b) malaria Ans: A

121 166.Increase hydrostatic pressure related question...... a).Constriction afferent arteriole b)Constriction efferent arteriole Ans: B 167)Which is pivot joint A)Atlantoaxial joint b) pubic Ans: A 168)C7 vertebra specific feature A)Longest Spine b) body Ans: A 169)Decrease osmotic pressure . ......infusion a)Isotonic saline b)20% albumin solution c) .Distilwater d)10% glucose Ans: c[ distalled water is istonic bec no salute) 170.Person come to pakistan and develop fever from 2 days and become unconcious a)P .falciperum b) p ovale Ans: A 171.athlete has infection of big toe..there is tenderness in groin.. which nodes involve. a. seperficial inguinal horizental. b..Ans: superficial inguinal vertical.. .c. deep inguinal d.. node of cloquet Ans: B 172.Acute renal failure related question which part of tubule involve..... a) PCT b) Dct c) Loop of Henle Ans: A 173)Wound strengthing collegen Type a) type 1 b.type 2 c.type3 d.type4 Ans: A 175.Imprtnt regarding wound... aCollagen points a)Inc wound tensile strength.... Collagen I b)Imprtnt in wound healing III

c) Deposited first in wound healing... Collagen III d)Collagen I takes 3 weeks or more 176 Right border of heart?? Ans:> Right atrium 177.l-dopa inhibit .. a)prolectin b)oxytocin Ans: A 179..Primary perstalsis different from secondry in a)Orophargeal phase Ans: a 180..Virus in cell cause alter....... A)DNA b)Oncogen Ans: A 181.60 year gardner present with caushing syndrom a.renal cell carcinoma b.adrenal adenoma c.Thyroid carcinoma Ans:wer B 183.Heanz body presnt in • a)hereditry spherocytosis • b)sickle cell • c).G6pd • Ans: C 184.Methicilin resistance staph aureus sensitive to .a).vancomycin b) gentamycin c)lincomycin Ans: A 185..Mineralocorticoid....Adlosteron secretion stimulate raise ..related question tha A)Potassium b)Renin Ans: A 186..Staff nurse and hospital acquired infection related question.. .A. Left Gloves on floor after blood sample B.) Not hand washing after glove removed Ans: B 187..superior parathyroid lies superficial to .A).pretracheal fascia Ans: A 188..CSF related question a) Drain arachnoid villi b) Absorb choroid plexus Ans: A

122 189,filiform papillae of tongue a.. mushroom like b imparts red colour to tongue c.. mostly present on tip Ans: a

8TH NOVEMBER 2016 SURGERY PAPER -1 AND 2 MORNING 1) Pretracheal fascia completely covers a-Thyroid b-Trachea c-Esophagus d-Laynx e-Pharynx Ans:A(ref snell) 2) Long scenerio of fibroadenoma showing features of firm fibrous mass. Gross appearance of fibroadenoma a-Metaplasia b-Dysplasia c-Desmoplasia d-Aplasia -Anaplasia (It is a feature of Carcinoma) Ans:C 3) Proto oncogenes converted to oncogenes a-Gene amplification b-Point mutation (ans:B ref robbins) 4) Tumour Metastasis: A. migration B. degradation of extracellular matrix C.degradation of "E-cadherin" D. loss of attachment of tumor cells with each other Ans:c 5) Sertoli cells-remove blood testis barrier (exact words) a-produce androgen binding protein b-secrete testosterone Ans:A ref first aid 6) Regarding parotid gland a-contain serous acini B- duct opens in the second molar tooth(oposite upper 2nd molar tooth) C-pierces masseter(pass over messater pierces buccinator) Ans:A ref first aid+klm

7) Histology slide shows rete ridges with nuclear atypia and nuclear pleomorphism with infiltrating malignat epithelial cells a-Verroceous Carcinoma b-Squamous Carcinoma c-Transitional Cell carcinom Ans;B ref robbins 8) Three days after delivery a women died because of pulmonary thrombo embolism. There was blockage at pulmonary artery which result in a-V/Q ratio achieve infinity b-V/Q ratio becomes zero(shunt) c-V/Q ratio is unchanged d-Shunting of the alveoli occurs Ans:B ref brs 9)A substance freely filtered and actively reabsorbd by the nephron a-Clearance will be increased in prescence of metabolic poison -b-Clearance will be decreasd in presence of metabolic poison c-It's concentration in urine is more than plasma d-It's concentration in urine is less than plasma Ans:D 10) Low grade fever with cervical lymphadenopathy. Histology shows diffuse infiltrating macrophages with spaces and increased mitotic index a-Tuberculosis b-Burkitt lymphoma c-Lymphocytic Lymphoma d-Sarcoidosis Ans:B ref first aid 11) 24 years old male presents with fever and weakness. Non-lactose fermenting organism isolated with greenish metallic colonies -a-Pseudomonas Aeuroginosa(answer)first aid b-Staphlococcus Aeureus c-Streptococcus d-Klebsiella Ans:A ref first aid 12) Histolgy slide shows circular and longitudional arrangement with skeletal chaned into smooth muscle fibres a-Upper esophagus b-Middle esophagus

123 c-Pharynx d-Palatine tonsil e-Lower esophagus Ans:B 12) Patient presents with signs of low grade fever, non- healing ulcer on tongue and cervical lymphadenopathy. Biopsy shows epitheliod cells with casseous necrosis a-Tuberculosis b-Sarcoidosis c-Burkit Lymphoma d-Lymphocytic Lymphoma Ans:A ref robbins and first aid 13) The functional residual capacity ??? a is the sum of residual volume and inspiratory reserve volume b. Volume of air in the lungs at the end of tidal expiration c. Vital Capacity + Expiratory Reserve Volume d. Expiratory capacity +tidal volume e. Can be measured by spirometry Ans:B ref brs 14) Scenerio of Hemorrhage , patient was transfused with 150mmol/L Normal Saline. Most likely change occurs a-Inc in ECF Na level b-Inc in intracellular volume c-Inc Urinary osmolarity d-Inc ICF Osmolalite Ans:B 15)28 years lady presented with vaginal bleeding and show signs of cervical hyperplasia a-Inc estrogenic stimulation b-Dec estrogenic stimulation c-Inc leutinizing Hormone Ans:A 16)Potassium secretion decreases in -Inc dietary potassium(low k diet) -Inc aldosterone secretion(true) -Decrease Na absorption Ans:B 17)Sodium is primarily regulated via a-Osmoreceptors b-Aldosterone(reabsorption) c-Renin Ans:A 18)Which of the following has the capacity of regeneration

a-Mitochondria b-Golgi Bodies c-Endoplasmic Reticulum d-Nucleolus Ans:A ref guyton 19)Which of the following is absent from the axon a-Mitochondria b-Nissil Bodies c-Microfilament d-Microvilli Ans:B ref kaplan 20) ECF and ICF a-Na and Ca inc in ECF as compared to ICF b-ECF more than ICF c-ECF has high K and Na concentration Ans:A ref guyton+brs 21)Blood Group A+ receipient cross matched blood group with every other blood group. Cross matching failed due to the presence of antivodies a-Anti-M b-Anti e c-Anti gG d-Kell e-Duffy Ans:E 22)Nerve damage resulting in loss of sensations in lateral three and a half finger. a-Ulnar b-Median c-Musculocutaneous d-Accessory e-Axillary Ans:B ref snell Note:If dorsum of hand and lat 3 and ½ fingers than choose superficial branch of radial nerve. 23) Greater tuberosity of humerus a-Medial side of shoulder b-Short head of bicep arise from it c-Attachment of Sub-scapularis d- Embedded beneath Teres major e-tere minor attached at inferior facet of greater tubercle Ans:E ref snell 24)Girl with type 1 diabetes hve mostly hyperglycemia in morning .which is appropriate option?

124 a-regular insulin before meal B- shrt acting insulin twice b4 meal c- long acting insulin twice a day b4 meal D-long acting and intermediate insulin twice a day E-intermediate insulin twice a day Ans:D 25) Breast Carcinoma undergo surgery with axillary clearance done during surgery. Winging of scapula scenario due to surgery complication. Nerve damage a-Thoracodorsal b-Long Thoracic c-Dorsal Scapular d-Sub-scapular Ans: B ref Snell 26)Sciatic nerve severed, dorsum of foot supplied by which nerve?? a.saphenous nerve b.Sural nerve c.Common peroneal nerve Ans:A ref snell 27)After hemorrhage which of the following is the last to return to normal a-RBC in peripheral smear b-Hemoglobin c-Reticulocytes Ans:A 28) Circle of Willis is formed by A-Vertebral artery and Middle cerebral B-2 internal carotid and Basilar artery(answer) c-Basilar artery with external Carotid D-Anterior cerbellar with Posterior Communicating artery E-Between 2 vetebral arteries and internal carotid arteries Ans:B ref kaplan ==>I check again and again Posterior cerebral was not in the option. 29)Vitamin C deficiency A-Defective Collagen Formation B-Decreased Collagen Formation C-Fibrillin/Collagen ratio deranged Ans:A ref lippncott 30)Fetal movement detected by mother in which month A-6 B-7 C-5

D-4 E-8 Ans:C 31)35 year old man bilateral crepts, on xray chest heart fills completely ,mass in RT ventricle , What will be the main cause a.mesothelioma b.rhabdomyosarcoma c.angiosarcoma d.myxoma Ans:D ref cmdt 32) Organism associated with bladder squamous cell carcinoma A-Schistosoma hematobium B-Schistosoma Mansoni Ans:A ref robbins +cmdt 33)26 years old boy presents with pallor and weakness. Fever of 101°F. Echymosis and Bruises were there. Patechial leision was mentioned. Definitive diagnosis via investigation ? Options were A-Serology B-CBC C-Bone marrow aspiration D-Molecular biology Ans:B 34) Infratemporal fossa is also known as Ans> #Parapharyngeal #Space 35) Hematopoetic function of endothelium?? A-machanical barrier B) synthesis of fibronectin C) synthesis of anti plasminogen inhibitor factor D) synthesis of prostacycline Ans:D 36) 62 years old lady presents with backache. 30% monoclonal plasma cells with features of joint pain. Most likely diagnosis A-Multiple Myeloma B-Waldernstorm -C-Monoclonal gammopathy D-Plasmacytome Ans:A ref robbins and cmdt 37) Extensive Lesions in the cerebrum with involvement of brocas and wernicke area -A-Global Aphasia -B-Transcortical Aphasia

125 C-Sensory-Motor aphasia ANS:A ref kaplan 38)A patient presents with type 2 diabetes mellitus. Most likely complication in limb will be A-Dry Gangrene B-Wet Gangrene C-Raynaud's Phenomenon Ans:B Diabetes=Dry Diabetes with complication=Wet 39)Absolute contraindication of propofol :-A-Hypersensitivity reaction -B- Cardiac problems - c-Porphyria D-Pregnancy Ans:A ref katzan 39)Xylocaine given to a patient with multiple drug allergy. Sign of increase sensitivity? a Rash. b Inc blood pressure c Rise in temp d Syncope e Tacchycardia Ans:D ref lippncott 40)Regarding neutrophil, correct statement is A. Migrate into and out of the blood B. Phagocytic in bloodstream C. Decrease in infective conditions D. Decrease when corticosteroids were given Ans:B ref guyton 41)Rt kidney differentiated from lft kidney by..? A. Relation of structure within hilum B. Surface and pole Ans:A ref snell Rest of the Options were changed but A is the most appropriate one ! 42)Adrenal gland A -has same size in adult n infants B- arterial supply from 2 arteries C -pyramidal in shape d -has seperated from kidney by perinephric fat. Ans:D ref snell Question was only about adrenal gland. Right adrenal gland is pyramidal in shape,,

left is crescent. Supplied by three arteries and one vein. Only suitable option is D. 43) A patient presents in E.R with a hemorrhage. Blood loss (500m/l). Compensatory mechanisms were activated. Which will decrease after compensation A-HeartRate B-Venous Capacitance C-Urine Output Ans:A What inc and what decreases( Decided on group) :Hemorrhage During compensation 1)venous capacitance decrease 2)urine output decreases 3) h.rate increases due to sympathetics. After compensation 1)h.rate decreases and returms to normal. 2)urinary output increases and returns to normal 3)v.capacitance increases and returms to normal. 44) A patient had difficulty in abducting and adducting his fingers and has lost abduction of his thumb. The nerve most likely involve A)Sup. Branch of Ulnar Nerve B)Deep. Branch of Ulnar Nerve C)Dorsal Branch of Ulnar Nerve D)Median Nerve E)Anterior Cutaneous branch Ans:B Rabia ali Page 30 MCQ No 12 with little changed option. 45)Non-Pregnant , 26 years lady presented with FBS:122mg/dl, After one hour 198mg/dl, After two hours 194mg/dl. Most likely A-Normal Glucose Toleramce B-Overt Diabetes C-Secondary Diabetes D-Gestational Diabetes E-Lag storage curve Ans:B 46) No of somite at 30th day A’25-30 B-32-34 C-34-35 D-42-44 E-44-46

126 Ans:A (21-29 somites on 30th day ref keet moor embryo) 47)Which one is essential amino acid? {A} serine B} tyrosine C} Phenylalanine D} proline Ans:C ref lippncott 48)Best example of pharmacodynamics drug drug interaction: a: tetracycline with calcium b: toxicity of lithium with thiazide diuretics c:reverse the action of aspirin by NAHCO3 d:reverse the action of morphine by naloxone Ans:D ref lippncott 49)Regarding the cardiac reserve A. Decreases in atheletes B. Increases in vitamins deficiency C. Is increased in ischemic heart disease D. Is the maximum percentage by which cardiac output can be increased above normal. Ans:D 50)Individuals have the independent and same chance of being selected: Asimple random sampling B.stratified sampling C.systemic sampling Ans:A past paper bcq 51)Fatty acid oxidation is helped by? -A-Biotin B-vit c C-vit D D-vit B12 Ans:A ref lipncott 52)Most likely about PONS -A-Facial colliculus formed by facial nerve nucleus B-nuclei of pons in on ventrolateral area C-spinal leminiscus originates from tegmemtum _Poor ReCALL 53) If the whole movement of abduction of arm is lost, the most likely damage part of brachial plexus will be A-Upper trunk and posterior cord B-Upper trunk and Medial Cord C-Lower trunk and medial cord D-Lower Trunk and Posterior cord

ANS:A ref snell Rabia ali mcq no 23 page 31 54) Increased bleeding time scenerio. Von willebrand disease. 55)Most common cause of inc bleeding time -Aspirin ingestion(past paper bcq) 56)Bundle of HIS. Blood supply A-Right marginal branch of LCA B-LCA C-Interventricular branch of LCA D-Right Marginal branch of RCA E-RCA Ans:E ref snell 57) Patient on ATT. Develop narrow vision amd difficulty in night vision. Drug responsible A-Ethambutol B-INH C-Rifampicin D-Diazepam Ans:A ref cmdt 58) 3 years old child presents with rash, facial flushing and tacchycardia after eye drop instillation. Fever of 101°C . Drug responsible A-Atropine B-Homoatropine C-Tropicamide D-Pilocarpine E-Hyoscine Ans:C Tropicamide eye Drops causes rash, facial flushing and tachycardia. Atropine eye drops causes blurred vision, eye pain and stinging amd hypotension. 59)A nursing mother presents with herpes labialis. Drug of choice A-Acyclovir B-Triflouridine C-Ranitidine D-Amantadine Ans:A 60)Most likely feature present in spinothalamic tract but absent in dorsal column medial leminiscus -Long receptive field A- sensory rapidly adatping or slow adapting but never both B-cam supply 1 or more than 1 limbs It’s a poor recall.

127 61)Resection of ileum absorption effected A-Zinc B-Iron -C-Vit b12 Ans:C past paper bc1 62)At the start of fifth week A-Stomach rotates B-Limb formation occurs C-Definitive heart is formed D-Ventral mesentry ka koi chakar tha :-P Ans:A 63)Left sixth aortic arch forms A-Ligamentum venosum B-Ligamentum arteriosum C-Subclavin artery -D#None of the above Ans:B ref first aid 64) Drug absorption in gut is increased by -A-Inc gut motility B-ionized state of drug C-Lipid solubility(lippncott) Ans:C 65) Ear ossicles are the example of A-Flat bone B-Irregular bones C-Sesamoid Bones D-small bones Ans:D ref snell 66) Infant's joint between two mandible is A-Gomphosis B-syndosmosis C-synchondrosis D-symphysis Ans:D ref snell 67) 42 years old women with metaplasia cervix. Organism responsible A-HSV-2 B-HPV C-Trichomonas Vaginalis D-Gonorrhea Ans:B ref snell 68) Most rapidly adapting receptors A-Touch B-Alph-1 Ans:A 69) 45 years old female with biliary colic, rash and cholelithiasis. Enzyme most likely raised AAlanine aminotransferase B-Glutamate Dehydrogenase CLDH

D-Aspartate Aminotransferase E-Alkaline Phosphatase Ans:E 70)Most likely in Sarcoidosis A-Erythema Nodosum B-Hypercalcemia C-Hepatic granuloma D-Pleural effusion Ans:B ref cmdt Erythema nodosum only 7% 71)Bronchogenic Crcinoma. Most likely cause -Benzidine -Tobacco(robbins+cmdt) -Asbestosis 71)Sympathetics via alpha adrenergic effect A-Vasodilation B-Bronchodilation C-Pupillary dilation D-lipolysis Ans:C ref brs n kaplan 72)A six year old girl with history of fever and sorethroat. Cultures reveal tiny colonies with complete hemolysis on blood sheap agar plate. Organism responsible A-Beta hemolytic group A Streptococcus(strept.pyogens:bacitracin sensitive) B-Staphloccus Aureus C-Bordetella D-Klebsiella E-Corynebacterium diphtheria Ans:A ref first aid 73)Normal angle is 125° with shaft in lower limb. In Cox Vera A-Head Neck Shaft Angle increases B-Head Neck Shaft Angle decreases C-Head Neck Shaft remain same D-Head of femur displaced laterally Ans:B Cox vera:angle dec 《120 74) Patient unable to turn eye laterally. Most likely cause A-Infection of right cavernous sinus B-Tumour of left cavernous sinus C-tumour of pituitary gland D-tumour of anterior cranial fossa Ans:A 75)The characteristic lesion in primary hereditary telengiectasia is on

128 A-Palate B-gingivae C-Lips D-Nose E-Tongue Ans: C (ref cmdt ref taken from a dentistry mcq book. Key was LIPS there). 76) I/V drug abuser most likely develop A-Infective endocarditis B-Marantic Endocarditis C-Libman Sac Endocarditis -D-Rheumatic Disease Ans:A ref cmdt as marantic occurs in cancer patients and limbman in sle 77)Most common nerve injury in lower limb in anesthesized patient A-Ant. Tibial B-Saphenousnerve C-Common Peroneal Nerve D-Femoral E-Obturator Ans:C ref snell 78)Sensory supply of uterus travels in A-Broad Ligament B-Round Ligament C-Uterosacral Ligament D-Tranverse Cervical Ligament Ans:A ref snell 79)62 years old female with change in bowel habits. DRE negative and sigmoidoscopy shows 6cm mass near caecum. Diagnosed by pathologist as Adenocarcinoma. Most likely -K-RAS with neoplastic cells(cmdt 2016) This is true as K-RAS and B-raf associated with CA Colon. H-RAS with Bladder Cancer. 80) Slow growing and longstanding tumour of parotid gland A-Pleomorphic adenoma B-Warthin C-Mucoepidermoid tumour D-Mucous Cystic adenocarcinoma Ans:cmdt 81)Regarding urinary bladder A-Sympathetic causes relaxation of bladder muscle wall(brs +lippncot) B-Parasympathetic causes relaxation of internal sphincter(lippmcott+brs) C-Pundendal nerve relaxes external sphinter

D-atonic bladder result from filling sensation Ans:both A and B correct 82) Subcostal incision. Rectus abdominis not necrosed even after damage of superior epigastric artery because it anastamoses with A-deep circumflex iliac B-inf. Epigastric artery C-subcostal artery D-external pudendal artery Ans:B ref snell 83)Which hormone is responsible for brain maturation of fetus?? A-Thyroxine B-Growth Hormone C-Cortisol Ans:A 84) Which hormone stores carbohydrates and protein?? A-Insulin B-Cortisol Ans:A 85) Biopsy of endocervix shows squamous epithelium. this is called. A Dysplasia b. Metaplasia c Anaplasis Ans:B ref robbins 86) Maximal level of urinary 17 ketosteroids is presnt in??? -normal male(ans) 87)Ovarian cancer will first metastas to whch lymph node?? Para-aortic Lymph Node(snell+past paper bcq) 88)Shoulder pain referred due to?? c3 c4 c5(ans:snell) 89)A patient with T4N1M0 with a very long stem(long stem was to distract) they wanna ask wat else could be seen... A. Cardiac murmur B. Cachexia Ans:B past paper bcq 90) Uterine artery branch of illiac artery crosses which structure A-Ureter B-Kidney Ans:A past paper bcq 91) Head of pancreas lie anteriorly to A-Inferior Vena Cava

129 B-Aorta C-Sup. Mesenteric Artery D-Inferior Mesenteric Artery Ans:A ref snell 92)Regarding Cardiac Cycle A- P wave is before ventricular contraction(p for atrial depolarization:ref first aid) B- Second heart sound corresponds to t wave C- first heart is during iso-volumetric contraction(ref brs) Ans:Both A and C appears to be true. 93) 30 years old female, with fever 101 F , with pus cells in urine, ct kidney showed parenchymal changes , diagnosis A. Acute tubular necrosis B. Necrotizing C. Chronic pyelonrphritis D. Acute on chronic Ans:C 94) 30 years old female, with fever 101 F, fever unresponsiveness for weeks with pus cells in urine, ct kidney showed parenchymal changes, , diagnosis A. Acute tubular necrosis B. Necrotizing nephritis C. Chronic nephritis D. Chronic acute nepritis Ans:D 95)Aldosterone secretion increasd by : A-hypokalemia B-hyperiosmolality C-dehydration D-Ace inhibitors E. Dec plasma volume Ans:E 96)Anterior Abdominal Wall between subcostal and umbilicus A-External oblique and anterior layer of internal obliquE B-Anterior layer of internal oblique with transversalis muscle C-External oblique aponeurosis D-Outer fibres of external oblique Ans:A ref keet moor 97)Regarding gall bladder Has squamous epithelium Fundus lies at edge of 6th costal cartilage Completely covered by peritoneum

Supplied by cystic artery, branch of right hepatic artery(snell) 98) regarding small intestine A. jejunum thin wall n is longest B drain by both celiac n sup mesenteric c. Small intestine is suplied by both sma and ima d. none of it is retroperitoneal Ans:B ref snell 99)Myocardial Ischemia A-contraction band(12-24 hours)first aid B-Clumping of nuclear chromatin Ans:A》B 100)RBC have A-Biconvex B-Glycolytic activity Ans:B guyton 101)peroxidation of lipids A amyloid b lipofuscin Ans: B 102)Recurrent laryngeal nerve passes between A-Cricoid and thyroid B-Thyroid and hyoid Ans: B 103) Relation of common peroneal nerve with biceps femoris in politeal fossa A.anteroir B-posterior C- Medial D- Lateral(to peroneus longus) Ans:C 104)study presented in 2×2 table Chi test(past paper bcq) 105) Facial nerve enters into mastoid part of temporal bone via A-Stylomastoid foramen B-carotid canal C-petrous temoral bone D-foramen spinosum E-Internal acoustic meatus Ans:E 106)Most appropriate about hyperacute rejection A-occurs within minutes B-by preformed cytotoxic T cells C-treated with immunoglobulin Ans:A 107)Enzyme used in break down of non epinephrine and serotonin secretion

130 1.MAO 2.COMT 3 dopamine Ans:A ref lipncott 108) Right Coronary artery blockage after giving marginal artery. Which is affected A-AV node B-SA node C-Purkinjee fibres D-IV septum Ans:A past paper bcq 109) sub mucosal glands in. ileum. A-duedonum B-jejenum. C-Stomach Ans:A past paper bcq+snell 110)8 years old boy with difficulty in swallowing and mass on left side of esophagus. Desmin negative and Vimentin + . He had a history of fetal alcohol syndrome and not doing well in school. Most likely A-Wilms tumour B--Rhabdomyosarcoma C-Metaplastic Lymphoma D-Retinoblastoma Ans:B Rhabdomyosarcoma is desmin and vimentin positive, associated with fetal alcohol syndrome but mass in the neck is controversial. Wilms tumour is desmin negative and vimentin positive but it is common in 2-5 years of age and is a renal tumour. Nothing wrong in the stem and options. 111)Rearding seminal vesicles A- Storage of sperms B-opens with ejaculatory duct into prostatic urethra C-Anterolateral to ureter Ans:A snell 112)Property of cardiac muscle that prevent genesis of A-tetanisation B-Refractory Period Ans:A its automaticity 113) Subcostal incision. Rectus abdominis not necrosed even after damage of superior epigastric artery because it

anastamoses with A-deep circumflex iliac B-inf. Epigastric artery C-subcostal artery D-external pudendal artery Ans:B 114) Iritant receptor in airway A-cause coughing and broncho-constriction B-among epiyhelium -rapidly adapting C-Only response to chemical irritants Ans:A 115)Abdominal angina likely cause of narrowing Superior Mesenteric artery(snell:past paper bcq) 116)Removal of gastric antrum, what happens A-Decrease emptying time for solids B-Dec gastric compliance Ans:B Discussed on group Remove fundus- Decrease receptive Relaxation Remove Gastric Antrum- Dec gastrin release and decrease gcompliance Remove Pylorus- Inc emptying time for solids 117) intercostal muscle supplied by :ventral rami.(snell) 118)Trichmonas vaginalis :is sexually transmitted.(first aid) 119)Maximum reabsorbed from renal tubule to plasma A-Glucose B-Sodium C-Potassium Ans:A ref brs 120) Free nerve ending A- encapsulated B-un-encapsulated Ans:B ref kaplan) 121) fecal oral route sy spreading. hep E* (Hep A not in options) 122)Chorda tympani nerve a- AutoNomic function(preganglionic parasympathetic secretomotor fibers for submandibular and salivary gland) b- post 1/3 rd of tongue(ant 2/3rd) c- pain fibers from middle ear d- supplies stapedius

131 Ans:A ref snell 123) Medical meniscus of knee attachement A-Medial collateral ligament B-Anterior cruciate Ligament C- Posterior Cruciate Ligament Ans:A ref snell 124)Supinator alternative or secondary role A-Bicp Brachii B-Brachialis c-Extensor Carpi Ulnaris (Brachioradialis was not in the options) Ans:A ref snell 125)Widdrawal of cerebrospinal fluid needle moves in through Epidural-dura-sub dural-arachnoid(ans) 126) About polycythemia rubra vera A-Person goes to himalyas B- Cancer of lung. C- myloproliferative disorders Ans: C ref cmdt 127)Septic meningitis A-chlymidia. B-streptococcus Ans:B ref first aid) 128)Posterior inferior cerebellar artery blockage. Part of brain affected A-Dorsolateral part of medulla B-Lateral area of Pons Ans:A ref kaplan 129)Retrograde amnesia A:Trauma or blow to head B:Damage to limbic system C:Can be reduced by dopamine agonist Ans: B? Not sure 130)Surgically lobes of breast Ans:Separated by interlobar septa/stroma 131) type of collegn in dermis of skin. A-1 B-2 C-3 D-4 Ans:A ref first aid 132) most imp buffer of body. A-phosphate. B-protein C-hco3 Ans:C ref cmdt 133)Tunica Vaginalis is derived from A-External Oblique aponeurosis

B-Internal Oblique and Transversalis Fascia C-Peritoneum Ans:C ref :(snell:it is expanded part of process vaginalos which is detived from peritoneal folds) 134) Aldosterone is increasd by A:Hypokalemia B: Increasd plasma osmolarity C.low blood volume Ans:C 135)Left recurrent laryngeal nerve Ans:-Branch of Vagus Nerve(snell) 136)Diarrhea -Alkalosis(it causes acidosis) -increased Sodium(cmdt) 137)DELTA ANTIGEN... a -acute resolvinng hep B- previous infection of hep C- chronic hep Ans:C ref first aid) 138) 36 Years old woman with high grade fever and chills. Prescribed anti-malarial presents with cola coloured urine. Hb 6mg/dl. A single test was done and was diagnosed as A-G6PD B-Malaria Falciparum Ans:A past paper bcq 139)45 yr old basiines man travelng far east complaing of abdominal discomfort. Tip of spleen palpable hilar lymphadenopathy and caseating granuloma in lungs Ans-Wegners Granulomatosis 140) A young boy presents in OPD with cholesterol deposits (something like that) on extensor surfaces of arms. Serum cholesterol was 800. Family history of same problem was also present. Mutations in gene of which protein involved in cholesterol METABOLISM A-HDL B-LDL C-Chylomicron Ans:B ref first aid type 2 familial hypercholestrolemia 141)Correct sequence of events ? A. Damaged valve, thrombus , perforation , emboli B. Damaged valve , perforation , thrombus , bacteremia

132 C. Damaged valve , bacteremia , thrombus , perforation D. Bacteremia , thrombus , perforation , emboli Ans:A 142) LCX blocked. M.I in affected area A- ant surface of left venticl B- ant surface of right ventrical C- posterior surface of left ventricle d-anterior 2/3 of IV septum Ans:C ref snell lat and post wall of left ventricle 143) perinephric fat and kidney is enclosed by. a.triangular ligament. b.gillsons fascia. c.gerotas fascia d.peritoneum. e.coronary ligament. Ans:C ref snell gerotas fascia is called renal fascia 144)350 plasma osmolality, urine 1250 A-SIADH B-Water Deprivation C-Diabetes Insipidus D-Central Diabetes Insipidus E-dehydration Ans:A ref cmdt 145)Surfactant secreted from A-Type 2 pnemocytes B-Type 1 pneumocytes Ans:A ref guyton+brs+first aid 146) Left 6th Aortic arch derivative. A-hemizygous vein B-Ductus venousum C-Ductus arteriosum D-coronary sinus E-none Ans:C 147)A+ blood group. Transfusion reaction will occur with which blood group A-AbB-A+ C-AD-O+ E-OAns:A past paper bcq 148)Skin disinfection -Wash hands with alcohol plus chlorhexidine(past paper bcq)

149) Temperature regulatory centre is present in A-Thalamus B-Hypothalamus C-Basal Ganglia Ans:B ref kaplan 150- Regarding thyroid a)parathyroid lies posterior to anterior b)epithelium changes shape acording to function c)collecting ducts are present in gland epithelium Ans:B past paper bcq 151) Anemia in pregnancy is due to: hemodilution(past paper bcq) 152)In Emergency A-Only patient should stay, family outside B-Physchosocial support to the patient and family plays an imp role Ans:B 152)Urachus---Patent Allantois(first aid) 153)Cerebral Blood Flow regulation A-Low systemic pCO2 B-Low pH C-High Systemic PO2 Ans:A 154)open wound healing=>Myofibroblast 155)Clostridium Tetani A-Lethal exotoxin B-Chronic reaction Ans:A past paper bcq 156) Eating Raw or undercooked Meat -Tapeworm ingestion(first aif:taenia solium) 157)Regarding thirst A-Inc plasma volume and dec osmolarity B-Dec Plasma volume Ans:B past paper bcq) 158)Change in nuclear size, shape and polarity -Dysplasia(past paper bcq) 159) Open woumd healing Contraction of Myofibroblast 160) Antigen-Antibody complex. T-cells reacts with A-Antigen presenting cells B-Major Histocompatibility Complex Ans:B 161Patient wth renal stones of A-furosamide, B-Hydrochlorohiazide C-Acetazolamide.

133 Ans-B past paper bcq 161) Most common cause of HCC -Both hep B and C Africa and asia hep b》hep c for hepatoma In usa and western countries hep c》hep b for hepatoma ref cmdt 2016 page 1595 162)Hep B most common transmission is via A-Through Contaminated Needles B-Orofecal route Ans:A ref cmdt 163) Medial Leminiscus -Nucleas Cuneatus and Gracilis(kaplan) 164-A patient comes in ER in shock. Initial feature A-Bradycardia B-Rapid Pulse C-Increase Respiration Ans:B past paper bcq 165-Pt with cancer T4N1M1 , weak, lethargic and pallor +, such that these pts have low survival rate. What will be common to such pts ? A. Cachexia B. HCC C. cervical CA Ans:A past paper bcq 166-Peroxidation of lipids A-Amyloid b-Lipofuscin C-Hemosiderosis D-Melanin or Melatonin Ans:B 167)A businessman travels to fareast, develops abdominal discomfort, jaundice, spleen just palpable? A-Tropical Spleenomegaly Syndrome B-Tropical Sprue C-Infectious Mononucleosis D-Ulcerative Colitis Ans:A 168-Surgically Lobes of breast are? A-Glandular tissue B-Stroma containing Gland C-Separated by interlobar stroma/septa Ans:C 169-Discharging Sinus , yellow colour pus -A-Actinomycosis B-Staph Aureues C-E.Coli

Ans:A ref first aid+past paper bcq 170-Patient Doctor Relation strengthen's by "Active Listening"(rabia bcq) 171-Urine dribbling from anterior abdominal wall defect A-Patent urachus B-Patent Allantois C-Median Umbilical Ligament Ans:B 172)Teratogenic among following A-Coffee B-Alcohol Ans:B ref first aid fetal alcohol syndrome 173-Morphine is used in "Terminal Cancer Patient Pain" 174-Patient presented with weakness, lethargy. Lab shows HB 5.6mg/dl, WBC 3.5, Platelets 12000. Diagnosis -A-Aplastic anemia B-Iron Deficiency Anemia C-Thalassema Major Ans: A ref first aid n cmdt 175)Lysosomes Contain "Hydrolyic Enzymes"(past paper bcq 176-A player injured his foot. First step to stop bleeding Ans-Vasoconstriction 177)Pt has pain between upper lip and lower eyelid. Nerve involved Ans-Maxillary branch of trigeminal

SURGERY 7.11.16 EVENING 1. Osteocyte a. periosteum for osteocyte for injury in bone Ans:::: A 2. Microscopic feature of compact bone A haversian canal arranged obliquely B lacunae have osteoblast C haversian canal are not interconnected collagen D lamellae are regularly arranged Ans::: D 3.which one is pivot joint a.median atlantoaxial joint b.sternoclavicular ANS::: A 4.Difference b/w pharyngeal and smooth muscle of esophagus. a. ca bind to troponin C in pharyngeal muscle

134 b.smilar ANS::: A 5.Mylenation in CNS a.Oligodendrocyte b.kuffer cells ANS::: A 6.Postganglionic parasympathetic nerve receptor are a. Cholinergic b. adrenergic ANS::: A 7. During surgical removal of axillary lymph node a nerve damage, pt is unable to flex and supinate at elbow? A. Radial nerve b. axillary nerve ANS::: B 8.During surgical removal of lymph node in post triangle f neck, nerve injured? a. spinal Accessory nerve b.12th ANS::: . A 9.After tonsillectomy , pt can’t taste senatation of post 1/3 of tongue which nerve damage a. glossopharyngeal b.7th ANS::: A 10.After fall pt can’t abduct above the head which muscle paralysed a. supraspinatus muscle b .infraspinatus Ans::: .A 11.Dermtome of medial 1/3 of hand a.C8 b.T1 C.C6 D.C7 (Ans::: a) 12.C8 contribute to a. lateral cord b. posterior cord/ (Ans::: b.posterior and medial mainly) 13.Breast lie on A Pectoralis minor b. superficial pectoral major fascia (Ans::: b] 14.Sparing of thenar eminence all muscle paralysed nerve damage? a. radial nerve b. c8-t1

Ans::: b[c8 t1 is same as ulnar] 15.Tumor in superior lobe of lung may involve which bronchopulmonary segment? a. apical superior b. superior basal (Ans::: a) 16.Phrenic nerve supply a. mediastinal pleura Ans::: A 17.SA Node located at A. upper part of sulcus terminalis [koch triangle] b. entrance of ivc [Ans::: A] 18.Superficial inguinal ring formed by a. aponeurosis of external oblique b. internal oblique Ans: A 19. In anterior duodenal perforation the contents will go to A. right iliac fossa B. left iliac fossa C. anterior subhepatic space D. posterior subhepatic space E. small bursae Ans::: A[if posterior lesser sac ] 20.4th part of duodenum a. completely retroperitoneal b. partial retroperitoneal Ans::: A. ref FA 21.Appendicular artery a. lies posterior to cecum b. enter in mesoappendix ANS::: b 22.right gastroepiploic artery branch of a. Gastrodudenal artery b. proper hepatic artery Ans::: A 23.In porta hepatic which structure damage by compression of hepatoma a. Portal vien b. cbd Ans::: A 24.Tumor at head of pancrease compress?? a.portal vein b.. CBD ANS::: b 25.Oblique fissure at lung a.T3-T6

135 b.. t2..t4 ANS::: A 26.How to differentiate between right and left kidney a. by arrangement of structure b. no of vessel ANS::: A[or by the length of renal vein ..left is longer ] 27.spleen true statement a. Length is 12 cm ANS::: A 28.Regarding esophagus appropriate stem a. contain rt vagal trunk posteriorly ANS::: a 29.Regarding CSF true a. Drain into subarchnoid villi b. absorbed by ependymal cells ANS::: A 30.Regarding the Filliform papillae: a. These have no fibrous core b. These are present at dorsum of tongue up to the tip[upper anterior 2/3] c. These are covered by soft epithelium d. These are mushroom like in shape e. These are present on posterior 1/3 of tongue ANS::: .b 31.During procedure at the submandibular nerve damage a. Marginal branch of facial nerve b. mandibular ANS::: A 32. During swallowing food prevented from entering to trachea by 1) Epiglottis elevation over naso pharynx 2) Close /-tight approximation of vocal cord 3) Movement of uvulva 4) Contraction of pharyngeal muscles 5) Position of tongue ANS::: B 33. true about superior parathyroid gland A. is at mid of posterior border of thyroid at first tracheal ring B. its position changing C. is not enclosed in pretracheal fascia D. all of above ANS::: A 34.Major support of uterus a. cardinal ligament b. broad ligament

ANS::: A[also known as trAns:::vers cervical ligament ] 35.Aftr surgical procedure at uteus female develops the urinary incontinence most likely due to a. urinary bladder and vaginal fistula b.fecovaginal ANS::: A 36.injury at back pt unable to void , nerve damage may be a.S234 b. L5s1 ANS::: A 37.Atonic bladder due to a. loss of inhibitory signal from midbrain b. damage of pelvic splanchnic nerve (Ans::: b) 38.rt testicular ca dain into a. b/l paraaotic lymph node b. ipsilateral inguinal and ipsilateral aortic ANS::: A 39.yolk sac tumor marker a. HCG b. afp c. ldh ANS::: B 40.sympathetic function a. bronchodilation b. bradycardia ANS::: a 41.facial nerve cervical branch supply to a. platysma b .sternocleidomastoid ANS::: a 42.infraorbital artery is the terminal branch of a. facial artery b. maxillary artery c. sup temporal artery (Ans::: b) 43.sarcomere a. b/w two z lines b. A band ANS::: A 44.fiber for fast pain a. A delta b.c fiber ANS::: A 45.tuninng fork 256 hz frequency vibration perceive by a. pacinian corpuscle

136 b. miessner corpuscle ANS::: A 46.Most important response in shock as a whole a. CNS ischemic response b. carotid body ANS::: A 47.Carotid sinus a. sense rapidly decreasing arterial pressure b. dec in volume ANS::: A 48.In action potential , depolarization by a. k efflux b.Rapid Na influx ANS::: B 49.In cardiac cycle QRS complex prior to onset of a. Isovolumetric relaxation b. Atrial systole c. ventricular systole ANS::: c 50.Most important body mechanism in maintaining te,prature in cold a. tachycardia b. sweating c. Shivering ANS::: c 51. During treadmill exercise if heart rate is increased to double the normal rate, then cardiac cycle time will be: A. 0.2 second B. 0.4 second C. 0.8 second D. 1.2 second E. 1.6 second (ANS::: b will be half ] 52.parasympathetc action of heart a. increase PR interval b. increase systole time more than diastole ANS::: A 53.fracture at greater trochanter after 7 days developed lower leg swelling ,pain and edematous most likely next complication a. fever b.pulmonary thromboembolism ANS::: B 54.patient is unable to invert damage may occur a. posterior and anterior tibial muscle b.gastrocenamius

ANS::: A 55.true about great saphenous nerve a. passes in front of medial malleolus b. passes in front of lat malleolus ANS::: A 56.Nerve supply of anterior scalneus muscle a. Ans:::a cervicalis b. brachial plexus c. C6,C7,C8 ANS::: [They are innervated by the fourth, fifth, and sixth cervical spinal nerves (C4C6).] 57.nerve damage in ant tibial syndrome??? a. common perroneal b. sciatic Ans::: A 58.Lymphatic drainage of toe a. verticle group of sup inguinal b. medial group of sup inguinal (ANS:::)a 59.which molecule trAns:::ported by secondry active trAns:::port a. glucose b. protien [Ans::: A..protein is a part of trAns:::porter ] 60.If a man loss 1 liter blood in 5 min most important body response a. inc vasomotor tone b. inc heart rate Ans::: is a 61.sound produce by ventricular vibration during rapid inflow a. s4 b. s3 Ans::: b 62.Most important stimulant for release of aldosterone a. Renin b. k ANS::: b 63.substance dec the plasma osmolarty a. dec vasopressin b. aldosterone ANS::: A.inc plasma water content 64.hormone act ia steroid mechanism a. gh b. t4 Ans::: b

137 65.tidal volume 500ml r-r 10 calculate alveolar ventilation? a.3200 b.3500 c.4200 ANS::: b 66.in bronchial asthma a. fev1/fvc<75 b. residual volume decreased ANS::: A 67.O2 –hb dissociation curve shift to left a. acidosis b. inc temp c. alkalosis ANS::: C 68.main factor that shift k from intracellular and extracellular a. sternous exercise b. insulin ANS::: A 69.major intracellular buffer a. hb b. protein c. po4 Ans::: B protein …. … The most important buffer in the ECF and the body is HCO3(bicarbonate) which combines with excess H+ ions to form carbonic acid. 70.pt died due to CRF , autopsy show which hormone function is increase a. pth b. erythropoiten ANS::: A 71.kidney important in activation of? a.b12 b. cholecalciferol ANS::: b 72.most important part of nephron affected in shock a.pct b.dct c.loop of henle ANS::: A[ pct is the most susceptible to ischemic and toxic injury] 73.60% na reabsorbed in a.dct b.pct ANS::: b…. majority of all stuff is absorbed in pct even water… 74.GFR a. increased by afferent arteriolar

constriction b, increased by efferent arteriolar constriction ANS::: b..[ gfr inc and plasma flow dec ] 75.41% ht emeAns::: a. 41% rbc ,wbc and platelet b.59 prcnt RBC ANS::: A 76.In severe dehydration volume would be replaced by a.0.9 % n/s b.ringer lactate c.5 % glucose ANS::: C ganong bcq key is c there 77.19 y ear boy sitting in well ventilated room at temp 26degree he has sweating ,increase heart rate , main factor for these changes a.exercise b heat stroke c.high bp ANS::: A 78.complement help in phagocytoss a.c3b b.c5 ANS::: A 79.pt receiving chemotherapy for cancer most imp mechanism to reduce the size of tumor a. apoptosis b. atrophy ANS::: A 80.cushing syndrome by a. adrenal adenoma b. lung cancer ANS::: A..ref FA 81.ant pituitary tumor a. are chromphobes ANS::: a 82.dopamine given in parkinsonism will inhibit a. prolactin b. oxytocin ANS::: a 83.after partial hepatetomy important early hisytologic statement a.inc GH alpha b.EGF ANS::: ..B 84.During defecation imp reflux is A. rectoanal

138 b. mass movement ANS::: A 85.major diff in b/w primary and secondary peristalsis a. in primary oropharyngeal phase occur b. speed ANS::: A 86.icam and vcam are a. leukocyte adhesion b. antigen ANS::: a 87.in heamophilia if cryoprecipitate is not available , alternative source of factor viii a.FFP b. stored blood ANS::: a 88.a mother brought a boy for circumsission he has history of bleeding during umblical cord cut in both male and female , on investigation PLT normal , bt is normal , only APTT increased , diagnosis a. vwb b. Heamophilia ANS::: b 89.heparin actionas an anticoagulant it inhibits clot a. propagation b.formation ANS::: A 90.a young boy short limbs ,normal trunk ,normal intellectual what would be the pattern of inheritance a.autosomal dominat ANS::: A 91.mediator of allergy a. 5ht b. lymphokines c. serotonin d. bradykinin ANS::: A 92.A pt admitted in oncology ward with hypotention with investigation on he has positive blood cultutre of pseudomonas which is mediator for his condition a. tnf alpha ANS::: A 93.most important prerequisite for renal trAns:::plant a. HLA compatibility b. ABO comptabilty

Ans::: b 94.young girl with parasites , rashes which type of allergy a. TYPE 2 b. type 1 ANS::: B 95. a pt undergoes kidney trAns:::plant on first day he was unable to produce urine there is also tenderness at trAns:::plant site A} Graft versus host rejection {b} hyper acute graft rejection {c} blockage of ureter {d} renal artery stenosis) {E} thrombus at the side of trAns:::plant ANS::: b 96. young girl with ambiguous genitalia having karyotype 46 xx , diagnosis a. androgen insensitivity syndrome b. adrenocortical hypersensitivity c. mixed gonadal dysgenesis Ans::: b… while in androgen insensitivity karyotype will b XY) 97. chemical carcinogen a. propyl alcohol b. methyl alcohol c. benzidine Ans::: c 98.nasopharyngeal ca caused by a. rhinovirus b.EBV ANS::: b 99.A young boy complain of abdominal pain , investigation show haing mass with blue cells and inc level of catecholamines,dianosis a. neuroblastoma .b.adrenal adenoma Ans::: a 100.ca colon 1 metastasize to a.liver b.mesentric lymph node Ans::: a 101.virus act by oncogenesis by acting on A.rNA B.DNA C.protooncogen Ans::: b 102.54 year old male came with dyspnea with h/o cogh ,trAns:::lucency of upper lobe , no infiltration prominent

139 pulmonary arterial marking a.arteriosclerosis Ans::: A 103.10 year old boy while playing suddenly collapsed and died what will be diagnosis a.kawasaki disease b.gian cell arteritis ANS::: a 104.ECG changes on lead v4 ,which artery will infarct a. LAD b.rca Ans::: A… septal infarct 105.Case of infective endocarditis, definitive diagnosis by a. blood culture ANS::: a 106.myxometous degeneration seen in A. aortic valve B .Mitral valve prolapse ANS::: b 107.2 year boy raised mc 109 , lethargic , fatigue , diagnosed by a. serum b12 b. antiintrinsic antibody Ans::: A 108.multiple drug therapy in tb due to a. dec side effect b.decreas resistant Ans::: b 109.pt with deranged lfts and ring in eye a. Wilson disease Ans::: A ..keyer fishcher ring.. 110.a sexulay active male complainof burning micturition , no ulcer on penis urine culture negative for n gonorrhea a. candida b. chlaymdia Ans::: b 111.hormone inhibit release of gnrh release in darkness’ a. melatonin Ans::: a 112.sarcoma feature a. vimentin Ans::: A 113.sqamous cell ca feature a. keratin 114.pregnant lady infected with rubella , baby born will have risk of

a.cataract b.imperforate anus Ans:::a A 115. Data is collected for some clinical trial based on presence or absence of Vomiting. What type of data it is: A. Nominal B. Ordinal C. Parametric Ans::: A 116.type of collagen in reticularis a.type 2 b. type 3 ANS::: b 117.down syndrome trisomy a.21 ANS::: A 118.Thiamine deficiency cause a. Wernicke korsakoff syndrome b. polyateritis ANS::: a 119.high cardiac output failure a. thiamine deficiency b. b12 def ANS::: a 120.Glycolysis a.in aerobic glycolysis pyruvate is an intermediate b/w gglycolysis and TCA b. energy isnt produced Ans::: A 121.daily requirement of helathy man of 70 kg a.25-30kcal b.15-20 kcal Ans::: a 122.1 gm of hb bind to a.1.34 ml of o2 ANS::: a 123.clostridia is a. spore forming b. easy to destory Ans::: A 124.pt live at river and got eyes problem ,which nematode involve a. oncocerca volvulus b. leshmania ANS::: A… o for optic 125.regarding MRSA a. sensitive to vancomycin b. not treatable Ans::: a

140 126.regarding aminoglycoside A .monitor in elderly pt with renal problem b. high safety profile Ans::: A 127.NSAIDS A. Inhibits cox pathway b. inhibit phospholipase Ans::: a 128.Drug receptor interaction A. heparin with protamine calcium b. manitol diuresis ANS::: b heparin antidote is protamine sulphate 129.male siting with eyes closed which waves dominant on EEG a. alpha wave b. beta c.theta ANS::: A..ref FA 130.cholinergic fiber a. all postganglionic parsympathic b.. all preganglionic

ANS::: a 131.Melanocyte develop from a. neural crest cell b. endothelium ANS::: a 132.left sided lower limb paralysis with left face and left sided homonomys hemianopia lesion at a.midbran b.pons c.medulla ANS::: b 133.tremor at end of movement in left hand , hypotonia of left hand with ataxia Lesion at a. left cerebellar hemisphere Ans::: A 134.loss of GABA b/w G.pellidus and sustantia nigra a.athetosis b.hemibalismus Ans: a

141

GYNAE/OBS

2016 PAPERS SEPTEMBER, OCTOBER, NOVEMBER Gynae and Obs 7th September 2016 (evening) 1.Lateral attachment of urogenital diaphragm; a. Sarum n coccyx b. Obturator internus membrane c. Inferior ramus of the pubis and ischial ramus d.ischial tuberosities e. tip of coccyx Ans: : cc 2. The most appropriate therapy for endometriosis is a.Danazol b.Norgesterol c.Oral androgens d.Oral contraceptive e.Raloxifine Ans: : aa 3. important buffer of blood? a. Hb b.hco3

c.Acetate d.protien e.phosphate Ans: : bb 4.Content of fat in male is? a.10% b.15% c.40% d.60% e.17% Ans: : bb 5. Erythropoetin is secreted by? a. endothelial cells of the peritubular capillarie b. jg cells c. adrenal cortex d. glomerulus e. loop of henle aAns: : aa 6. 1.The richest source of vitamin ...? a. whole milk b. butter c. margarine

142 d. liver cord oil Ans: : dd 7. concentration of which of the following hormone doesnt increase during pregnancy: a) PTH b) cortisol c) GH d) prolactin Ans: : cc 8. Size of ureter is a. 20-25 cm b.25-30 cm c.30-35cm d.35-40 cm e. 15-20cm Ans: : bb ( 25 cm is correct ) 9.Blood supply in midline of flap of forehead ?? a.frontal branch of frontal artery b. paired supra trochlear artery c. optahlmic branch of trigeminal artery d.facial artery e. paired Supra orbital artery Ans: : Bb 10.A fetus was aborted .it was sent to forensic lab .how can the viability be checked .that for how long the fetus was viable.? a.Fetus weight 200g b.Eyes laterally c.Fetus CRL >250 mm d.Absence of wrinkles e.Absent toe nails Ans: : cc 11.what is absent in C1? a.Spinous process b.Archc.trAns:verse process d.Lateral mass Ans: : aa 12. difference between roots of right lung from left lung? a.No of primary bronchi b.No of arteries c.No of veins d.No of nerves plexus Ans: : aa 13. cisplatin has the following side effect a.Nephrotoxicity b.Neurotoxicity c.hepatotoxicity

d.nausea , vomiting Ans: : aa 14.How does body adapt to cold temperature in long term. a .Internal pyrogens b .Chemical thermogenesis c. Formation of brown fat d. peripheral vasodilation e.pheripheral vasoconstriction Ans: : c 15.which of the following factor deficiency cause coagulation? a.5 b.7 c.8 d.10 e.12 Ans: : aa 16. hyperbaric oxygen treatment is most useful in? a.CO poisoning b.Anemia c.Polycythemia d.decompression sickness e.Gas gangrene Ans: : aa 17. 2 feet away from ileocolic junction at antimesentric border a.urachal fistula b.meckel diverticulum Ans: : bb 18. tunica albuginea a.Contain testes only b.Derivative of parietal peritoneum c.derivative of visceral peritoneum d.covers ductus deferens Ans:: a & b both correct 19. Costo diaphragmatic recess a.seen only on left side b.Wide open space in centre Option missing 20. deep ring of inguinal canal attaches a.Fascia tranvesalis b.Tranversua abdominus c.External oblique d.internal oblique Ans: : aa 21.An anesthetist wants to give his patient most potent muscle relaxant who is about to undergo cesearation so that she may relax completly.he would give

143 a.Bupivacaine b.Lidocaine Ans: : aa 22. Root value of femoral nerve a.L2 ,L3, L4 b.L2,L3,L4,L5,S1,S2 c.L1 ,L2 Ans: : aa 23. A patient complains of arthritis and discoid rashes. Which of the following would be specific? a. Ds DNA b. ANA c. AMA d.Anti ccp Ans: : aa 24. At the end of marathon race , a person has ? a. high insulin low glucagon b. high glucagon & low insulin c. high cortisol d. high insulin and glucagon e.low insulin and glucagon Ans: : bb 25. A 25 years old married female came to you with history of amnoerhea for 3 weeks her beta hcg shows that her pergnanacy test is negative. What could be the cause? a. ectopic pregnancy b. anovulation c. primary hypothalamic disorder d.Anorexia Nervosa/ normal ? Ans:: dd 26.test for difference of two meAns: A. T – test B. SNOVA c. ANOVA Ans:: aa 27. In Tuberculosis, antibodies are a. cell bound b. inside cell c. absent Ans: : a 28. Toxoplasmosis is a. parasite b helminth c.bacteria d. virus e.Fungi Ans: : a

29.Virus acts on cells by: a:Damaging cell membrane, b:Damaging nuclear membrane, c:Invloving protien synthesis, d:Changing protien structures. e:Changing protien metabolism Ans: : c 30. Aspirin and other NSAIDS work by acting on a.Lipoxygenase b.Cyclo-oxygenase A2 c.HMG Co A reuctase inhibitor Ans: : b 31. Which of the following is a spore forming bacteria? a. clostridium b.cornybeacterium c. Chlamydia Ans: : a 32. microscopic feature of malignancy a.Invasion b.Pleomorphism c.N/C ratio Ans: : a 33. A patient with pulmonary embolism and extensive tissue injury. What is the most probable cause of his dx. a. Dvt b. Thromboembolism c. Atril myxoma Ans:: bb 34. Aldosterone acts on? a. Collecting duct b. Dct c. Cortical collecting tubule Ans: : aa 35.Plasma proteins return to normal after a.2 -3 days b..7-8 days c.10-12 days d.12-14 days Ans: : c 36. An abnormally tall boy of 14 year age consults his physician for his abnormal height and secretions from his mammary gland. X ray skull shows enlarged pituitary gland. Histological examination of pituitary is likely to show: a. Decreased number of pituicytes b. Hessing bodies seen in karge number. c. Increased no of basophill cells.

144 d. Increased no of chrimophobes e. Increased no of eosinophills Ans: : e 37. prolactin raised with infertility what will be cause a.microprolactinoma b.macroprolactinoma c.pituitary tumor Ans: : bb 38. Diagnostic test for typhoid in 2nd week is : a.Blood culture and widal test . b. blood culture c. stool culture d.widal test e. urine culture Ans: :aa 39. Substance which act as both hormone and nurotrAns:mittor? a. adrenaline b.cortisole c . acetylcholine d. thyroid e. Aldosterone Ans: : aa 40.Stimulation of parasympathetic nerves to the heart decreases heart rate by a. Increase in permeability of K thru ventricles b. Incraese in permeability of k through sa node c. prolonging of AV nodal delay Ans: : cc 41..Starling's law of the heart: a.Does not operate during exercise b.Is not obeyed by the failing heart c.Explains the increase in heart rate during exercise d.Explains the increase in cardiac output when venous return increases e.Explains the increase in cardiac output due to sympathetic stimulation Ans: : d 42.A lady presented with per vaginal bleeding,history of bleeding disorder and having a dead fetus ..first step of management A.blood TrAns:fusion B. Evacuation of uterus C.platelet trAns:fusion d.iv heparin

Ans: : a 43. Platelets a.half life of 10 days b. Increase after spleenectomy c.granules contain serotonin c. Stimulated by growth hormones Ans: : b( remember half life is 4to 5 days life span is 10 days a big differnce) 44.The ascent of kidney is prevented by A.external iliac artery B.internal iliac artery C.superior mesenteric art D.ivc E.inferior mesentric artery Ans: : e 45.which of the following structure has both intra & extra pelvic extension A.broad ligament B.ovarian ligament C.trAns:verse cervical ligament D.round lig of ovary E.round ligament of uterus Ans: : e 46.most potent chemotactic factor A.c3a B.c5a C viruses D.arachidonic acid metabolites Ans:: b 47.Rbcs are A.biconvex B.nonfragile C.largest of all cells D.have glycolytic enzyme activity Ans: :d 48.lysosome contains A.glucose B.Na+ C.ca + D.acid phosphatase E. Water Ans: : d 49.gram -ve infection causes septic shock in which of the following tracts A.biliary tract B.colon C.small bowel D.urinary tract E.genital tract Ans: :d

145 50.flexor reflex by A.Muscle spindle B.Monosynaptic C.messner corpuscle D.noxious stimulus E. By Tapping with hammer Ans: : d 51.study on sexually trAns:mitted disease,one group of boys had penile ulcers and the other group had no ulcers,non ulcerating leisons were d/t? A.treponema paliidum B.chlamydia C.HPV D.ducreyi E.Gonorhea Ans: : ee 52..barr body test in saliva is diagnostic for A.klinfelter syndrome B.gonadal dysgenesis C.turner syndrome Ans: : c 53..You are testing efficacy of unknown unnamed drug A and drug B in an infection...then randomnly allocated to pts recieving both of them..it is an example of A.double blind/placebo B.single blind C.triple blind. D.cross sectional study Ans: : a ( RCT is selected in the case when a new drug is being tested for its efficacy ) 54.anemia of maturation failure..immature cells in periphery A.iron def anemia B.normocytic C.pernicious anemia D.microcytic anemia Ans: : c 55.the difference of osmolarity b/w interstitial fluid and plasma is A.1mosm/L B.1.5Mosm/L C.3 Mosm/L D.5 Mosm/L E.no difference Ans: : a 56local cause of delayed wound healing A.infection

B.ischemia C.vit c deficiency D.cortisol Ans: : a 57.tricuspid auscultation area A.2nd right intercostal space B.4th left intercostal space C.mitral area D.lower half of right border of sternum Ans: :d 58.aortic hiatus aneurysm compresses A.thoracic duct and vagus nerve B.ayzgous vein C.phrenic nerve Ans: : b 59.pulmonary wedge pressure A.measure of pulm artery pressure B.measure of rght atrial pressure C.measure of left atrial pressure Ans: :c 60.about csf A.It is an ultra filtrate of plasma B.provides nutrition to cns C.formed by arachnoid granulation D.separated from interstitial fluid by blood brain barrier Ans: :d 61.growth hormone stimulation A.rem sleep B.wakeful condition C.hypoglycemia D.free fatty acids Ans: : c 62.which one of the following is mesenchymal in origin A.choristoma B.hemartoma C.sarcoma D.carcinoma E. Choriocarcinoma Ans: : c 63.in the first step of thyroid hormone synthesis iodine attach with A.albumin B.thyroxin C.thyroglobulin D.tyrosine E.hb Ans: : d 64.tumor with all three germ layers A.teratoma

146 B.adenocarcinoma C.hamartoma D.sarcoma E.seminoma Ans: : a 65.female child,breast fed developed jaundice on 2nd day hb 9 bilirubin 8 unconjugated A.mother is RH Postive B.breast fed jaundice C.physiological jaundice Ans:: c 66..bifurcation of common carotid artery is felt at A.anterior border of sternocleidomastoid at the superior border of thyroid cartilage B. post border of sternocleidomastoid in posterior traingle C.at level of trapezius D. post border of sternocleidomastoid at the level of hyoid Ans: : a 67.an adult in sitting position,resonance can be auscultated from back of the chest till A.6th rib B.8th rib C.10th rib D.11th rib E.12th rib Ans: :bb ( clinical methods: Bedside) 68.rbcs a.lack mitochondria b.half life of 180days c.released from bone marrow as mature erythrocyte d. Present on mhc 2 e.are the largest cells of the body Ans: :a 69.scenario of increase serum calcium,irregular bone,also bone pain...which of the following causing it A.PTH B.GH C.vit D deficiency Ans: : a 70.lysosomal storage disease with mental retardation and blindness A.taysachs disease B.granulomatous disease Ans: : a

71.measured by spirometry A.residual volume B.Total lung capacity C.residual volume D.vital capacity Ans: : d 72.the obsteteric nerve block cause urinary incontinence,anaesthetized muscle is located in A.pelvic diaphragm B.urogenital diaphragm C.superficial perineal pouch D.none of the above Ans: : b 73 true about superior parathyroid gland A.is at mid of posterior border of thyroid at first tracheal ring B.its position changing C.is not enclosed in pretracheal fascia D.all of above Ans: :a 74.which structure has 3 layers of muscels A.fundus of stomach B.esophagus C.one of the above D.all of the above Ans: : a 75.which of the following is an example of primary active trAns:port A.H+K pump B.Na glucose co trAns:port in intestine C.Na-Amino acid co trAns:port in intestines D.none of the above Ans: : a 76.alpha receptors effects A.contraction of radial muscle of iris B.vasodilation in skeletal muscle C.increase in activity of gut D.decreased heart rate Ans: : aa 77.regarding parasympathetic supply of heart A.Left superficial cardiac plexus B.vagus nerve C.sympathetic system D.Left deep cardiac plexus Ans: : b 78.which of the following supresses growth hormone release A.Puberty B.sleep

147 C.somatomedin D.starvation E.stress Ans: : c 79.A female with regular cycle of 21 to 23 Days will most likely be ovulated on which of the following days A.12-14 B.14-18 C.7-9 D.10-12 Ans: : c 80.trypsinogen is activated by A.enterokinase B.peptidase C.amylase D.acid phosphatise E. HCL Ans: : aa 81.most early sign of vit A deficiency is A.keratomalacia B.cheilosis C.hyperkeratosis D.diminshed dark adaptation/night blindness Ans: : d 82.a female presents with jaundice her lab reports show Alt 17,alk phosp.75 and retics 10.cause of jaundice A.hemolytic anemia B.pernicious anemia C.iron def anemia D.all of the above Ans:: a 83.true regarding cavernous sinus A.occulomotor trochlear and opthalmic of trigeminal present in lateral wall B.connected posteriorly to facial vein C.drain directly into straight sinus D.has abducent nerve in medial wall E.Related Laterally to sphenoid Sinus Ans:: aa 84.A man with bee sting,develop swelling and acute infalmaation.mediator for it? a.Histamine b.Bradykinin c.prostaglandin d.leukotrine e. Esinophills Ans: : a 85. severe blood reaction occur a.A+ To 0+

b.O+ To B+ c.A- To A+ Ans: : a 86. Pregnant lady with mitral stenosis, pt.developed dysponea.orthopnea and difficulty to breath. Most probable cause would be? a.Right heart failure b.Pulmonary htn Ans: : Bb 87.on a study about diabetes between 2 groups it was found Women with mean 200±50 Men with mean 205±10, this meAns:’ a. Women with low mean and high standard deviation b.Man with high mean and low standard deviation c.Both have no link d. Case control e. Cross Sectional study Ans:: aa 88. which is obsevatinal study a.Case control b.Cohort c.cross sectional study Ans: : b 89. breast outer quadrant lymph drainage a.Ant./pectoral b.central c. apical d. posterior Ans: :a 90. testicular lymph node drainage a. Paraaortic b. superficial inguinal c. deep inguinal d. para umbilical Ans: : a 91. superficial perineal poch content a.Bulbourethral gland b.Perineal nerve c.Sup.trAns:verse perineal muscle d. internal pedundal vessels Ans: : c 92. branchial cleft a.Open laterally b.Open laterally ant.to sternocleido mastoid Ans: : b 93. draining sinus and granuloma is the feauture of

148 a. aspergillus b. nocardia c. actinomycosis Ans: : c 94. Which of the following helps in formation of the bones? a. osteoblast b.osteoclast c.osteoprogenitor cells d.fibroblast Ans: : a 95. more central value of no. In series a.Mean b.Median c.Mode Ans: : b 96. A child since he was born was having problem of urine leaking from ventral surface it may be due to? a. Epispadiasis b. Failure of urogenial folds to close Ans: : b 97. Lacunar ligamnt forming medial boundary , fibres enrolled from a.Inguinal ligament b.TrAns:verse ligament c.Relected ligament Ans: : a 98. Malignacy is caused by which of the following a.Silicosis b. Asbestosis c.Smoking Ans: : b 99. ADH is inhibited by A.Alchol B.increase Body water Ans: : a 100. Sympathetic autonomic system true is.... a.Adrenergic preganglionic b. Thoracolumbar outflow c.Short postgang fibres d.pre and post ganglionic short fibres Ans: : b 101. Most common bacteria involved in STDs is a.hpv bChylmydia c.Gonnorhea d.Hsv

Ans: : b 102. Person wid bloating abdominal pain very foul smelling stools .First Investgation would be a.Cumplete stool d/r b.Stool cs c.Lipid profile Ans: : a 103.most Comon mediator of fever a.1L-1 b.Tnf. c.. IL 6 Ans: : a 104. increase in ECF osmolarity Lead to a. stimulate plasma vol and osmoreceptors And stimulate adh b. Inc adh and also nhibit osmoreceptors nd vol c.inhibits adh and stimulate vol and osmoreceptors d.stimulate adh and inhibit vol and osmoreceptors Ans: : a 105. Person having history of trauma on the left upper arm he develop swelling in arm that was painful for 3months it was well circumscribed 4 cm and on xray appeared bright then it gradually deceased in one year most Likely .... a.Metaplasia ... b. Hyperplasia .... c,.Neoplasia ..... d.Dysplasia ... e.Hypertrophy Ans: : e 106. Irregular growth of bone is due to the effect of which of the following? a.leydig cells .... b.Ant pituary hormone effct c.Pth d.GH Ans: : d

PAPER B GYNAE

1.Drug via oral route having aqueous absorption .... a.Digoxin b.ephidrine c.Phenobarbital d.diazepam e.carbamizapine

149 Ans: :a 2.Cerebellum .... a.Iniates movemnt .... b.stabalizes movement and coordination Ans: : b 3.Most common cause of bronchogenic carcinom A Tobacco B Asbestos Ans: : a 4.Female pale lethargic .didnot take any antenatal care .... During delivery she bleeded heavily .... Which nutrient deficiency would be present in both baby And mother ? a .Vit k b.Iron c.calcium Ans: : b 5. permeation is defined as a.tumor invasion to blood vessels b.tumor invasion to lymphatics c.tumor spread to adjacent tissues d.distant metastasis Ans: : b 6.Pt presented wid leg ulcer and severe pain And on blood examination howell jolly bodies present wats the diagnosis ? a.Sickle cell b.Hereditory spherocytosis c.beta thalasemia d.alpha thalasemia Ans: : a 7.Longst acting insulin ? a.Porcine lente b.Human lente .... c.bovine Ultra lente d. glargine Ans: : d ( if glargine not in option select Ultralente) 8. 7yrs child anemic for two years with blood investigations shows decreased hb 6.5 also Hbf 97% , Hba2 4% no history of blood trAns:fusion . spleen palpable a.Thalasemia major b.thalasemia trait c.herditory sphero d. hereditary persistence of Hbf Ans: : a 9.Female wid Warty lesion on vulva.

a.condyloma b.Papilloma c.Herpes Ans: : b 10.Long tubular structure having mucosa and trAns:itional epithelium a.ureter b.kidney c.urethra Ans: : a 11.During sexual act secretion from vagin are effect of a.Bombesin b.Relaxin c. Progsterone d.gnrh e. Parasympathetic stimulatiom Ans: : e 12.Measurement of amount of lead in body a.absorbed from git b.absorbed from respiratory tract lower lobes. c.absorbed from skin d.absorbed from nasal passages. Ans: : a 13.About zygote implantation a.28cell stage. b.3-4days after fertilization c.invasion of endometrium by syncitiotrophoblast Ans: :c 14.trigone of bladder a.ureters entering as oblique slits b.devoid of peritoneum. Ans: : a 15. Carbonic anhydrase act on which part of kidney a.pct. b.early dct c.cortical collecting ducts d.calyces. Ans: : a 16. person given NaCl one litre infusion wat will happen a.osmolality increases in plasma b.osmolality decreases in urine c.na increases in plasma d.na increases in urine Ans: : D

150 17. isonizid not metabolised in people with deficiency of a.glucoronyl tranferase b.hyloroinidase c.pseudocholinesterase d.xanthine oxidase e. amine oxidase Ans: : a 18. cells having oxident stress as oxygen radicals were produced which one of the following can decrease this effect a.nadph oxidase b.glutathione oxidase Ans: : b 19. In end stage renal failure whats affected on prolong basis a.erythropoeitin decreased b.thyroid hormone decreased Ans: : a 20. propofol contraindicated in a.hypersensitivity b.renal disease c.cardiac disease Ans: : a 21. Toxic effect of lignocaine? a. Prolong seizures for 9-7 hours b.hypotension c.Perioral anesthesia d.rash Ans::aa 22. data from smaller study on one group of people is combined with data from larger study what type of data is this a.case control b.randomised control trial c.cohort d.meta analysis Ans: :d 23. cerbral metabolic rate of oxygen a.15-20ml/min/kg b.2-3 ml/min/kg c.3-4ml/min/kg Ans: :c 24. hyaline disease of new born a.maternal diabetes b.maternal eclampsia c.prematurity Ans: :c 25. urea is formed by liver in the body to get rid of a.ornithine

b.arginine c.ammonia Ans: :c 26. fracture of clavicle inner side and medial half , upward displacement takes place it is due to which muscle a.sternocleidomastoid b.subclavicular c.trapizius d.perctoralis major e.deltoid Ans: : a 27. inner medial lower quadrant of breast doesnt drain into a.anterior axillary b.inf phrenic c.supraclavicular Ans: : a 28. which nerve in urogenital triangle is damaged a.post cutaneous nerve of thigh b.ilioinguinal c.genitofemoral d.femoral e.Pudental nerve Ans: E 29. protamine side effect a.hypotension b.bleeding tendency Ans: : b 30. after abdominal hysterectomy on 3rd day post op there is serosanguinous amount of fluid from the wound wats the cause a.wound infection b.wound dehiscence c.wound hematoma Ans: : b 31. 7yrs old child with coughing and mucus inc eisoniophills plug on xray it shows hyperinflation and infiltrates whats the diagnosis a.cystic fibrosis b.bronchial asthma c.gastric aspiration d. hypersensitivity pnumonitis aAns: : b 32. normal menstrual blood flow is less than a.80ml b.120ml

151 c.40ml d.60ml Ans: :a 33. women during menstruation uses tampons presents .she develops vomiting diarrhoeahypotension high grade fever.which organism is involved in it? a.staph aureus b.e colli c.gardnerela vaginalis d.chlaymidia e.gonnorhea Ans: : a 34. remnant of epiphiseal plate a.cortical bone b.trabecular bone c. spongy bone Ans:: c 35. locally malignant cancer a.melanoma b.adrenoblastoma c.pleomorphic adenoma of parotid d. Adenoid cystic Carcinoma e. BCC Ans:: ee 36. blood pumped by heart per min a.750ml/min b.6lit c.5lit Ans: :c 37. esmolol is given in anesthesia because a.its selective beta blocker effect b.decreses pain’ c. doenot increase icp d. doesnot cause rebound hypertension Ans: : a 38. bladder ca a.schistosomia hematobium b.japonicum c.mAns:oni Ans:: a 39.Cotisol decreases.... a...glucose utilization in tissues Ans:: aa 40. before Ovulation maturation of follicles occur by action of a.Fsh b.Estradiol c.Gnrh d.Lh Ans:: a

41. Superior and inferior mesenteric arteries anastomose on a. marginal artery b. colic c. left ileocolic d. sigmoidal arteries Ans: : a 42. after amoebic infection , rectal biopsy was done now to confirm which test would be done a. pas b. electron microscope c. perl reaction d. antibodies against amoeba Ans: : d 43. lethal hepatitis in pregnancy. a. Hep E 44. Thickish white curdy discharge. a. Candidasis 45. Erythroblastosis. ........ a. Type 2 Reaction 46. HCG secreted by a. Syncytotrophoblast b. Cytotrophoblast Ans:: aa 47. trachea ? a. related ant to thyroid. b.fibrocartilagenous ring c. Ends by dividing at the lower border of T4 Ans: : c 48. Blood supply to rectum is from.... a. superior and inferior rectal arteries. 49. Captopril... mechanism of action? a. Inhibits angiotensin II b. ACEI Ans:: bb 50.Tetanus a.exotoxin has lethal effect b.exotoxin afect on cerebral c.no role of active immunity d.trismus is rare. Ans:: a 51 Which one is precancerous condition a..myelodisplastic syndrome b.cervical erosion c.osteoporosis d.psoriasis Ans: : a 52.Parasympathetic effect on lungs a.inc compliance

152 b.inc restrictive work c.inc anatmicl dead space d.inc elastic work Ans:: b 53. cell responsible for innate immunity are activated most commonly by a carbohydrate seq in bacterial cell wall b cytopasmic protein of bacteria c eosinophil d glucocorticoid e pollen Ans:: a 54.regarding cilia: a) stereo cilia are a type of cilia b) can be seen on light microscopy c) can be seen on electron micscopy d) has absorptive function e) cannot be seen under light microscopy Ans:: d 53.Tb is closely associated with .... a. .hypersensitivity 4 56. Alpha adrenergic antagonist... a. doxazocin b. clonidine Ans:: a 57. intermediate mesoderm give rise to a. Somites b.Muscle c.Heart tube d. genitourinary system Ans::d 58.Paramyxovirus; in control group causes for which aso titre was being done a.Glomerulopnephritis b.Sydenham chorea c.Arthritis d.Carditis e.erythema marginatum Ans:: cc 59. New anti hypertensive drug want to check potency. a.RCT/ Metaanalysis b. Double blind Ans:: aa 60.Stroke volume increases? a.Venous dilation b.Inc in heart rate at constant cardiac output c.venous compliance d.inc in blood volume Ans::dd

61. a 7 years old boy had pharyngitis with halitosis and peritonsillar abcess a. cornybacterium diphtheria b. streptococcus c.staphylococcus Ans::b 62.Spinal cord with dura arachnoid and pia mater in adults terminates at the level of lowr border of: L1 63. coccygeal ligament at the level of? a.L1 b.L2 c .S1 d.coccyx Ans::a 64. u did a study and it rejected null hypothesis it mean a.There z difference b. there z no difference. c. study of research is not valid. d. ur research is not powerful enough Ans::a 65.Abducent arise from a.midbrain b.pons c.medulla Ans::b 66.Which structure connects skeletal muscle to bone ? a.Tendon b.ligament c.joints Ans::a 67.true about spinal nerve ? a. derived entirely from nueral crest b. pas trough intervertebral formina c. dorsal root contain both sensory n motor supplies Ans: :b 68.Notochord forms: a.Nucleus pulposis b.Annulus fibrosis Ans:: aa 69. In females neck of urinary bladder a.lies above urogenital diapgrahm b.lies above urogenital triangle Ans:: aa 70 nerve damage in surgical neck of humerous fracture a. Axillary b. Radial

153 Ans:: aa 71 Regarding carcinoma tissue used for staining and for diagnosis a. Cyto keratin b. Vimentin Ans:: aa 72.Mensturation occurs when a. progesterone decreases b. progesterone increases Ans:: aa 73 Lateral relation of lower vagina a. ureter b.uterin tube Ans:: aa 74 most common cause of bronchogenic carcinoma a. tobacco b. Asbestos Ans:: aa 75 Regarding lignocaine a. causes Na channel blockage b. causes Cl channel blockage Ans:: aa 76 osmotic diuretic acts on a. proximal convulated tubules b. DCT Ans:: aa 77 NO derived from a. ornithine , arginine, citruline b. leucine,isoleucine, lysine Ans:: aa 78 Reactive hyperemia a. red infarct b. white infarct Ans:: aa 79 lysosomal storage disease a.tay sach disease Ans:: aa 80 regarding superficial parathyroid a. located at mid of thyroid posteriorly at 1st tracheal ring Ans:: aa 81. S1 = isovolumetric contraction 82 man with progressive dyspnea, bilateral pitting edema upto knee , bilateral crypts, bilateral pleural effusion on x ray what will be the lab finding a. increase Na b. decrease Na Ans:: aa 83 Superficial perianal pouch

a. perineal body 84 Ip3 mechanism is example of a. Adh Ans:: aa 85 Content of superficial perineal pouch a. perineal body b. bulbourethral glands c. membraaneous urethra d. internal pudendal nerve e.perineal body Ans: E

GYNAE OBS 14TH OCT MORNING Q1.During surgery ovary removed from body in which lymph nodes ovary drain? A)External iliac B)Internal iliac C)Para aortic D)Sup.inguinal nodes ANS..C Q2.UroRectal septum divides into? A) Rectum and urogenital sinus b) Anus and urogenital sinus ANS…A Q3.Levator ani supplied by? A) Hypogastric plexus B)Pudendal nerves ANS..B Q4.Bladder supplied by? A) Superior hypogastric plexus B) Inferior hypogastric plexus ANS…B Q5.Detrusor muscle control by? A) Pudendal nerve B) parasympathetic and pelvic nerve C) ilioinguinal nerve ANS….B Q6.Toxoplasmosis is? A) Parasite B) Bacteria ANS….A Q7.Pseudomembranous colitis caused by? A) Cl.difficile B) Cl.botulism ANS….A Q8.Detrusor supplied by?

154 A) Sup inguinal B) Pudendal nerve ANS….B Q9.In a female during during labour mediolateral episiotomy performed which muscle cut? A) Bulbospongiosis B) Levator ani C) Sup transverse perineal muscle ANS…..A Q10.Structure damage close to phrenic nerve? A) Pericardiophrenic artery B) Musculocutaneous nerve ANS….A Q11.An 11 year old boy present with fever for 5 days.He also c/o malaise and nausea for 2 weeks.He recently visit to village few days back where sanitation not proper.which causative factor? A)Hep A B)Hep B C)Hep C D)Hep D E)Hep E ANS….A Q12.A female at 38 weeks of gestation with fever nausea ALP=275,SGPT=178,ALK=258.She visit a remote village 6 weeks back where sanitation not proper.what’s the causative factor? A)Hep A B)Hep B C)Hep C D)Hep D E)Hep E ANS….E Q13.Which of the following hepatitis leads to chronic liver disease? A)Hep B B)Hep C ANS….B Q14.Most initial sign of bupivacaine toxicity? A) Nausea B) Ringing in ears ANS….B Q15.Regarding acute inflammation which is true? A) WBC increase upto 40,000

B) Eosinophil not change C) Steroids cause no change in eosinophils D) Lymphocytes decrease ANS…..A Q16.Regarding immunological response? A)Palatine tonsils covered by simple squamous epithelium B)Antibodies produce and release by plasma cells ANS….B Q17.Hymen epithelium? A) Stratified squamous epithelium B) Simple squamous epithelium ANS…..A Q18.A boy on examination a structure which is fail tu develop on ventral surface of penis.In girls which is similar to this structure develop? A)Labia majora B)Labia minora C)Clitoris ANS..B Q19.A female has low MCH,MCV,MCHC.What type if anemia is? A)Iron def anemia B)Megaloblastic anemia ANS…A Q20.Regarding B-thalassemia? A)Defect in globin chain B)Defect in globin ratio ANS….A Q21.In 60 year old man chronic diarrhea for 6 weeks causes? A) Alkalosis rather then acidosis B)K is predominant increase C)Osmolality increase ANS…A Q22.Vitamin D3? A)Animal vitamin B)1,25 cholecalciferol C)Plant vitamin D)25.hydroxy cholecalciferol ANS….B Q23.Rough endoplasmic reticulum involved in? Ans)Protein synthesis Q24.In a boy B.P270/180 & creatinine 8%.which structure is damaged? A)Macula B) JG cells

155 ANS….B Q25. GFR dec due to? A) Afferent arteriolar constriction B)Efferent arteriolar dilatation ANS….A Q26. Which metabolic effect caused by thyroid? A) Dec gluconeogenesis B) Protein synthesis C) Dec fatty acid oxidation D) Dec fatty acid synthesis ANS….C Q27. Which effect caused by reduction of p450 mechanism? A) Acetylation of isoniazid B) Oxidation of phenytoin ANS….A Q28.Glucocorticoid causes? A)Dec peripheral glucose utilization Q29. Increase respiratory effect in pregnant women due to? A) Estrogen B) Progesterone C) LH ANS….B Q30.A man with subfertility has Anti FSH antibody against FSH receptors.which of the following increase in plasma? A) Anti mullerian B) Inhibin C) Testosterone ANS….B Q31. Increase protein synthesis in muscle caused by? A) Testosterone B) Progesterone ANS……A Q32.Uterus is prolapsed but membrane intact.which is due to? A) Pelvic diaphragm B) Broad ligament ANS….A Q33.Cervix drain by? A) Internal iliac nodes B) External iliac nodes ANS…..A Q34. Bladder drainage due to? A) Hypogastric plexus B) Inf.phrenic nodes C) Internal iliac nodes

ANS….. C Q35.Corticostriate fibers contain A) Dopamine B ) GABA C) Glycine D) Epinephrine E.Glutamate ANS….E Q36.In a pregnant female with sugar level 340mg/dl.what is the treatment? Ans) Insulin Q37. Due to hydrocarbon which occur? A)Aplastic anemia B)Bronchogenic carcinoma ANS…..B Q38. A man working in mine with hilar lymphadenopathy. A) Silicosis B) Asbestosis ANS….A Q39.Man with pleural plaques show malignancy. A) Asbestosis Q40. Principle of ethics? A)Autonomy,beneficence,non-malificence and justice Q41.A female pt presented in ER in state of shock.she had transfused 10 pints of blood 2 yrs back.B.P80/50,pulse 60/min. A)Sheehan’s syndrome B)Addison’s disease ANS….A Q42. A female present with fever and chills.No h/o burning micturition.On staining gram –ve organism seen. A)E.Coli B)N.gonorrhea ANS…..B Q43.In a female major breast duct terminate into terminal ducts from terminal ducts it forms? A) Lactiferous duct B) Lactiferous sinus ANS….B Q44.A female with vasculitis and fever.on examination gram –ve endotoxin present.which is most likely causes ths condition. A) TNF B) TGF-B C) IL-1

156 ANS…..C Q45. Number of observation fall on a set point or on given scale known as? A) Data B) Distribution C) Rate D) Frequency ANS…..D Q46.In a child diphtheria vaccine is given.what type of hypersensitivity present? A) Type 1 B) Type 2 C) Type 3 ANS….C Q47.Tranfusion reaction to A+ve if we transfuse with A) AB –ve B)O+ve ANS….A Q48. Pregnant lady with gall stones? A) GGT Q49.Most likely feature in HELLP Syndrome? A)Hemolysis,elevated liver enzyme and low platelet count Q50.Amino acid which is not produced in body but essential in diet? A)Phenylalanine Q51.Resection of pons causes? A)Ceasation of respiration B)Dec rate of respiration C)Loss of respiration ANS….A Q52.Corticostriate fibers decussate at? A) Pyramid B) Internal capsule ANS….A Q53. Cushing syndrome in old age due to? A) Thyroid adenoma B) Adrenal adenoma ANS…..A Q54.Most salient feature regarding ANS: A)All parasympathetic supplied by cholinergic B)All sympathetic by adrenergic C)All parasympathetic by vagus nerves ANS….A Q55.A female with PPH now present with Acute renal failure.Defect present in?

A) PCT B) DCT C)Loop of henle ANS….A Q56.A person lose 2L water in sweat then he takes 2L of tap water.what occur? A) Inc ICF B) Inc ECF C) Inc osmolality ANS….A Q57.A pt with CO2=44,pH= 7.33,HCO3=19,What acid base abnormality? A)Metabolic acidosis B)Acute resp acidosis C)Metabolic alkalosis ANS…. A Q58.Most characteristics features of TB? A)Caseating granuloma B)Multinucleated giant cells on microscopy ANS…..A Q59.Which is DNA respiratory virus? A)Resp syncitial virus B)Adenovirus C)Herpes virus ANS….B Q60.Most closely related to lymphocytes? A) Only arise from bone marrow B) Play major role against cancer in body ANS….B Q61.Regarding heart: A) RCA supply SA node,AV node,bundle of HIS,purkingi fibers and its branches B) LCA divide into Left circumflex artery and anterior interventricular artery ANS…B Q62.In a patient dec calcium,inc phosphate,inc PTH and normal alkaline phosphatase? A)Osteomalacia B) CRF C)Vit D deficiency ANS…..B Q63.During surgery of colon CA first site of metastasis into? A) Liver B) Lung ANS…..A Q64. A female with narrow subpubic arch and inverted ischial spine.what type of pelvis?

157 A) Anthropoid B) Android C) Gynecoid ANS…..A Q65.Nasopharangeal CA associated with? A) EBV B) CMV C)HPV ANS…..A Q66.Cervical CA due to? A)HPV B)HSV ANS….A Q67.HIV female pt with associated with malignancy? A)Cervical CA Q68.Major stress hormone during surgery? A)ACTH Q69.A female met an accident had lost her son and husband.Afterward she had amenorrhea due to? A)Inc ACTH B)Low CRH C)Low dopamine ANS…..C Q70.Breast atrophy in adult male due to? A) Estrogen B) Estrogen and progesterone ANS….A Q71. Correct about thyroid gland? A) Drain into deep cervical lymph nodes Q72. Longest pro-erythrocyte stage? A)P.Malaria B)P.Falciparum C)P.ovale ANS….A Q73.Shortest pro-erythrocyte stage? A)P.Falciparum B)P.Malaria C)P.ovale ANS….A Q74. A girl with fever and black water urine? A) G6pD deficiency B) Plasmodium falciparum ANS….B Q75. Origin of oxytocin and vasopressin? A)Post.hypothalamus Q76. Auscultation of tricuspid valve best at?

A) Right lower end of sternum Q77.GVE fiber of tenth nerve contain which nucleus? A) Nucleus ambigus B) Dorsal nucleus ANS….B Q78.A female with infection of HPV comes after 2 years.Cell shows prominent nucleoli and in size? A) Dysplasia B) Metaplasia ANS….A Q79.Virus which cuases metaplasia then dysplasia? A) HPV in cervix Q80.Benign neoplasm: A) Adenoma B)Hepatoma ANS….A Q81.In dark granules containing cell IgE attaches to? A) Basophils Q82. Pulmonary artery supply to? A) Alveoli Q83. In young boy dyspnea on lying down? A) Retrosternal goiter Q84.DNA replication occur in? A)Interphase Q86.Collagen fibers? A)Acidophils B)Basophils C)Major elastic tissue in body ANS….C Q87. Protrusion of mandible by? A)Lateral pterygoid Q88.Isotonic and isometric difference is that? A) Isotonic contain more phosphate bond Q89.A boy with lesion on nose.He also has granuloma with vasulitis.what is the cause? A) Lepsrosy B) Wegener’s granulomatosis ANS…..B Q90. Pt with intensional tremors and facial paralysis lesion at? A)Pons B)Cerebello pontine angle ANS…..B Q91.Major best investigation in ITP?

158 A)APTT B)CBC C)Platelet count D)BT ANS…..B Q92.A female with CRF.she has been advised renal transplant from twin sister.The most important pre requisite for matching is? A)HLA compatibility B)ABO compatibility ANS….B Q93.Steroids cause its action on cell by? A)Activation of genes Q94.Class 3 Anti arrhythmic? A)Amiodarone Q95.Histology of cornea? A)Stratified squamous non-keratinized epithelium Q96.Women after delivery,platelet 70,000.APTT normal PT normal.Diagnosis? A)DIC B)ITP ANS…A Q97.Mechanism of DIC? A)Degradation of fibrinolytic products B)Dec platelet function C)Vit K deficiency ANS….A Q98.Pyruvic acid is used in intermediate in which step of cell cycle? A)Carbohydrate to Acetyl co A B)Glucose to Acetyl co A ANS….B Q99.A female diabetic and hypertensive best drug of Rx? A)Captopril Q100.Cataract in newborn due to which maternal infection? A)Rubella Q101.Aphasia due to lesion in? A)Frontal lobe B)Parietal lobe C)Hippocampus ANS….A Q102.Virus cuase cancer by mutation in? A)DNA B)RNA C)Proto oncogene D)Oncogene ANS….C

Q103.Sub acute endocarditis cuased by? A)Strep.viridans Q104.At day 30 fetal life,number of somites? A)40-44 Q105.Boy after RTA,diplopia and proptosis? A)Basilar artery thrombosis Q106.High cardiac output in? A)Just after delivery B)In labour C)In first trimester ANS….A Q107.Thyroid gland A)Both lobes symmetrical B)Tracheal ring lie at level of cricoid cartilage ANS..A Q108.Delayed type hypersensitivity seen in? A)TB Q109.Method detecting Gonococcus infection? A)Gram staining B)ZN staining ANS…A Q110.10% formaldehyde is used for sending biopsy specimens why? A)To prevent autolysis B)Sterilization C)To denature protein ANS…..C Q111.In female thick whitish curd like discharge present.causative organism? A)Candida albicans Q112. Cause of edema? A) Lymphatic obstruction Q113. Tachyphylaxis? A)Diminshed response developing quickly Q114. A diabetic pt with organ involvement cause of death? A)Mucor 115.During surgery management of acute high BP? A)Hydralazine Q116.Epidural anesthesia given to increase its duration? A)Inc dose B)Inc conc. C)Give with 1:20000 epinephrine ANS….C

159 Q117.Ureter most narrow at? A)Ischial spine B)Ischial tuberosity C)Sacral promontory ANS…A Q118.Artery remain in true pelvis? A)Middle rectal artery Q119.Symphysis pubis: A)Sec.cartilagenous joint 120.Female with low MCV,MCH,MCHC and sr.ferritin.what the next best investigation? A)Sr.iron & TIBC Q121.Heinz bodies present in? A)G6PD deficiency Q122.Ovary held in position by? A) Broad ligament Q123. Hallmark of HIV? A) Immunodeficiency B) Proliferation of virus inside T lymphocytes ANS…..B Q124.A baby born with erythroblastosis fetalis with blood group A+ve.He should treat with? A)A+ B)AB+ C)AANS…..A Q125.Starling law of heart? A)Explain the inc in cardiac output when venous return increases Q126.In inferior wall MI artery involved? A)Right marginal artery Q127.On x-ray right border of heart formed by? A)SVC Q128.Drug to drug interaction? A)Both pharmacokinetic and pharmacodynamic Q129.In diabetic nephropathy best investigation? A)Urine albumin Q130.Highest diffusion capacity across across respiratory membrane? A)CO2 Q131.Plasma protein return to normal in? A)10-12 days

Q132.Patient with lower backache and pain at posterior thigh.DR tells him that longest nerve of the body is involved? A)Lumbosacral plexus B)Sacral nerve ANS….A Q133.Heart prone to fibrillation? A)At the end of action potential Q134.Donut shaped organism in HIV? A)Cryptococcus 135.Person taking normal saline? A)Inc urine sodium B)Dec urine osmolarity ANS….B Q137.A female on ATT taking contraceptive but still got pregnant due to? A)Rifampicin Q138.Corneal opacity in a person due to? A)Chloroquine Q139.Glucose transport across membrane due to its conc.difference is called? A)Simple diffusion B)Facilitated dissusion ANS…A 140.Cradiac muscle can’t tetanized becoz it has? A)Long refractory period Q141.Artery arises on substance of parotid gland and lies at posterior body of digastrics? A)Lingual artery Q142. Breast upper and outer quadrant drainage? A) Anterior axillary nodes B) Pectoral nodes ANS….A Q143.Radiation causes cancer? A)5-10 years Q144.Hematocrit of 41%means? A)RBC B)WBC C)RBC,WBC,Platelets ANS…C Q145.Drug not causes gynecomastia? A) Griseofulvin B) Androgen ANS….B Q146.In female offspring clear cell carcinoma of vagina due to?

160 A) DES Q147. Bladder Carcinoma due to which organism? A)Schistosoma hematobium Q148. Simple columnar partly ciliated and non ciliated epithelium? A) Fallopian tube Q149.48 Years old lady has CA cervix that involved cervix and vagina not lower third,lymph nodes palpable and some suspicion of metastasis.what stage? A) T2N1MO B) T2NIM1 ANS…B Q150.A pt in CCU: A)feel terror and discomfort B)Behave vigorously and shout on everyone C)Keep calm and cooperate with family members and staff ANS..C Q151.Mesonephric duct functional remnant? A)Ductus deference B)Uterus ANS…..A Q152.At the beginning of 5th week? A)Limb bud appers B)Stomach rotates ANS…..B Q153.Which of the following not receive innervations from pudendal nerve? A)Posterior fornix of vagina Q154.Inferior rectal artery branch of? A)Internal pudendal artery Q155.Needle inserted at 4th intercostals space it pierces? A)Intercostal muscle B)Intercostal membrane ANS…..B Q156. A young female with secondary infertilitycome to you on 18th day of his LMP for D & C.In which phase? A)Secretory B)Luteal ANS…..A Q157.Vibrio cholera A)Grow best at 25 C B)Grow best in acidic medium C)Resistant to alkaline medium ANS…C

Q158.A 60 year pt suffering from generalized lymphadenopathy for last 8 months.Hb=6g/dl,platelet and wbc low. A) CLL B) Hairy leukemia C)Hodgkin lymphoma ANS….C Q159.Trachea terminate at? A)C6 B)C4 C)T4 ANS….C Q160.Anterior fontanelle closes at? A)12-18 months Q161.During fetal life kindney infected due to which drugs? A)ACE inhibitors Q162.In fetus carotid artery gets blood from? A)Aorta B)Ductus arteriosus ANS….B Q163.In female after some antibiotic use develop bad taste,thin watery discharge and extreme itching.which antibiotic was cause of itching and bad mouth taste? A)Metronidazole Q164.After breast surgery best chemotherapeutic drug given to pt? A)Tamoxifen Q165.Most common organ affected by SLE? A)Kidney B)Lung C)Eyes D)Joints ANS…..A Q166.Most common 2nd messenger pathway?? A)AMP B)ADP C)ATP ANS…A Q167.Cervix drain by? A)Internal iliac nodes Q168.A farmer working in field become unconscious frothing from mouth.which drug u give? A) Atropine Q169. A child with severe diarrhea and bleeding.no family history.Cause?

161 A)Vit K deficiency

Gynae and obs 14 oct 2016(evening) 1.a female having two children ,after cholecystectomy she developed pallor her peripheral blood shows microcytosis type of anemia a blood loss b iron deficiency Ans B 2.lowest O2 content in umblical circulation is in a.pulm trunk b.right atrium c.right ventricle d ivc e.svc Ans |e 3.The principal difference between cytotoxic type 2 n immune complex type 3 hypersensitivity is a. Delayed hypersensitive reaction b .The class of immunoglobulin involved c. the participation of T cells d. The participation of complement e. The site where antigen antibody complexes are found Ans E 4. A female diabetic from last 4 years, anemic, admitted with pseudomonas infection, blood sugar was normal but ater 6 days she developed shock, how a.electrolyte imbalance b.acid base imbalance c.inc plasma osmolarity d.Increased vascular permeability Ans D 5.cryptorchoidism should be treated as there is increase risk of?? ANs> malignancy. 6.temporomandibular joint dislocation will lead to which muscle paralysis?? Ans>lat pterygoid 7.unable to open mouth which muscle paralyse? Ans>lat pterygoid 8. Airborne droplet infection can spread upto a.1 to 2 feet

b.3 to 6 feet ANS b 9.maximum ventricle is filled during A) diastasis B)atrial systole C)rapid inflow D)slow inflow Ans>C 10. The neural crest cells originate at the dorsal most region of the??? Ans> neural tube. 11.uterus main support ?? Ans>pelvic diaphragm. 12.sesamoid cartilage found in a. epiglotis b .nasal septum c .larynx d. ear pinna Ans>C 13.left renal vein drain into ??? Ans>ivc. 14.left renal vein anrerior to??? Ans> aorta. 15.max renal clearance of ??? Ans>PAH. 16. A boy collapsed 2 hr after tonsillectomy .what is the reason behind this A. Hypovolumic shock B. Septic schock C. Neurogenic shock D. Cardiogenix shock E. Toxic shock Ans>A 17. A patient had to undergo amputation of right leg. At the site of amputation ha present with a mass, biopsy shows haphazard pattern. a- neuroma b- schwanomma c- lymphoma Ans>A 18.post ganglionic fibres found in a - all spinal nerve b. mid thoracic Ans>A 19.diaphragmatic surface of heart form by??? Ans> right ventricle.

162 20. In a study 40% women recived HRT 20% women have not recived HRT. the best way to represent such study is??? Bar chart pie chart chi square Ans>B 21.scenario was of case control 22. regarding free water diuresis.. A.NEGATIVE WATER CLEARANCE B,positive water clearance, (ans) c.plasma osmolarity Ans>B 23. Pt had severe dehydration, what is the best treatment to increase body water a.5% dextrose b.10% dextrose c.Albumin d.Normal saline Ans>A 24.female presented in oncology depart hypotensive fever what is the cause of this condition? Ans >TNF (IL1 was not in options) 25.true hermaphrodite a.xx b.xxy Ans>B 26. The antimotabolite which is used for nonneoplatic diseasea and is also an immunosuppresant??? A.Azathioprime B.cytarabine C.Methotrexate Ans>C 27. Methotrexate is?? Ans> a folate antagonist 28. cyclophosphamide side effect ?? Ans>hematuria. 29.eye blunt trauma which structure will be more prone to damage=?? Ans> limbus?? 30. progesterone is a precursor of.?? a.de oxycortisone,Aldosterone,cortisol and corticosteriods b.hydrocortisone,pregnenolone;estriol c.cholestrol :cortisone,cholic acid d.aldosterone .deoxycholic acid .pregnenlone

e.corticosterone.testosterone.cholestryl sulphate Ans>A 31.nipple plus lat part breast drain Into ??? Ans>anterior axillary lymph nodes. 32.lat part breast drain into ?? Ans>anterior pectoral nodes. 33. boil on labia majora will drain Into? Ans> medial group of superficial inguinal lymph nodes. 34.star shaped lumen a. ureter b.fallopian tube Ans>A 35.sensitive to radiation = lymph nodes (germ cell , git epi also v. sensitive to radiation). 36.tumour removed from ovarian fossa which structure prone to damge ?? Ans>internal illaic artery. 37. 45 yrs old woman has invasive ductal carcinoma of left breast with metastasis in fur ipsilateral axillary lymph nodes. Tumor measures 6.5cm in its max diameter. Acc to Manchester's staging. Stage of this tumor would be A. Stage 1 B. Stage 2B C. Stage 2A D. Stage 3 E. Stage 4 Ans>D 38.structure posterior to ureter ??? ans>genitofemoral nerve. 39.change of one epithelium to other is ?? Ans>metaplasia. 40.an anesthetic drug given to pregnant lady which conc inc in mother blood as compared to fetus name of drug? a. lidocaine b. bupivaciane Ans>B 41. histological section of tissue sample shows normal mitoses,pleomorphism and hyperplastic cells are observed in different stages of cell cycle with no invasion of basement membrane it is most likely due to

163 A.metaplasia B.dysplasia C.anaplasia D.malignancy Ans>B 42.an anesthetic given to a patient thorugh which route it given will cause highest conc in blood a.spinal b. epidural Ans>B 43.ovarian artery is the branch of ??? Ans>abdominal aorta. 44.Ant wall of bladder devoid of?? Ans> peritonium. 45.medial boundry of ischiorectal fossa by a.ana canal b.levator ani Ans>B 46.SLE scenario investigation of choice ?? Ans>anti Ds DNA. 47.women developed cogulation defect on ultrasound having IUD baby management? Ans>evacuation of fetus 48.illiacus muscle supplied by ?? Ans>femoral nerve. 49. ischiorectal fossa abscess drainage will result in damage to a.inferior rectal nerve b.sacrotuberous ligament c.internal pudendal artery Ans A 50.female having complain of fever productive cough diagnosed case of staph aureus pneumonia now fever perisist but gradually increased dyspnea xray chest showed air fluid level what complication she have developed? a.granuloma b.fibrosis c.abscess formation ans>C 51. sphincter urethra supplied by? a. sacral splanchnic nerves b.pudendal nerve Ans>B 53.49 year female developed pain in right big toe with fever she had same episode of pain 6months back in knee joint what investigation will u do

a. uric acid b.xray c.ANA d.anti Ds Dna e.rheumatoid factor Ans>A 54.dont remember exact words ( The greater, lesser, and least splanchnic nerves arise from the thoracic part of the sympathetic trunk and carry much of the sympathetic and sensory supply of the abdominal viscera. The nerves pierce the diaphragm and reach the celiac and aorticorenal ganglia) 55. which hernia lies medial to inferior epigastric artery?? indirect inguinal Direct inguinal correct this one is answer femoral umbilical Ans>A 56.cause of epispadasis is urethral fold fusion defect. faulty position of genital tubercle 57.Hernial sac which is ramanant of processus vaginalis and carry all layer alonwith is which type of hernia?? a. direct inguinal hernia b indirect inguinal hernia Ans>B 58.vagina histology shows abundant a.collagen plus elastic b.elastic Ans A 59. on microscopy how carcinoma diagnosed = ?? Ans> local invasion 60.cells present in epidermis which give colour pigment =??? ANs> melanocytes 61. in hypoxemia which will respond a.carotid body b carotid sinus c.central chemoreceptors Ans>A 62. A patient on hypertensive drugs presents postural hypotension. in heart rate while standing is mediated by:

164 A. decrease fire rate of baroreceptors B. decrese sympathtic stimulation C. Inc vagal tone D. Inc vasomotor tone Asn>A 63.lady having carcinoma developed generalized edema cause is ?? Ans lymphatic obstruction 64.Edema due to renal disease cause ?? Ans> salt retention plus albuminurea 65.common cause of DVT a.post operatively b.prolong bed rest Ans>B 66.old male patient case of atherosclerosis was under your treatment with blockage of coronary artery suddenly brain ischemia occured cause = a. thrombus b embolus Ans>B 67. a lady, known diabetic for 5 yrs. she is on Glibenclamide. she had an acute asthma attack for which she is still taking medication. this morning she felt dizzy her blood sugar is 40. choose the drug which interacted with glibenclamide a. b-blocking agents b. steriods Ans>B 68.cynosis due to ?? Ans> deoxyhemoglobin 69.Ineffective erythropoiesis in a. thalsemia b.sickle cell anemia Ans>A 70.when volume of water decrease all of following will increase except a. ADH b. aldosterone c estrogen Ans>C 71.ventilation perfusion ratio zero in a. Pulmonary embolism b.pleural effusion c fibrosis Ans>A 72.worst from to do immunosuppression ??

Asn>full body irradiation ( poor recall of statement but ans was full body radiation) 73.girl diagnosed with rubella what ishe indication of terminating pregnancy ?? Ans: presence of IGM 74.most abundant antibody ?? Ans:IgG 75.Established autosomal dominant disease a. heridatory tubular adenoma familial adenomatous b. hemochromatosis Asn:A 76.automatic bladder when lesion is a.above sacral segment b.below sacral spine segment Asn:A 77.bladder detrussor muscle contract due to ?? Ans:pelvic splanchnic plus hypogastric plexus 78. 1 gm of normal Hb can bind a maximum of?? Ans: 1.34 ml O2 79.scenario of hemolytic anemia 80. A patient has hb 6 .platelets 450000 e.hypercellular Bone marrow.diagnosis is A. acute blood loss? B.leukemia Ans:B 81.vitamin c required for which step in collagen synthesis ?? Ans: hydroxylation of proline and lysine residues. 82. Renal failure feature ?? Ans: oligourea 83.Pulmonary artery supply?? Asn: alveoli 84.How to differentiate b/w iron def and thalassemia . what lab investigation will u ask a. hb F b. serum ferritin c. hb electrophoresis d. mcv Ans:C 85.Single best investigation for iron deficiency anemia ??? Ans: serum ferritin 86. pregnant lady with jaundice n bile duct stone , investigation??

165 Ans:GGT 87.Bleomycin ?? Ans: pulmonary fibrosis 88.insulin secretion increase ?? a. secretin b. somatostatin C.GIP Asn:C 89.vit k plus c deficiency causes ?? Ans: bleeding disorder 90.A 14 yr child with bleeding and also diarrhea no family hostry of any bleeding disorder cause of bleeding a. Liver faliure b. Hemophilia c. Vwf d. vit k deficiency Ans:D 91. Fetal brain development is caused by a. Growth hormone b. Thyroid hormone c. ADH d. Prolactin ____________________________________ ____ Ans. b. Maturation of CNS of the fetus requires the presence of thyroid hormone in the perinatal period. Thyroid hormone deficiency causes irreversible mental retardation. Because there is only a brief prenatal period when thyroid hormone replacement therapy is helpful, screening for neonatal hypothyroidism is mandatory. 92. sertoli cell?? Ans: androgen binding protein 93. Mean arterial pressure ?? Ans: diastolic pressure plus 1/3 rd pulse pressure. 94.Hormone which produce smooth muscle of git relaxation and also increase git secretion a. substance p b.VIP Ans:B 95.Femlae presented with complain of sever headache , uncontrolled hypertension lab shows k 3.4 most probably cause of hypertension a. aldosterone

b.catecholamines (scenario of pheochromocytoma) Ans:B 96.self replicative a. mitochondria (ans) b.smooth ER c.nucleus d RER 97.essential amino acid a. tyrosine b. threonine Ans:B 98.PTH normal in a. osteoprosis b pseudohyperparathyroidism Ans:A 99.Total body water a. 60% of body weight b. ECF +ICF+ transcellular fluid Ans:B 100. Tetanus vaccination ?? Ans:Toxoid is used. 101.Hypervitaminosis ?? Ans:Scally dermatitis 102.Widest space in vertebral coulm?? Ans:L2( dis question has come many times in exam) 103.child ribs a. vertical b. on cross section almost circular Ans:b 104. Micturation reflex ?? Ans: self regenrative 105. Hypothyroidism ?? Ans: Inc cholesterol level 106.malaria in which stage inactive in body a.sporozite b.merozite c .shizont d.hypnozoite Ans:D 107.mother o negative blood group father o postive child devlope erythroblastosis fetalis child blood group? Ans:B positive 108.Fibroblasts ?? Ans: amorphous extracellular matrix 109. Scenario of malaria, black water fever?? Ans: falciparum

166 110. Post fontanalle?? Ans: smaller than anterior 111.Trachea divide at c4-c5 (rest all irrelevant options). 112. Chronic liver disease on microscopy a. fibrosis b. lymphocytic infiltration asn:B 113.In complete heart block ?? Ans:atria and ventricle contract independently. 114. pain mediator ?? Ans:bradykinin 115.Megaloblastic anemia scenario was there ans was megaloblastic anemia. 116. 23 year old woman regular menstrual cycle of 21 to 23 days .ovulation is expected to occure between cycle days: a. 10,12 b.7,9 Ans:B 117. Mesonephric duct develop from ?? Ans:intermediate mesoderm. 118. Alpha 1 agonist cause ?? Anscontraction of radial muscle of iris. 119.Non healing diabetic ulcer due to?? Ans: angiopathy and neuropathy. 120.Cimitidine ?? Ans:Hepatic enzyme inhibitor 121. Most common non bacterial infection developed in burn patient?? Ans: candida.(ref Goljan patho) 122. Post ventricular branches 2 in number and supply ?? Ans:diaphragmatic surface heart. 123.How u differentiate chest lesion being or malignant?? Ans = recurrence after excision 124. prolactin raised with infertility what will be cause a.microprolactinoma b.macroprolactinoma c.pituitary tumor Ans:b 125. plasmodium malarie ?? Ans= quartan fever 126. Highest clearance ?? Ans: PAH 127. Gynecoid pelvis outlet ?? Ans= AP > transverse dia

128. Bacterial vaginosis lead to ?? Ans:preterm labour. 129.Deep inguinal ring ?? Ans: facia transversalis 130.true about pituitary?? Ans:Pituitary drain in dural venous sinuses. 131. other was about pitutary ans was Post sup relation of pituitary is sphenoid. 132. pain on defecation mediated by pudendal nerve. (inf rectal was not in options)' 133. fast pain A delta fibres 134. maximum inc cardiac ouput in pregnant lady is after delivery. 135. ocp prevent ovarian ca. 136. adrnelectomy done which replacement necessary for life a. aldosterone b. corticosteriod c. epinephrine Ans:B>A 137.How to check staph aureus virulence a. coagulase positive b. hemolysis Ans:A 138. GTO prevent Ans:tearing in overstretching. 139. Tension detect?? Ans: by golgi tendon. 140.Diabetic nephropathy confirm ?? Ans:by urine albumin 141. most common brain tumor in ?? Ans:adult glioblastoma multiform 142. uterus absent because of malformation of ?? Ans:paramesonphric duct. 143.paramesonephric duct male remnant a? As:ppendix testis. 144. second heart sound ?? Ans:closure of Aortic and Pulmonary valve. 145.excessive hemolysis bilirubin bind with albumin and go to liver. 146.EBV associated with ?? Ans:nasopharyngeal carcinoma. 147. Antibodies againt ACH receptors = ? Ans:mysthenia gravis 148. first pass effect avoided by giving drug a. IM (ans)

167 b. rectal c oral d oral suspension form iv was not in options 149.Endometriosis treatment danazole. 150. Highest bioavailability metachlopromide.

GYNE OBS 8TH NOVEMBER 2016

1- Tumor suppressor gene A.N-myc B.C-myc C.P 53 D. C- erb E.ras Ans:C 2-Shortest acting local anesthetic A.Procaine B.Lignocaine C.Ropivacaine Ans:A3-Risk measurements, exposed group vs non exposed A.Cohort study B.Case control C.Prospective D.None of above Ans:A 4-Data from smaller study on one group of people iscombined with data from larger study.what type of data is this A.Case control B.Cohort C. Randomized control trial D.Metaanalysis Ans:D 5-Chi square is used to measure A.Statistical analysis B.Confidennce interval C.Sensitive D.Specificity E.Valadity Ans: A 6-Automatic blader cause A.lesion above sacral segment B.lesion below sacral segment C.Frequentmictuiration D.loss of sensory supply Ans:A

7-..Increase in respiration during luteal phase of menstrual cycle is due to A. Fsh B.lh C. Estrogen D.Testosterone E.Progesterone Ans E 8.Regarding lesser pelvis A.has a cavity whose post wall is much shorter than ant B.Outlet is bounded by sacrotuberousand sacrospinous C. has relatively longer antposterior diameter at the inlet D.has a small sub pubic angle E. Is generally circular in cross section Ans:B 9-Effect of estrogen on cervix A.Cervicaladenosis B.Sqamous cell metaplasia. Ans:A10-Pt on ot table heat loss through A.Conduction and radiation b.Conduction c.Radiation d.Evaportion e.Convenction. Ans:A 11-Pt having butterfly rash n multiple miscarriage a..sle b.rehmatoid arthritis c. scleroderma d. rheumatic fever e.sjogren syndrome Ans:A 12-True about implantation of zygote A. Occurs 3-5 days after fertilization B. Occur when 128 cells structure something formed C.invasion of endometrium by syncitiotrophoblast D.progesterone inhibit implantation by acting on corpus luteumAns:c (asim bcq). 13-Pregnat female at 20 weeks gestation withhyperthyroidism.best treatment A.propylthiouracil B.thyrodectomy C.no treatment till delivery Ans:A 14-Nerve supply of levatorani

168 A.hypogastric B.illioinguinal C.pudendal nerve Ans:C15-Contents of superficial perinealpoch A. Perineal body. B. Pudendal vessels C. Bulbo urethral glands Ans:A 16-Pt with pallor... mcv raised cause A.folic acid deficiency . B. Sideroblastic anemia C.IDA Ans:A 17-On histopathology partially celiated, partially non ciliated A.Uterus B.Cervix C.Uterine tubes Ans:C 18-Breast milk secretions contain A.IgA B.Igg C.IgM D.Igd E.IgE Ans:A 19-At what age the head of fetus become half of CRL length A.3rd week B.strt 3rd month C.strt 4th month D.6th month E. 7 month Ans:c 20-Inflamation of the portal triads which spills into adjacent lobules bands of inflammation extending between portal areas and terminal hepatic veins and foci of liver cell necrosis throughout the lobule is most likely pic of. A.chronic acute hepatitis B.cirrohsis C.chronic passive congestion D.chronic persistent hepatitis E. fulminant hepatits AnsA: 21-Stucture between tranverselismuscle and extraperitoneal fat A.internal oblique B.ext oblique

C. Fascia transverselis. D.skin E. Sup Fascia Ans:C 22-Pt given harmonal therapy for metatstatic cancer. A .Apoptosis B. Atrophy C.hypertrophy D. necrosis Ans:A 23-Virus causing alteration in A.DNA B. RNA C. Proto oncogene D. Oncogene Ans: C 24-Diaphragmatic hernia is associated with A.lung aplasia B.surfactant deficiency Ans: A25-Trachoma.. A.chlymadia B.crcinoid C.condyloma D.lymph granuloma venerum, Ans:A 26-Pt with bicornuate uterus. Associated anomalies are A.git B.cvs C.resp D.utI E.Renal Ans:E 27-Pt in labour for 2 hours.on pv inverted spines,narrowsubpubic arch and sacral promontory not reachabletype of pelvis A.Gynaecoid B. Android C.platypalloid D.anthrapoid.. Ans:b Explanation :(Traditional obstetrics characterizes four types of pelvises: Gynecoid: Ideal shape, with round to slightly oval (obstetrical inlet slightly less transverse) inlet: best chances for normal vaginal delivery. Android: triangular inlet, and prominent ischial spines, more angulated pubic arch. Anthropoid: the

169 widest transverse diameter is less than the anteroposterior (obstetrical) diameter. Platypelloid: Flat inlet with shortened obstetrical diameter). 28-Niacin deficiency... A.Plegrra B.Dermatisis C.Chelosis D.Periperal neuropathy Ans: A 29-Plataeu is formed due to A. Na and CaChAnnels B.Voltage gated Na channel. c.k channels. Ans:A(ca chaneels wasn’t in options)30which part of blood test should be sent for matching histocompatibility of the recipient and donar A.Wbc B.Rbc C.Sserum D.Bone marrow E.Plateles Ans:a 31-cortisol req for ? A. Thyroxine B. Epinephrine C.Glucagon and Epinephrine Ans: c 32-Increase glucose, fatty acid and protein in serum A.Insulin . B.Growth hormone. C.Cortisol Ans:c 33-Well differentiated sq cell tumor a.Loss of polarity. b. Keratin or something. Ans.B 34-Maximally filling ventricle with blood during which phase A. Diastasis B.Rapid inflow C.Atrial systole D.isovolumetriccontraction Ans:B 35-Atropine same action. A.physostigmine B.Scopolamine Ans: B 36-lack of lymphoid nodules

A.thymus B.spleen C.lymph node Ans: A 37-most common cause of malignancy in workers A,tobacco industry b.silicone C.asbestosis Ans: A38-maximum bioavailability ? A.I\M B.sublingual C.per rectal .D.Orall Ans:B(i/v wasn’t in option) 39-most common cause of fatty liver in our country A, hep b and c B.alcohol C.fatty food D.protein malnutrition Ans:D 40-Lacunar is extension of inguinal ligament..notreflectd 41-poliomyelietis . A.anterior horn cell injury B.Virustraveles along peripheral nerve. Ans:A 42.Ocp A.inhibiting LH B. inhibition of ovulation C.thickening cervical muous D.hepatic adenoma Ans:b... 43-In pregnancy A.muscle bulk increase cz hypertrophy andhyperplasia B. uterus remain relaxed till labour C.breast size inccoz of prolactin D.BMR increases by 10% Ans: A 44-Infection from pretacheal region can travel to... A.superior mediastinum B. anterior.mediastinium. C. Middle mediastinum D. posterior mediastium Ans:B 45-step which is same in glycogeneolysis and glycogenesis?? A. glucose6 phosphate

170 B.fructose 6 to 1,6 bi phosphate... Ans:B 46-Pelvic outlet most appropriate A. AP diameter more than transverse B.Tranverse diameter more than AP diameter Ans:a 47-Stratified squamous epithelium A. renal tubular cells B.duct of salivary gland C.corneal epithelium D.bulbular conjunctiva E.ovarian follicle Ans: B 48-sympathetic stimulation causes?? A. bronchodilation.. B. vasodilation of skin Ans:A 49-Which is the content of superficial perineal pouch A. perineal body B. membranous part of urethra C.bulbuourethralsphincture D.internalpudenal vessels Ans:A 50-Radiation effect. A.Protooncogenes B.Oncogenes C.DNA D.RNA Ans: A51-Uterine artey arises from internal ilac artery cross above? A.ureter.. B.fallapion tube C.ligament of ovary D. urinary bladder Ans:A 52 most commonuti... A.staphlococci B.Echoli C.streptococci Ans:B 53-structure posterior to ureter? A.genitofemoral nerve.. B. gonadal vessels C.illeocoelicvessesls D.left colic vessels E.root of mesentry Ans:A

54-Platelets ridges/lines of zahn are present inA.Propagated clot B.Coraline thrombus.. Ans: B 55-platelets . A.fragments of megakarocytes Ans:A 56-Prostacyclin formed from A.edothelial cells B.plateles C.peyerpathes D. mast cell E. fibroblast Ans: A57-cardiac output increase by A. venous compliance B.Inrease Blood vol... C.Increase interstitial fluid D. increase oncotic pressure. E.increase capillary permeability Ans: B 58-Length of adult ureter. A.25-30cm B.20-25cm C.30-35cm D.35-40 cm E.15-20 cm Ans:A (kLM) 59-Herniation of meninges from skull A.craniomeningeocele B.anencephaly C.cranioschisis D.craniosynostosis E. encephalocele Ans A 60-Oxygen hemoglobin curve shift to right side? A.alkalosis B.acidosis Ans:B 61-Oxygen hemoglobin curve shift to left in A. Hyperthermia B. CO poisining C. Ans.B 62-Radiation late effect..... A.endarteritis obliteration. B.Damaged of epithelium Ans:A 63-Endocervix epithelium change. A. Metaplasia

171 B. dysplasia C.hpv D.papilloma E.carcinoma Ans:A 64-Radio sensitive A.Ovary B.Cervix C.Endometrium D.Lymph node. Ans:D 65.secondary center of ossification in long bones develop in. A.epiphysis B.metaphysis C.epiphysis plate D.diaphysis E.synovial joints Ans:A 66-Structure developing from neural crest?? A. Autonomic ganglion B. adrenal gland C. Neural plate D.sensory ganglion Ans:A 67-Inc mean systemic filling pressure with A.decreased venous resistance B.exercise. C. Hypothermia D. temperature Ans: A 68-Mostly growth factor receptors present on A.plasma membrane(via tyrosin kinase) B.Neuleus C. DNA D.Roughenoplasmic reticulum E.mitochondial Ans:A 69-Highest PO2 is in A.arterial blood B.venous blood C.right atrium D.left ventricle E.pulmonary capillaries Ans:D 70-Oliguria... A.less than 500ml B.less than 50 ml C.less than 100 ml

D.0.5 to 1 L Ans:A 71-Left supra renal vain drain into A.left renal vein B.Abdominal aorta C. Iva D. Azygous vein Ans:A 72-Regarding CSF.. A.Provide nutrition B.prevent brain injury by cushion effect Ans:B 73-Internal oblique apponeurosis forms? A.Conjoint tendon B.Internal spermatic fasica C. Transcersalis fascia Ans:A 74-Drug similar in physiological functions like Atropine A.scopolamine B.physostigmine Ans:A 75-Coccygeal ligament most superior border where all three meningeal layers present A.L1 upper border. B. lower border of L1 Ans: B76-Structure not Pierced A.tunica albuginia B.skin C.cremasteric fascia D.internal muscle and colles fascia E. internal spermatic fascia Ans:A 77-In hypothyroidism... A. TSH B. T3 C. T4 D.T3 & T4 Ans:A 78-In pregnancy iron deficiency anemiadiagnostic investigation A.serum iron B. serum ferritin C. TIBc D. Ferritin n TIBC Ans: D 79-Structure between peritonial connective tissue and transversus muscle

172 is A. abdominis fascia B.transversalis fascia C. TRAnverses muscle Ans:B 80-Ectopic thyroid is secreted in which carcinoma A.sqauamous cell ca larynx B. small cell carcinoma of lung C.Small cell carcinoma of larynx D. adenocarcinoma of colon Ans:A 81-Milk not produced in pregnancy despite increased prolactin hormone inhibiting it isA..estrogen b.progesterone c.oxytocin Ans:B 82-Triglycerides high in A.chylomicrons B. HDL C .LDL D. vLDL Ans:A 83-Acidosis A. Increases ventilation B.decreases ventilation Ans: A 84-Nissel granules absent in A. axon B. hillock C. dendrite D.cell bodies Ans:B 85-Hypokalemia causes A.Increased h ion in plasma. B. decreased ph of plasma. C. increased negative base in plasma. D .increased bicarbonate in plasma. Ans: D 86-Cervical malignacny. A.hpv B.hsv

C.cancer D. metaplasia E.dysplasia Ans:A 87-Pt with rigors n fever suprapubic pain, gram negative rods on stainA.E.coli B.streptococci C.H. influenza D.Nesseriamenigitidies Ans:A 88-Circle of willis A.Post. Communicating artery connect basilar to internal carotid Ans:A 89- Cerbral cortex is supplied by which artery A. anterior and middle cerebral arteries B.ant and post cerebral artery C.basillar artery. D.vertebral artery E.middle and posterior cerebral Ans:A 90-Autosomal dominant= A. both parent of affected child have disease B.only 1 parent of affected child have disease C.only males are affected D. only females are affected Ans:B 91-diaphragmatic hernia is associated with A. pulmonary agenesis B.Surfactant deficiency C.Bronchial obstruction Ans:A 92-Primigravida 2nd trimister, gelt sudden lower abd severe cramp followed by watery foul smelling discharge, after 2 days deliverd a still born child. On ex chorion and placenta have neutrophilic infiltration, cause by which organism? A. Mycobac B. Hsv C,Toxoplasmosis Trponema E. E.coli

173 Ans: E 93-vulva is supplied by A. Pudendal nerve B.illioinguinal nerve Ans:A 94-potassium depletion is associated with?? A. bakers dystrophy B. mysthenia gravis C.duschene muscular dystrophy D. Familial periodic paralysis Ans: D 95-blood pic shows macrocytosis hypochromic pic? dx is? A. folate deficiency B.irondeficiency C.sideroblastic anemia D.megaloblastic anemia Ans:D 96-Pt with blood grp A has... A.Anti A antibodies B.Anti AB antibodies C.anti B antibodies D.no antibodies Ans:c 97-messeter is supplied by which nerve.. .A. mandibular v3 B. maxillary nerve C. facial nerve D. trigerminal nerve Ans: A98-.which one has transient adhesive property. A. cytokines B. imunoglobins C. c5-9 D. integrins E. selectin Ans:E 99- 2 liter fluid loss in sweat and person takes 2 liter distilled water what will happen..A.decicv B.dec intracellular osmolarity C.inc intracellular osmolarity D.inc ecfvoloume E.dececfosmolarity

Ans:E100- Severe dehydration causes.. A.total body water loss B.loss of ecf volume c.Loss of icf volume D.acidosis Ans:A 101-Abnormal over growth and irregular thickening of bone caused by A.chief cells of parathyroid B. Cells of adenohypophysis C.cells of parsintermedia D. Leydig cells AnsB102-Foramen Ceacum is embryonic derivative of? A. Thyriod. B.Thymus C. Ant pituitary D. Post pitutry Ans:A 103-lady, known diabetic. she is on Glibenclamide. she had an acute asthma attack for which she is still taking medication. this morning she felt dizzy her blood sugar is 40. choose the drug which interacted with glibenclamide A.-blocking agents B. steriods C.insulin D.beta blockers Ans:B 104-Tb vaccination…. A. hypersenstivity type 4 B. cell mediated C.antibody mediated D.type 3 Ans:A 105-Tubular structure with star shaped lumen lined by transitional epithelium.. A. ureter B.fallopian tube C.oesophagus D.stomach Ans:A 106-call exner bodies present in a.sertoli cell tumor b. ledying cell tumor c.granulosa cell tumor d.dysgerminoma

174 Ans:C 107-12yr old grl passes pieces in menstrual flow...worried about tiny piece of tissues she passed in her menstrual flow.she is reassured and explain that thsese piece are likely to be A.endometrial sheds B. Unfertilized ovum C.small clots D.cervicalepithjelium E. pieces of spiral artery Ans: A108-Collagen type in skin A. 1 B.2 C.3 D.4 Ans.A 109-Free nerve endings ... A.non encapsulated receptors.... B. detect pain only C. type of special visceral enteroceptor D.found in volountry muscle E. not present in epidermis of skin Ans: A 110-Lymphatic drainage around anus. A.Medial group of horizontal superficial nodes B. lateral group of horizontal superficial group C. vertical group of node D.internal iliac nodes E.external iliac nodes Ans: A111-Which hormone regulate prolactin ..... A.dopamine(Prolactin inhibitory hormone) B.cortisol C.growth hormone Ans: A 112-Pregnent lady 34weeks with vaginal bleeding and coagiulation problem . frist step of managment was A.. evacuation of uterus B.blood transfusion C.heparin therapy D.platelates transfusion E.streptokinase

Ans: A113-Uterine tube epithelium. .A. ciliated columnar ans.A 114.secreti0n of prostate contains a. Acid ph0sphatase B. Alkaline ph0sphatase C.Fructose Asc0rbic acid Ans: A 115-HCG has alpha and beta units.each unit has amino acids A.Alpha subunit 92 and beta 145 B.alpha 102 and beta subunit 210 C. 100 alpha and 160 beta Ans:A 116- Tenth intrc0stal nerve enter in rectus sheath through A. Xiph0id pr0ces B.Pubictubercle C.Pyramidalismuscle D.umbilicus Ans:D 117-male narrowest part of urethra A. externa meatus B.prostatic part C.membranous part D.pelvic brim Ans:A 118-One molecule of glucose how many atp A.38 B.30 C.10 D.20 E.4 Ans:a 119-Avidin A.Biotin Ans:A 120-Esr increase in A. anemia B,polycythemia, C.infection D.ccf Ans:C 121-Structure arches over Root of left lung A. Aorta

175 B.azygous vein C.phernic nerve D.vagus nerve Ans:A 122-2% lidocaine A. 4ml B.8ml C.20ml D .10ml Ans:B 123-48 hrs child meningitis ? A e.coli B.Gp A strep Ans A GBS was not in option so ECOLi here It is ASIM SHOAIB Mcq 124-Oestrogen increase risk of A. thromboembolism B.dvt C.hepatic carcinoma D. adenoma Ans:A 125-Beta thalasemia trait A.5%... B.10% C.1% D.15% Ansa:A126-Purpra on whole body A.Defect in capillary permeability B. Dec. Platelet count LESS THAN 90000 C.dec platelets less than 20000 D. itp Ans:C 127-Baby born..apnoicspells.no breath sounds on left sideimprove on upright postureA.pDA B.VSD C.Diaphragmatic hernia D.ASD E.TOF Ans:E 128-1st enzyme to appear after 2 hr after mi. A. ckmb B. LAD

C.tropinin T D.tropnin I E. alkal;aine phosphate Ans: A129-Muscle of mastication A. ant and post belly of diagastric B.stylohoid C Lateral Ptergoid D.stapedus E.tensor tympani Ans:C 130-Milk has excessive. A. igA B..igg C.ig m D. ig e Ans:A 131-True abt testes A. para aortic node B.superficial inguinal node C.internal iliac node D.celiac node Ans:A 132-Chemotectic factor Is A.nitrous oxide B. C3b C. C5-9 complex D. leukotrine B4 E.histamine Ans: D133-Abt doctor patient relation and how good a doctor is known by= A.his degree B.his records C.good bedside manners and polite behavior D.punctuality Ans:C 134-Severe mg deficiency causes.. A. hypocalcmia B.Hypophosphate C.Hypokalemia Ans: A135-Regarding thyroid drain into A.deep cervical lymph nodes B.superficial cervical lymph node C.submental lymph node Ans:A 136-Upper lateral quadrant of breast is drained by which grp of nodesA.anterior

176 axillary lymph node B.post group of axillary lymph node C.internal mammary lymph node D.postintercostals lymph node Ans:A137-Symphysis pubis A.secondry cartilaginous joint B.primary cartilaginous joint C.fibrocartilagenous D.syndesmosis E.synarthrosis Ans:A 138-Costochondral joint A.primary cartilaginous joint B.fibrocartilagenous joint C.secondarycartilahenous joint Ans:A 139-Sensations of urinary bladder before emptying is carried by which tract... A.Anteriorspinothalamic track B.Postspinothalamic track C.Dorsal col medial D.leminicus lateral E.lateral sphinothalamic tract . Ans:E 140-Superficial temporal artery is accompanied by which nerve= A.auriculotemporal nerve B.glossopharngeal nerve C.deeppetrosal nerve D.vagus nerve Ans:A 141-Pt having chills n fever 2 days later she complained of weakness and passage of black colouredurine.Amobile health team was in the area .did a sample test and diagnosis it as the most likely cause A. falciparum malaria B.renal stone C. G6ph deficiency D.paroxymal nocturnal hamoglobinuria E.poststrtetococcalglomerculonepharitis Ans:A 142-Which is benign a.Lymphoma b. Leiomyoma c.Melanoma

d.hepatoma Ans:B 143-Double peritoneal ligament is A.round ligament B.BroadLiagamnet C.ovarian ligament D.tranverse cervical (cardinal ) liagament . Ans:B 144-During hypoxia changes will occur??? A. Increase oxidative phosphorylation B. Reduced pH due to acidosis C. all the processes will be blocked due to lack of ATP or due to hypoxia D. anaerobic glycolysis forming lactic acid. Ans:D 145-Tumour regression by A.necrosis B.Infraction C.apoptosis D.ischemia Ans:C 146-maternal mortality calculation 50 mother died n 10000 thousand live births A.MMR=5/1000 live births ans:A (asim bcq) 147-which has anabolic action and increases lipolysis and gluconeogenesis A.thyroxine B.Insulin C.GH Ans:A 148-neurogenic shock A.dec vasomotor tone B.occur in comatose pt C.tachycardia Ans:A 149-BP 80/50 ,Inc LDH, DEC CVP A.Hypovolemic shock B.Septic shock C.Pericardialtamponade C.CCF D.cardiogenic shock Ans:A

177 150- Patient with some haematological disorder developed dyspnea after pronged hospital stay On choincpulm gradient something The primary cause is A.MI B.Primaryhaematological disorder C.Pulm.HTN Ans:C 151-increase ICP A.Halothane B.Isoflurane C.Sevoflurane ANS:A 152- follicle with oocyte surrounded by zonapellucida and multiple granulosa layers plus antrum containing fluid A.Primordial B.Primary follicle C.Sec follicle D.Tertiary follicle Ans:C 153-how to prevent congenital rubella A. avoiding contact B. vaccination in young females C.vaccine to babies D. family education Ans:B 154- Gfrdec in A.afferentartiole constriction B.afferentartiole dilatation C.efferent arteriole constriction D.dec angiotensin Ans:A 155-Lysosomes A.Hydrolytic enzymes B. oxidase enzyme C.produce from golgi D. formed from ribosome Ans:A 156-regarding fetal heart A.Endocardial cushions divide it into rt and left atriovencular chambers Ans:A 157-chorioamniotis cause A. E.COLI

B. toxoplasma Ans:Aq 158-destroyed in substantianigra A.Dopamine B.Acetycholine C.epinephrine and norepinephrine ANS:A 159-actinomycin D mechanism of action A.Binds DNA with hydrogen bond B.Inhibits mRNA and RNA Polymerase C.Inhibits t RNA D.Inhibits ribosomal RNA Ans:A 160-Gonorhea easy method of detection A.gram stain B.z N stain C. biochemical method D.il6 Ans:A 161-sepsis diagnosis Criteria A.Wbc<3000 or >15000 B.Blood culture postive C.Heart>100 D.Heart<100 Ans:b 162-preganglionic fibres A.A alpha fiber B.Abeta fiber C.B fiber D.C fiber E.A gamma fiber Ans:c 163-Dec plasma osmolarity by infusion of A.NS B.VASOPRESSIN C.aldosterone Ans:B 164-MOST IMP SOURCE OF CREATININE A.SKELETAL MUSCLE B.Kidney C.Heart D.liver Ans:A

178 165-B12 Absorption in A. Terminal Ileum B.colon C.jejunum Ans:A 166-Twave appear A.ventricular repolarization B.ventricle repolarization C.atrial systole D.ventricle systole Ans:A 167-milk is notoriously deficient in A.pantothenic acid B. vitamin c C. vitamin a D.iron E.riboflavin ANS:a (cpsp key) 168- There is variation in incidence of pre eclampsia .most significant change in fall of dis is d/t A. Dec induction of labour B.dec edema of preg women C.fall in maternal and perinatal mortality D.inc c/s rate E. unanimous definition of disease Ans:c 169-c AMP mediated mechanism in membrane A.Receptors B.Pumps C.Carrier proteins D.Enzymes Ans:a 170-erytropoitein synthesis A.cells in peritubular capillary B.maculadensa C.juxtaglomuler cells Ans:a 171- 30 years old lady after cholcystectomy..she complains that her wound is not healing. Her diet contains increase protein intake and lacks fruits and vegetables. .factors responsible for delayed wound healing A. Dec formation of granulation tissue

B. Dec synthesis of type 3 collagen C. Dec tensile strength of collagen Ans:c 172- of 20 women with neonates getting tetanus 19 had not taken a tetanus toxiod vaccine and of 40 whose babies did not have tetanus.30 had taken two tetanus toxiod shots during pregnancy .the odd ratio will be?? A.10 B.11 C. 7 D. 8 E.9 Ans:e 173-highest resistance in A.Arteries B.Veins C.Arterioles D.Capillaries Ans:c 174- thymus A. regress after birth B.formed by 4th pharyngeal arches C.produced T cell D.none of above E. B cell maturation occur in thymus Ans:c 182- Sub arachnid space contain A. s2 s3 B.S1-S2C.S3-4 Ans.A 175.pregnant lady in her late trimester came to opd …pale…noprevious antenatal checkups done..to find out type of anemia which ix. Is best a.ferritin b.tibc c.hb electrophoresis Ans. B 176.least role in healing A.proteins b.vit c c.ascorbic acid d.factor 7,9Ans.A

179 177.side effect of antihyperlipemic drugs A.gi effects b.peripheral neuropathy 178.50 yr old para7 hving 3rd degree prolapsed ..weakness of which ligament A.transverse cervical ligament B.pelvic diaphragm C.uterosacral ligament Ans: B 179.longest pro erythrocytic time A.vivax B.malaria C.falciparam Ans.B 180.decrease venous return A.leg muscles paralysed Ans.A 181.muscles derived from A.mesoderm B.,ectoderm C.endoderm Ans.A 182.sriatum A.putamen plus caudate B.putamen plus globus pallidus C.caudate plus globus pallidus Ans.A 183.dynamic support of uterus A.pelvic diaphragm, B.transverse cervical ligament C.uterosacral ligament Ans.A 184.female with vaginal discharge given treatmentnow presented with bad mouth taste and thick vaginal discharge with itching drug given was? A.metronidazole B.clindamycin Ans.A 185.respiratory symptoms A.ascariasis Ans.A 186.Good analgesic bad anaesthetic A.NO Ans.A 187.depletion of potassium in body

A.inc.Na2co3 Ans.A(asim bcq) 196.pacinian corpuscle A.pressure B.high frequency vibration C.touch Ans.B 188.roof of ant.horn of lateral ventricle A.body of corpus callosum Ans.A 189.volume of distribution not affected by A.age B.sex Ans.B 190.pregnant lady using antihypertensive drugs..fetal kidney affected..drug used A.captopril Ans.A 191.incidence A.noof new cases in a total population at risk Ans.A 192- Pseudomonas toxin A.endotoxin B.exotoxin C.erythrogenic toxin d.toxic shock syndrome Ans. B

GYNAE 9 NOVEMBER MORNING PAPER

Q1. Irritant receptors in the airways a.carries by myelinated fibres b.rapidly adapting c.stimulate cough and bronchoconstriction ANS: C Q2.a 45 years man, smoker working in tyre factory developed transitional cell ca. cause A.smoking b.aromatic amines ANS:.A Q3.cimitidine main side effect a.effect liver mtabolism (Ans:) Q4.ovary lymph drainage a.paraaortic lymph nodes

180 b.iliac nodes c.inguinal nodes ANS:.A Q5.epithelium in intestine a.simple columnar non ciliated epi b.ciliated columnar epi c.stratified columnar ANS:.A Q6.True hermaphrrodite a.xxy b.xx/xy c.xx d.yy ANS:. A CPSP KEY Q7.Increase carbohydrate consumption requires a.thiamine b.riboflavin c.biotin ANS:.A Q8.Man had lipid profile after having burger and fatty food.. his blood shows whitish frothy appearence a.chylomicron b.vldl c.Hdlp ANS:.A Q9.Cervix lymph drainage a.internal iliac b.paraaortic ANS:.A Q10.max effect of 5% lignocaine in spinal anaesthesia can b achieved by a.injecting while pt is sitting b.coughing c.giving adrenaline ANS:.C Q11.amount of o2 binding to 1g of hb a.1.34gm b.2g c.2.34g ANS:.A Q12.safe dose of fio2 in pregnant lady a.0.5 b.0.65

c.0.8 Ans:>A Q13.L1-L2 forms a.inferios hypogastric plexus b.mesenteric plexus c.lumbar plexus ANS:.A Q14.detrousor muscle contraction is controlled by?????? a.parasympthetic from pelvic sphlanchic b.pudendal s1 s2 s3 ANS:.A Q15.In RTA due to trauma to sacrum and ischeal spine it can rupture which viscera??? a.urethra b.urinary bladder c.anal canal d.rectum ANS:.A Q16.no of functional ova after 2nd meiotic division a.1 b.2 c.3 d.4 e.8 ANS:.A Q17.structure to ryt side of trachea in sup mediastinum???? a.Right vagus b.aorta c.lymph nodes ANS:.A Q18.location of submandibular gland in neck a.bounded by two bellies ofdiagastric and mandible b.other options were anterior n posterior triangle boundries ANS:.A Q19.after emerging from jugular foramina immediate relation of internal jugular vein a.accesory nerve b.carotid sheath

181 ANS:.A Q20. granulosa cell tumor contain Ans:.call exener cells Q21.safest site of pleural effusion drainage a.lower border of intercostal space in mid axilary line b.lower border of intercostal space in ant axilary line c.upper border of intercostal space in mid axilary line d.lower border of intercostal space in mid clavicular line ANS:.A Q22.pudendal nerve root valu a.s1 s2 s3 b.s2 s3 s4 c.l1 l2 l3 ANS:.B Q23.pregnant lady e- fever chills rigors and black urine cause a.plasmodium falciparum b.G6pd dficiency ANS:.A Q24.nitrous oxide when given wid o2 in 50:50 best acts as a.good analgesics b.good anasthetic ---------------------------------------------------------------------------c.relaxes uterus well ANS:.A Q25.in theory ur null hypothesis is rejected it meAns: a.there is difference btw observed values b.there is no difference btw observed values ANS:.A Q26.relation of left kiidney posteriorly a.body of pancreas b.Diaphram Ans:>b Q27.relationship of supra renal gland a.supplied by thoracis sympthetic/ b.left lies on left central tendon c.right lies on right central tendon Ans:>A

Q28.period btw remission and reappearence of symptms a.recovery b.incubation c.latent ANS:.C Q29.malignant tertian malaria a.falciparum b.malarae c.vivax ANS:.A Q30.etiologic agent of disease best determined by a.incidence b.prevalence c.disease index Ans:>C Q31.systemic hemosiderosis due to a.excessive iron intake b.decrease iron utilization c.hemolytic anemia ANS:.C Q32.thelesemia trait investigation a.dec MCH MCV B.INC HBF C.normal HBA2 D.decrease ferritin AND.A Q33.Hormone given as adjuvant in depression therapy???? a.ACTH B.Cortisol c,.thyroxin d.prolactin ANS:.C Q34.about left kidney a.separated from psoas by quadratus lumborum b.PCT lined y cuboidal epithelium An>A Q35.oocyte surrounded by cell layer called a.zona pellucida b.comulus oophorus ANS:.A Q36.a long scnerio asking at the end b/w

182 trAns:versus muscle n parietal peritonium structure a.trAns:versus fascia b.internl oblique c.visceral peritonium ANS:.A Q37.specific side effect of cyclophosphamide ANS:.hematuria Q38.ramnant of urachus ANS:.allantois Q39.birth canal is supplied by a.pudendal nerve b.parasympthetic c.obturator nerve ANS:.A Q40.urogenital triangle is supplied by a.ilioimguinal nerve b.pudendal nerve c.obturato ANS:.A Q41.inferior rectal artery is branch of a.internal pudendal b.internal iliac .femoral ANS:.A Q42. anterior interventricular artery is accompanied by a.greater cardiac vein b.lesser cardiac vein ANS:.A Q43.ca ovary ..surgically ovary removed contains epithiliod cells and lymphocytes a. granuloma pyogenes b.granulomata formation Ans:>B Q44.non caseating necrosis.cause ANS:.Sarcoidosis q.45.tractus solitarios carries ANS:.taste fibres Q46.bp falls belo 50 mmhg .main control by Ans:>Cns ischmic response Q47.Proliferative epithelium containing tortous glands thickened epithelium which phase

a.secretory phase b.proliferative phase ANS:.A Q48.mid luteal phase surge a.LH b.progeterone ANS:.A Q49.scnerio young girl ulcers on body ,howell jolly bodies/ a.sickle ceell anemia b.thalasemia ANS:.A Q50.patient lost2 litres of blood what will decrease a.stroke volume b.vascular resistance ANS:.A Q60. What prevents lactation during pregnacy a.high prolactin level b.high estrogen and progestrone c.high level oh HPL D.low growth harmone ANS:.B Q61.HCG is a.91 ,145 aminoacids b.44,67 ANS:.A Q62.what effects the positive predictive value a. incidence b.prevalence c.specifity d.sensitivity ANS:.B Q63.2+2 (2 into 2)is a.chi-square b.t table c.student test ANS:.A Q64.histolgy of tissue shows partly ciliated and some non ciliated columnar epithelium the tissue is a.uterine tubes b.uterine cervix c.cervix

183 d.intestine ANS:.A Q65. A test which help in diagnosis n eliminating ths disease a.sensitive b.specific c.positive predictive prevalence ANS:.A Q66.lymph drainage of upper outer quardrant of breast a.pectoral lymph nodes b.anterior axillary c.internal thoracis d.clavicular ANS:.B Q67.medial quadrant of breast drains into a.internal thoracis nodes b.anterior axillary c.infraclavicular nodes ANS:.A Q68.necrosis in brain a.coagulative b.liquefective c.fat necrosis ANS:.B Q69.most common cause of pulmonay embolism a.DVT b.mural thrombus c.MI d.surgery ANS:.A Q70.in chronic hmolysis ANS:..decrease heptoglobin Q71.the antihypertensive best to use in diabetics a.captopril b.b blockers c.ca channel blockers ANS:.A Q72.about vibrio cholera a.vaccination provides specific protection b.trAns:mitted by contaminated water and food(Ans:)

c.not dangerous ANS:.B Q73.cause of edema in renal dsease a.dec albumin and salt retention b.inc hydrostatic pressure c.dec hydrostatic pressure ANS:.A Q74 .cause of NA retention a.kidney failure b .heart failure c.liver failure ANS:.B Q75 .long scnerio asking both obese and non obese has the equal risk of developing which disease a.cirrhosis b.osteoarthritis c.hypertension d.diabetes Ans:>C Q76.in dehydration which will have low concentration in body a.ADH b.water concentration c.ANP d.aldosterone ANS:.C Q77.locally malignant tumor a.adenoid cystic carcinoma b.pleomorphic adenoma ANS:.A Q78.Toxin produce in scarlet fever a.erythrogenic toxin b.exotoxin c.endotoxin d.invasive ANS: A Q79.lypmhocytes provide protection against a.bacteria b.cancer cells ANS:.B Q80.structure present above pirformis muscle a.inferior gluteat b.superior gluteal

184 c.pudendal nerve d.femoral nerve ANS:.B Q81.lymphocytes provide protction against a.exogenous factors b.endogenous factors ANS:.A Q82.what is absent in 1st cervical vertebrae a.spine b.foramina trAns:verserum c.antereior arch d.posterior arch ANS:.A Q83.deep inguninal ring is formed by a.internal oblique muscle b.trAns:versus muscle c.fascia trAns:versalis d.externl oblique Ans:>C Q84.hairy leukoplakia is characteristic of a.HBV B.HCV C.HIV D.squamos cell ca ANS:.C Q85.about tendineous insertion of linea alba a.one is present at xiphoid process b.total in no c.present at symphysis pubis ANS:.A Q86why atropine is given before anasthesia a.to dec heart rate b.to decrease tone of muscle c.to provide good analgesia d.to decrease the secretion of GIT tract e.to decrease the tone of git sphincter ANS:.D Q87.MMR definition a.no of materna deaths/1000 population b.no of maternl deaths /100000 c.no of maternal deaths /100 ANS:.B

Q88.pseudomona infection (stem not remembered exactly) a.TNF Q89.thiopentene action remains for only 15 mintues a.bcz of renal excretion b.bcz it is rapidly distribued throughout the body c.bc zof liver metabolism ANS:.B Q90.in elderly dose of gentamycin is reduced bcz a.because of decrease renal clearence b.because of decrease distribution of action c.bcz of rapid clearence froom liver metabolism ANS:.A Q91.sceondary centre of ossification is present in a.metaphysis b.epiphysis c,diaphysis d.cotrex ANS:.B Q92.cardiac muscle doesnotdevelop tetany because a.long duration of action potential b.long pleateu phase c.increase amount of calcium in cardiac cells d.decrease amount of calcium in cadiac cells ANS:.B Q93.about compact bone a.lamellae are regularly arranged b.haversian canal are present in medullary rays c.rich blood supply ANS:.A Q94.breastr milk is rich in a. immunoglobins b.calcium c.vit c d.vit k ANS:.A Q95.tumor causing homonymous hemianopia is present in a.middle of chiasma

185 b.lower part of optic chiasma c.upper part of optic chiasma ANS:.C Q96.etilogy of cervical carcinoma a.HPV B.herpes virus c.chronic irritation d.cervical metaplasia ANS:.A Q97.metaplasia is a.functional change in one normal tissue to other b.increase in nuclear cytoplasmic ratio c.[pleomorphism d.irreversible ANS:.A Q98.histological examination shows squamous cells is a.dysplasia b.ca in situ cervix c.metaplasia d.ca cervix ANS:.C Q99.a lady lost his family in an earth quake she would be having A.situational crisis b .depression c.anxiety ANS:.A Q100.about hyaline cartilage a.present in knee meniscus b.present in pinnae c .not present in nose d.develop from enchondral ossification e.larynx Ans:>E Q101.most potent stimulus for erythropoitin secretion A.hypoxia b.anemia c.acidosis d.erythrocytosis ANS:.A Q102.uterus is prolapsed but anus is in its positiion due to intact a.urogenital diaphargm

.b.pelvic diaphragm c.perineal body d.levator ani ANS:.B Q103.minimum caloric requirment in adult male is a.25-30 cal b.30-60cal c.40-80 cal d.80 -100 cal Ans:>A Q104.which of following acts on both central and peripheral chemoreceptors a.PH B.oxygen c.co2 d.hypoxia ANS:.C Q105.blood flow through brain is regulated by a.oxygen b.PH c.co2 d.acidosis e.hypoxia ANS:.C Q106.a long scnerio with woman with hyperpigmentation moon face central obesity. What will be increased in this women a.ACTH b.cortisol c.aldosterone d.renin ANS:.A Q107 about active trAns:port a.NA is always involved b. is uphill movement c.requires co trAns:porte d.counter trAns:port ANS:.B Q108.Erythropoitin is released by a.peritubular capillary cells b.juxtaglomerular cells c.cuboidal cells d.mesangial cells

186 ANS:.A Q109. Inferior vena cava commences at a.l5 b.l4 c.s5 d.l1 ANS:.A Q110 ) epithelium of vagina and uterus is derived from a. endoderm b. ectoderm c.endo plus mesodem d.ecto plus mesoderm ANS:.C Q111.cause of necrosis in most of tissues a. atheroma b. thrombosis c. mechanical ANS:.B Q112.protrusion of jaw by a.lateral pterygoid b. medial pterygid c. temporalis d.stylomastoid ANS:.A Q113 a person sustained a temporal blow with rupture of artery, artery damage is a.middle meningeal artery b. anteriorr cerebral artery c.posterior cerebral artery d.lateral cerebral ANS:.A Q114.most imp factor causing delayed wound healing a.ischemia b.infecton c.thrombosis d.metabolic ANS:.B Q115.location of SA node a.upper part of crista terminalis b.lower part of crista terminalis ANS:.A Q116.pyogenic meningitis cause a.bacterial I b.viral

c.fungal d.protozoal ANS:.A Q117increase ejection fraction what Will decrease??? a.EDV B.ESV ANS:.B Q118 best example of drug pharmakokinetic ANS:..antagonising naloxone on morphine Q119.scnerio of young male child presented with dark colour urine anemia,no hx of fever cause a.GG6PD Deficincy b.malaria ANS:.A Q120chromosome are arranged of equator in which phase a.prophase b.anaphase c. metaphase d.telophase e.prometaphase ANS:.C q121 thiamine deficiency will cause .a.subacute combined degnration of spinal cord b.peipheral neuropathy c.mallory bodies ANS:.B Q122. Regarding platelets what is true a. fragmentation of megaaakaryocyte b eosinophils has half life of 1 month c. platelets are increased in bacterial infections ANS:.A Q123 biguanides side effects a. flaulance and bloating stomach b. hypoglycemia c.metabolic alkalosis ANS:.A Q124.NeurotrAns:mitter present at presynaptic cleft?? a. calcium b.acetycholine

187 c.epinephrine d.dopamine ANS:.B Q125.which surface of bladder not covered wid peritonium a.anterior b.superior c. posterior ANS:.A Q126 paramesonephric duct forms a.uterus uper part of vagina and uterine tubes b.lower part of vagina ANS:.A Q127.Part of segmental bronchi with its artery is called a.broncho pulmonary segment b.tertiary bronchi c.secondary bronchi ANS:.A Q128.Regardind micturation reeflex??? a.intergrated in sacral segment of spinal cord b. once begin is self regenrative c.control by pudendal nerve ANS:.B Q129.Flight and fight response will cause??? a.brochoconstrion b.pupilary constriction c.bronchodilation ANS:.C Q130.lymphoid follicle is present in a.cortex of lymph nodes b.spleen c.medulla pf lymph node thymus ANS:.A Q131. regarding mast cellls a.it release histamine b.increase in acute in flammatory response ANS:.A Q132.mid gut supply a.t12-l2 b.l2-l4 c.t8-t10

ANS:.A Q133scnerio of molar pregnancy what will increased thn normal pregnancy a. b-HCG b.alpha-HCG c.alpha fetoprotein ANS:.A Q134 scnerio of acute panctreatitis ,wht will be increase a. lipases Q135 vasoconstrictor a. cocaine b.lignocaine ANS:.A Q136. Local anasthesia given which fibrs will be anasthesized first a. A-DELTA b. A-beta c. c fibres ANS:.A Q137 vit c causes ANS:.hydroxylation reaction in collagen formation Q138.1st bone to ossify a.clavicle b.femur c.tibia ANS:.A Q139. Cause of septic meningitis a.Streptococcus b.h influenza c.crypococcus ANS:.A Q140 fats proteins and lipid deposit harmone involve is a. insulin b.glucagon c.corticosteroid ANS:.A Q141 CNS myelination is caused bby a. oligodendrocytes b. schwan cells c, astrocytes ANS:.A Q142.Pus contains a.dead bacteria

188 b.dead neutrophils ANS:.B Q143 smoking plus asbestos has highest risk of a.mesothelioma b. bronchogenic carcinoma ANS:.B Q144 microscopy of malignant tumor a. invasion of othr tissue b.pleomorphism ANS:.A Q145 characteristic of malignant tumor a.metastasis b.pleomorphism c.inc nucler cytoplasmic ratio ANS:.A Q146 skeletal muscle cannot sythesize glucose bcz it lack which enzyme a.glucose 6 phosphorylase b.glucose phosphatase ANS:.A Q147 old lady wid ca endometrium cause a.exogenous estrogen b.HPV C.HSV d.irradiation ANS:.A Q148 .LP done containing blood cause a.subarachnoid hemmorhage b.extradural hemmorhage c. mengitis ANS:.A Q149 counterpart of labia major a.scrotum b. testis c.penis ANS:.A Q150.scnerio of iron deficiency anemia, cause A.chronic blood loss b.folic acid deficiency C.vit B12 def ANS:.A Q151 eosinphils are raiseed in all except a.igE b.asthma

c.iGM d.parasite ANS:.C Q152counselling a.help patient themselves b. help family c.help doctor to treat pt ANS:.A Q153most commn cause of hep b and c a. needles b.blood c. mother to fetus ANS:.A Q154.Immunological test is done fr a.amebic cyst b. hyadatid cyst ANS:.B Q155. Glucose appear in urine of pt at blood sugar level of 100mg/dl, cause a. dec renal threshold fr glucose b.decrease reabsorption of glucose c. diabets mellitus d.impaired glucose tolerane ANS:.A Q156.Pt presented with ammenorhea and weight gain cause??? a.hypperthyroidism b.hypothroidism c.hypoparathroidism ANS:.B Q157 leprosy initial investigation sample is taken from a.blood b.urine culture c.nasal scrapings ANS:.C Q158 which closely covers the viscera a.visceral peritoneum b.parietal peritonium c.fascia ANS:.A Q159. Scnerio od emphysema etilogy ANS:..dec alph 1antitrypsin

7TH NOVEMBER MORNING RADIOLOGY

189 1. infraorbital artery is a branch of A) Maxillary artery B) Superficial temporal artery C) Mandibullar artery ANS A 2. Sacral hiatus is located at A) In S5 and S1 B) Between S1 and S2 C) Between S5 to coccyx ANS C The sacral hiatus corresponds to the posterior caudal opening at the end of the sacral canal, which usually occurs at the fifth sacral vertebra (S5), at the posterior surface of the sacrum. 3. Superior parathyroid gland location A) Anteior to thyroid gland B) posterio lateral to thyroid gland c) posterio inferior to thyroid gland ANS B 4. function of baroreceptors in blood pressure is A) To decrease arterial pressure B) to increase arterial pressure C) hypertension D) hypotension ANS A 5. In brain, major blood vessels are located in A) Subdural space B) sub arachnoid space C) within pia D) extradural space ANS B 6. Melanocytes in skin are derived from A) ectoderm B) emdoderm C) mesenchyme ANS A 7. In anterior wall MI artery damage is A) Right coronary artery B) left coronary artery C) Left Anterior descending artery D) circumflex artery ANS C 8. In inferior wall septum myocardial

infarction artery damage is A) Anterior coronary artery B) posterior coronary artery C) circumflex artery D) left anterior descending artery ANS C.(both RCA n circumflex can cause ) 9. Most important chemical mediator in allergy is A) Interleukins B) Bradykinin C) Monocytes D.Histamine ANS D 10. Oblique fissure of lung is located at A) T3 spine to T5 costochondral junction B) T3 spine to T6 costochondral junction ANS B 11. patient with flu like symptoms with fever, test for infectious mononucleosis is positive likely ?? ANS monospot test 12. most potent chemical carcinogen is A) Methyl alcohol B) ethyl alcohol C) xylol ANS B 13. patient with low grade fever, night sweats , weight loss shows A) Caseous granuloma B) macrophages C) epithelial cells ANS A 14. Structure passing through aortic opening of diaphragm is A) vagus nerve B) thoracic duct C) Recurrent laryngeal nerve ANS B 15. important feature of 7 cervical vertebrae is A) massive body B) heart shaped body C) short cervical spine ANS All incorrect 16. Lymphocytes Have A) important role to prevent body against

190 cancer B) convert to monocytes ANS A? (poor recall of dis bcq) 17. Pendular knee jerk is present in A) upper motor neuron lesion B) lower motor neuron lesion C) cerebellar dysfunction ANS C 18. function of ICAM and VCAM is A) direct endthelial injury B) margination C Adhesion/tight binding Ans :C 19. In CNS myelin sheath is formed by A) schwan cells B) glial cells C) oligodentrocytes ANS C 20. the potent intracellular buffer is A) Phosphate B) bicarbonates C) CO2 D) Hb ANS A 21. Contents present at free border of lesser omentum A) CBD, cystic duct, hepatic artery B) CBD, cystic duct , hepatic artery C) CBD, hepatic artery, portal vein ANS C 22. Patient with whole body patechea, platelets decrease, rest of labs normal with no previous history ??? A) thrombocytopenia B) hemophilia C) thalessemia D) ITP ANS A 23. patient who got injury and his arm hangs on the side winging of scapula nerve damage is A) Suprascapular nerve B) axillary nerve C) long thoracic nerve ANS C 24. Appendiectomy done, at MC burneys

point artery likely to be damage is A) iliiohypogastric nerve B) inferior epigastric artery C) inferior mesentric artery D) deep circumflex iliac artery ANS D 25. Patient is suffering from meningitis organism involved after causing pygenic lung abscess and then spread to brain is A) Ecoli B) Streptococcus C) staph aureus D) pnumococcus ANS C 26. Virus that causes cancer are likely to cause damage to A) RNA B) protooncogenes C) Oncogenes D) DNA ANS C 27. Respiratory zone is situated A) Between terminal bronchioles and alveoli B) Between bronchioles and alveoli C) Between alveolar ducts and alveoli ANS B 28. Patient with markedly decrease TSH increase T3 What is the diagnosis A) Hypothyroidism B) Hyperthyroidism C) hyperparathyroidism D) hyperparathyroidism ANS B 29. Patient with occasional diplopia dizziness suddenly collapsed. A) hemianopia B) basillary artery thrombosis C) pontine hemorrhage ANS C 30.Location of SA node in right atrium A) Subepicardium B) Subendocardium C) endocardium ANS A

191 31. IN which lobe of the lung has 3 beonchopulmonary segments A) right upper lobe B) Left lower lobe C) right lower lobe D) right medial lobe ANS A 32.Hemochromotosis diagnosed by A) Ceruloplasmin level B) antimitochondrial antibodies C Serum Ferritin+transferrin saturation ANS C 33. WILSON disease is diagnosed by A) Urinary ceruloplasmin level B) serum ceruloplasmin level ANS B 34. Worst reaction presents by transfusion of A) A- TO A+ B) A+ TO ABC) O+ TO A+ D) O+ TO B+ ANS A 35. Caput medussa is formed by A) abdominal vein to paraumblical veins B) portal vein to greater splancnic vein ANS A 36. vascular segment of kidney is A) 5 B) 7 C) 4 D) 6 ANS A 37. In which disease splenectomy is best responded A) hemophilia B) thalessemia C) G-6PD deficiency D) Hereditary spherocytois ANS D 38. Beta -thalassemia A) deficient synthesis of alpha chain B) decrease synthesis of beta chain C) increase synthesis of beta chain D) deficient synthesis of beta chain ANS B 39. In brown sequard syndrome

A) pain an temperature is lost on the same side vibration and position sensation is lost on the opposite side B) Only vibration ans position sense is lost on the same side C) Pain and temperature is lost on opposite side and vibration and position sense is lost on same side. ANS C 40. one gram of Hb carry how much O2 A) 7.34 ml B) 5.34 ml C) 3.34 ml D) 1.34 ml ANS D 41. Penetrating wound at left side of margin of sternum, structure damage is A) Right atrium B) right ventricle C) intercostals membrane D) Intercostals muscle ANS C 42 IN anterior duodenal ulcer perforation fluid will accumulate in A) Right illiac fossa B) pouch of dougles C) hepatoduodenal pouch ANS A 43. Apoptosis A) Councilman bodies ANS A 44. ambigious genitalia with 46 XX karyotype diagnosis is A) klinefelter syndrome B) turner syndrome C) mixed gonadal dysgenesis ANS c? 45. A 50 years is short stature due to short limbs but intellegence is normal this disorder is A) autosomal dominant B) autosomal recessive C) X linked ANS A 46. lesion at optic chiasma its effect will be

192 A) bitemporal hemianopia B) binasal hemianopia ANS A 47. Trauma at medial epicondyle results in ulnar nerve damaged, loss of function is A) medail 1/3 of palmer surface of hand B) medial 1/3 of palmer and dorsal surface of hand C) lateral 2/3 of dorsal surface of hand ANS B? 48. Location of sternal angle A) at meaniburo sternum B) in accordance with costocondral joint ANS A 49. A 40 year old female has prominent neck swelling, hoarseness of voice A) Throid scan B) T4 stimulation C) biopsy ANS A 50. Male with sterility due to azospermia what is dignostic test A) FSH and LH B) Prolactin C) Testosterone D) progesterone ANS A 51.On Xray right Border of heart?? A..SVC B..right atrium Ans A 52. blood supply of left atrium A) Left circumflex artery B) left anterior descending artery C) right anterior descending artery ANS A 53. A 55 Year old female with cervical cancer, lymph node involved. A) external iliac lymph node B) inguinal lymph node C) internal iliac lymph node D) external and internal lymph node ANS D 54. A 60 year old man with testicular tumor lymph node first involved are ??? A) testicular lymoh node

B) iliohypogastric lymph node C) Para aortic ANS C 55. In young males mobile cilia are present in??? A) ductus difference B) epidydmis C) testis D) seminal vesicle ANS B 56. IN males narrowest part of urethra is A) bulbous B) penile C) memberanous D) prostatic ANS C 57. A 6 year old child while playing suddenly falls down and died on the spot on autopsy there is fresh thrombosis in coronary artery, all cardiac chambers are normal A) subacute endocarditis B) Kawasaki disease ANS B 58. Location of cruciate ligament of knees A) subcapsular ANS A 59. IN adult female breast atrophy is caused by A) prolactin B) estrogen and progesterone C) estrogen D) progesterone ANS C 60.in RTA there is fracture of left 8TH rib, the fracture most likely to damage A) left kidney B) spleen C) Pancreas D) Stomach ANS B 61. In splenectomy the surgeon seperates which structure to reach spleen A) transverse mesocolon ANS A 62. secondary active transport which

193 substance is transported A) glucose B) Sodium C) Co2 D) bicarbonates ANS B 63. Immunological test done for diagnosis of A) Liver abscess B) hydated cyst of liver C) hemoochromatosis ANS B 64. 41% hematocrit means there are A) 41% of rbc is present B) 41% of platelets and Rbc are present C) 41% of rbc wbc and platelets are present ANS C 65. abccess in first web space drainage is A) supra clavicular Lymph node B) infraclavicular luymph node C) Axillary Lymh Node D) supratrochlear lymph node ANS C 66. feature of malignant Tumor A) Pleomorphism B) metastasis ANS B 67. IN a severely dehydrated patient fluid replacement is done by A) 5% glucose solution B) 10% glucose solution C) Isotonic saline solution D) distilled water ANS C 68. During wound healing strength to wound is given by A) type 3 collagen B) type 1 collagen C) type 2 collagen ANS B 69. part of pleura supplied by phrenic nerve A) Visceral pleura B) parietal pleura C) Diaphragmatic pleura ANS C 70. QRS complex occurs prior to A) atrial depolarization

B) ventricular depolarization ANS B 71. Highest caloric value is of A) proteins B) carbohydrates C) Fats ANS C 72. Kidney activates which substance A) erythropoietin B) renin C) Carbohydrates ANS B 73. Most common carpal bone fracture A) Scaphoid B) lunate C) Trapezium D) Pisiform ANS A 74. Which antigen is common in featus and adult with colonic cancer ???? A) CEA B) alpha fetoprotein ANS A 75. in terminal ileum resection there is loss of ?? A) iron B) B 12 C) Calcium ANS B 76. in a 70 kg man minimun caloric requirement per day is A) 25-30 g/Kg B) 35-50 g/Kg C) 50-60 g/Kg D) 70 g/ Kg ANS A 77. example of piviol joint is A) shoulder joint B) axial joint ANS B 78 how will you break bad news? Ans:Separatel session with close relative or close friend 79. diabetic patient on a diuretic presented with hyperkalemia and coma Cause A) diuretic intake

194 B) insulin ANS B.(lack of insulin can present with DKA and hyperkalemia) 80. At cold exposure bodys first reaction to heat production is by A) piloerection B) Increase release of catecholamines C) shivering ANS C 81. boy presents in room temperature 28c pulses 120/min, BP 150/ 95 mmhg with hyperventilation ?? A) hyperthyroidism B) Excercise ANS A 82. lymphatic drainage of greater curvature of stomach is A. celiac node ANS A 83. inguinal canal posterior wall reinforced by A) conjoint tendon ANS A 84.Phrenic nerve origin A) c4 ANS A 85.Carcinoma which filaments involve A) desmin B) keratin ANS B 86. svc and ivc drain into A) right ventricle B) left atrium C) right atrium ANS C 87.Two ureter and two calyses are due to A) premature divison of mesonehric duct ANS A 88. Structure pierced lateral to sternum A) intercostal membrane B) intercostal muscle ANS A 89. Nasopharygeal carcinoma caused by A) EBV ANS A 90. in surgery supply to to molar lower lip

and alveolar arch of one side is gone nerve involved is A) inferior alveolar nerve ANS A 91.Narrowest part of constriction of ureter A Ishcial spine B Ishcial tuberosity C. Sacroiliac joint| Ans C 92.Fastest conduction fibres of heart Ans: purkinje 93.Scenerio of Cervical CA,lymphatic drainade first to which node..??? A.Paracervical nodes B.Intrnal iliac nodes Ans>B 94.Bare area of liver not having ?? ans)peritonium 95.Regarding lesser omentum? Double layered 96.Thoracic wall formed by which muscles ?? A internal intercostal B diaphargm Ans>A 97.Weight of liver Ans>1.5 kg 98.Right bronchus measurement Ans>2.5cm 99.Structure between celiac trunk n superior mesenteric .. Ans>Pancreas 100,Celiac trunk arteries Right gastroepiploic is branch of??? Ans> gastroduodenal 101.Select true statement regarding blood supply of stomach: Ans>right gastroepipoloic artery is branch of gastroduodenal artery 102.First branch of abdominal aorta ??? Ans> inferior phrenic 103.Surgeon recongnize the left kidney??? A.brown colour B.anterior relation with stomach,pancrease,descending colon,spleen,jejunum

195 Ans >B 104.Thoracodorsal supplies? Ans>Latismus dorsi 105.Rhomboid supply? Ans> Dorsal sapular 106.12 year old boy with obesity hyperphagia short stature mental retardation (some other things) A.Turner b.Klinefelter c.Prader villi d.Angleman Ans C 107.Relation of structures in femoral triangle from medial to lateral?? Ans>Femoral vein, femoral artery femoral nerve. 108.Structures below inguinal ligament , from medial to lateral ? Ans.Femoral Vein artery and nerve 109.Lymphatic drainage of upper outer quadrant of breast..??? Ans>Ant axillary 110.Hdl which lipoproteins??? Ans.B100 111.Reticulocytes decreased in ? A.Chronic renal disease B.Bleeding from gut Ans A 112.Which tumor invades nerve sheath ? A.Adenocarcinoma B.Expiliomorfic carcinoma c.Adinoid cyctic carcinoma Ans:B 113.About highest level of iliac Crest. Asn:Lies at the level of l4 114.Triceps reflax lost wd index and large fingees parestesia C6. 7 C7. T1 C5. 6 AnsA 115.Stop bleeding of internal carotid at??? Ans: C6 116.Lateral calf muscle weaknes wd los of reflaxes

Lmn Umn Lmn wd ant cells Ans:C 116:In hypertension .. Sensory loss ta shayad .. Which area affected repeat q Ant part of post limb wd genu Post part of ant limb wd genu Ans:A 117:Total No of Lower pulmonary airways? Ans: 23 118.Lunate fractured,nerve demaged is?? Ans: ulnar 119.Footballer was hit by ball on Right side of head on x-ray parietal bone fracture and hematoma at temporal region.????? Structure damaged Asn:Middle meningeal artery 120.Rapidly adapting sensory receptora? A. Baroreceptors B.touch receptors. C.pain receptora Ans:A 121.Embryo se...neural tube is formed in ??? Ans:4th to 5th week 122.Radial nerve damage? Asn: loss of extension of wrist joint 123.Child with edema of lower limbs. best initial investigation A.Urine for proteins B.Serum album i Ans:A 124.Axon hillock related..? Asn:absence of ribosomes 125:Regardin Optic canal ,whic structure passing??? Ans:Opthalmic artery n nerve 126.Maxillary nerve exit from rotandum then enter into A.Temporal fosa B.Infra temporal C.Pterygopalatin fosa Ans :C 127.Which fatty acids have coronary risk...something like that was asked? A.Omega f.a,

196 B. polyunsaturated, C momunsaturated, D.transaturated Ans:C 128.Regarding post cord of brachial plexus branches suppy ? Ans:Extensors 129.Zinc def leading ??? Ans:scaly dermatitis 130.S.A node..located ?? Ans:in upr part of R.A Which structure recieves input from cerebral cortex but don't give input to cerebral cortex in return(something like that) Ans:Caudate nucleus 131.Steroid has anti inflammatory action mechanism?? Ans:Inhibit phospholipase 132:Regarding Lesser sac ? Asn:Hep artery and portal vein 133:Surgeon operting lower part of ureter , surface marking like that Asn:common iliac artery bifurcation 134:Inferior surface of central portion of diaphragm is supplied by. Ans:Phrenic nerve 135.Urethral Sphincter is located at a.above prostate b.sup perineal pouch c.deep perineal pouch Ans:C 136:Hypotension amd vasodilation in which shock? Ans:Septic 137.Transitional cell Ca in a smoker working in tyre factory due to a.Tobacco b.Aromatic Amines ans:a (ref goljan patho) 138.Regarding abdominal aorta Ansinf phrenic is 1st branch 139.Highest transverse point? A.Pubic tubercle B.Poster sup iliac spine Ans:B 140.Narrowest part of airway in child

a.Cricoid b.Nostrils c.Vocal cord Ans:A 141.Microscopic featire of malignant tumor was asked? AnsInvasion. 141.Transmission of herpea simplex... Ans.droplets 142.Bitemporal hemianopia aneurism is at??? Ans:anterior communicating artey 143.Hormone release in stress ? A.Gh B.Catcolamn C cortisol Ans:C 144.Strongest ligament??... Ans:illeofemoral 145.Middle thyroid vein drains into A.Internal jugular B.External jugular C.Anterior jugular Ans:A 145.Genital fold ?? Ans:clitoris in female 146:Irreversible cell injury?? Ans:Massive influx of calcium. 147.Medial arcuate ligament formed by? A.Lumbar facia B.Rectus abdom muscle C.Diaphragm Ans:C 147.Edema in CCF??... Ans:inc hydrostatic pressure 148.One regarding spleen a.length of spleen 12cm b.more echogenic than liver on UsG Asn:A 149.Cornary sinus drains ? Ans:Rite atrium 150.Ek cervical lymphadenopathy, aur on biopsy kuch follicles lkha tha, aur mutation poochhee thee, I did 8:14 151.Adh is stimulated by ...?? Ans:inc plasma osmolarity amd dec plasma

197 vol 152.HEAT STROKE..CONTRIBUTING FACTOR A.EXCESS SWEATING B.DECREASED SWEATING C.POSTERIOR HYPOTHALAMUS UNRESPONSIVE Ans:c 153.Fungal sinusitis in nose most common?? Ans: aspergillus.. 154.A baby boy..recurrent resp inf...dec b cells and dec immunoglobulins? a. x linked agamaglobinemia b. brutons agaamaglobulinemia Ans:B 155.Ligamentum teres...?? Ans:umbilical vein 156.Viral infection? Ans:Inc lymphocytes 157.During intrauterine life testosterone secretion by stimulated by A.FSH B.LH C.Placental estrogen D.Androstenedione E.Placental Hcg ans:B 158.Arterial supply to cbd ? Asn:gastroduodenal and hepatic 159.Letral upper breast drain ? A.Anterior axilary b.Lateral axilary Ans:A 160.Infront of lower part of head of pancreas? A.Sup mess art b.Left colic C.Ivc Ans A 161.Theophylline most commonly causes a.nausea b. seizures c. apnea Ans:A 162.Foot turns inward?

Ans: Tib ant and Tib post 163.Co2 passes more readily actoss cell membrane thn o2 because I did diffusion co efficient 164.Sartorias supply by ?? Ans:femoral nerve 165.Superficial Dorsal vein of penis drains into ASantorini's plexus b(Superficial or great) saphenous vein Ans:b 166.Atrial systole ? Ans:”a’ wave on JVP 167.Thoracolumbar nerves ... Ans.Sympathetic NS 168.Ek pregnant lady wid obstructive jaundice best test ?? Ans :GGT 168.Parasympathetic outflow frm segents a.L1 L2 b.S2 S3 S4 c.S1 S2 S3 Ans:b 169.One mcq of horner syndrome chooses one A.Anhydrosis b.Anhydrosis and Ptosis ANS:b 170.Sensation of distention from rectum " 1. Parasympathetic afferents. 2. Superior rectal nerve 3. Inferior rectal nerve 4. Middle rectal nerve Ans.3 171.Achpndroplasia ( autosomal dominant scenerio) 172.One question on cause of hydrocele in 1 year old child : A. Filariasis B Testicular inflammation C. Scrotal wall something Ans:A 173.Electrolyte whose variability in Ecf affects heart the most??? a.Ca b.K+

198 Ans:B 174.Diff b/w primary and sec intention healing a.keloid b.Granulation tissue c.Wound contraction Ans:C 175.Cingulate gyrus is supplied by Ans anterior cerebral artery 176. touch n vibration by AVentral spinothalamic bSpinothalamic CMedial laminiscus Ans:C 177.Unlocking of knee joint by??? Ans: popliteus

199

RADIOLOGY 2016 PAPERS SEPTEMBER, OCTOBER, NOVEMBER RADIOLOGY5SEP MORNING2016 1. Earliestsignof VitaminA deficiencyis a.Night Blindness. b.Keratitis c. Bitot’ spots. Ans:: aa 2. Young boywithpainlessswelling ofrightorbit, other eyeis normal , what is the Dx ? a. Optic nerve glioma. b. Retinoblastoma. c. Cataract d.Syphilis Ans:: aa 3. Hypermagnesemia causes a. decreasedAchrelease

b. increasedAchrelease c. tetany d. arrhythmia Ans:: aa 4. Originof thyroidfollicular cell is a.Endoderm b.Mesoderm c.Ectoderm Ans:: aa 5. Epithelium ofUterusandVaginaisderivedfrom a.Endoderm andMesoderm. b.Mesoderm alone c. Endoderm alone Ans:: aa 6. A womanhadfallopiantubeligature,sheisfoun dtobeinshockafter6hours,Laparotomy is doneandintraperitonealhemorrhageisfoun d. bleeding ismostlikelyfrom branches of a. uterineartery

200 b. commoniliacvessels c. external iliacvessels d. aorta Ans:: aa 7. A manwalks100yardsandfeltspainincalf,reli evedafterstopping,arteryinvolvedis a. Popliteal artery. b.Peroneal artery. c. Posterior tibial artery. Ans:: aa 8. Leftrenal veinisposterior to a. headofpancreas b. 3rdpart ofduodenum c. aorta d. IVC e. Portal vein Ans:: bb 9. Nick during surgery onrightside ofhepatoduodenal ligamentwill cause damageto a. Cysticduct b. IVC c. hepaticveins d. hepaticartery e. portal Vein. Ans: : E 10.Massatportal hepatiswill compress a.Cystic duct. b.IVC. c.Hepatic vein d.Hepatic artery. e.Portal vein, Ans:: ee 11.Regarding Cephalicvein a. startsfrommedial dorsal surface ofhand b. connectswithbasilicveininelbow. c. Runsindeltopectoral groove d. piercesClavipectoral fascia Ans:: cc 12.whichcranial nerve nucleilieslateral torhomboidfossa/sulcuslimitAns: a. dorsal vagal nucleus b. CN6 c. CN3 d. CN11 e. CN9 Ans:: aa 13.Posteriormostfibersinposteriorpartofth erim ofinternal capsulecontain a. opticradiations b. corticospinal fibers ofupperlimb c. CSfibers oflowerlimb d. frontopontinefibers

e. thalamocortical tract. Ans:: ee 14.Manduring running developsaching paininankle,nextdayswhenhewakes uphefounds ecchymosedaroundanklejoint,hecanstand ontiptoes butithurts. a.Ruptured Plantaris tendon. b. Ruptured Achiles tendon. Ans:: aa 15.structureanterior topancreas, A. hepatoduodenal ligament b. leftcolic c. portal vein d. superiormesentericvessels e. splenicvessels Ans:: dd. 16.vessel dilatedmostlikelyinportal HTN a. Leftcolicvein b. inf. epigastricvein Ans:: aa 17.Neuronsofvestibularnucleiendsat a. cortex b. dentate nucleus c. granularlayer of cerebellum d. cochlearnucleus Ans:: cc 18.IrreversiblechangeinMI is a. Contractionbands b. appearance of neutrophils c. Mac callum patch formation. Ans:: aa 19.Featheryappearancein a.Jejunum . b.Duodnum c. Ileum Ans:: aa 20.Childof2years age presentsinopdwithFever,diarrheaand pallorfor2days. Hob 5.6%,MCV 107MCHC inc. Inv of choiceis a. trAns:cobalaminlevels b. intrinsicfactorantibodiesassay c. serum b12level d. rbcfolate. Ans:: bb , ( if it were an adult than Ans:wer would be ccc). 21.RighttesticularCA 1stmetastasizes to A. Aortocavolumbarnodes b. renal hilarnodes c. sup. inguinal d. commoniliac

201 e. paraortic Ans:: ee 22.Extraembryoniccoelom isderivedfrom a. Epiblast b.Hypoblast Ans:: bb ( langmann embryo). 23.TrueaboutTrigeminal ganglion A. liesinAnt. cranial fossanear cristagalli B. liesinapex ofpetrouspartoftemporal boneinmiddlecranial fossa c. coveredinganglion d. bathesinCSF Ans:: bb ( if bb option not present prefer dd) 24.Young adultinvolvedinRTA,gotinjurytomiddlecr anial fossa,bleedingfrominternal ear.After recovery presentswithlossof tearformation. Injurytowhichstructure during RTA causedthis a. Greatersuperficialpetrosal nerve b. deeppetrosal nerve c. interaortocaval plexus d. cilliaryganglion e. carotidplexus Ans:: aa 25.A manpresentswithgradual lossofvision, radiologicalstudiesshowgrowing clotinavessel,the mostlikelyinvolvedvessel is a. vertebralartery b. basilarartery c. MCA d. PCA e. posterior communicating artery Ans:: dd 26.A womanpresentswithbleeding withsuperficial cuts. O/Inv. onlyBTtimeisprolonged. defectis dueto a. thrombocytopenia b. factor8def c. factor9def Ans:: aa 27.Regarding mostcorrectstatementaboutbundle ofHIS a. only pathwayconducting impulsesfromatria toventricles b. situatedinmembranouspartofIVseptum nearatrioventricularjunction Ans:: bb 28.mostprominentstructure during IVU ofkidneyis a.Calyces b.Renal pelvis.

Ans:: aa 29.Amino glycoside halflifeisincreasedinOldage patientsdueto a. hepaticmetabolismisdecreased b. decrenal function c. decVol ofdistribution Ans:: bb 30.A patientcomeswithswelling ofbig toe onrightsideoffoot. nopainwae there,, what investigation would you do ?? a. serum uricacid b. serum ureaanduricacid c. microscopy ofjointfluid d. urine uricacid e.Synovial fluid for polarized light. Ans:: ee 31.Clara cellsarepresentin a.terminal Bronchioles b. alveoli. c. alvelar duct. Ans:: aa 32.Foreheadisformedfrom A. FrontoNasal Process B. maxillary process c. palatonasal process Ans:: aa 33.A footballer getsinjuredduring match,presentsinERwithlimbinstability. O/Ethereisswelling inprepatellarregionandhis TibiaisdrawnforwardonFemur. Thestructuremostlikelyinjured is A. AnteriorCruciateLigament b.Posterior cruciate ligament tear. Ans:: aa 34.8patientswerereportedinacasestudy,the iragewas20,25,25,30,40,20,30. Themedianof valuesis a.27.5 b.25 c.20 Ans:: aa medianis calculatedwithvalues putinascending orderand ifoddno. thenthecentral oneis median. Ifeven no. thenmiddletwovaluesaverage,inthis case(25+30)/2isAns: 35.Regarding anatomy ofInguinal Canal a. liesfromASIS to pubictubercle b. Superiorboundaryformedfromconjointtendon Ans:: bb

202 36.A studyinwhichdiseasedpeoplearecompared tonondiseasedhealthy people, looks for prior exposure for a risk faactor is called a. crosssectional b. cohort c. casecontrol d. RCT Ans:: cc 37.Ingrowing embryo,themesotheliumcovering theLung forms a. pleuroperitoneal membrane b. pleuropericardial membrane c. parietal pleura d. visceral layer ofpleura e. fibrous pericardium Ans:: dd 38.FoodIntake/reflex a. ispromotedifhungercenterisdamaged b. isinhibitedifsatietycenterisdamaged c. inhibitedbyleptinssecretedfromadipocytes d. inhibitedbyleptins secretedfrommyocytes Ans:: cc 39.A patientpresentswithcough. CXRshowsB/L peri-hilarinfiltrates. Biopsy of skin lesionsshows granulomatousinflammation. Diagnosismostlikelyis a. silicosis b. TB c. sarcoidosis d. asbestosis e. histoplasmosis Ans:: cc 40.Extensormuscles ofarmsaresuppliedby a. PosteriorCord b. Medial cord Ans:: aa 41.Infracture ofhamate,whichisinvolved? a. ulnarartery b.ulnarnerve Ans:: bb 42.Manwithlymphomawas being treatedwithanti cancer drugs,biopsy ofalymphnodesshows clumping ofnuclearmaterial andfragmentation. Themostlikely mechanismis A.Apoptosis b.necrosis Ans:: aa 43.Ininfectiousmononucleosisorganatrisk ofruptureis a.Spleen.

b.Liver Ans:: aa 44.P. Malaria trAns:mits tohumaninform of a.Sporozoite b.Merozoite Ans:: aa 45.A malefromBaluchistanpresentswithletharg y,jointpainsandleg ulcers. Diagnosiswill most likely A. sicklecell b. thalassemiamajor c. HbC Ans:: bb 46.A childpresentswithpallor,lethargyanddecac tivity. O/I shows decHbdecMCVandBone changesareprominentonFace a. thalassemiamajor b. sicklecell disease c. thalassemiaalpha Ans:: aa 47.Regarding RBC antigenonsurface A. glycolipids B. Glycoprotein c.Glycosphingolipids Ans:: cc 48.Regarding anklejoint a. surfacecoveredfromfibrocartilage b. deltoidligamentonMALLEOLARSIDE(medi al)wasnotwritten c. superficial peroneal nerveisonlysupplytoit d. talofibularligamentisonmedial side Ans:: bb 49.Youareallowed tobreachpatient’sconfidentialitywhen a. patientallowstodoso b. insuranceclaim Ans:: bb 50.A patientpresentswithcentral chestpainfor4-6hours. Painisradiatingtoarmandjaw. Best markertoconfirmMI atthisstageis a. CPK b. CKMB c. LDH Ans:: bb( Trop T was not in options). 51.A patienthadheart-lung trAns:plantationandwas onimmunosuppressive drugsnow presents withhighgradefever,neckrigidity+vekerni

203 g.CSFstudiesshow anorganismwithclear cytoplasmwithahaloaroundit. a. CRYPTOCOCCUS b. EBV c. staphaureus d. klebsiella e. CMV Ans:: aa 52.InShockwhichis themostimportantmechanismwhichconser vesthe organismas awhole a. baroreceptor b. sympathoadrenergicsystem c.CNSischemicresponse Ans:: bb,,,,,, isbestbczitconserves perfusionto vital orgAns:whileCNSischemicresponsecomesat endstagewhenperfusiontobrainislow,itcauses widespreadvasoconstriction,whichcauses organfailure. 53.RemnantofUmbilical Veinis a. falciformligament b. ligamentof teres Ans::bb 54.Patienthadhypovolemicshock,presents withsignsofAcuteRenal failure. Mostlikely damage towhichpart ofthe nephronisgreatest a. PCT b. descending c. ascending d. DCT e. Collecting duct Ans:: aa 55.A school goingboy developsRightsidedflankswelling andlumbarpain,USshows Hydronephrosisof therightkidneywhilethe otherisnormal.Whichof the belowismostlikely thecause ofit a. Urethral stricture b. ureteral stricture c. posteriorureteral valves d. vesicouretericreflux Ans:: cc 56.A patienthadanAccidentwithfractureofsurgi cal neck ofhumerus.Mostlikelyinvolvednerve is a. axillary b. radial Ans:: aa

57.Lymphnode ofinner breastdrainsintoall except a. infphrenicnodes b. supraclavicularnodes c. internal thoracicnodes d. pectoral nodes Ans:: dd 58.Mostcommonfracture offemuratage of60is A. femurheadfracture b. intra capsularfracture c. neck offemurfracture d. shaftfracture Ans:: cc 59.Epidural SpaceEndsat a. S2 Ans:: aa 60.Leftgastricarteryisabranchof a. CeliacArtery b.Hepatic artery Ans:: aa 61.Structureaccompanying esophagusindiaphragmis a. leftvagus b. rightvagus/vagal trunk c. phrenicnerves d. azygousvein Ans:: bb 62.InhibitorsofProstaglandinsare A. Corticosteroids b. aspirin Ans:: aa 63.Hall markofAdrenocortical insufficiencyis a. Hyponatremia b. hypokalemia c. metabolicalkalosis d. hypocalcemia Ans:: aa 64.FastPain travelsby A. A deltafibers b.C fibers Ans:: aa 65.Sympatheticnervesending atInferiorhypogastricplexushaverootvalue of A. L1-L2 . B. L3-L4 Ans:: aa 66.Detrusormuscle ofbladdersuppliedby A. pelvicSplanchnicnerves Ans:: aa 67.Pectoral groupofaxillary nodes drain a. Upper halfof trunk

204 b. Upper halfofupperlimb c. Majorpart ofthe breast d. Lowerhalfof trunk Ans:: cc 68.Regarding VagusNerve a. Exits throughpassing middle partofjugular canal b. Exitsthorax by passing withAorta Ans:: aa 69.FragileXsyndrome is a. Trinucleotiderepeatdisorder Ans: ; aa 70.Mostcommoncause ofcongenital inheriteddefects a. Multifactorial b. Autosomal dominant Ans:: aa 71.A patientpresentsinOPDwithvesicularerupti verashwhichisstarting fromlefttemporal side andendsnearmedial side oflefteye. Rashispainful,diagnosisofShinglesismade. Whichof the following nervemay beinvolved a. Ophthalmicnerve b. Superiororbital nerve c. Supra trochlearnerve d. Maxillary e. Mandibularnerve Ans:: dd 72.Acrosomeisformedby a. Golgi bodies b. Endoplasmicreticulum. Ans:: aa 73.External urethral sphincterrelated toprostatic: a. Apex b. Base c. Posterior d. anteriorside Ans::bb 74.Small particlesare clearedby A. Ciliainterminal bronchioles. B. Macrophages Ans:: bb 75.HighestPCO2isinwhichvessel a. Pulmonaryartery b. IVC c. SVC Ans:: aa 76.Notisotonictoplasmais a. 5% dextrosewater 77.Lumbar triangleanteriorly boundby a. Posteriorborder of External Oblique

Ans:: aa Refence Snell 78.Ulcer 2ndpart ofduodenumatposteriorsidearteryerodedi s a. Gastroduodenal artery Ans:: aa 79.Slowestgrowing tumorof thyroid A. Papillarycarcinoma Ans:: aa 80.Commonestdislocationof TMJ is A. Anterior Ans:: aa 81.A manhasshorttrunkandlongerarms,thisdise aseis ? a. Autosomal dominantdisorder Ans:: aa 82.Type3reactionis a. Arthusreaction Ans:: aa 83.MyelinformationinCNSisfrom a. Oligodendrocytes b.Schwann cells Ans:: aa 84.Mandibular branchoftrigeminal nerveexits cranial cavitythrough a. Formanovale Ans:: aa 85.Hemeinbloodbindswith a. Haptoglobulin b. Hemopexin Ans:: aa 86.Cerebral bloodflowisregulatedby a. PCO2invenousblood b. pH ofCSF Ans:: aa 87.Eparterial bronchusis a. Rtsuperior. Ans:: aa 88.Epidural sinusinspinal canal a. Longitudinal venousplexus b. CSF c. Isbetween Duraandarachnoidsspace Ans:: aa 89.Mostcommoncongenital anomaly ofheadis a. Cleftlip&palate . Ans:: aa 90.A womanheadRadical mastectomyafterrecovery unabletoraisearmabove her head, structure damageis a. Long thoracicnerve

205 Ans:: aa 91.Anteriorrelationofleftkidneywhichisdir ectlyincontactwithkidneywithoutanymem brane a. Lessersac b. Pancreas c. Duodenum d. Leftcolicflexure. Ans:: bb 92.Structure passing abovelowerheadofPancreas a. SMA . Ans:: aa 93.ICAMandVCAMare. a. adhesionmolecules. Ans:: aa 94.Derivative of2ndarchis a. Stylohyoid Ans:: aa 95.Adverseeffectinlong term use ofglucocorticoidsis a. Osteoporosis Ans:: aa 96.Branches ofbrachiocephalictrunkare. a. RtcommoncarotidandRtSubclavian 97.If LCXisoccludedwhatwill betheeffect a. Posteriorofleftventricle bemoreaffected b. SA nodal blockwill occur c. Postinterventricularseptum1/3will beinfarcted Ans:: aa 98.SA nodeispacemaker ofheartbecause A. Itliesatthe bottom ofatrioventricularjunction b. Itliesatthetopof RTatrium c. Ithasaplateau d. Itsfibers havefastestconduction. e.it has automaticity n generates impulses at a faster rate. Ans:: e 99.Heinzbodiesarepresentin a.G6PD 100. 2yearoldchildwas playing withtoyswhenhesuddenly becomesuffocatedandcyanosedfor fewseconds. Mothertook himahospital onCXR he hadateasmall coin,wherewill itbelodged mostlikely A. Rtlowerbasal segment. Ans:: aa Ans:: aa 101. A young childinhaledapeanutwhereitwill belodgedmostlikely Ans:. Lowerlobe ofrightlung

102.Intrinsicfactorissecretedfrom a.Gastricfundus Ans:: aa 103.Virus causes a.Alterationof proteinsynthesis. Ans:: aa 104.Moral supporttopatientinoursocietycomesfrom a.Family b.Eidhi. Ans:: aa 105.A patientisbinge drinkerofAlcohol presents toERwithC/ORTside hypochondrial pain. US showsfattyliver,Biopsyshowscouncilmanbod iesinhepatocytes. A. Apoptosis Ans:: aa 106.Feature ofAtlasis a.AbsentBody Ans:: aa 107.A patientisbeing evaluatedfor TB. The bestinvestigationwhichwill diagnoseTBis a. CaseousnecrosisonBiopsy b. EpitheloidCells c. DemonstrationofAcidfastbacilli onZNstain Ans:: cc 108.Middlemeningeal artery Enterscranial CavitythroughforamenSpinosum 109.WhatparameterisincreasedinPregnan twomen a. TIBC b. Serumferritin c. PCV d. Hb Ans:: aa 110.Nervesupplytopericardiumisfrom a.phrenicNerve Ans:: aa 111.Highestenergycompoundis a. Fats 112.Pubicbonesjoinanteriorly by a. Secondarycartilaginous joints 113.Veinhaving novalvesis a.SVC 114.Following veinisa tributary ofPortal vein a. Superiorrectal vein

206 b. Hemiazygousvein c. Infepigastricvein d. Leftrenal vein e. Lumbarveins Ans:: aa 115.A ladyhasdifficultyinstanding fromasitting positionbutcanwalknormally. Muscleinvolvedis a. Gluteus Maximus Ans:: aa 116.A detachedembolusfrom deepveinsofleg will gofirstto a. IVC b. RtAtrium c. Ltventricle d. Pulmonaryveins e. Pulmonaryartery. Ans:: ee 117.DefectofRBCsinHereditarySpherocyt osis a.Cytoskeletonabnormality b.Defect in spectrin and ankyrin Ans:: aa 118.Mixedvenousbloodsamplecanbe obtainedfrom a. SVC b. IVC c. Rtatrium d. Rtventricle e. Pulmonary arteries Ans:: ee. 119.Long term glucosecontrolcanbe bestdetectedby Ans:. HbA1C 120.Glycocalyxis a. Polysaccharide moiety onbacterial cell wall b. Carbohydratechainoncell wall Ans:: bb. 121.Basicdrug bindswith Ans:: Alphaglycoprotein 122.Thyroiddrains to a. deepcervical nodes. Ans:: aa 123.Pregnantwomenwithviral hepatitis. Bestinv. todetectis A. GGT Ans:: aa 124.1/3rdofTBWis A. ECF Ans:: aa

125.Whichhormonelevelwill fall after drinking water a. ADH b. Angiotensin Ans:: aa 126.Maximum bloodpressureisat a. Renal artery Ans:: aa 127.Mostcommoncause ofKshiftfromICF toECFA. Vigorous exercise Ans:: aa 128.Glucoseismainsourcefor a.Neurons b. Liver Ans:: aa 129.OxytocinandADH originatein a. Hypothalamus 130.MostcommonassociationofADPKDis a. Renal failure b. Cerebralhemorrhage Ans:: bb ,, cause of death is renal failure. 131.Musclethat causessadness a. Platysma 132.Clostridium botulinumcauses a. Flaccidparalysis 133.Nerveaccompanyingsuperficialtempo ral arteryis Auriculo-Temporal nerve. 134.SarcomadiffersfromCarcinomain IncreasedVascularity 135.Patientontable heatlossmainly by a. Conduction b. Radiation c. conduction and radiation. Ans:: cc 136.AcidicurineinTubuleswill cause a. increasedsecretionofCLb. DecreasedHCO3c. DecK+secretion d. ReducedH+reabsorption e. Aldosteroneinc Ans::dd 137.Baby bornwithcongenitalcataract, mother hadmostlikelywhichintrauterineinfection A. Rubella 138.Leftadrenal veindrains to leftrenal vein 139.Lower Esophageal Veinsdrainto A. Portal Vein

207 140.Leftlung hasgroovefor a. Ascending aorta b. Descending aorta c. Archofaorta d. Azygousvein Ans:: cc 141.Deepveinthrombus1stlodgesin a. Branches ofpulmonaryartery b. Rtatrium c. Rt. Ventricle d. IVC Ans:: aa 142.PatienthadTB, developedpleural effusion. Needleinsertedwill bealong upper border oflowerrib 143.Cause ofpyogenicperitonitisis a. E. Coli b. Bacteriodes c. Klebsiella d. Staphaureus Ans:: aa 144.Metaplasiais onecell typereplaces otherwithfunctional change 145.A patienthadacoronary bypassdevelopedA-fibnow presentsinERwithsignsofstroke. Brains showswhichtype ofnecrosis a. Liquefactive 146.HIVaffects CD-4CellsT 147.Laborworking inhotweathercollapses. Mostlikelyabnormalityfoundinhimwill be a. Hyponatremia b. Hypervolemia c. Hyperkalemia d. Hypercalcemia Ans:: aa due to excessive sweating. 148.Carcinoma compared toadenomahas a. Capsule b. Invasion c. Nolossofdifferentiation d. Vascularity Ans:: bb 149.TBresistsimmunity. Inthis caseAntibodiesarefound a. Attached tocell b. Boundtoorganism. c.No ntibodies. Ans:: cc 150.Ironisstoredinparenchymaunder normal conditionsis Ferritin 151.Regarding saliva a. HasincNa as compared toplasma

b. LowK&HCO3incomparisontoplasma c. Hypotonicity ismorethanplasma. Ans:: cc 152.Osmoticpressure dependsupon a. Noofparticles b. Diffusionpotential c. Charge onmembrane oneitherside Ans:: aa 153.Diffusionpotential dependsupon a. Surfaceareaofmembrane b. Charge onmembrane. c. Charge oneitherside d. Fromlow conc. tohigh conc. Ans:: aa 154.From 3rdto9thmonthofpregnancy,source ofestrogenis a. Corpusluteum b. Granulosa cells c. Placenta Ans:: cc 155.Strengthprovided towoundduring healing isby a. Collagen1 b. Collagen3. Ans:: bb ,, if only mentioned tensile strength than it is type 1 156.Granulosa cellsarefoundin a. Corpusluteum b. Groundsubstance c. Graffianfollicles d. Follicularepithelium. Ans:: cc 157.Esophageal constrictionisby a. Leftatrium 158.Regarding RtAtrium a. Formslowerborder ofsternocostal surface b. Is the posteriormoststructure c. IVC Ans:SVC junctionisvisibleatoutersurface d. IssuppliedbyLCX Ans:: aa 159.Regarding boneformation A. Newlylaidboneisintheform ofosteon 160.Vibrationsduring RapidVentricularfilling produces A. 3rdheartsounds b. 4thheartsound Ans:: aa

208 161.TPRisdirectlyrelated to a. Capacitance b. HR c. Vasomotor tone Ans:: cc 162.Exposuretohydro-carbonscause a. Acute nonlymphocyticleukemia b. CML c. AML d. Livercancer e.Bronchogenic carcinoma. Ans:: ee 163.Cause ofmesotheliomais Asbestos 164.Subacutebacterial endocarditisis causedby A. StrepviridiAns: 165.Cornayae diphtheriamode of trAns:missionis a. Oro-fecal b. Respiratory droplets c. Skincontact d. Fromspores Ans:: bb 166.Inhibinisproducedby a. Sertoli cells 167.Lymphaticnodulesabsentin A. Thymus B. Spleen c. Payer’spatches Ans:: aa 168.Rapidlyadopting fibersare. a. Pacinian b. Meissners c. Merkel d. Ruffini e. Free nerveendings Ans:: aa 169.Regarding vagusnerveinGIT a. Stimulationcausesrelaxationofano-rectal sphincter b. Contractionoflongitudinal musclegroupforperistalsis c. Decreasedacidsecretion d. Constrictionofoddi sphincter Ans:: bb 170.Incase ofAnteriorduodenal perforation,fluidwill goto a. Leftparacolicgutter b. Rtsubhepaticspace c. Rtsubphrenicspace d. RtPara colicgutter Ans:: dd,, if this is not in option than rt. Iliac fossa.

171.Incase ofposteriorperforationof1stpart ofduodenum,fluidwill becollectedinto a. Lessersac b. Rtsubphrenicspace c. RtSubhepaticspace d. Rtparacolicgutter Ans:: bb 172.Antibodiesareformedby Ans:. Plasma cells 173.Glucocorticoids cause a. Increasedproductionofprotein b. Hepaticgluconeogenesis c. Decaminoacidmetabolism Ans:: bb 174.Whichhormoneactsthroughphospholi paseC a. Hormonecausing increasedbloodflowinpenis b. Causing waterreabsorptioninkidneysAdhV2 c. Constrictionofvasculature byADH V1 d. ANP e. Angiotensincausing vasoconstriction Ans:: cc 175.Sensorysupplytoskinoverlowerhalfofd eltoidissuppliedby A. Axillary nerve 176.Preganglionicautonomicfibersare A. Bfibers 177.Cellswhichareenlargedinadenomaofp arathyroid a. Chiefcells b. Oxyphil cells c. Both Ans:: aa 178.Genital branchofgenito-femoral nerve a. Passesthroughdeepinguinal ring b. Gives branchestoanteriorscrotal wall. Ans:: aa 179.CharacteristicofIgM A. Isalarge molecule (pentamer) 180.Highesttriglycerides%isin A. Chylomicrons 181.Efficiency ofadrug is a. Efficacy b. Potency Ans::a 182.Tensioninmuscleissensedby A. Golgi tendon 183.A patientpresents to youinemergency. HispH is 6.95HCO34,glucose500mg/dl. You administer insulin

209 A. Regularinsulin 184.Isthmusofuterinetubesis A. Belowthefundus b. Is the narrowestpart of tube. Ans:: bb 185.Genioglossussuppliedby A. C1fibersthroughhypoglossal nerve 186.Chondorcyteschangeintoosteocytes A. Thyroxin b. Somatotrophs Ans:; bb 187.Patientwithlowerbackacheandpainra diatingtoposterior thigh. Themostlikelycauseis A. DischerniationatL4-L5 b. DischerniationatL5-S1 Ans::bb

Radiology Paper 5 September 2016 Evening Session

1)Structure in right free margin of lesser omentum? Bile duct,hepatic vein,portal vein Bile duct,hepatic artery,hepatic vein Bile duct, hepatic artery,portal veinANS: 2)Parts of bile duct? Supraduodenal,infraduodenal,retroduodenal, intraduodenal ANS: Supraduodenal,retroduodenal,intraduodenal Supraduodenal,infraduodenal,retroduodenal 3)Lymph drainage to superior mesenteric nodes? Stomach Duodenum Jejunum ANS: Liver 4)Regarding lymph drainage of testes? Intrtnal iliac nodes Preaortic nodes External iliac nodes Paraaortic nodes ANS: 5)Regarding posterior communicating artery what is correct? a) Connects ICA to basilar artery b) Connects ICA to posterior cerebellar artery above occulomotor nerve ANS: c) Connects ICA to posterior cerebral artery below occulomotor nerve d) Connects ICA to posterior communicating artery

6)Blood trAns:fusion reaction will occur if we trAns:fuse? a) O+ to A+ b) A+to AB+ c) B- to AB+ d) AB+ to A+ ANS: 7)Pain of ovary on medial side of thigh is felt by? a) Femoral nerve b) Illiohypogastric nerve c) Obturator nerve ANS: d) Sural nerve 8)Lady with hepatoma and dilated abdominal veins? a) IVC obstruction ANS: b) Internal iliac vein obstruction c) Superior mesenteric vein obstruction 9)Structure connects ECM to Cytoskeleton? a) Inermediate filament b) Integrin ANS: c) ProteoglycAns: 10)Lymph drainage of prostate? a) Internal iliac ANS: b) External iliac c) Internal iliac+External iliac 11)Which of the following structure is drained to inguinal nodes? a) Cervix b) Ovary c) Part of body of uterus ANS: d) Urinary bladder 12)Patient can stand on toes with severe pain which of the following is ruptured? a) Achillis tendon b) Plantaris ruptured ANS: c) Fracture of medial melleolus 13)Hormone required for conversion of bHcg to estradiol? a) FSH ANS: b) Gonadotropins c) LH 14)Blood supply of shaft of humerus by? Radial artery a) Profunda brachii artery ANS: b) Ulnar artery 15)Main supply of head of femur by? Femoral artery a) Medial and lateral circumflex artery b) Retinacular artery Ans: c) Obturator artery

210 16)Loss of suppination and flexion of arm..nerve damaged? a) Axillary nerve b) Median nerve c) Musculocutaneous nerve ANS: d) Ulner nerve 17)Loss of sensation on medial half of hand in front and behind.the structure damaged is? a) Supracondylar fracture b) Fracture of medial epicondyle ANS: c) Fracture of shaft of humerus d) Fracture of scaphoid bone 18)Physiological fuction of vit A? RhodopsinANS: a) Other options were weired dnt recall those. 19)Dorsal nucleus of vagus is located in? Midbrain a) Pons b) Medulla oblongata ANS: c) Cerebellum 20)Remanant of urachus? a) Medial umbilical ligament b) Mickels diverticulum c) Median umbilical ligament ANS: d) Medial arcuate ligament 21)Child with eating disorder,represents with weight gain and hypogonadism,the most likely due to? a) Craniopharyngioma ANS: b) Basal ganglia disorder c) Cerebral disease 22)arch of aorta compresses? Azygous vein a) Esophagus b) Left bronchus ANS: c) Trachea 24)Abdominal aorta at diaphragmatic opening compresses? a) Phrenic nerve and Thoracic duct b) Azygous vein and Thoracic duct ANS: c) Inferior vena cava and azygous vein 25)Inferior vena cavastarts at the level of? a) L3 b) L4 c) L5 ANS: d) L6 26)Superficial temporal artery accompanies in parotid gland? a) Lesser occipital nerve

b) Facial nerve c) Buccal nerve d) Auriculotemporal nerve ANS: 27)Submandibular gland surgery done,which nerve is damaged? a) Cervical nerve b) Facial nerve ANS: c) Maxillary nerve d) Submandibular nerve 28)In axilla which part of brachial plexus is present? a) Trunks b) Devisions c) Cords ANS: d) Roots e) All of above 29)Spastic paralysis occurs in? a) Upper motor neuron lesion ANS: b) Lower motor neuron lesion c) Cerebellar lesion d) Cerebral lesion 30)Pendular knee jerk is present in? a) Cerebellar lesion ANS: b) Basal ganglia disorder c) Upper motor neuron lesion d) Cerebrum lesion 31)Counter part of rete testes cords is? a) Mullerian ductsANS: b) Mesonephric ducts c) Ductus deference 32)Femoral triangle does not contain? a) Femoral nerve b) Obturator nerve ANS: c) Lymphatics d) Adductor magnus 33)Muscle which flexes knee and extends hip? a) Sartorius b) Tensor fascia lata c) Semitendinous ANS: 34)Vessel which runs obliquely upward in parotid gland? a) Facial artery b) Maxillary artery c) Lingual arteryANS: d) Occipital artery 35)Optic canal is present in which part of fascial bone? a) Lesser wing of sphenoid b) Greater wing of sphenoid ANS: c) Vomer

211 d) At junction of sphenoid and ethmoid 36)Germ cells incorporate at? a) 3rd week b) 6th week ANS: c) 3rd month d) 6th month 37)Drainage of bare area of liver into? a) Celiac nodes b) Posterior mediastinal nodes ANS: c) Lymph nodes in porta hepatis 38)In tetralogy of fallot what is present? a) Overriding of right ventricle b) Ventricular septal defects ANS: c) Left ventricular hypertrophy d) Pulmonary hypertension 39)Behind head of pancrease which structure is present? a) Superior mesenteric artery b) Lesser omentum c) Inferior vena cava ANS: d) Aorta 40)In front of uncinate process which structure passes? a) Aorta b) Superior mesenteric artery ANS: c) Splenic artery d) Left renal vein 41)Fracture of neck of fibula which structure damages? a) Ulner nerve b) Brachial nerve c) Common peroneal nerve ANS: d) Radial nerve 42)Biceps Brachii attaches to? a) Medial epicondyle of humerus b) Lateral epicondyle c) Supraglenoid tubercle of scapula ANS: 43)Aortic area is present in? a) Left 2nd intercostals space b) Right 2nd intercostals space ANS: c) At lower border of sternum d) Right 3rd intercostals space 44)What causes hypercalcemia in renal failure? a) Hypervitaminosis D b) Parathyroid adenoma ANS: c) Thyroid cancer 45)Metastatic calcification occurs in? a) Follicular adenoma thyroid b) Tuberculosis c) Malarial parasite

d) Normal kidney ANS: 46)About esophagus what is correct? a) Cardiac orifce at 10th costal cartilage b) Lies on right side throughout its course c) Serosa is thickened at lower endANS: d) Supplied by thoracic aorta only 47)About intercostals nerves what is correct? a) Supply skin of whole body wall b) 12 in number ANS: c) Supply abdominal muscles only d) Part of sympathetic chain 48)Patient with sciatica compression on 5th lumber segment causes? a) Absent ankle jerk b) Absent babinski c) Brisk ankle d) Weak dorsiflexion ANS: e) Weak plantar flexion f) Confirm it by yourself. 49)About spleen what is correct? a) Develops in ventral mesogasrium ANS: b) Tail of pancrease is related to its lower border c) Cant recall other options,bohat ajeeb aur confusing options thay. 50)Another question about spleen? a) On CT concave surface on medial side I did b) Can’t recall other options bohat he confusing aur fazul options then.bus aik isi ki sense ban rahe the… 51)What is correct about bile duct? a) 6 inches long b) Present on right side of hepatic artery in hepatogastric ligament ANS: c) Present on left side of portal vein 52)Arterial supply of jejunum is by? a) Celiac artery b) Splenic artery c) Superior mesenteric artery ANS: d) Inferior mesenteric artery 53)Sciatic nerve passes through? a) Lesser sciatic foramen b) Greater sciatic foramen ANS: c) Obturator foramrn Behind inguinal ligament 54)Number of segments of kidney are? a) 5 ANS: b) 6 c) 7

212 d) 8 55)Sequence of renal artery? a) Lobar artery-Arcuate artery-Interlobar artery b) Segmental artery-Lobar arteryInterlobar artery-Arcuate arteryInterlobular artery ANS: c) Lobar artery-Interlobular artery-Arcuate artery-Interlobar artery 56)Length of thoracic duct? a) 38cm b) 40cm ANS: c) 45cm 57)Patient unable to turn foot medially but can turn laterally…muscles paralyzed? a) Tibialis anterior and extensor hallusis longus b) Tibialis anterior and tibialis posterior ANS: c) Tibialis posterior and flexor hallusis longus 58)Pancreatic fluid accumulates in? a) Paracolic gutter b) Subphrenic space c) Lesser sac ANS: d) Subdiaphragmatic space 59)Question about hemophilia? a) X chromosome ANS: b) Y chromosome c) Chromosome 6 60)A patient has history of myocardial infarction one year back…which artery supplies the inferior portion of posterior interventricular septum? a) LCA b) LAD c) RCA ANS: 61)Which segment of kidney has more diluted urine? a) Proximal convoluted tubule b) Distal convoluted tubule ANS: c) Thick ascending limb of loop of henle d) Collecting ducts 62)Lymph drainage from nipple of breast is to? a) Posterior axillary nodes b) Apical nodes c) Anterior axillary nodes ANS: 63)What is correct about inferior vena cava relations?

a) Lies close to 3rd part of duodenum I DID b) Crosses by illeal mesentry c) Related to left side of aorta 64)What lies close to subepicardium?cant recall rest of scenario… a) SA node ANS: b) AV node c) Bundle of HIS d) Purkinje fibers 65)Breast atrophy in adult female is due to? a) Estrogen ANS: b) Progesteron c) Estrogen and Progesteron 66)Cause of fatty liver in Pakistan? a) Hep B and C ANS: b) Fatty diet Malnutrition 67)A patient is taking fatty diet…which organ is most likely effected? a) Kidney b) Skin c) Brain d) Liver ANS: 68)What lies posterior to periequiductal grey area? a) Substantia nigra b) Red nucleus c) Tectum ANS: d) Pineal glnd e) Colliculus 69)At what level glucose begins to appear in urine? a) 180 b) 200 c) 250 ANS: 70)Maximum number of sodium channels are present in? a) Initial segment of axon b) Node of ranvier ANS: c) Collecting tubules d) Distal convoluted tubules 71)Most important cells of chronic inflammation? a) Lymphocytes b) Macrophages ANS: c) Eiosinophils d) Neutrophils 72)Surgery of parotid gland is done,which muscle will be paralysed? a) Medial pterygoid b) Lateral pterygoid

213 c) Buccinator ANS: d) Temporalis Maseter 73)Content of anterior mediastinum? a) Trachea b) Vagus nerve c) Phrenic nerve d) ThymusANS: 74)Bundle Of His is supplied by? a) LCA ANS: b) RCA c) Carotid artery 75)A very very long scenario ended up by asking parasympathetic supply of descending and sigmoid colon? a) L4 L5 b) S1 S2 S3 c) S3 S4 S5 d) S2 S3 S4 ANS: 76)Lymphatics of gall bladder will first drained into? a) Pre aortic nodes b) Para aortic nodes c) Porta hepatis nodes ANS: 77)Inferior epigastric artery lies lateral to? a) Direct inguinal hernia ANS: b) Inguinal hernia c) Indirect inguinal hernia 78)Patient with sensory loss of left side of face and left side of body…lesion is at? a) Thalamus ANS: b) Pons c) Internal capsule d) Medulla 79)Motor loss of right side of body,something like that?Lesion is at? a) Anterior limb of internal capsule b) Genu+Posterior part of posterior limb of internal capsule c) Genu d) Anterior part of posterior limb of internal capsule e) Genu+Anterior part of posterior limb of internal capsule ANS: 80)Radial artery is accompanied by? a) Median nerve b) Musculocutaneous nerve ANS: c) Deep branch of radial nerve d) Superficial branch of radial nerve

81)A structure at L1 level having coils with grey colour at outer side and black colour in middle? a) Colon b) PYLORUS of Stomach ANS: c) Jejunum d) Ieium 82)The following organism is resistant to stomach acid? a) Vibrio cholera b) Pneumococcus c) Mycobacterium Tuberculosis d) h.pylori ANS: 83)Multifactorial inheritance? a) Sickel cell anemia b) Cystic fibrosis c) Omphalocele ANS: d) Alkaptonuria 84)a patient while walking…pelvis sink to left side is due to? a) Right side of gluteus maximus and medius b) Right sided gluteus maximus medius and minimus ANS: c) Left sided gluteus medius and minimus d) No separate option of right sided gluteus medius and minimus is present there.. 85)A patient having strangulated hernia,strangulation occurs due to? a) Excessive movement of lower limb b) Pressure by inguinal ligament c) Less space of femoral canal ANS: d) Tight boundaries of femoral ring 86)About right brachiocephalic vein,what is correct? a) Begins behind right sternoclavicular joint b) No valves c) Shorter than left d) Vertical ANS: e) More than one options are correct. 87)Descending thoracic aorta passes behind? a) Median arcuate ligament b) Lateral arcuate ligament c) Medial arcuate ligament ANS: 88)A very very long confusing scenario is given,only hint of 9:21 chromosome is given… a) CML ANS: b) Mylodysplastic syndrome

214 c) Muscular dystrophy d) 9:21 89)A patient with history of RTA,CT scan shows a mass in petrous part of temporal bone..where such mass is present? a) Base b) Inferior surface c) Apex d) Anterior surface e) Roof ANS: 90)Superficial dorsal vein of penis drains into? a) Great sephanous vein ANS: b) Internal iliac vein c) Femoral vein d) Santorini plexus

RADIOLOGY OCT 2016

1) Fibrillation commonly occur during A) start of action potential B) End of action potential C) Peak of action potential D) Refractory period ANS B 2) Right testes ca drans to A) Bilateral paraortic B) Ipsilateral pelvic C) Bilateral pelvic D) Bilateral paraortic and ipsilateral pelvic E) Ipsilateral pelvic and ipsilateral paraortic ANS A 3) a docter prescribe vit d3 to pt what it actually is A) Inactive B) 1,25 dihydrocholecalciferol C) 25 hydroxycholecalciferol D) Animal source E) Plant source ANS B 4) Difference in plasma and interstitium A) Proteins B) Calcium C) Sodium ANS A 5) Multigravida women presented with thigh swelling painless expansile and pulsatile below inguinal ligament A) Femoral hernia B) Femoral artery aneurysm C) Saphena varix ANS B

6) Tail of pancrease Ans) Lie in splenorenal ligament 7) Corneal opacities caused by A)Ethambutol B)Chloroquine C)Quinidine ANS B 8) RTA scenario presented with proptosis of eye rushing sound in ear .episcleral vessels dilated A)Retro opthalmic hemorhage B)Floor of or bit damage C)Roof of orbit damage D)Carotid cavernous fistula ANS D 9) Paraneoplastic syndrome caused by A) Small cell ca of lung B) Adenoca stomach C)Renal ca D)Thyroid ca E)Parathyroid ca ANS A 10) Most potent stress hormone A) Acth B)Epinephrine C)Norepinephrine ANS A 11) Raw egg ingestion avidin which biochemical reaction hampered A)Decarboxylation B)Transamination ANS A 12) Glycolysis A)Reversible B)Hexose phosphate converted to lactate and pyruvate ANS B 13) Sarcoidosis associated with A)Pulmonary embolism B)Increased calcium C)Erythema nodosuum D)Hepatic granuloma ANS D 14) Posterior mediastinum desending aorta Ans) Gives 9 post intercostal arteries 15) Extensor and lateral rotation of hip A)Gluteus maximus B)Tensor fascia lata ANS A

215 16) Lesser omentum A) Attach to duodenum B) Forms falciform ligament C) Related to right renal gland D) Left renal gland and left kidney ANS D Note …post wall related to left adrenal and upper pole of left kidney… 17) Extension of epidural space A)Foramen magnum to sacrococcygeal membrane B)Foramen magnum to s3 C) C1 TO s4 D) foraman megnum to l5 ANS A 18) Within kidney renal vessels run in A)Colum of drummond B)Columns of bertin ANS. B 19) 10 th rib fracture scenario A) Spleen damage ANS. A 20) Posterior boundary of epiploic foramen ANS. Ivc 21) nystagmus loss of pain and general sensation on left side of face left sided weakness damage to A)Pica B)Superior cerebellar C)Basilar D)Post communicating artery ANS .A 22) Number of renal segments A). 5 23)Ureter narrowest area A)Sacral promontary B)Sacroiliac joint C)Ischial tuberosity D)Ischial spine E)Just below kidney ANS. D 24) Neural crest derivative A)Ventral horn motor neurons B)Accessory nerve C)Retina D)Vestibular and cochlear nerve nucleus E)Cerebellar nuclei ANS. B 25) Anatomical conduction of heart A)Intercalated disc

B)Gap junction C)Desmosomes D)Nerve fiber ANS. A 26) Prostate weight A)18 gram B) 50 gm C)40 gm D)30 gm E)10 gm ANS .A 27) Four pillar medical ethics A) Beneficence autonomy non maleficence justice ANS. A 28) Symphysis pubis A)Secondary cartilag joint ANS. A 29) Breast lies on A)Pect major B)Serratus anterior ANS. A 30) Solider recieved shrapnel wound in neck after 3 yrs presented with swelling in neck bulges on coughing and sneezing A)Damage to supra plueral membrane B)Unhealed parts of first rib C)Damage to deep facsia in roots of neck ANS. A 31) Wound greenish pus discharge A) Pseudomonas ANS. A 32) Primary sign of shock A) Dec o2 perfusion B) Inc pulse C) Bradycardia Ans. B 33) Important feature of malignant tumor A) Metastasis B) Pleomorphism C) Invasion D) Inc n/c ratio ANS. A 34) Inc mitochondrial activity occurs in apical region of A) Ciliated cells B) Hepatocytes ANS. A 35) In coronary careunit patient expected to show

216 B)Behave calmly without psychiatric problem ANS I marked b 36) Values coming on point or range of measurement scale is called A) Rate B) Data C) Distribution D) Frequency E) Prevalance ANS. D 37) Thirst partially inhibited by A) Gastric distension B) Adh C) Inc blood volumes ANS C( see renal chapter for details) 38) Tachphylaxis A) Unresponsiveness develop rapidly B) Unresponsiveness develop gradually ANS A 39) Free nerve endings A) Non.encapsulated ANS A 40) Hogkin lymphoma A) Contagious spread Lymph nodes B) Extranodal stage common C) Lymphocytes dec in young age ANS A 41) Lesion in temporal lobe A) Memory loss ANS A 42) Dec calcium.dec phosphate normal pth A) Crf B) Inc prarthyroid C) Vit d dec D) Metastatic calcification ANS C 43) Lesion in right parietal lobe A) Memory loss B) Weakness on C) Left right disorientation D) Visual defects E) Olfactory ANS C 44) Franks Starling law A) Inc venous return leads to inc cardiac output B) Inc Venous return leads to inc heart rate ANS A 45) Most immediate response to shock

A) Baroreceptors B) Cns ischemic response ANS A 46) Easy way to diagnose n.gonorhea in exuadate A) Gram stain B) Blood culture C) Zn stain ANS A 47) Vibrio cholera A) Grows best at slight acidic ph B) Nutritionally fastidious C) Tolerance to alkaline ph D) 24 degree ANS A 48) Achalsia scenario A) LES tone dec B) Absent myenteric plexus ANS B 49) Ca cervix drains first into A) Parametrial lymph nodes B) Paracervical lymph nodes C) Internal iliac nodes D) Superficial E) Paraortic ANS C 50) Bitemporal hemianopia due to compression of optic chaisma b/c of anuerysm of A) Ant communicating artery B) Post communicating artery C) Basilar artery ANS A 51) Right cornary artery supplies A) SA node B) Infundibulum of right ventricle C) I/v septum ANS A 52) Anterior fontanella closes at A) 12- 18 months B) 18-24 ANS A 53) Child presented with sudden onset of generalized purpura A) Plt 80000 B) Plt 20000 C) Capillary damage D) Factor 8 deficiency E) Vwf deficiency ANS B

217 54) During swallowing food prevented from entering to trachea by A) Epiglottis elevation over naso pharynx B) Close /-tight approximation of vocal cord C) Movement of uvulva D) Contraction of pharyngeal muscles E) Position of tongue ANS B 55) Cephalic vein A) Pierces clavipectoral fascia B) Lie between deltoid and pect major C) Arise from medial side of dorsal venous arch ANS B 56) Nasopharyngeal ca A) Ebv ANS A 57) Meningitis with rash A) Meningiococemia ANS A 58) Pancreatic fluid drains into A) Lesser sac ANS A 59) Esophageal opening in diaphragm A) T10 ANS A 60) Metabolic function of thyroid at physiological state A) Dec fatty acid synthesis B) Inc fatty acid oxidation C) Dec gluconeogenesis D) Inc synthesis of protein ANS B 61) Orofecal transmission A) Hep e B) Hep b C) Hep c D) Hep d ANS A 62) Peudo hyperparathyroidism A) Pthrp B) Inc secretions from parathyroid C) Other condition secreting paratharmone secretions ANS C 63) Middle age female with epigastric pain radiating to back after fatty meal and to right shoulder A) Gall bladder B) Pancrease ANS A

64) regarding Ovary A) supplied by Ovarian artery B) Drain into paraortic nodes C) Covered by squamous epithelium D) Attached to lateral margin of uterus by mesoovarium E) Both a and b ANS E 65) Infection in pretracheal fascia spread to A) M.sterni B) Retropharyngeal space C) Anterior to pericardium in thoracic cavity ANS C 66) Failure of fusion of median nasal and maxillary process A) Cleft lip B) Cleft palate ANS B 67) Which nerve arises from trunk of brachial plexus A) Thoracodorsal B) Supraclavicular C) Long thoracic nerve D) Medial cutaneous nerve E) Musculocutaneous nerve ANS B 68) Rotator cuff formed by A) SITS ANS A 68) Esinophilic infiltrates characteristics of Ans) Parasitic infestation 69) Bladder A) Devoid of peritonium B) Drains to superficial inguinal lymph nodes C) Drain to internal iliac ANS C 70) Cause of edema in nephrotic syndrome is A) Hypoalbuminemia B) Inc hydrostatic pressure ANS A 71) Superficial petrosal sinus lie in A) Falx cerebelli B) Tentorium cerebella ANS B 72) Disorganized cell size shape. Strucure A)Metaplasia B) Dysplasia

218 ANS B 73) Tb confirm by A) Acid fast ANS A 74) Cd4+ cells A) Helper t lymphos B) Cytotoxic lymphos ANS A 75) Dec ach function, ptosis lethargy weakness A) M.gravis B) Lambert eaton syndrome ANS A 76) Acute inflamation characterized by??? A.Lecocytosis ANS A 77) Detrosor muscle contraction controlled by A) Inf hypogastric plexus+parasym pelvic ANS A 78) Dna virus causing respiratory. Infection A) Adenovirus ANS A 79) bare area of liver is limited by A) coronary ligament ANS A 80) pt can stand on feet but hurting A) plantaris tendon rupture ANS A(see my post in details for dis) 81) area of tricuspid auscultation Ans) right side of lower body of sternum 82) trachea bifurcation level, A) c5_ c6 ANS A 83) trachea extension level, A) t5 to c6 ANS A 84) dependent edema scenario

A) ivc obstruction ANS A 85) somites number AT 30 DAY Of development A) 42 to 44 B) 32-36 ANS B 86) vit b12 deficency scenario person feels that she is walking on wool coton floor, posterior cruciate A) ligament tibia moves forward ANS A 87) nucleus of vagus, A) Dorsal nucleus 88) Obstruction in inferior vena cava superior to azygous vein leads to diversion of blood to A) Hepatic vein B) Ivc C) azygous vein ANS C 89) Example of delayed type 4 reaction A) Tb B) Leprosy ANS A 90) Virus causes neoplastic changes by alteration in Dna Rna A) Protooncogenes B) Oncogenes ANS B 91) Metastatic calcification A) Ghon focus B) Peudocyst pancrease C) Atherosclerosis calcification D) Nephrocalcinosis ANS D

Golden file 3, september, oct , november 2016 all papers.pdf ...

Golden file 3, september, oct , november 2016 all papers.pdf. Golden file 3, september, oct , november 2016 all papers.pdf. Open. Extract. Open with. Sign In.

3MB Sizes 31 Downloads 603 Views

Recommend Documents

File 352 Golden apple 3.pdf
Retrying... Download. Connect more apps... Try one of the apps below to open or edit this item. File 352 Golden apple 3.pdf. File 352 Golden apple 3.pdf. Open.

#Protiva___Quarterly Activity Plan-September to November, 2016.pdf ...
batch fellowship &. Review (4th) fellows monthly. work plan. 1 Sat Weekend 1 Tue Review fellows monthly. work plan. 2 Assist to Capacity Building (CB) Project.

September 24, 2014 16:3 WSPC/INSTRUCTION FILE ...
Sep 24, 2014 - Optimal allocation is an increasingly important area in credit market modeling. ... chain is composed of a “good” state, which is the portfolio .... The P&L, at time t ≤ T, of a buy and hold position on the asset k ∈ {1,...,K}

SY 16-17 PTC Schedule #1 (September, October, & November 2016 ...
Retrying... SY 16-17 PTC Schedule #1 (September, October, & November 2016).pdf. SY 16-17 PTC Schedule #1 (September, October, & November 2016).pdf.

Advert. - All Four Courses (Revised November 3 2017) (003).pdf ...
3 days ago - Retrying... Advert. - All Four Courses (Revised November 3 2017) (003).pdf. Advert. - All Four Courses (Revised November 3 2017) (003).pdf.

Teacher August September October November ...
Megan Scott. Taylor Watkins. Orlowski. Kathryn Nelson. Lauren Snyder. Elijah Burns. Kolbie Mason. Emma Hepler. Ousley. Isabelle Schmucker. Emily Miller. Montana ... Jason Slisz. Olivia Shively. Kelsey Hamilton. Hope Miller. Slein. Mindi Chapman. Chas

40 Nieuwsbrief VH -3 september 2016.pdf
There was a problem previewing this document. Retrying... Download. Connect more apps... Try one of the apps below to open or edit this item. 40 Nieuwsbrief ...

TZ PTO Meeting Minutes November 3, 2016.pdf
Announcements December 5th POTH parents meeting6:30 in cafeteria. X. Adjournment 7:12 p.m.. Page 2 of 2. TZ PTO Meeting Minutes November 3, 2016.pdf.

Minutes of the COMP meeting 3-4 November 2016 - European ...
Jan 9, 2017 - Send a question via our website www.ema.europa.eu/contact ...... that the best standard of care was employed by using combination therapies ..... the treatment of patients with graft vs host disease for orphan designation.

Minutes of the COMP meeting 3-4 November 2016 - European ...
Jan 9, 2017 - access to documents within the framework of Regulation (EC) No 1049/2001 as ... Applications for orphan medicinal product designation ...... To establish correctly if there exists a scientific rationale for the development of the.

DH Issue 3 Volume 18 November 2016.pdf
Athlete Leadership Academy strives to. make an impact in the school, community. Winter Sports Schedules. SPORTS. -Photo courtesy of Ms. Robtison.

Minutes of the CAT meeting 3-4 November 2016 - European ...
Dec 9, 2016 - Send a question via our website www.ema.europa.eu/contact ..... Intended for acute graft versus host disease grades III and IV resistant to the ...

November 7, 2016 November 8, 2016
Nov 8, 2016 - All submissions should be emailed to .... All students, whether or not they purchase a pumpkin, will receive a cup of apple juice and a treat.

September 2016
March 2018 Day Cycle Calendar. Sun. Mon. Tue. Wed. Thu. Fri. Sat. 1. 6. 2:50 Student Council. Meeting. 2:50 Math Club. 2:50 iCARE Student. Committee (Best Buddies). 2. 1. Choices Field Trip for. Selected 8th Graders. 7:00-9:00 Grade 7/8. Fun Night. 3

September 2016
All are welcome. From the President .... Celebration welcome table. Space is limited .... Stay in touch with the Cayuga Bird Club through our Facebook page and ...

September 2016 - Snell & Wilmer
Sep 27, 2016 - about my role in building an excellent Education Law practice and continuing to serve so many clients who work day in and day out to improve ...

Minutes of the CAT meeting 3-4 November 2016 - European ...
Dec 9, 2016 - 30 Churchill Place ○ Canary Wharf ○ London E14 5EU ○ United Kingdom. An agency of ...... Meeting run with support from relevant EMA staff.

DH Issue 3 Volume 18 November 2016.pdf
Page 1 of 3. Avon Grove High School, West Grove, Pa. The Devil's Herald. Friday, November 18, 2016 Volume 18 Issue 3. By. Gabby Pino. Editor. One of the goals of the new I/E. periods is to let students expand. in areas they wouldn't be able to. in a

November 2016 - Snell & Wilmer
Nov 22, 2016 - the beneficent author of all the good that was, that is, or that will be; that ... such a degree of temporal prosperity as He alone knows to be best.

September 2016
Feb 1, 2018 - 4:15 Girls Basketball at. Bethlehem. 9. 4. Jump Rope for Heart. Ends. 7:00 pm The Addams. Family. 10. 7:00 pm The. Addams. Family. 11. 2:00 pm. The Addams. Family. 12. 5. 13. 6. 14. 1. 15. 2. 2:50 iCARE student Committee. –Cafeteria.

Oct 2016.pdf
approval of John Forney sport event worker, and Donald Kochel, volunteer softball coach. Roll call vote, all members voting yes. Motion carried 7-0. I.U. #13 ...

OCT. 2016.pdf
029201 10-14-2016 GENERAL BINDING CORP. 43.78 N .... 2016.pdf. OCT. 2016.pdf. Open. Extract. Open with. Sign In. Details. Comments. General Info. Type.

september (3).pdf
SMK8! Win the race to reach. your reading goal each nine. weeks with lots of great. reading! We can't wait to. celebrate all of you that. read to the finish line and.

Oct 2016.pdf
our concessions at the Chuck Wagon for the season. Although we had much fewer home. games to sell and fund raise, we had record sales and are well on our ...